Teme si probleme pentru concursurile studentesti de matematica ...

258

Transcript of Teme si probleme pentru concursurile studentesti de matematica ...

Page 1: Teme si probleme pentru concursurile studentesti de matematica ...
Page 2: Teme si probleme pentru concursurile studentesti de matematica ...

Cuprins

II. Analiza matematica 0

7 Siruri si serii numerice 1

8 Calcul diferential pentru functii de o variabila reala 43

9 Calcul integral pentru functii de o variabila reala 62

10 Functii de mai multe variabile reale 93

11 Siruri si serii de functii: serii Taylor, serii Fourier 122

12 Functii complexe 170

III. Matematici discrete 195

13 Combinatorica si grafuri 196

14 Aritmetica si teoria numerelor 232

Page 3: Teme si probleme pentru concursurile studentesti de matematica ...

Lucrarea a fost elaborata dupa cum urmeaza:

Capitolul 7. Vasile Pop, Mircea Olteanu

Capitolul 8. Liliana Popa

Capitolul 9. Dorian Popa, Vasile Pop

Capitolul 10. Dorian Popa

Capitolul 11. Mircea Olteanu, Radu Strugariu

Capitolul 12. Liliana Popa

Capitolul 13. Monica Burlica, Mihai Ispas

Capitolul 14. Gabriel Mincu

Page 4: Teme si probleme pentru concursurile studentesti de matematica ...

Prefata

Cartea de fata a fost elaborata ın cadrul proiectului POSDRU/56/1.2/S/32768, ”For-marea cadrelor didactice universitare si a studentilor ın domeniul utilizarii unor instru-mente moderne de predare-ınvatare-evaluare pentru disciplinele matematice, ın vedereacrearii de competente performante si practice pentru piata muncii”.Finantat din Fondul Social European si implementat de catre Ministerul Educatiei,Cercetarii, Tineretului si Sportului, ın colaborare cu The Red Point, Oameni si Com-panii, Universitatea din Bucuresti, Universitatea Tehnica de Constructii din Bucuresti,Universitatea ”Politehnica” din Bucuresti, Universitatea din Pitesti, Universitatea Tehnica”Gheorghe Asachi” din Iasi, Universitatea de Vest din Timisoara, Universitatea ”Dunareade Jos” din Galati, Universitatea Tehnica din Cluj-Napoca, Universitatea ”1 Decembrie1918” din Alba-Iulia, proiectul contribuie ın mod direct la realizarea obiectivului generalal Programului Operational Sectorial de Dezvoltare a Resurselor Umane - POSDRU si seınscrie ın domeniul major de interventie 1.2 Calitate ın ınvatamantul superior.Proiectul are ca obiectiv adaptarea programelor de studii ale disciplinelor matematicela cerintele pietei muncii si crearea de mecanisme si instrumente de extindere a oportu-nitatilor de ınvatare.Evaluarea nevoilor educationale obiective ale cadrelor didactice si studentilor legate de uti-lizarea matematicii ın ınvatamantul superior, masterate si doctorate precum si analizareaeficacitatii si relevantei curriculelor actuale la nivel de performanta si eficienta, ın vedereadezvoltarii de cunostinte si competente pentru studentii care ınvata discipline matematiceın universitati, reprezinta obiective specifice de interes ın cadrul proiectului. Dezvoltareasi armonizarea curriculelor universitare ale disciplinelor matematice conform exigentelorde pe piata muncii, elaborarea si implementarea unui program de formare a cadrelordidactice si a studentilor interesati din universitatile partenere bazat pe dezvoltarea siarmonizarea de curriculum, crearea unei baze de resurse inovative, moderne si functionalepentru predarea-ınvatarea-evaluarea ın disciplinele matematice pentru ınvatamantul uni-versitar sunt obiectivele specifice care au ca raspuns materialul de fata.Formarea de competente cheie ın matematica si informatica presupune crearea de abilitatide care fiecare individ are nevoie pentru dezvoltarea personala, incluziune sociala si insertiepe piata muncii. Se poate constata ınsa ca programele disciplinelor de matematica nu auıntotdeauna ın vedere identificarea si sprijinirea elevilor si studentilor potential talentati lamatematica. Totusi, studiul matematicii a evoluat ın exigente pana la a ajunge sa accepteprovocarea de a folosi noile tehnologii ın procesul de predare-ınvatare-evaluare pentru aface matematica mai atractiva. In acest context, analiza flexibilitatii curriculei, ınsotita deanaliza metodelor si instrumentelor folosite pentru identificarea si motivarea studentilortalentati la matematica ar putea raspunde deopotriva cerintelor de masa, cat si celor deelita.Viziunea pe termen lung a acestui proiect preconizeaza determinarea unor schimbari ınabordarea fenomenului matematic pe mai multe planuri: informarea unui numar cat mai

Page 5: Teme si probleme pentru concursurile studentesti de matematica ...

Prefata

mare de membri ai societatii ın legatura cu rolul si locul matematicii ın educatia de baza,ın instructie si ın descoperirile stiintifice menite sa ımbunatateasca calitatea vietii, inclu-siv popularizarea unor mari descoperiri tehnice, si nu numai, ın care matematica cea maiavansata a jucat un rol hotarator. De asemenea, se urmareste evidentierea a noi motivatiisolide pentru ınvatarea si studiul matematicii la nivelele de baza si la nivel de performanta;stimularea creativitatii si formarea la viitorii cercetatori matematicieni a unei atitudini de-schise fata de ınsusirea aspectelor specifice din alte stiinte, ın scopul participarii cu succesın echipe mixte de cercetare sau a abordarii unei cercetari inter si multi disciplinare;identificarea unor forme de pregatire adecvata de matematica pentru viitorii studenti aidisciplinelor matematice ın scopul utilizarii la nivel de performanta a aparatului matematicın construirea unei cariere profesionale.

Page 6: Teme si probleme pentru concursurile studentesti de matematica ...

Introducere

Concursurile de matematica, nationale si internationale pentru elevi au o traditieındelungata, primul concurs international fiind organizat la initiativa Romaniei, ınRomania ın anul 1959 (Olimpiada Internationala de Matematica). In toti acesti ani, lanivelul matematicii preuniversitare s-a ajuns la o programa de concurs comuna, unanimacceptata de toate tarile participante la OIM (ın prezent peste 120 de tari) iar concur-sul reprezinta pentru multi dintre participanti cel mai important test de verificare alnivelului pregatirii matematice si ın acelasi timp un barometru pentru nivelul matematiciicompetitionale al tarii din care provin.

Este de dorit ca si la nivel universitar competitiile internationale sa urmeze modelulOIM, ın special ca forma de organizare si ca programa de concurs general acceptata sicunoscuta.

La nivel universitar concursurile de matematica s-au desfasurat foarte mult timp doarla nivel national ın diverse tari si ın multe cazuri sporadic. Cea mai veche competitienationala cu desfasurare neıntrerupta este concursul Putnam, organizat ın Statele Uniteale Americii ıncepand cu anul 1938. In Romania, Concursul National Studentesc ”TraianLalescu” s-a desfasurat la mai multe discipline, s-a ıntrerupt ın perioada 1992-2006 si afost reluat din 2007 la matematica.

Cea mai importanta competitie internationala de matematica pentru studenti esteIMC (International Mathematics Competition for University Students) care se organizeazaitinerant din 1994 fiind echivalentul Olimpiadei Internationale de Matematica la niveluniversitar. In ultimii ani la aceasta competitie participa peste 300 de studenti din peste 70de universitati si peste 30 de tari. Competitia este individuala iar fiecare echipa reprezinta ouniversitate (nu o tara). Dificultatea problemelor date ın concurs este deosebit de ridicata,iar rezultatul este edificator: concursul se desfasoara pe durata a doua zile si se dau 5 sau6 probleme ın fiecare zi.

Incepand din 2007 se desfasoara Concursul International Studentesc SEEMOUS (SouthEastern European Mathematical Olympiad for University Students), analogul OlimpiadeiBalcanice de Matematica pentru elevi, la care au participat ın fiecare an studenti de launiversitati din Romania (Bucuresti, Cluj-Napoca, Iasi, Timisoara).

Aceasta culegere de probleme a fost gandita pentru a pune la dispozitia studentilordin Romania un material necesar pentru o buna pregatire matematica ın vederea ridicariinivelului pregatirii obisnuite la nivel competitional (national sau international). Laelaborarea cartii au fost implicati profesori cu experienta la concursurile nationale siinternationale studentesti.

In elaborarea programei care sta la baza culegerii am decis, dupa discutii cureprezentanti ai majoritatii universitatilor din tara, sa folosim curricula concursurilorinternationale de matematica la care studentii de la universitatile din Romania participacel mai frecvent.

Problemele au fost ımpartite pe teme ın 14 capitole:

Page 7: Teme si probleme pentru concursurile studentesti de matematica ...

Introducere

• Algebra - capitolele 1 si 2,• Algebra liniara - capitolele 3, 4, 5,• Geometrie analitica - capitolul 6,• Analiza reala (functii de o variabila) - capitolele 7, 8, 9,• Analiza matematica (functii de mai multe variabile) - capitolul 10,• Siruri si serii de functii - capitolul 11,• Functii complexe - capitolul 12,• Matematici discrete - capitolele 13 si 14.

Fiecare capitol ıncepe cu o prezentare a notiunilor si rezultatelor necesare rezolvariiproblemelor, urmata de un numar suficient de probleme rezolvate, unele clasice, dar sem-nificative, altele pentru antrenament si altele selectate din concursurile internationale saunationale ale altor tari ca: Rusia, Franta, Iran, S.U.A., Ungaria, Cehia, Israel.

Culegerea contine peste 600 de probleme cu rezolvari complete, o lista de peste 50 detitluri bibliografice (carti editate ın tara sau ın strainatate), precum si o lista de adresede Internet ale diverselor concursuri internationale studentesti. Dupa cunostinta autoriloraceasta culegere este prima ın lume care trateaza o astfel de tematica la modul general,nefiind dedicata doar unui anumit concurs.

Fiecare capitol al culegerii a fost elaborat de unul sau doi dintre cei 11 autori si fiecarea putut contribui cu probleme la orice alt capitol. De coordonarea ıntregii culegeri sifinalizarea ei s-au ocupat conf. dr. Vasile Pop de la Universitatea Tehnica din Cluj-Napocasi conf. dr. Cornel Baetica de la Universitatea din Bucuresti.

Page 8: Teme si probleme pentru concursurile studentesti de matematica ...

Capitolul 7

Siruri si serii numerice

Definitii si rezultate

Teorema Stolz-Cesaro 1. Fie (an)n≥0, (bn)n≥0 doua siruri de numere reale cu pro-prietatile urmatoare:

1) (bn)n≥0 este strict monoton si nemarginit;

2) exista limn→∞

an+1 − anbn+1 − bn

= l, l ∈ R.

Atunci limn→∞

anbn

= l.

Teorema Stolz-Cesaro 2. Fie (an)n≥0, (bn)n≥0 doua siruri de numere reale cu pro-prietatile urmatoare:

1) limn→∞

an = limn→∞

bn = 0;

2) sirul (bn)n≥0 este strict monoton;

3) exista limn→∞

an+1 − anbn+1 − bn

= l, l ∈ R.

Atunci limn→∞

anbn

= l.

Corolar. Fie (an)n≥0 un sir de numere pozitive cu proprietatea ca exista

limn→∞

an+1

an= l, l ∈ R. Atunci lim

n→∞n√an = l.

Teorema. Fie (an)n≥1 un sir de numere pozitive cu proprietatea ca exista limn→∞

an+1

an=

l. Atunci, daca l < 1 ⇒ limn→∞

an = 0, iar daca l > 1 ⇒ limn→∞

an =∞.

• Fie

∞∑n=1

an o serie de numere reale. Sirul (sn)n≥1, unde sn =

n∑k=1

ak, se numeste sirul

sumelor partiale ale seriei.

• Daca exista limita sirului (sn)n≥1, atunci ea se numeste suma seriei.

• Daca sirul sumelor partiale este convergent si limn→∞

sn = s, atunci se spune ca seria∞∑n=1

an este convergenta si se scrie

∞∑n=1

an = s.

• Daca seria∞∑n=1

|an| este convergenta se spune ca seria∞∑n=1

an este absolut conver-

1

Page 9: Teme si probleme pentru concursurile studentesti de matematica ...

2

genta.

• O serie care este convergenta, dar nu este absolut convergenta se numeste seriesemiconvergenta.

Observatii. a) Dintr-o serie data∞∑n=1

an se pot obtine alte serii, prin schimbarea ordinei

termenilor (∞∑n=1

aσ(n), σ : N∗ → N∗ bijectiva) sau prin asocierea unor termeni (

∞∑n=1

(af(n)+1+

af(n)+2 + · · · + af(n+1)), unde f : N∗ → N∗ este o functie strict crescatoare). In general,aceste transformari pot schimba suma seriei si chiar natura seriilor.

In cazul seriilor absolut convergente avem:Teorema. Daca ıntr-o serie absolut convergenta schimbam ordinea termenilor sau asociemsecvente de termeni, seria obtinuta are aceeasi suma cu seria initiala.

In cazul seriilor semiconvergente situatia este complet diferita dupa cum arataurmatoarea:Teorema (Riemann). Intr-o serie semiconvergenta se poate schimba ordinea termenilorın asa fel ıncat seria sa fie divergenta sau sa fie convergenta cu suma un numar real ar-bitrar.

b) Pentru fiecare numar natural m ∈ N∗ definim seria rest de ordin m prin Rm =

∞∑n=m

an.

Seria

∞∑n=1

an are aceeasi natura cu orice serie rest a ei.

c) Daca seria∞∑n=1

an este convergenta, atunci sirul (an)n este convergent la zero.

Un criteriu de divergenta este urmatorul:

C0. Daca sirul (an)n nu converge la zero, atunci seria∞∑n=1

an este divergenta.

Seria geometrica

Daca q este un numar real, atunci seria∞∑n=0

qn se numeste seria geometrica de ratie q.

Pentru q ∈ (−1, 1) seria geometrica este convergenta si suma ei este∞∑n=0

qn =1

1− q.

Pentru q ≥ 1 seria este divergenta si are suma ∞.Pentru q ≤ −1 seria este divergenta si nu are suma.

Seria armonica generalizata

Daca α este un numar real, atunci seria∞∑n=1

1

nαse numeste serie armonica generalizata

de exponent α.

Pentru α > 1 seria armonica∞∑n=1

1

nαeste convergenta si suma ei se noteaza

∞∑n=1

1

nα=

ζ(α). Functia ζ : (1,∞)→ R se numeste functia ”zeta” a lui Riemann. Pentru α ≤ 1 seria

Page 10: Teme si probleme pentru concursurile studentesti de matematica ...

Siruri si serii numerice 3

armonica

∞∑n=1

1

nαeste divergenta si are suma ∞.

Criterii generale de convergenta

C1. (Criteriul general al lui Cauchy) Seria

∞∑n=1

an este convergenta daca si numai

daca, pentru orice ε > 0 exista un rang N(ε) ∈ N astfel ca pentru orice n ≥ N(ε) si oricep ≥ 1 sa avem:

|an+1 + an+2 + · · ·+ an+p| < ε.

C2. (Criteriul lui Abel-Dirichlet) Daca seria

∞∑n=1

an are sirul sumelor partiale

marginit, iar sirul (bn)n este descrescator la zero, atunci seria

∞∑n=1

anbn este convergenta.

C3. (Criteriul lui Abel) Daca seria

∞∑n=1

an este convergenta iar sirul (bn)n este

monoton si marginit, atunci seria∞∑n=1

anbn este convergenta.

C4. (Criteriul lui Leibniz) Daca sirul (bn)n≥1 este monoton si convergent la zero,

atunci seria∞∑n=1

(−1)nbn este convergenta.

Criterii de convergenta pentru serii cu termeni pozitivi

In urmatoarele criterii (C4-C10) termenii seriilor care apar sunt strict pozitivi.

A. Criterii intrinseci

C4. Criteriul raportului (d’Alembert)

a) Daca exista q ∈ (0, 1) si N ∈ N∗ astfel caan+1

an≤ q pentru orice n > N , atunci seria

∞∑n=1

an este convergenta.

b) Daca exista N ∈ N∗ astfel caan+1

an≥ 1 pentru orice n > N , atunci seria

∞∑n=1

an

este divergenta.

C4’. Daca exista limita limn→∞

an+1

an= l atunci:

a) pentru l ∈ [0, 1) seria

∞∑n=1

an este convergenta;

b) pentru l ∈ (1,∞) seria∑n≥1

an este divergenta;

c) pentru l = 1 criteriul este ineficient.

C5. Criteriul radicalului (Cauchy)

Page 11: Teme si probleme pentru concursurile studentesti de matematica ...

4

a) Daca exista q ∈ (0, 1) si N ∈ N∗ astfel ca n√an ≤ q pentru orice n > N , atunci seria

∞∑n=1

an este convergenta.

b) Daca exista o infinitate de termeni pentru care n√an ≥ 1 atunci seria este divergenta.

C5’. Daca exista limn→∞

n√an = l atunci:

a) pentru l ∈ [0, 1) seria

∞∑n=1

an este convergenta;

b) pentru l ∈ (1,∞) seria∑n≥1

an este divergenta;

c) pentru l = 1 criteriul este ineficient.

C6. Criteriul Raabe-Duhamela) Daca exista un numar real c > 1 si un numar natural N ∈ N∗ astfel ca

n

(anan+1

− 1

)≥ c, pentru orice n ≥ N,

atunci seria

∞∑n=1

an este convergenta.

b) Daca exista un numar natural N pentru care

n

(anan+1

− 1

)≤ 1, pentru orice n ≥ N,

atunci seria∑n≥1

an este divergenta.

C6’. Daca exista limita limn→∞

n

(anan+1

− 1

)= l atunci:

a) pentru l > 1 seria

∞∑n=1

an este convergenta;

b) pentru l < 1 seria∑n≥1

an este divergenta;

c) pentru l = 1 criteriul este ineficient.Observatie. In general criteriul Raabe-Duhamel se aplica la serii la care criteriul

raportului sau radicalului este ineficient.

C7. Criteriul condensarii (Cauchy)

Daca sirul (an)n este descrescator, atunci seriile

∞∑n=1

an si

∞∑n=1

2na2n au aceeasi natura

(sunt simultan convergente sau divergente).

B. Criterii de comparatie

C8. Daca exista N ∈ N∗ astfel ca 0 < an ≤ bn pentru orice n > N , atunci:

a) Daca seria∞∑n=1

an este divergenta, atunci seria

∞∑n=1

bn este divergenta.

Page 12: Teme si probleme pentru concursurile studentesti de matematica ...

Siruri si serii numerice 5

b) Daca seria

∞∑n=1

bn este convergenta, atunci seria

∞∑n=1

an este convergenta.

C9. Daca exista N ∈ N∗ astfel caan+1

an≤ bn+1

bnpentru orice n > N , atunci:

a) Daca seria

∞∑n=1

an este divergenta, atunci seria∞∑n=1

bn este divergenta.

b) Daca seria∞∑n=1

bn este convergenta, atunci seria

∞∑n=1

an este convergenta.

C10. Daca exista limn→∞

anbn

= l atunci:

a) pentru l ∈ (0,∞) seriile

∞∑n=1

an si

∞∑n=1

bn au aceeasi natura;

b) pentru l = 0 avem implicatiile:∞∑n=1

an divergenta ⇒∞∑n=1

bn divergenta;

∞∑n=1

bn convergenta ⇒∞∑n=1

an convergenta;

c) pentru l =∞ avem implicatiile:∞∑n=1

bn divergenta ⇒∞∑n=1

an divergenta;

∞∑n=1

an convergenta ⇒∞∑n=1

bn convergenta.

Observatie. In general pentru a decide natura unei serii∞∑n=1

an prin criteriul C10 se

folosesc pentru comparatie serii armonice generalizate. Se obtine criteriul 10’.C10’. Daca exista α ∈ R astfel ca

limn→∞

nαan = l ∈ (0,∞)

atunci:

a) pentru α > 1 seria

∞∑n=1

an este convergenta;

b) pentru α ≤ 1 seria∞∑n=1

an este divergenta.

Produsul Cauchy a doua serii

Definitie. Daca∞∑n=1

an si∞∑n=1

bn sunt doua serii, atunci seria∞∑n=1

cn cu termenul general

cn = a1bn+a2bn−1 +a3bn−2 + · · ·+anb1, n ≥ 1, se numeste produsul Cauchy al celor douaserii.

Observatie. In general produsul Cauchy a doua serii convergente nu este neaparat o

serie convergenta (an = bn =(−1)n−1

√n

).

Teorema (Mertens). Daca seriile

∞∑n=1

an si

∞∑n=1

bn sunt convergente, iar una din ele

Page 13: Teme si probleme pentru concursurile studentesti de matematica ...

6

este absolut convergenta, atunci produsul lor Cauchy

∞∑n=1

cn este o serie convergenta si

daca∞∑n=1

an = A,

∞∑n=1

bn = B, atunci

∞∑n=1

cn = AB.

Siruri. Probleme

Problema 7.1 Fie I ⊆ R si f : I → I. Definim sirul (an)n≥0 prin relatia an+1 = f(an),n ≥ 0, a0 ∈ I. Sa se arate ca:

1) Daca f este crescatoare, atunci (an)n≥0 este monoton;2) Daca f este descrescatoare, atunci sirurile (a2n)n≥0, (a2n+1)n≥0 sunt monotone si

au monotonii diferite.

Solutie. 1) Daca a0 ≤ a1 rezulta ca f(a0) ≤ f(a1), adica a1 ≤ a2 si apoi prin inductiese arata ca an ≤ an+1 pentru orice n ≥ 0. Daca a0 ≥ a1 rezulta analog ca sirul estedescrescator.

2) Avem

a2n+1 = f(a2n+1) = (f f)(a2n), n ≥ 0

si

a2n+2 = f(a2n+1) = (f f)(a2n), n ≥ 0.

Cum g = f f este crescatoare, din punctul 1) rezulta ca (a2n)n≥0 si (a2n+1)n≥0 suntsiruri monotone. Daca presupunem ca (a2n)n≥0 este crescator, din relatia a2n ≤ a2n+2

obtinem f(a2n) ≥ f(a2n+1) echivalent cu a2n+1 ≥ a2n+3, n ≥ 0, ceea ce arata ca (a2n+1)n≥0

este descrescator. Presupunerea ca (a2n)n≥0 este descrescator conduce ın mod analog lafaptul ca (a2n+1)n≥0 este crescator. Deci sirurile (a2n)n≥0 si (a2n+1)n≥0 au monotoniidiferite.

Problema 7.2 a) Sa se arate ca limn→∞

(1

n+ 1+

1

n+ 2+ · · ·+ 1

2n

)= ln 2;

b) Sa se calculeze limn→∞

n

(1

n+ 1+

1

n+ 2+ · · ·+ 1

2n− ln 2

).

Solutie. a) Fie cn = 1 +1

2+ · · ·+ 1

n− lnn, n ≥ 0. Avem

xn =1

n+ 1+

1

n+ 2+ · · ·+ 1

2n− ln 2 = (c2n − cn) + ln 2n− lnn =

= c2n − cn + ln 2,

de unde obtinem limn→∞

xn = ln 2.

b) Fie yn =

1

n+ 1+

1

n+ 2+ · · ·+ 1

2n− ln 2

1

n

, n ≥ 1,

an =1

n+ 1+

1

n+ 2+ · · ·+ 1

2n− ln 2, bn =

1

n.

Page 14: Teme si probleme pentru concursurile studentesti de matematica ...

Siruri si serii numerice 7

Conditiile celei de-a doua teoreme a lui Stolz-Cesaro sunt ındeplinite si avem

limn→∞

an+1 − anbn+1 − bn

limn→∞

− 1

n+ 1+

1

2n+ 1+

1

2n+ 21

n+ 1− 1

n

= −1

4

de unde rezulta ca limn→∞

yn = −1

4.

Problema 7.3 Fie f : [1,∞) → R o functie descrescatoare si marginita inferior. Sa searate ca sirul (an)n≥1 de termen general

an = f(1) + f(2) + · · ·+ f(n)−∫ n

1f(x)dx

este convergent.

Solutie. Studiem monotonia lui (an)n≥1. Avem

an+1 − an = f(n+ 1)−∫ n+1

1f(x)dx+

∫ n

1f(x)dx =

= f(n+ 1)−∫ n+1

nf(x)dx =

∫ n+1

n(f(n+ 1)− f(x))dx ≤ 0,

tinand seama ca f este descrescatoare. Rezulta ca sirul (an)n≥1 este descrescator. Demon-stram ca sirul este marginit inferior. Avem

an =

(f(1)−

∫ 2

1f(x)dx

)+

(f(2)−

∫ 3

2f(x)dx

)+ · · ·+

+

(f(n− 1)−

∫ n

n−1f(x)dx

)+ f(n) =

=

∫ 2

1(f(1)− f(x))dx+

∫ 3

2(f(2)− f(x))dx+ · · ·+

+

∫ n

n−1(f(n− 1)− f(x))dx+ f(n),

de unde rezulta ca (an)n≥1 este marginit inferior, tinand seama de monotonia lui f side faptul ca f este marginita inferior. Prin urmare sirul (an)n≥1 este convergent, fiindmonoton si marginit.

Observatie. Pentru functia f : [1,∞)→ R, f(x) =1

x, rezulta imediat ca sirul (cn)n≥1,

cn = 1 +1

2+ · · ·+ 1

n− lnn

este convergent.

Problema 7.4 Sa se calculeze limn→∞

[(n+ 1) n+1√n+ 1− n n

√n].

Page 15: Teme si probleme pentru concursurile studentesti de matematica ...

8

Solutie. Consideram functia f : [n, n+1]→ R, n ∈ N∗, f(x) = x1+ 1x , careia ıi aplicam

teorema lui Lagrange. Rezulta ca exista cn ∈ (n, n+ 1) astfel ca

f(n+ 1)− f(n) = c1cnn

(1

cn+ 1− ln cn

cn

).

Din cn > n rezulta ca limn→∞

cn =∞ si ın continuare

limn→∞

[f(n+ 1)− f(n)] = 1.

Problema 7.5 Demonstrati ca daca sinx 6= 0, atunci sirul (sinnx)n≥0 nu are limita.

Solutie. Sa presupunem ca sirul (sinnx)n≥0 este convergent. Din

cosnx =sin(n+ 1)x− sin(n− 1)x

2 sinx

rezulta ca limn→∞

cosnx = 0.

Tinand seama de relatia

sinnx =cos(n+ 1)x− cos(n− 1)x

2 sinx

deducem ca limn→∞

sinnx = 0, prin urmare limn→∞

(sin2 nx+cos2 nx) = 0, contradictie. Rezulta

ca sirul (sinnx)n≥0 este divergent.

Problema 7.6 Sa se determine cel mai mic numar real pozitiv x pentru care sirul (an)n≥1,

an =

(1 +

1

n

)n+x

este descrescator.

Solutie. Consideram functia f : [1,∞)→ R, f(t) = (t+x) ln

(1 +

1

t

), t ≥ 1. Evident

an = ef(n), n ≥ 1. Avem

f ′(t) = ln

(1 +

1

t

)− t+ x

t(1 + t),

f ′′(t) =t(2x− 1) + x

t2(1 + t)2.

Daca x ≥ 1

2rezulta f ′′(t) ≥ 0 pentru orice t ≥ 1, deci f ′ este strict crescatoare

pe [1,∞). Cum limt→∞

f ′(t) = 0 rezulta f ′(t) < 0, t ≥ 1, deci f este descrescatoare pe

[1,∞). Rezulta ca (an)n≥1 este un sir descrescator pentru x ≥ 1

2. Daca x <

1

2, atunci

ecuatia f ′′(t) = 0 are radacina t0 =x

1− 2xsi f ′′(t) ≤ 0 pentru t ≥ t0. Rezulta ca f ′

este descrescatoare pe [t0,∞) si cum limt→∞

f ′(t) = 0 avem f ′(t) > 0 pentru t ≥ t0. Prin

urmare sirul (an) este crescator pentru n > t0. Cel mai mic numar pentru care (an)n≥1

este descrescator este x =1

2.

Problema 7.7 Sa se arate ca daca limn→∞

ann = a, limn→∞

bnn = b, a, b > 0, atunci pentru orice

p ≥ 0, q ≥ 0 cu p+ q = 1, are loc relatia

limn→∞

(pan + qbn)n = apbq.

Page 16: Teme si probleme pentru concursurile studentesti de matematica ...

Siruri si serii numerice 9

Solutie. Aratam mai ıntai ca limn→∞

an = 1 si limn→∞

bn = 1. De aici deducem ca

limn→∞

(pan + qbn) = 1.

Apoi avemlimn→∞

n(an − 1) = ln a, limn→∞

n(bn − 1) = ln b

si ın continuare

limn→∞

(pan + qbn)n = elimn→∞

n(pan+qbn−1)=

= elimn→∞

[pn(an−1)+qn(bn−1)]= ep ln a+q ln b = apbq.

Problema 7.8 Sa se calculeze limn→∞

(e1+ 1

2+···+ 1

n+1 − e1+ 12

+···+ 1n

).

Solutie. Fie cn = 1 + 12 + · · ·+ 1

n − lnn. Avem

xn = e1+ 12

+···+ 1n+1 − e1+ 1

2+···+ 1

n = e1+ 12

+···+ 1n

(e

1n+1 − 1

)=

= ecn+lnn(e

1n+1 − 1

)= ecn · n

n+ 1· e

1n+1 − 1

1

n+ 1

.

Rezulta ca limn→∞

xn = ec, unde c este constanta lui Euler.

Problema 7.9 Sa se arate ca urmatoarele siruri sunt convergente, folosind problema 7.3.

a) an = 1 +1

2+ · · ·+ 1

n− lnn;

b) an =1

2 ln 2+

1

3 ln 3+ · · ·+ 1

n lnn− ln(lnn);

c) an = 1 +1

2α+ · · ·+ 1

nα− 1

1− αn1−α, α ∈ (0, 1);

d) an = 1 +1

2α+ · · ·+ 1

nα, α > 1.

Solutie. a) Se ia f(x) =1

x;

b) f(x) =1

x lnx;

c) f(x) =1

xα;

d) f(x) =1

xα.

Problema 7.10 Sa se calculeze limitele urmatoarelor siruri:

a) an =1

lnn

(1 +

1

2+ · · ·+ 1

n

), n ≥ 2;

b) an =1

ln(lnn)

(1

2 ln 2+

1

3 ln 3+ · · ·+ 1

n lnn

), n ≥ 3;

c) an =1

n1−α

(1 +

1

2α+ · · ·+ 1

), α ∈ (0, 1).

Solutie. Se utilizeaza prima teorema a lui Stolz-Cesaro obtinandu-se:a) lim

n→∞an = 1;

b) limn→∞

an = 1;

c) limn→∞

an =1

1− α.

Page 17: Teme si probleme pentru concursurile studentesti de matematica ...

10

Problema 7.11 Daca notam cu a limitele sirurilor de la exercitiul 7.9 sa se calculezelimitele urmatoare:

a) limn→∞

n

(1 +

1

2+ · · ·+ 1

n− lnn− a

);

b) limn→∞

n lnn

(1

2 ln 2+

1

3 ln 3+ · · ·+ 1

n lnn− ln(lnn)− a

);

c) limn→∞

nα(

1 +1

2α+ · · ·+ 1

nα− 1

1− αn1−α − a

), α ∈ (0, 1);

d) limn→∞

nα−1

(1 +

1

2α+ · · ·+ 1

nα− a), α > 1.

Solutie. Se aplica a doua teorema a lui Stolz-Cesaro.

a) xn = 1 +1

2+ · · ·+ 1

n− lnn− a, yn =

1

n, n ≥ 1. Avem

limn→∞

xnyn

= limn→∞

xn+1 − xnyn+1 − yn

= limn→∞

1

n+ 1− ln(n+ 1) + lnn

1

n+ 1− 1

n

=

= limx→∞x∈R

1

x+ 1− ln(x+ 1) + lnx

1

x+ 1− 1

x

= limx→∞x∈R

− 1

(x+ 1)2− 1

x+ 1+

1

x

− 1

(x+ 1)2+

1

x2

=1

2;

b) Se obtine limita 1;c) Aplicand teorema a doua a lui Cesaro-Stolz obtinem

limn→∞

nα(

1 +1

2α+ · · ·+ 1

nα− 1

1− αn1−α − a

)=

= limn→∞

1

(n+ 1)α− 1

1− α[(n+ 1)1−α − n1−α]

1

(n+ 1)α− 1

=

= limn→∞

1− α−[(n+ 1)− n

(n+ 1

n

)α](1− α)

nα − (n+ 1)α

=

= limx→0x∈R

(1− α)x− (1 + x) + (1 + x)α

x[1− (1 + x)α](1− α)=

1

2

aplicand regula lui l’Hospital de doua ori;

d) Se obtine limita1

1− α.

Problema 7.12 Sa se arate ca daca p, q ∈ N∗, p < q, au loc relatiile:

a) limn→∞

qn∑k=pn

1

k= ln

q

p;

b) limn→∞

1

n

qn∑k=pn

1

k= ln

q

p;

Page 18: Teme si probleme pentru concursurile studentesti de matematica ...

Siruri si serii numerice 11

c) limn→∞

1

lnn

nq∑k=np

1

k= q − p;

d) limn→∞

qn∑k=pn

1

k ln k= ln

(ln q

ln p

);

e) limn→∞

nq∑k=np

1

k ln k= ln

q

p;

Solutie. Fie (an)n≥1 un sir de numere reale, sn = a1 + a2 + · · ·+ an, n ≥ 1 si (bn)n≥1

un sir cu proprietatea ca sirul (sn − bn)n≥1 este convergent. Daca (pn)n≥1, (qn)n≥1 suntdoua siruri de numere naturale, pn ≤ qn pentru n ≥ 1, atunci

qn∑k=pn

ak = sqn − spn + apn = (sqn − bqn)− (spn − bpn) + (bqn − bpn) + apn .

De aici obtinem

limn→∞

qn∑k=pn

ak = limn→∞

[(bqn − bpn) + apn ]

ın ipoteza ca limita din dreapta exista.a) pn = pn, qn = qn, bn = lnn,

limn→∞

(ln qn− ln pn+

1

pn

)= ln

q

p.

b) pn = pn, qn = qn, ak =1

k, bn = lnn.

Pentru c), d), e) procedam analog.

Problema 7.13 Fie (an)n≥1 si (bn)n≥1 doua siruri de numere ıntregi cu proprietatea0 < an ≤ bn, n ≥ 1. Sa se arate ca

limn→∞

anbn

bn∏k=an

e1k = 1.

Solutie. ln

anbn

bn∏k=an

e1k

=

bn∑k=an

1

k+ ln an − ln bn =

=

(bn∑k=1

1

k− ln bn

)−

(an∑k=1

1

k− ln an

)+

1

an→ c− c+ 0 = 0.

Problema 7.14 Demonstrati ca

limn→∞

[1000

√1 +

√2 + · · ·+

√n

]= 1757.

Solutie. Consideram sirurile (an)n≥5, (bn)n≥5,

an =

√1 +

√2 + · · ·+

√n, bn =

√1 +

√2 + · · ·+

√n+√

2n.

Page 19: Teme si probleme pentru concursurile studentesti de matematica ...

12

Se arata usor ca (an) este crescator, iar (bn) este descrescator si an < bn, n ≥ 5, prinurmare

1, 7575 < a6 < limn→∞

an < b6 = 1, 7579,

deci

limn→∞

[1000

√1 +

√2 + · · ·+

√n

]= 1757.

Problema 7.15 Fie a, b > 0 si (xn)n≥1, (yn)n≥1 doua siruri de numere reale cu pro-prietatile:

limn→∞

xnna

= A, limn→∞

ynnb

= B, A,B ∈ R.

Sa se calculeze

limn→∞

(x1 + x2 + · · ·+ xn)(y1 + y2 + · · ·+ yn)

n(x1y1 + x2y2 + · · ·+ xnyn).

Solutie.(x1 + · · ·+ xn)(y1 + · · ·+ yn)

n(x1y1 + · · ·+ xnyn)=

x1 + · · ·+ xnna+1

· y1 + · · ·+ ynnb+1

x1y1 + · · ·+ xnynna+b+1

si

limn→∞

x1 + · · ·+ xnna+1

= limn→∞

xn+1

(n+ 1)a+1 − na+1=

A

a+ 1,

limn→∞

y1 + · · ·+ ynnb+1

=B

b+ 1,

limn→∞

x1y1 + · · ·+ xnynna+b+1

= limn→∞

xn+1yn+1

(n+ 1)a+b+1 − na+b+1=

= limn→∞

xn+1

(n+ 1)a· yn+1

(n+ 1)b

(n+ 1)a+b+1 − na+b+1

(n+ 1)a+b

=AB

a+ b+ 1.

Limita ceruta este egala cua+ b+ 1

(a+ 1)(b+ 1).

Problema 7.16 (Transformarea Toeplitz) Fie cn,k : 1 ≤ k ≤ n, n ≥ 1 un sir dublude numere reale cu proprietatile:

i) limn→∞

cn,k = 0 pentru orice k ∈ N∗;

ii) limn→∞

n∑k=1

cn,k = 1;

iii) exista c > 0 astfel ca

n∑k=1

|cn,k| ≤ c pentru orice n ≥ 1.

Atunci pentru orice sir convergent de numere reale (an)n≥1, sirul (bn)n≥1 definit prin

bn =

n∑k=1

cn,kak, n ≥ 1, este convergent si limn→∞

bn = limn→∞

an.

Page 20: Teme si probleme pentru concursurile studentesti de matematica ...

Siruri si serii numerice 13

Solutie. Daca an = a pentru orice n ≥ 1, atunci din ii) avem

limn→∞

bn = a limn→∞

n∑k=1

cn,k = a.

Astfel este suficient sa consideram cazul cand sirul (an)n≥1 converge la zero. Pentrum > 1 si n ≥ m avem

(1) |bn − 0| =

∣∣∣∣∣n∑k=1

cn,kak

∣∣∣∣∣ ≤m−1∑k=1

|cn,k| · |ak|+n∑

k=m

|cn,k| · |ak|

Fie ε > 0. Din limn→∞

an = 0 rezulta ca exista n1 ∈ N astfel ca |an| <ε

2cpentru n ≥ n1.

Sirul (an)n≥1 este marginit si presupunem ca |an| ≤ D, pentru orice n ≥ 1. Din i) rezultaca exista n2 ∈ N astfel ca pentru n ≥ n2

n1−1∑k=1

|cn,k| <ε

2D.

Punand m = n1 ın (1), obtinem

|bn| ≤ Dn1−1∑k=1

|cn,k|+ε

2c

n∑k=n1

|cn,k| <ε

2+ε

2= ε

pentru n ≥ maxn1, n2. Prin urmare limn→∞

bn = 0.

Problema 7.17 Sa se demonstreze ca daca ın exercitiul precedent cnk > 0, 1 ≤ k ≤n, ∀n ≥ 1, atunci pentru orice sir (xn) cu limita ∞, rezulta ca si transformata sa Toeplitz,(yn), are limita ∞.

Solutie. Fie (xn) cu xn →∞; se poate presupune ca toti termenii termenii sirului (xn)

sunt strict pozitivi. Fie C > 0; din conditia limn→∞

n∑k=1

cnk = 1, rezulta ca exista N1 ∈ N

astfel ıncat:n∑k=1

cnk >1

2, ∀n ≥ N1.

Sirul (xn) fiind nemarginit, exista N2 ∈ N astfel ıncat xn ≥ 2C,∀n ≥ N2. Fie N3 =maxN1, N2; atunci, pentru orice n ≥ N3, avem:

n∑k=1

cnkxk =

N3∑k=1

cnkxk +

n∑k=N3

cnkxk ≥

≥N3∑k=1

cnkxk + C > C,

ceea ce ıncheie demonstratia.

Problema 7.18 Demonstrati ca daca limn→∞

an = a, a ∈ R, atunci

limn→∞

na1 + (n− 1)a2 + · · ·+ ann2

=a

2.

Page 21: Teme si probleme pentru concursurile studentesti de matematica ...

14

Solutie. Se aplica teorema lui Toeplitz cu cn,k =2(n− k + 1)

n2sau se aplica teorema

Stolz-Cesaro de doua ori.

Problema 7.19 Daca limn→∞

an = a, limn→∞

bn = b, a, b ∈ R, atunci

limn→∞

a1bn + a2bn−1 + · · ·+ anb1n

= ab.

Solutie. Daca b 6= 0, luam cn,k =bn−k+1

nbın teorema lui Toeplitz.

Daca b = 0, punand cn,k =1 + bn−k+1

n, avem

limn→∞

a1(1 + bn) + a2(1 + bn−1) + · · ·+ an(1 + b1)

n= a

si tinand seama ca limn→∞

a1 + · · ·+ ann

= a rezulta concluzia.

Problema 7.20 Presupunem ca limn→∞

an = a, a ∈ R. Sa se calculeze:

a) limn→∞

(an1

+an−1

2+ · · ·+ a1

2n−1

);

b) limn→∞

(a1

1 · 2+

a2

2 · 3+ · · ·+ am

n(n+ 1)

);

c) limn→∞

(an1− an−1

2+ · · ·+ (−1)n−1 a1

2n−1

).

Solutie. Se obtin, aplicand teorema lui Toeplitz, rezultatele:

a) 2a; b) a; c)2

3a.

Problema 7.21 Determinati multimea punctelor limita ale sirului (an)n≥1, unde:

a) an =[1− (−1)n] · 2n + 1

2n + 3;

b) an =(

cosnπ

3

)n;

c) an =2n2

7−[

2n2

7

].

Solutie. a) a2n =1

2n + 3, a2n+1 =

2n+1 + 1

2n + 3. Avem lim

n→∞a2n = 0 si lim

n→∞a2n+1 = 2,

deci L(an) = 0, 2;b) L(an) = −1, 0, 1;c) a7k = 0, a7k+1 =

2

7, . . . , a7k+6 =

2

7. Se obtine

L(an) =

0,

1

7,2

7,4

7

.

Problema 7.22 Fie (an)n≥1 un sir de numere reale cu proprietatea ca limn→∞

(an+1−an) =

0. Aratati ca multimea punctelor limita ale lui (an)n≥1 este un interval ınchis.

Solutie. Fie a < b puncte limita ale sirului (an)n≥1 si c ∈ (a, b). Vom construi prinrecurenta un subsir (ank)k≥1 avand limita c. Presupunand (ank)k≥1 ales, fie n0 ∈ N astfel

Page 22: Teme si probleme pentru concursurile studentesti de matematica ...

Siruri si serii numerice 15

ca |an+1 − an| <1

k, pentru n ≥ n0. Din faptul ca a, b sunt puncte limita ale lui (an)n≥1,

rezulta ca exista p, q ∈ N, p, q > maxn0, nk cu proprietatea ca ap < c < aq. Notam cunk+1 cel mai mare indice cuprins ıntre p si q astfel ca c < ank+1

+ 1. Rezulta ca

|ank+1− c| ≤ |ank+1

− ank+1+1| ≤1

k.

Aceasta constructie arata ca multimea punctelor limita ale lui (an)n≥1 este un interval.Fie a o extremitate a acestui interval. Exista deci un sir (xn)n≥1 format din puncte limitapentru sirul (an)n≥1 astfel ca lim

n→∞xn = a. Este suficient sa alegem un subsir (ank)k≥1

astfel ca |ank − xk| ≤1

k. Avem lim

n→∞ank = a, ceea ce ıncheie demonstratia.

Problema 7.23 Fie f : R→ R o functie periodica cu perioada T > 0, continua ın punctulx ∈ R. Fie (Sn)n≥1 un sir satisfacand conditiile:

(i) limn→∞

Sn =∞;

(ii) limn→∞

(Sn+1 − Sn) = 0.

Atunci f(x) este un punct limita al sirului (f(Sn))n≥1.

Solutie. Deoarece f este continua ın x, exista δ1 > 0 astfel ıncat |t− x| < δ1 implica|f(t)−f(x)| < 1. Cum lim

n→∞(Sn+1−Sn) = 0, exista N1 ∈ N astfel ıncat pentru orice n ≥ N1

sa avem |Sn+1−Sn| < δ1. Fie k1 ∈ N cu proprietatea ca x+ k1T ≥ SN1 . Din (i) rezulta caexista n1 ∈ N, n1 ≥ N1, astfel ıncat Sn1 ≤ x+k1T < Sn1+1. Avem ca |x+k1T −Sn1 | < δ1

si atunci |(Sn1 − k1T )− x| < δ1, de unde |f(Sn1)− f(x)| = |f(Sn1 − k1T )− f(x)| < 1.Deoarece f este continua ın x, exista δ2 > 0 astfel ıncat |t − x| < δ2 implica |f(t) −

f(x)| < 12 . Cum lim

n→∞(Sn+1 − Sn) = 0, exista N2 ∈ N astfel ıncat pentru orice n ≥ N2

sa avem |Sn+1 − Sn| < δ2. Fie k2 ∈ N cu proprietatea ca x + k2T ≥ Smax(N2,n1+1). Din(i) rezulta ca exista n2 ∈ N, n2 ≥ max(N2, n1 + 1), astfel ıncat Sn2 ≤ x + k2T < Sn2+1.Avem ca |x+ k2T − Sn2 | < δ1 si atunci |(Sn2 − k2T )− x| < δ2, de unde |f(Sn2)− f(x)| =|f(Sn2 − k2T )− f(x)| < 1

2 .Continuand procedeul de mai sus vom obtine un sir strict crescator (np)p≥1 care are

proprietatea ca |f(Snp)− f(x)| < 1p si trecand la limita obtinem lim

p→∞|f(Snp)− f(x)| = 0,

deci sirul (f(Snp))p≥1 converge la f(x).

Problema 7.24 Fie En = 1 +1

1!+

1

2!+ · · ·+ 1

n!, n ≥ 1.

Demonstrati ca:

a) 0 < e− En <1

n · n!, n ≥ 1;

b) e 6∈ Q;c) lim

n→∞(n!e− [n!e]) = 0.

Solutie. a) Em+n − En =1

(n+ 1)!+

1

(n+ n)!+ · · ·+ 1

(n+m)!<

<1

(n+ 1)!

[1 +

1

n+ 2+

1

(n+ 2)2+ · · ·+ 1

(n+ 2)m−1

]<

1

(n+ 1)!· n+ 2

n+ 1

Fixand n si facand m→∞ obtinem

e− En ≤1

(n+ 1)!· n+ 2

n+ 1<

1

n · n!.

Page 23: Teme si probleme pentru concursurile studentesti de matematica ...

16

b) Sa presupunem ca e =p

q∈ Q, p, q ∈ N, q 6= 0. Avem 0 < e−Eq <

1

q · q!si ınmultind

cu q! obtinem 0 < p(q − 1)!− q!Eq <1

q, contradictie, pentru ca (p(q − 1)!− q!Eq) ∈ Z.

c) Din punctul a) rezulta ca pentru orice n ≥ 1 exista θn ∈]0, 1[ astfel ca

e = En +θnn · n!

,

deci

[n!e] =

[n!En +

θnn

]= n!En,

decilimn→∞

(n!e− [n!e]) = 0.

Problema 7.25 Sa se arate ca limn→∞

n sin(2πen!) = 2π.

Solutie. Din problema 7.24 a) rezulta ca pentru orice n ∈ N exista θn+1 ∈ (0, 1) astfelca

e = En+1 +θn+1

(n+ 1)(n+ 1)!.

Avem

xn = n sin(2πen!) = n sin

[2π

(En+1 +

θn+1

(n+ 1)(n+ 1)!

)n!

]=

n sin

[2π

(En +

1

(n+ 1)!+

θn+1

(n+ 1)(n+ 1)!

)n!

]= n sin

(2πEnn! +

1

n+ 1+

θn+1

(n+ 1)2

)si cum n!En ∈ N obtinem

xn = n sin

[2π

(1

(n+ 1)!+

θn+1

(n+ 1)2

)]=

sin[2π( 1

(n+1)! + θn+1

(n+1)2)]

2π(

1(n+1)! + θn+1

(n+1)2

) (2π

n

n+ 1+

nθn+1

(n+ 1)2

),

deci limn→∞

= 2π.

Problema 7.26 Fie (an)n≥1 un sir de numere reale cu proprietatile: 0 < an ≤ 1 pentruorice n ≥ 1 si lim

n→∞(a1 + a2 + · · ·+ an) =∞.

a) Sa se arate ca pentru orice l ∈ [1,∞) ∪ ∞ exista o functie strict crescatoareL : N∗ → N∗ astfel ca

limn→∞

a1 + a2 + · · ·+ aL(n+1)

a1 + a2 + · · ·+ aL(n)= l.

b) Sa se determine functia L pentru an =1√n

, n ≥ 1.

Solutie. a) Fie sk = a1 + · · · + ak, k ≥ 1. Intervalele [sk, sk+1), k ≥ 1, determina opartitie a intervalului [a1,∞).

1. Daca l > 1, atunci pentru orice n ≥ 1 exista un unic k ∈ N∗ astfel ca ln ∈ [sk, sk+1)si definim functia L(n) = k, deci ln ∈ [sL(n), sL(n)+1). Cum ln+1− ln > 1 > aL(n)+1 rezultasL(n+1) ≥ sL(n)+1 si atunci L(n+ 1) > L(n), deci L este functie strict crescatoare.

Avem:sL(n) ≤ ln < sL(n)+1 = sL(n) + aL(n)+1 < sL(n) + 1

Page 24: Teme si probleme pentru concursurile studentesti de matematica ...

Siruri si serii numerice 17

sL(n+1) ≤ ln+1 < sL(n+1)+1

din care deducemln+1

ln<sL(n+1)

sL(n)<

ln+1

ln − 1,

de unde obtinem limn→∞

sL(n+1)

sL(n)= 1.

2. Daca l = 1, alegem L(n) = n si obtinem

limn→∞

sn+1

sn= 1 + lim

n→∞

an+1

sn= 1.

3. Daca l =∞ alegem L(n) astfel ca nn ∈ [sL(n), sL(n)+1) si avem

sL(n+1)

sL(n)≥ (n+ 1)n+1 − 1

nn→∞.

b) Sirul (an)n≥1, an = 1 +1√2

+ · · ·+ 1√n− 2√n este convergent. Avem

sL(n+1)

sL(n)=aL(n+1) + 2

√aL(n+1)

aL(n) + 2√aL(n)

,

limn→∞

sL(n+1)

sL(n)= lim

n→∞

√aL(n+1)√aL(n)

.

Pentru l = 1 alegem L(n) = n.Pentru l > 1 alegem L(n) = [l2n].Pentru l > 1 alegem L(n) = nn.

Problema 7.27 Fie a si b doua numere reale astfel ıncat 0 < a < b. Definim sirurile:

a1 =√ab, b1 =

1

2(a+ b)

a2 =√a1b1, b2 =

1

2(a1 + b1)

. . . . . . . . .

an =√an−1bn−1, bn =

1

2(an−1 + bn−1).

Sa se arate ca sirurile an si bn sunt convergente si au aceeasi limita (numita mediaaritmetico-geometrica a numerelor a si b).

Solutie. Evident, din inegalitatea mediilor rezulta an ≤ bn,∀n ∈ N si a < a1 < b1 < b.Vom arata ca sirul (an) este crescator, iar sirul bn este descrescator. Avem:

an+1 − an =√anbn − an =

an(bn − an)√anbn + an

> 0,∀n ∈ N,

bn+1 − bn =an + bn

2− bn =

an − bn2

< 0,∀n ∈ N.

Rezulta ca sirurile sunt convergente; daca notam L1 = limn→∞

an si L2 = limn→∞

bn, atunci,

trecand la limita ın relatia an+1 = 12(an + bn), rezulta L1 = L2.

Page 25: Teme si probleme pentru concursurile studentesti de matematica ...

18

Problema 7.28 Fie (xn) un sir de numere reale astfel ıncat exista L ∈ R cu proprietatea:

limn→∞

(2xn+1 − xn) = L

Sa se demonstreze ca limn→∞

xn = L.

Solutia 1. Fie ε > 0; din ipoteza, exista N(ε) astfel ıncat:

L− ε < 2xn+1 − xn < L+ ε, ∀n ≥ N(ε).

Fie n ≥ N(ε) fixat si fie k ∈ N; ınsumand inegalitatile:

L− ε < 2xn+1 − xn < L+ ε, ∀n ≥ N(ε).

2(L− ε) < 4xn+2 − 2xn+1 < 2(L+ ε)

......................

2k−1(L− ε) < 2kxn+k − 2k−1xn+k−1 < 2k−1(L+ ε),

Obtinem:

(1 + 2 + ...+ 2k−1)(L− ε) < 2kxn+k − xn < (1 + 2 + ...+ 2k−1)(L+ ε),

sau, echivalent (ımpuartind la 2k):(1− 2−k

)(L− ε) < xn+k − 2−kxn <

(1− 2−k

)(L+ ε).

Alegem acum k astfel ıncat:

|2−kxn| < ε si |2−k(L± ε)| < ε.

Atunci, pentru orice p ≥ n+ k (alesi ca mai sus), rezulta:

L− 3ε < xm < L+ 3ε,

ceea ce ıncheie demonstratia.Solutia 2. Scriem

L = limn→∞

(2xn+1 − xn) = limn→∞

2n+1xn+1 − 2nxn2n+1 − 2n

.

Din teorema Cesaro-Stolz

limn→∞

2n+1xn+1 − 2nxn2n+1 − 2n

= limn→∞

2nxn2n

= limn→∞

xn,

deci limn→∞

xn = L.

Problema 7.29 Fie a si b doua numere pozitive. Sa se calculeze limita sirului (xn) definitde relatia:

xn+1 =√a+ bxn, ∀n ≥ 1, x1 =

√a.

In particular, sa se calculeze:

limn→∞

√1 +

√1 +

√1 + · · ·+

√1, (n radicali).

Page 26: Teme si probleme pentru concursurile studentesti de matematica ...

Siruri si serii numerice 19

Solutie. Demonstram prin inductie faptul ca (xn) este marginit, mai precis:

0 < xn <b+√b2 + 4a

2, ∀n ≥ 1,

numarul b+√b2+4a2 fiind solutia pozitiva a ecuatiei x2 − bx − a = 0. Evident, x1 = a <

b+√b2+4a2 ; presupunand ca xn <

b+√b2+4a2 , rezulta

xn+1 =√a+ bxn <

√a+ b · b+

√b2 + 4a

2=b+√b2 + 4a

2.

Demonstram ca xn este strict crescator; este evident ca:

x2 =

√a+ b

√a >√a = x1.

Relatia xn+1 > xn este echivalenta cu x2n− bxn− a < 0. Ultima inegalitate este adevarata

deorece xn ∈ (0, b+√b2+4a2 ).

Sirul (xn) este deci convergent si prin trecere la limita ın relatia de recurenta, rezulta

limn→∞

xn =b+√b2 + 4a

2.

Problema 7.30 Sa se demonstreze formula lui Ramanujan:√1 + 2

√1 + 3

√1 + 4

√1 + . . . = 3

Solutie. Fie sirul de functii

f1(x) =√

1 + x, f2(x) =

√1 + x

√1 + (x+ 1), . . . ,

fn(x) =

√1 + x

√1 + (x+ 1)

√1 + · · ·+ (x+ n− 2)

√1 + (x+ n− 1) (n radicali)

Vom demonstra ca sirul (fn(x)) converge pentru orice x ≥ 1. Fie x ≥ 1, fixat; evident,(fn(x)) este crescator. Aratam ın continuare ca este marginit. Evident:

fn(x) ≥

√x

√x

√. . .√x ≥ x

Pentru orice n ∈ N∗ si x ≥ 1, avem:

fn(x) ≤

√(x+ 1)

√(x+ 2)

√(x+ 3) . . .

√(x+ n) ≤

√2x

√3x

√4x . . .

√(n+ 1)x ≤

√2x

√4x

√8x . . .

√2nx =

= 2∑nk=1

k

2k x∑nk=1

1

2k ≤ 4x.

Page 27: Teme si probleme pentru concursurile studentesti de matematica ...

20

Fie f(x) = limn→∞

fn(x); din inegalitatea f(x) ≥ x, rezulta f(x) ≥ 2−1(x+ 1) si deci:

1

2(x+ 1) ≤ f(x) ≤ 4x, ∀x ≥ 1.

Inlocuind x cu x+ 1, rezulta:

1

2(x+ 2) ≤ f(x+ 1) ≤ 4(x+ 1), ∀x ≥ 1.

Trecand la limita ın relatia de recurenta si apoi ridicand la patrat, obtinem:

(f(x))2 = 1 + xf(x+ 1)

Din dubla inegalitate de mai sus rezulta

x1

2(x+ 2) + 1 ≤ (f(x))2 ≤ 4x(x+ 1) + 1

Dupa calcule simple, obtinem:

2−12 (x+ 1) ≤ f(x) ≤ 2(x+ 1)

Repetam procedeul anterior, i.e. scriem inegalitatea anterioara pentru x+1, apoi ınmultimcu x si adunam 1:

2−12x(x+ 2) + 1 ≤ (f(x))2 ≤ 2x(x+ 2) + 1

si dupa calcule rezulta:

2−122 (x+ 1) ≤ f(x) ≤ 2

12 (x+ 1)

Iterand de n ori, rezulta:

2−12n (x+ 1) ≤ f(x) ≤ 2

12n−1 (x+ 1), ∀n = 1, 2, 3 . . .

Trecand la limita (n → ∞) obtinem f(x) = x + 1. In particular, pentru x = 2, se obtine

formula lui Ramanujan:

√1 + 2

√1 + 3

√1 + 4

√1 + . . . = 3.

Problema 7.31 Sa se calculeze limita sirului:

n∑k=1

(k ln

(2k + 1

2k − 1

)− 1

).

Solutie. Termenul general se scrie:

n∑k=1

(k ln

(2k + 1

2k − 1

)− 1

)= ln

(2n+ 1)n

1 · 3 · 5 · · · (2n− 1) · en=

= ln

(2n+ 1

2n

)n+ ln

(2n)n

1 · 3 · 5 · · · (2n− 1) · en=

= ln

(2n+ 1

2n

)n+ ln

4n · nn · n!

(2n)! · en

Primul termen tinde la 12 ; ın al doilea termen ınlocuim n! si (2n)! cu expresiile core-

spunzatoare din formula lui Stirling. In final obtinem limita 12 − ln

√2.

Page 28: Teme si probleme pentru concursurile studentesti de matematica ...

Siruri si serii numerice 21

Serii. Probleme

Sa se determine sumele seriilor:

Problema 7.32

∞∑n=1

(a+ 1)(a+ 2) . . . (a+ n)

(b+ 1)(b+ 2) . . . (b+ n), a > 0, b > a+ 1.

Solutie. Avem

an =(a+ 1) . . . (a+ n)

(b+ 1) . . . (b+ n)= an−1

a+ n

b+ n,

din care rezultaan−1(a+ n) = an(b+ n)

sauan−1(a+ n) = an[(a+ n+ 1) + (b− a− 1)],

decian−1(a+ n)− an(a+ n+ 1) = (b− a− 1)an.

Suma primilor termeni ai seriei este

Sn =

n∑k=1

ak =1

b− a− 1

n∑k=1

(f(n− 1)− f(n)) =

=1

b− a− 1(f(0)− f(n)) =

1

b− a− 1(a0 · a− an(a+ n+ 1)) =

=1

b− a− 1

(a2

b− (a+ 1)

(a+ 2) . . . (a+ n+ 1)

(b+ 1) . . . (b+ n)

).

Deci

limn→∞

Sn =a2

b(b− a− 1)− (a+ 1) lim

n→∞

(a+ 2) . . . (a+ n+ 1)

(b+ 1) . . . (b+ n).

Ultima limita o determinam astfel:

(a+ 2) . . . (a+ n+ 1)

(b+ 1) . . . (b+ n)=

=1(

1 +b− a− 1

a+ 2

)(1 +

b− a− 1

a+ 2

). . .

(1 +

b− a− 1

a+ n+ 1

) <

<1

b− a− 1

a+ 2+b− a− 1

a+ 3+ · · ·+ b− a− 1

a+ n+ 1

=

=1

b− a− 1· 1

1

a+ 2+

1

a+ 3+ · · ·+ 1

a+ n+ 1

care are limita zero caci seria

∞∑n=2

1

a+ neste divergenta (comparand-o cu seria armonica).

Deci∞∑n=1

an =a2

b(b− a− 1).

Page 29: Teme si probleme pentru concursurile studentesti de matematica ...

22

Problema 7.33

∞∑n=1

1n∑k=1

k3

.

Solutie.n∑k=1

k3 =n2(n+ 1)2

4. Avem Sn =

n∑p=1

4

p2(p+ 1)2=

= 4n∑p=1

[−2

(1

p− 1

p− 1

)+

1

p2+

1

(p+ 1)2

]=

= −8

(1− 1

n+ 1

)+ 4

[1 +

1

(n+ 1)2+ 2

(1 +

1

22+ · · ·+ 1

n2

)]limn→∞

Sn = −4 + 8 limn→∞

(1 +

1

22+ · · ·+ 1

n2

)= −4 +

4

3π2

(suma seriei

∞∑n=1

1

n2este

π2

6).

Problema 7.34∞∑n=1

[a+ 2n

2n+1

], a ∈ R.

Solutie. Este cunoscuta identitatea:[a+

1

2

]= [2a]− [a], a ∈ R.

Avem

an =

[a+ 2n

2n+1

]=[ a

2n

]−[ a

2n+1

],

Sn =

n∑k=1

ak = [a]−[ a

2n+1

]si

Sn =

[a], daca a ≥ 0[a] + 1, daca a < 0.

Problema 7.35∞∑n=1

1

2ntg

a

2n, a ∈ R \

2n(π

2+ kπ

)| k, n ∈ Z

.

Solutie. Avem identitatea tg x = ctg x− 2ctg 2x si

an =1

2ntg

a

2n=

1

2n

(ctg

a

2n− 2ctg

a

2n−1

)=

=1

2nctg

a

2n− 1

2n−1ctg

a

2n−1.

Sn =n∑k=1

ak =1

2nctg

a

2n− ctg a,

limn→∞

Sn = −ctg a+ limn→∞

1

2n

tga

2n

= −ctg a+1

a.

Page 30: Teme si probleme pentru concursurile studentesti de matematica ...

Siruri si serii numerice 23

Problema 7.36

∞∑n=0

(−1)ncos3 3na

3n, a ∈ R.

Solutie. Avem identitatea 4 cos3 x = cos 3x+ 3 cosx din care:

cos3 3na

3n=

1

4

[cos 3n+1a

3n+

cos 3na

3n−1

]Suma primilor n termeni este

Sn =1

4

(3 cos a+ (−1)n

cos 3n+1a

3n

)si

limn→∞

Sn =3

4cos a,

care este suma seriei.

Problema 7.37∞∑n=0

arctg2n

1 + 22n+1.

Solutie.

arctg 2x = arctg x+ arctgx

1 + 2x2,

din care

arctg2n

1 + 22n+1= arctg 2n+1 − arctg 2n.

Sn =

n∑k=0

(arctg 2k+1 − arctg 2k) = arctg 2n+1 − arctg 1 = arctg 2n+1

limn→∞

Sn =π

2− π

4=π

4.

Problema 7.38∞∑n=3

arctg3

n2 − n− 1.

Solutie. Avem identitatea:

arctg a+ arctg b =

arctg

a+ b

1− ab, daca ab < 1

π + arctga+ b

1− ab, daca ab > 1

an = arctg3

n2 − n− 1= arctg

3

1 + n2 − n− 2=

= arctg(n+ 1)− (n− 2)

1 + (n+ 1)(n− 2)= arctg (n+ 1)− arctg (n− 2).

Sn =n∑k=3

(arctg (k + 1)− arctg (k − 2)) =

= arctg (n+ 1) + arctg n+ arctg (n− 1)− arctg 1− arctg 2− arctg 3.

limn→∞

Sn = 3π

2− π

2− (arctg 2 + arctg 3) =

= 3π

2− π

4−(π + arctg

2 + 3

1− 2 · 3

)= 3

π

2− π

4− π +

π

4=π

2

Page 31: Teme si probleme pentru concursurile studentesti de matematica ...

24

Problema 7.39

∞∑n=1

(−1)n+1

n.

Solutie. S2n =

(1 +

1

2+ · · ·+ 1

2n

)− 2

(1

2+

1

4+ · · ·+ 1

2n

)=

=

(1 +

1

2+ · · ·+ 1

2n− ln 2n

)−(

1 +1

2+ · · ·+ 1

n− lnn

)+ ln 2n− lnn =

= c2n − cn + ln 2,

unde cn = 1 +1

2+ · · ·+ 1

n. Sirul (cn)n este convergent la constanta lui Euler c si atunci

limn→∞

S2n = c− c+ ln 2 = ln 2

Analog

S2n+1 = S2n +1

2n+ 1→ ln 2

deci∞∑n=1

(−1)n+1

n= ln 2.

Problema 7.40 1 +1

3+ · · ·+ 1

2p− 1− 1

2− 1

4− · · · − 2

2q+

1

2p+ 1+

+1

2p+ 3+ · · ·+ 1

4p− 1− 1

2q + 2− 1

2q + 4− · · · − 1

4q+ . . . ,

unde p, q ∈ N.

Solutie. Notam cu S(p, q) suma seriei,

an = 1 +1

2+ · · ·+ 1

nsi cn = 1 +

1

2+ · · ·+ 1

n− lnn,

sirul (cn)n fiind convergent la constanta lui Euler c. Suma primilor n(p + q) termeni aiseriei este

Sn(p+ q) = 1 +1

3+ · · ·+ 1

2p− 1+

1

2p+ 1+ · · ·+ 2

2np− 1−

−(

1

2+

1

4+ · · ·+ 1

2q+

1

2q + 2+ · · ·+ 1

2nq

)=

= a2np −(

1

2+

1

4+ · · ·+ 1

2p+ · · ·+ 1

2np

)−

−1

2

(1 +

1

2+ · · ·+ 1

q+

1

q + 1+ · · ·+ 1

nq

)=

= a2np −1

2anp −

1

2anq = c2np + ln(2np)− 1

2(cnp + ln(np))− 1

2(cnq + ln(nq)) =

= c2np −1

2cnp −

1

2cnq +

1

2ln

4n2p2

npnq.

Page 32: Teme si probleme pentru concursurile studentesti de matematica ...

Siruri si serii numerice 25

Trecand la limita obtinem:

S(p, q) = c− 1

2c− 1

2c+

1

2ln

4p

q=

1

2ln

4p

q

Observatie. 1) Daca q = 4p, atunci S(p, q) = 0, de exemplu

1− 1

2− 1

4− 1

6− 1

8+

1

3− 1

10− 1

12− 1

14− 1

16+ · · · = 0.

Cum 4pq | p, q ∈ N∗ = Q∗+, multimea ln 4p

q | p, q ∈ N∗ este densa ın R, deci pentruorice l ∈ R si pentru orice ε > 0 se poate alege p, q ∈ N∗ astfel ca l − ε < S(p, q) < l + ε.

2) In seria semiconvergenta ∑n≥1

(−1)n+1

n

s-a permutat ordinea termenilor astfel ıncat s-a obtinut o serie convergenta, dar cu o altasuma. Astfel s-a exemplificat teorema lui Riemann referitoare la serii semiconvergente.

Problema 7.41

∞∑n=1

(−1)nlnn

n.

Solutie. Sirul cu termenul general

xn =ln 2

2+

ln 3

3+ · · ·+ lnn

n− ln2 n

2

este convergent si notam limita sa cu l. Avem:

S2n =

2n∑k=1

(−1)kln k

k=

= − ln 1

1+

ln 2

2− ln 3

3+

ln 4

4− · · · − ln(2n− 1)

2n− 1+

ln 2n

2n=

= −(

ln 1

1+

ln 2

2+

ln 3

3+

ln 4

4+ · · ·+ ln(2n− 1)

2n− 1+

ln(2n)

2n

)+

+2

(ln 2

2+

ln 4

4+ · · ·+ ln(2n)

2n

)=

= −x2n + xn + ln 2

(1 +

1

2+ · · ·+ 1

n− lnn

)− (ln 2)2

2

limn→∞

S2n = −l + l + ln 2 · c− (ln 2)2

2= ln 2

(c− ln 2

2

)unde c = lim

n→∞

(1 +

1

2+ · · ·+ 1

n− lnn

)este constanta lui Euler.

Problema 7.42

∞∑n=1

(−1)n−1 1

n+ 1

(1 +

1

2+ · · ·+ 1

n

).

Page 33: Teme si probleme pentru concursurile studentesti de matematica ...

26

Solutie. Aratam ca seria

∞∑n=1

(−1)n−1 2

n+ 1

(1 +

1

2+ · · ·+ 1

n

)este produsul Cauchy

al seriei∞∑n=1

(−1)n−1 1

ncu ea ınsasi. Termenul general al produsului este

cn = (−1)n−1

(1

1 · n+

1

2(n− 1)+ · · ·+ 1

n · 1

)dar

1

k(n+ 1− k)=

1

n+ 1

(1

k+

1

n− k + 1

),

deci

cn = (−1)n2

n+ 1

(1 +

1

2+ · · ·+ 1

n

).

Deoarece seria produs este o serie alternanta iar sirul1 +

1

2+ · · ·+ 1

nn+ 1

este descrescator

spre zero, conform criteriului lui Leibniz, seria produs este convergenta si atunci suma eieste

S =

( ∞∑n=1

(−1)n−1 1

n

)2

= (ln 2)2.

Problema 7.43∞∑n=0

(−1)n

FnFn+1, unde F0 = F1 = 1, Fn+1 = Fn + Fn−1, n ≥ 1 (sirul lui

Fibonacci).

Solutie. Pentru matricea A =

[1 11 0

],

An+1 = An +An−1 si An+1 =

[Fn+1 FnFn Fn−1

],

det(An+1) = (detA)n+1,

deciFn−1Fn+1 − F 2

n = (−1)n+1, n ≥ 1.

Suma primilor n termeni ai seriei este

Sn =

n∑k=0

(−1)k

FkFk+1= 1−

n∑k=1

Fk−1Fk+1 − F 2k

FkFk+1=

= 1−n∑k=1

(Fk−1

Fk− FkFk+1

)= 1− F0

F1+

FnFn+1

=FnFn+1

din expresia lui Fn =1√5

(1 +√

5

2

)n+1

(1−√

5

2

)n+1 rezulta

limn→∞

Sn =2

1 +√

5=

√5− 1

2.

Page 34: Teme si probleme pentru concursurile studentesti de matematica ...

Siruri si serii numerice 27

Problema 7.44 Fie Fn sirul lui Fibonacci: F0 = F1 = 1, Fn+1 = Fn +Fn−1,∀n ≥ 1 si fie

σn =∑n

k=0 F2k . Sa se calculeze suma seriei:

∑n≥0

(−1)n

σn.

Solutie. Vom presupune cunoscute relatiile (se pot demonstra prin inductie):

Fn =1√5

(1 +√

5

2

)n+1

(1−√

5

2

)n+1 , ∀n ≥ 0 (1)

Fn−1Fn+1 − F 2n = (−1)n+1, ∀n ≥ 1. (2)

Din definitia lui Fk rezulta:

Fk+1Fk = F 2k + Fk−1Fk, ∀k ≥ 1.

Insumand egalitatile de mai sus pentru k = 1, 2, . . . , n, obtinem

σn = Fn+1Fn, ∀n ≥ 0. (3)

Din relatiile (2) si (3) obtinem:

Sn =

n∑k=0

(−1)k

σk=∑k=0

n (−1)k

FkFk+1= 1−

n∑k=1

Fk−1Fk+1 − F 2k

FkFk+1=

= 1−n∑k=1

(Fk−1

Fk− FkFk+1

)=

FnFn+1

.

Aplicand acum (1), obtinem suma seriei:∑n≥0

(−1)n

σn=

2

1 +√

5.

Problema 7.45∞∑n=1

arctg1

F2n, unde (Fn)n este sirul lui Fibonacci.

Solutie. Din problema anterioara avem relatia

Fn−1Fn+1 − F 2n = (−1)n+1

ın care ınlocuim unul din Fn cu Fn+1 − Fn−1 si obtinem:

Fn−1Fn+1 − Fn(Fn+1 − Fn−1) = (−1)n+1

sauFn−1(Fn+1 + Fn)− FnFn+1 = (−1)n+1

sauFn−1Fn+2 − FnFn+1 = (−1)n+1

Avem

arctg1

F2n+1− arctg

1

F2n+2= arctg

F2n+2 − F2n+1

F2n+1F2n+2 + 1=

Page 35: Teme si probleme pentru concursurile studentesti de matematica ...

28

= arctgF2n

F2nF2n+3= arctg

1

F2n+3,

deci

arctg1

F2n+1− arctg

1

F2n+3= arctg

1

F2n+2

Adunand relatiile de la n = 1 obtinem:

n+1∑k=1

arctg1

F2k= arctg

1

F1− arctg

1

F2n+3

Trecand la limita rezulta∞∑n=1

arctg1

F2n= arctg

1

F1=π

4.

Problema 7.46 Fie (xn)n un sir de numere reale astfel ıncat exista P ∈ (0,∞)∪∞ cuproprietatea:

limn→∞

((x1 + 1)(x2 + 1) · · · (xn + 1)) = P.

Sa se calculeze suma seriei∑n≥1

xn(x1 + 1)(x2 + 1) · · · (xn + 1)

.

Solutie. Descompunem termenul general al seriei:

xn(x1 + 1)(x2 + 1) · · · (xn + 1)

=xn + 1− 1

(x1 + 1)(x2 + 1) · · · (xn + 1)=

=1

(x1 + 1)(x2 + 1) · · · (xn−1 + 1)− 1

(x1 + 1)(x2 + 1) · · · (xn + 1).

Rezulta pentru sirul sumelor partiale al seriei date formula:

Sn = 1− 1

(x1 + 1)(x2 + 1) · · · (xn + 1),

deci suma seriei este 1− P−1 (cu conventia ∞−1 = 0).

Problema 7.47 Sa se studieze convergenta seriei∑n≥1

n!(an

)n, a > 0.

Solutie. Se aplica criteriul raportului:

limn→∞

xn+1

xn= lim

n→∞a

(n

n+ 1

)n=a

e

Daca a < e, atunci seria este convergenta; daca a > e, atunci seria este divergenta. Pentrua = e, aplicam criteriul lui Raabe-Duhamel:

limn→∞

n

(xnxn+1

− 1

)= lim

n→∞n

((n+ 1

n

)n 1

e− 1

)=

= n

((1 +

1

n

)n 1

e− 1

)=

1

elimn→∞

(1 + 1

n

)n − e1n

.

Ultima limita se calculeaza aplicand regula lui L’Hopital:

limx→0

(1 + x)1x − e

x= lim

x→0

(1 + x)1x−1[x− (1 + x) ln(1 + x)]

x2= −e

2;

rezulta ca seria este divergenta.

Page 36: Teme si probleme pentru concursurile studentesti de matematica ...

Siruri si serii numerice 29

Problema 7.48 Sa se studieze convergenta seriei∑n≥2

1

np lnq n, p > 0, q > 0.

Solutie. Daca p > 1, se aplica criteriul comparatiei: seria converge pentru orice q > 0deoarece

1

np lnq n≤ 1

np.

Daca p = 1, se aplica criteriul integral: seria converge daca si numai daca q > 1.Daca p < 1 se aplica criteriul de condensare: seria are aceeasi natura cu seria cu termenulgeneral 1

nq2n(p−1) lnq 2, care este divergenta pentru orice q > 0 (se poate aplica criteriul

raportului).

Problema 7.49 Fie (an)n un sir de numere reale si fie, pentru orice x ∈ R, seria∑

n≥1annx .

Sa se demonstreze ca daca seria data converge pentru x = x0, atunci ea converge pentruorice x ≥ x0.

Solutie. Vom aplica criteriul lui Abel; seria data se scrie:∑n≥1

annx

=∑n≥1

annx0· 1

nx−x0

Sirul 1nx−x0

este monoton (descrescator) si marginit, iar seria∑

n≥1annx0 este convergenta.

Problema 7.50 In seria convergenta:∑n≥1

(−1)n+1

n= 1− 1

2+

1

3− 1

4+ · · ·

sa se permute ordinea termenilor astfel ıncat sa se obtina o serie convergenta, dar cu oalta suma.

Solutie. Seria∑

n≥1(−1)n+1

n este convergenta si suma sa este ln 2. Fie deci:

1− 1

2+

1

3− 1

4+ · · · = ln 2

Inmultind egalitatea de mai sus cu 12 , rezulta:

1

2− 1

4+

1

6− 1

8+ · · · = 1

2ln 2

Insumam acum cele doua egalitati grupand termenii astfel:

1 +

(−1

2+

1

2

)+

1

3+

(−1

4− 1

4

)+

1

5+

(−1

6+

1

6

)+

1

7+

+

(−1

8− 1

8

)+

1

9+

(1

10− 1

10

)+

1

11+ · · · = 3

2ln 2.

Seria de mai sus este (dupa efectuarea calculelor din paranteze):

1 +1

3− 1

2+

1

5+

1

7− 1

4+

1

9+

1

11− · · · = 3

2ln 2,

si este o permutare a seriei initiale.Observatie. Solutia problemei se poate obtine folosind cazul particular al problemei

7.40 pentru p = 2, q = 1.

Page 37: Teme si probleme pentru concursurile studentesti de matematica ...

30

Problema 7.51 Sa se precizeze natura seriilor:

a)∞∑n=1

nn−2

enn!

b)∞∑n=1

nn

enn!.

Solutie. a)an+1

an=

(1 +

1

n

)n−2

e<

e

e

(1 +

1

n

)2 =

1

(n+ 1)2

1

n2

=bn+1

bn

Folosind criteriul de comparatie C9 pentru seria convergenta∞∑n=1

1

n2, rezulta ca seria

este convergenta.

b)an+1

an=

(1 +

1

n

)ne

>

(1 +

1

n

)n(

1 +1

n

)n+1 =

1

n+ 11

n

.

Folosind criteriul de comparatie C9 pentru seria divergenta

∞∑n=1

1

nrezulta ca seria este

divergenta.

Problema 7.52 Fie

∞∑n=1

an o serie convergenta cu termeni pozitivi. Sa se arate ca seria

∞∑n=1

n√a1a2 . . . an este convergenta si are loc inegalitatea:

∞∑n=1

n√a1a2 . . . an < e

∞∑n=1

an

(T. Carleman)

Solutie. (G. Polya) Definim numerele c1, c2, . . . , cn, . . . prin relatiile c1c2 . . . cn = (n+1)n pentru orice n ∈ N∗. Avem

∞∑n=1

n√a1a2 . . . an =

∞∑n=1

n√a1c1 · a2c2 · · · ancn

n+ 1

(∗)≤

≤∞∑n=1

a1c1 + a2c2 + · · ·+ ancnn(n+ 1)

=∞∑n=1

1

n(n+ 1)

(n∑k=1

(akck)

)(∗∗)≤

=∞∑k=1

(akck)

∞∑n=k

1

n(n+ 1)=∞∑k=1

(akck)∞∑n=k

(1

n− 1

n+ 1

)=

=

∞∑k=1

akck1

k=∞∑k=1

ak(k + 1)k

kk−1· 1

k=∞∑k=1

ak

(1 +

1

k

)k (∗∗∗)<

<∞∑k=1

ake = e

∞∑k=1

ak.

Page 38: Teme si probleme pentru concursurile studentesti de matematica ...

Siruri si serii numerice 31

In (∗) s-a folosit inegalitatea mediilor.In (∗∗) s-a folosit egalitatea:

∞∑n=1

an

(n∑k=1

bk

)=

∞∑k=1

bk

( ∞∑n=k

an

)

In (∗ ∗ ∗) s-a folosit faptul ca sirul ek =

(1 +

1

k

)keste crescator cu limita e, deci

ek < e, k ∈ N.

Problema 7.53 Fie (εn)n un sir astfel ıncat εn ∈ −1, 0, 1,∀n = 1, 2, . . . si fie sirul

xn = ε1

√2 + ε2

√2 + · · ·+ εn

√2.

(a) Sa se demonstreze egalitatea:

xn = 2 sin

4

n∑k=1

ε1ε2 · · · εk2k−1

), ∀n = 1, 2, . . .

(b) Sa se demonstreze ca sirul (xn) este convergent.

G. Polya, G. Szego

Solutie. (a) Daca ε1 = 0, atunci relatia este evident adevarata. Presupunem de aiciinainte ca ε1 6= 0. Demonstram egalitatea prin inductie; daca n = 1, egalitatea esteverificata. Presupunem acum adevarata relatia:

xn = 2 sin

4

n∑k=1

ε1ε2 · · · εk2k−1

).

Calculam, aplicand ipoteza de inductie:

x2n+1 − 2 = ε2

√2 + ε3

√2 + · · ·+ εn+1

√2 = 2 sin

4

n+1∑k=2

ε2ε3 · · · εk2k−2

)=

= −2 cos

2+π

2

n+1∑k=2

ε2ε3 · · · εk2k−1

)= −2 cos

2

n+1∑k=1

ε1ε2 · · · εk2k−1

),

ultima egalitate fiind evidenta pentru ε1 = 1; daca ε1 = −1, atunci egalitatea rezulta dinparitatea functiei cosinus. Evident, ipoteza de inductie a fost aplicata ın ipoteza ε2 6= 0,altfel egalitatea ceruta se verifica imediat: xn = ±

√2. Rezulta deci:

x2n+1 − 2 = 4 sin2

4

n+1∑k=1

ε1ε2 · · · εk2k−1

)− 2,

si ın concluzie

xn+1 = 2 sin

4

n+1∑k=1

ε1ε2 · · · εk2k−1

).

(b) Din relatia demonstrata la punctul (a), notand cu S suma seriei (convergente)n∑k=1

ε1ε2 · · · εk2k−1

, rezulta limn→∞

xn = 2 sin(π

4S).

Page 39: Teme si probleme pentru concursurile studentesti de matematica ...

32

Problema 7.54 Se considera sirul (an)n definit prin relatia de recurenta an+1 = ln(1 +an), n ≥ 1 si a1 = 1.

a) Sa se arate ca limn→∞

an = 0.

b) Sa se arate ca seria

∞∑n=1

an este divergenta.

c) Sa se arate ca seria∞∑n=1

a2n este convergenta.

Solutie. a) Prin inductie se arata ca an > 0, n ∈ N∗ si din inegalitatea ln(1 + x) ≤x rezulta ca sirul (an)n este descrescator (si marginit de zero) deci convergent. Dacalimn→∞

an = l atunci din relatia de recurenta rezulta l = ln(1 + l) cu singura solutie l = 0.

b) Comparam seria

∞∑n=1

an cu seria

∞∑n=1

1

n. Avem

limn→∞

an1

n

= limn→∞

n1

an

= limn→∞

n+ 1− n1

an+1− 1

an

=

= limn→∞

anan+1

an − an+1= lim

n→∞

an ln(1 + an)

an − ln(1 + an)= lim

x→0

x ln(1 + x)

x− ln(1 + x)=

= limx→0

x2 ln(1 + x)

xx− ln(1 + x)

= limx→0

x2

x− ln(1 + x)= lim

x→0

2x

1− 1

1 + x

=

= limx→0

2(1 + x) = 2 ∈ (0,∞),

deci seriile au aceeasi natura (divergente).

c) Aplicam criteriul comparatiei comparand cu seria

∞∑n=1

1

n2. Avem:

limn→∞

a2n

1

n2

= limn→∞

(nan)2 = 4 ∈ (0,∞),

deci ambele serii sunt convergente.

Problema 7.55 Fie seria convergenta cu termeni pozitivi

∞∑n=1

an. Sa se arate ca daca

exista limita limn→∞

nan, atunci ea este egala cu zero.

Solutie. Fie l = limn→∞

nan, l ≥ 0. Daca presupunem l > 0, atunci avem

limn→∞

an1

n

= l > 0

deci seriile∑

an si∑ 1

nau aceeasi natura, deci ambele divergente, contradictie.

Page 40: Teme si probleme pentru concursurile studentesti de matematica ...

Siruri si serii numerice 33

Problema 7.56 Sa se arate ca daca sirul (an)n este descrescator la zero si seria

∞∑n=1

an

este convergenta, atunci

limn→∞

nan = 0.

Solutie. Fie

xn = a1 + a2 + · · ·+ an − nan, n ≥ 1.

Sirul (xn)n este majorat de (Sn)n, sirul sumelor partiale ale seriei date, deci este marginit.Avem

xn+1 − xn = n(an − an+1) ≥ 0,

deci sirul (xn)n este crecator.In concluzie sirul xn = Sn − nan este convergent. Rezulta ca sirul (nan)n este convergentsi conform problemei 7.55 obtinem ca lim

n→∞nan = 0.

Problema 7.57 Sa se arate ca daca seriile∞∑n=1

a2n si

∞∑n=1

b2n sunt convergente, atunci seriile

∞∑n=1

anbn si

∞∑n=1

(an + bn)2 sunt convergente.

Solutie. Avem (n∑k=1

|akbk|

)2

≤n∑k=1

a2k

n∑k=1

b2k

saun∑k=1

|akbk| ≤

√√√√ n∑k=1

a2k

n∑k=1

b2k

din care rezulta∞∑k=1

|akbk| ≤

√√√√ ∞∑k=1

a2k

∞∑k=1

b2k

Avem: (n∑k=1

(ak + bk)2

)1/2

(n∑k=1

a2k

)1/2( n∑k=1

b2k

)1/2

din care∞∑k=1

(ak + bk)2 ≤

∞∑k=1

a2k

∞∑k=1

b2k.

(S-au folosit inegalitatile Cauchy-Schwartz si Minkowski.)

Problema 7.58 Sa se arate ca daca seria∞∑n=1

a2n este convergenta, atunci seria

∞∑n=1

ann

este convergenta.

Solutie. Luam ın exercitiul anterior bn =1

n.

Page 41: Teme si probleme pentru concursurile studentesti de matematica ...

34

Problema 7.59 Sa se arate ca seria

∞∑n=1

cosn

neste convergenta, dar nu este absolut con-

vergenta.

Solutie. Daca luam an = cosn si bn =1

n, sirul sumelor partiale ale seriei

∞∑n=1

an este

marginit, iar sirul (bn)n este descrescator la zero, deci conform criteriului lui Abel seriaeste convergenta.

Pentru seria valorilor absolute∞∑n=1

| cosn|n

, consideram functia

f(x) = | cosx|+ | cos(x+ 1)|, f : R→ [0,∞),

care este continua si are un minim diferit de zero, deci f(x) ≥ m > 0, ∀ x ∈ R (ısi atingeminimul pe intervalul [0, 2π]). Avem:

| cos 1|1

+| cos 2|

2+| cos 3|

3+| cos 4|

4+ · · ·+ | cos(2n− 1)|

2n− 1+| cos 2n|

2n≥

≥ | cos 1|+ | cos 2|2

+| cos 3|+ | cos 4|

4+ · · ·+ | cos(2n− 1)|+ | cos 2n|

2n≥

≥ m

2+m

4+ · · ·+ m

2n=m

2

(1 +

1

2+ · · ·+ 1

n

),

deci sirul sumelor partiale are limita ∞.

Problema 7.60 Fie

∞∑n=1

an o serie divergenta cu termeni pozitivi si (Sn)n sirul sumelor

partiale. Sa se arate ca:

a) Seria∞∑n=1

anSn

este divergenta.

b) Seria∞∑n=1

an

S1+αn

este convergenta pentru α > 0.

Solutie. a) Avem:

an+1

Sn+1+an+2

Sn+2+ · · ·+ an+p

Sn+p≥ an+1 + · · ·+ an+p

Sn+p=Sn+p − SnSn+p

.

Dar

limp→∞

Sn+p − SnSn+p

= 1 6= 0,

deci sirul sumelor partiale ale seriei∑ an

Sneste divergent conform criteriului general al

lui Cauchy (C1).

b) Consideram diferenta:lnSn−1

Sαn−1

− lnSnSαn

pentru care aplicam teorema lui Lagrange si

avem:lnSnSαn

− lnSn−1

Sαn−1

= (Sn − Sn−1)f ′(αn), αn ∈ (Sn−1, Sn)

Page 42: Teme si probleme pentru concursurile studentesti de matematica ...

Siruri si serii numerice 35

si f(x) =lnx

xα+1.

Avem:lnSn−1

Sαn−1

− lnSnSαn

= (Sn − Sn−1)α lnSn−1 − 1

Sα+1n

>

>Sn − Sn−1

Sα+1n

=an

Sα+1n

pentru n suficient de mare deci

an

Sα+1n

<lnSn−1

Sαn−1

− lnSnSαn

, n ∈ N.

∑n≥N

an

Sα+1n

<∑n≥N

(lnSn−1

Sαn−1

− lnSnSαn

)=

lnSN−1

SαN−1

deci restul de ordin N al seriei∑ an

Sα+1n

este marginit, deci convergent.

Problema 7.61 Fie seria convergenta∞∑n=0

an si Sn =n∑k=0

ak. Sa se arate ca pentru orice

a ∈ (−1, 1) seria

∞∑n=0

Snan este convergenta si

∞∑n=0

Snan =

1

1− a

∞∑n=0

anan.

Solutie. Seria∞∑n=0

Snan este produsul Cauchy al seriilor

∞∑n=0

an si

∞∑n=0

anan, ambele

convergente, iar suma primei serii este1

1− a.

Problema 7.62 Fie (an)n un sir cu termeni reali pozitivi. Sa se arate ca produsul

∞∏n=1

(1+

an) este convergent daca si numai daca seria∞∑n=1

an este convergenta.

Solutie. Avem inegalitatile

a1 + a2 + · · ·+ an ≤ (1 + a1)(1 + a2) · · · (1 + an)

si

(1 + a1)(1 + a2) · · · (1 + an) ≤ ea1+a2+···+an ,

(ex ≥ 1 + x, x ≥ 0).

Problema 7.63 Fie (εn)n un sir cu termenii εn ∈ −1, 1, n ∈ N. Sa se arate ca suma

seriei

∞∑n=0

εnn!

este un numar irational.

Page 43: Teme si probleme pentru concursurile studentesti de matematica ...

36

Solutie. Prin absurd presupunem ca

∞∑n=0

εnn!

=p

q∈ Q.

Inmultim cu q! si obtinem

(q − 1)! · p =

q∑n=0

q!εnn!

+∞∑

n=q+1

q!εnn!

Cum prima suma este numar ıntreg rezulta ca

∞∑n=q+1

q!εnn!

ar fi numar ıntreg.

Avem:∣∣∣∣∣∣∞∑

n=q+1

q!εnn!

∣∣∣∣∣∣ ≤∞∑

n=q+1

q!

n!≤ 1

q + 1+

1

(q + 1)(q + 2)+

1

(q + 1)(a+ 2)2+ · · · =

=1

q + 1· 1

1− 1

q + 2

=q + 2

(q + 1)2≤ 3

4.

Ramane de aratat doar ca suma nu poate fi egala cu zero.Dar: ∣∣∣∣∣∣

∞∑n=q+1

εnq!

n!

∣∣∣∣∣∣ ≥∣∣∣∣∣∣ 1

q + 1−

∞∑n=q+2

q!

n!

∣∣∣∣∣∣ > 1

q + 1− 1

q(q + 1)≥ 0.

Observatie. In particular rezulta ca e 6∈ Q.

Problema 7.64 Se considera sirul (an)n definit prin relatia de recurenta

an+1 = arctg an, n ≥ 1 si a1 = 1.

a) Sa se arate ca limn→∞

an = 0.

b) Sa se arate ca limn→∞

√nan =

√23 .

c) Sa se studieze convergenta seriei

∞∑n=1

aαn, α ∈ R.

Solutie. a) Se arata imediat ca sirul (an)n este strict descrescator si marginit inferiorde 0. Fie a = lim

n→∞an. Trecand la limita relatia de recurenta obtinem a = arctg a, deci

a = 0.b) Fie xn =

√nan =

√n1

a2n

. Cum sirul ( 1a2n

) este strict crescator si limn→∞

1a2n

=∞ putem

aplica Stolz-Cesaro si obtinem:

limn→∞

n1

a2n

= limn→∞

(n+1)−n1

a2n+1− 1

a2n

= limn→∞

11

a2n+1− 1

a2n

= limn→∞

(n+1)−n1

arctg2an− 1

a2n

= limn→∞

a2n−arctg2ana2narctg2an

=

limn→∞

a2n−arctg2ana4n

limn→∞

( anarctgan

)2 = limn→∞

an+arctganan

limn→∞

an−arctgana3n

= 2 limx→0

x−arctgxx3

= 23 si

de aici avem limn→∞

√nan =

√23 .

Page 44: Teme si probleme pentru concursurile studentesti de matematica ...

Siruri si serii numerice 37

c) Din limn→∞

aαn1

nα2

= (23)α rezulta, pe baza criteriului comparatiei C10’, ca seria

∞∑n=1

aαn

are aceeasi natura cu seria

∞∑n=1

1

nα2

, deci este convergenta daca si numai daca α > 2.

Problema 7.65 Fie

∞∑n=1

an o serie divergenta cu termeni pozitivi si a1 > 1.

Sa se arate ca:

a) Seria∞∑n=1

an+1

Sn lnSneste divergenta.

b) Seria∞∑n=1

an

Sn ln2 Sneste convergenta, unde Sn = a1 + a2 + · · ·+ an.

Solutie. a)

∞∑n=1

an+1

Sn lnSn=

∞∑n=1

Sn+1 − SnSn lnSn

Se aplica teorema lui Lagrange functiei f(x) = ln lnx pe intervalele [Sn, Sn+1] si avem

n∑k=1

ak+1

Sk lnSk= ln lnSn − ln lnS1 →∞

b) Se aplica teorema lui Lagrange functiei f(x) = − 1

lnx.

Problema 7.66 Fie∞∑n=1

an o serie divergenta cu termeni pozitivi astfel ca limn→∞

an = 0.

Sa se arate ca sirul (Sn)n∈N∗ este dens ın [0, 1], unde Sn = a1 + a2 + · · ·+ an si x estepartea fractionara a lui x.

Solutie. Daca 0 < a < b < 1 si b − a = ε, exista Nε ∈ N astfel ca an < ε, n > N(ε).Fie SNε = m + bN , m ∈ Z, bN ∈ [0, 1). Daca bN ∈ (a, b) am terminat. Daca bN < amai adaugam un numar minim de termeni din serie pana intram ın intervalul (a, b). DacabN > b, mai adaugam un numar minim de termeni pana ajungem la m+1+α cu α ∈ (a, b).

Problema 7.67 Fie (pn)n∈N∗ sirul numerelor prime. Sa se arate ca seria

∞∑n=1

1

pneste

divergenta.

Solutie. Daca seria ar fi convergenta, atunci ar exista N ∈ N astfel ca

∞∑n=N+1

1

pn<

1

2.

Orice numar de forma 1 + kp1p2 . . . pN = 1 + kP este un produs de numere prime(eventual cu exponenti) care nu fac parte din numerele p1, p2, . . . , pN . Multimea numerelor

de forma1

Mın care M este format doar cu numerele prime pN+1, pN+2, . . . este

1

pn1

,1

pn1pn2

,1

pn1pn2 . . . pn3

, . . .

Page 45: Teme si probleme pentru concursurile studentesti de matematica ...

38

suma lor fiind∞∑m=1

( ∞∑n=N+1

1

pn

)m<

∞∑m=1

(1

2

)m= 1

si atunci∞∑k=1

1

1 + kP< 1

deci seria

∞∑k=1

1

1 + kPar fi convergenta, contradictie.

Problema 7.68 Sa se studieze convergenta seriei∑n≥1

sinnx

n, x ∈ R.

Generalizare la seria∑n≥1

sinnx

nα, x ∈ R, α ∈ R.

Solutie. Fie xn = sinnxn . Daca x = kπ, k ∈ Z, atunci xn = 0,∀n ∈ N; presupunem ın

continuare ca x 6= kπ, k ∈ Z. Aratam mai ıntai ca seria nu este absolut convergenta:

|xn| =| sin(nx)|

n≥ sin2(nx)

n=

1− cos(2nx)

2n,∀n ∈ N?.

Deci, presupunand prin absurd ca seria data ar fi absolut convergenta, ar rezulta (cu

criteriul de comparatie) ca si seria∑

n≥11−cos(2nx)

2n ar fi convergenta. Seria∑

n≥1cos(2nx)

2n

este convergenta (criteriul lui Dirichlet): fie an = 12n si un = cos(2nx). Atunci (an) este

descrescator la 0, iar (un) are sirul sumelor partiale marginit:∣∣∣∣∣n∑k=1

cos(2nx)

∣∣∣∣∣ =

∣∣∣∣sin(nx) cos(n+ 1)x

sinx

∣∣∣∣ ≤ 1

| sinx|.

Rezulta ca si seria∑

n≥11

2n ar trebui sa fie convergenta, fiind suma a doua serii conver-gente, contradictie.

Seria∑

n≥1sinnxn este convergenta (ca mai sus, cu criteriul lui Dirichlet).

Problema 7.69 Sa se studieze convergenta seriei∑n≥1

(−1)n n√n sin

1

n.

Solutie. Seria nu este absolut convergenta (se compara la limita cu seria armonica).Seria este alternata; vom demonstra ca sirul an = n

√n sin 1

n este descrescator la 0, deciseria converge.Evident an → 0; pentru a arata ca an este descrescator (ıncepand de la un rang), fie

functia f(x) = x1x sin 1

x . Calculam

f ′(x) = x1x−2

((1− lnx) sin

1

x− cos

1

x

).

Pentru a studia semnul derivatei (pentru x ”mare”), calculam:

limx→∞

(1− lnx) sin1

x− cos

1

x= −1 + lim

x→∞

sin 1x

1x

· 1− lnx

x= −1,

deci f ′(x) < 0 pentru x suficient de mare, deci sirul an este descrescator.

Page 46: Teme si probleme pentru concursurile studentesti de matematica ...

Siruri si serii numerice 39

Problema 7.70 Fie α, β, γ trei numere strict pozitive. Folosind criteriul lui Gauss, sa sestudieze convergenta seriei:∑

n≥1

α(α+ 1) · · · (α+ n− 1)

n!· β(β + 1) · · · (β + n− 1)

γ(γ + 1) · · · (γ + n− 1).

Solutie. Criteriul lui Gauss: daca (an)n este un sir de numere strict pozitive, astfelıncat exista λ > 1, µ ∈ R si un sir (θn)n astfel ıncat

an+1

an= 1− µ

n− θnnλ,

atunci seria∑n≥1

an converge daca µ > 1 si diverge daca µ ≤ 1.

Aplicand criteriul lui Gauss seriei date, obtinem:

an+1

an=n2 + (α+ β)n+ αβ

n2 + (1 + γ)n+ γ= 1− 1 + γ − α− β

n− γ − αβ

n2

Rezulta ca daca γ > α + β atunci seria data converge, iar daca γ ≤ α + β atunci seriadiverge.

Problema 7.71 Sa se dea un exemplu de doua siruri (an)n si (bn)n astfel ıncat:limn→∞

anbn

= 1, dar seriile∑

n an si∑

n bn sa aiba naturi diferite.

Solutie. Fie, de exemplu, an = (−1)n

n si bn = (−1)n

n + 1n lnn . A doua serie este divergenta

(criteriul integral).

Problema 7.72 Fie (an)n si (bn)n doua siruri reale astfel ıncat seria∑

n≥1(bn − bn+1)este absolut convergenta si seria

∑n≥1 an este convergenta.

Sa se demonstreze ca seria∑

n≥1 anbn este convergenta.

Solutie. Facem mai ıntai urmatoarea observatie generala. Pentru orice siruri de numere(xn)n si (yn)n are loc urmatoarea identitate (”sumare prin parti”):

n∑k=1

xkyk =

n−1∑k=1

(x1 + x2 + · · ·+ xk)(yk − yk+1) + (x1 + x2 + · · ·+ xn)yn

Fie An =∑n

k=1 ak si Sn =∑n

k=1 akbk sumele partiale asociate seriilor∑

n an si respectiv∑n anbn. Insumand prin parti, obtinem:

Sn =n∑k=1

akbk =n−1∑k=1

Ak(bk − bk+1) +Anbn

Demonstratia se ıncheie daca aratam ca seria∑

nAn(bn − bn+1) si sirul (Anbn)n suntconvergente. Din convergenta seriei

∑n(bn−bn+1) rezulta convergenta sirului (bn)n si deci

(Anbn)n este convergent. Sirul An este marginit si din faptul ca seria∑

n≥1(bn − bn+1)este absolut convergenta rezulta ca seria

∑nAn(bn− bn+1) este absolut convergenta, ceea

ce ıncheie demonstratia.Observatie. In ultima implicatie ar fi fost suficient ca seria

∑n≥1(bn − bn+1) sa fie

(doar) convergenta?

Page 47: Teme si probleme pentru concursurile studentesti de matematica ...

40

Problema 7.73 Sa se calcuze sumele urmatoarelor serii:

(i)∑n≥0

cn, unde cn =

n∑k=0

xkyn−k, |x| < 1, |y| < 1.

(ii)∑n≥1

cn, unde cn =

n∑k=1

1

(n− k + 1)!k(k + 1).

Solutie. Vom aplica teorema lui Mertens pentru produsul Cauchy a doua serii.

(i) Evident,∑n≥0

cn =

∑n≥0

xn

∑n≥0

yn

= (1− x)−1(1− y)−1.

(ii) Seria data este produsul seriilor∑n≥1

1

n(n+ 1)si∑n≥1

1

n!. Prima serie are suma 1, iar a

doua e− 1, deci raspunsul este e− 1.

Produse infinite

Fie (an)n un sir de numere reale strict pozitive.

Produsul infinit∏n≥1

an se numeste convergent daca limn→∞

n∏k=1

ak exista si este nenula, sau,

echivalent, daca seria∑n≥1

ln an este convergenta. Este usor de observat ca o conditie nece-

sara (dar nu si suficienta) pentru convergenta produsului infinit∏n≥1

an este an → 1.

Dam ın continuare o lista cu cateva formule-produs uzuale:

sinx = x∏n≥1

(1− x2

π2n2

)= x

∏n≥1

cosx

2n

sinhx = x∏n≥1

(1 +

x2

π2n2

)

cosx =∏n≥1

(1− x2

π2(n− 12)2

)

cosx =∏n≥1

(1− x2

π2(n+ 12)2

)

Produse. Probleme

Problema 7.74 Sa se calculeze produsele:

(i)∏n≥1

a(−1)n

n , a > 0.

(ii)∏n≥1

e1n

1 + 1n

.

Solutie. (i) limn→∞

n∏k=1

a(−1)k

k = limn→∞

a∑nk=1

(−1)k

k = a− ln 2.

(ii) limn→∞

n∏k=1

e1k

1 + 1k

= limn→∞

e∑nk=1

1k

n+ 1= lim

n→∞

e∑nk=1

1k

n· n

n+ 1=

Page 48: Teme si probleme pentru concursurile studentesti de matematica ...

Siruri si serii numerice 41

= limn→∞

n

n+ 1· e

∑nk=1

1k−lnn = ec, unde c este constanta lui Euler.

Problema 7.75 Fie (an)n un sir cu termeni strict pozitivi. Atunci:

(i) produsul infinit∏n≥1

(1 + an) converge daca si numai daca seria∑n≥1

an converge.

(ii) produsul infinit∏n≥1

(1 − an) converge daca si numai daca seria∑n≥1

an converge (cu

ipoteza suplimentara an 6= 1, ∀n).

Solutie. (i) Deoarece an > 0 si din inegalitatea 1 + x ≤ ex, ∀x ≥ 0, rezulta:

n∑k=1

ak ≤n∏k=1

(1 + ak) ≤ e∑nk=1 ak ,

ceea ce demonstreaza prima afirmatie.

(ii) Presupunem mai ıntai ca seria∑n≥1

an converge. Rezulta ca restul seriei tinde la 0, deci

exista m ∈ N suficient de mare (fixat de aici ınainte) astfel ıncat∑k≥m

ak < 2−1. Rezulta

(pentru n > m) inegalitatea:

n∏k=m

(1− ak) ≥ 1−n∑

k=m

ak >1

2.

Daca notam Pn =

n∏k=1

(1− ak), atunci din relatia Pn = Pm−1

n∏k=m

(1− ak), rezulta ca sirul(PnPm−1

)n

este descrescator si minorat de 2−1, deci are o limita P ∈ [2−1, 1]. Rezulta ca

Pn este convergent la o limita nenula, deci produsul infinit∏n≥1

(1− an) converge.

Pentru a demonstra reciproca, presupunem ca seria∑n≥1

an este divergenta (are suma ∞).

Putem presupune ca an → 0 (altfel 1 − an 6→ 1) si deci exista m ∈ N astfel ıncat an ∈[0, 1), n ≥ m.Din inegalitatea 1− x ≤ e−x, x ∈ [0, 1), rezulta:

0 ≤n∏

k=m

(1− ak) ≤ e−∑nk=m ak → 0 (pentru n→∞),

ceea ce ıncheie demonstratia.

Problema 7.76 Notam cu ζ(p) =∞∑n=1

1

np, p > 1 functia lui Euler si cu

c = limn→∞

(n∑k=1

1

k− lnn

)

constanta lui Euler.

Page 49: Teme si probleme pentru concursurile studentesti de matematica ...

42

Sa se arate ca∞∑p=2

(−1)p

p+ 1ζ(p) = 1 +

c

2− ln

√2π.

Concurs Ukraina

Solutie. Pentru x ∈ (0, 1) consideram functia

f(x) =∞∑p=2

(−1)p

p+ 1xp.

Avem:

f(x) =1

x

∞∑p=2

(−1)p

p+ 1xp+1 =

1

x

∞∑p=2

(−1)p∫ x

0tpdt

=1

x

∫ x

0

∞∑p=0

(−t)p − 1 + t

dt =1

x

∫ x

0

(1

1 + t− 1− t

)dt

=1

x

[ln(1 + t)− t+

t2

2

] ∣∣∣x0

=1

x

(ln(1 + x)− x+

x2

2

)= −1 +

x

2+

ln(1 + x)

x.

Acum suma ceruta o exprima cu functia f :

S =∞∑p=2

(−1)p

p+ 1ζ(p) =

∞∑p=2

(−1)p

p+ 1

∞∑n=1

1

np= lim

m→∞

m∑p=2

(−1)p

p+ 1

∞∑n=1

1

np

= limm→∞

∞∑n=1

m∑p=2

(−1)p

(p+ 1)np=

∞∑n=1

∞∑p=2

(−1)p

(p+ 1)np=

∞∑n=1

f

(1

n

)

=∞∑n=1

(−1 +

1

2n+ n ln

(1 +

1

n

))

=1

2

∞∑n=1

(1

n− ln

(1 +

1

n

))+∞∑n=1

((n+

1

2

)ln

(1 +

1

n

)− 1

)

=1

2limm→∞

m∑n=1

(1

n− ln(n+ 1) + lnn

)+

∞∑n=1

ln

((n+ 1

n

)n+ 12

· 1

e

)

=1

2limm→∞

(1 +

1

2+ . . .+

1

m− ln(m+ 1)

)+ limm→∞

lnn∏k=1

((k + 1

k

)n+ 12

· 1

e

)

=1

2e+ ln lim

m→∞

(n+ 1)m√n+ 1

(n+ 1)!en

=1

2e+ ln lim

n→∞

(n+ 1)n√n+ 1

nn√

2πn=

1

2e+ ln

e√2π

= 1 +1

2e− ln

√2π

Page 50: Teme si probleme pentru concursurile studentesti de matematica ...

Capitolul 8

Calcul diferential pentru functii deo variabila reala

Definitii si rezultate

Definitie. Functia f : D → R se numeste uniform continua pe D daca pentru oriceε > 0 exista δ > 0 astfel ıncat oricare ar fi x′, x′′ ∈ D cu |x′−x′′| < δ, avem |f(x′)−f(x′′)| <ε.

Teorema lui Fermat. Daca x0 este punct de extrem (local) pentru functia f : I → IR,I ⊂ IR interval, daca x0 este punct interior pentru I si daca functia f este derivabila ınx0, atunci f ′(x0) = 0.

Teorema. Fie f : I → R o functie derivabila pe intervalul I. Atunci functia f ′ : I → Rare proprietatea Darboux.

Teorema lui Rolle. Fie f : [a, b] → R continua pe [a, b] si derivabila pe (a, b) astfelıncat f(a) = f(b). Atunci exista c ∈ (a, b) astfel ıncat f ′(c) = 0.

Teorema lui Lagrange. Fie f : [a, b]→ R. Daca f este continua pe [a, b] si derivabilape (a, b), atunci exista c ∈ (a, b) astfel ıncat

f(b)− f(a) = f ′(c)(b− a).

Teorema lui Cauchy. Fie f : [a, b] → R, g : [a, b] → R doua functii care ındeplinescconditiile: f, g sunt continue pe [a, b], f, g sunt derivabile pe (a, b), g′(x) 6= 0 pentru oricex ∈ (a, b). Atunci exista c ∈ (a, b) astfel ıncat

f(b)− f(a)

g(b)− g(a)=f ′(c)

g′(c).

Formula lui Taylor cu rest Peano. Fie I un interval deschis si f : I → R o functiede n ori derivabila ın x0 ∈ I. Atunci exista o functie ω : I → R astfel ıncat

f(x) = f(x0) +f ′(x0)

1!(x− x0) +

f ′′(x0)

2!(x− x0)2 + . . .

+fn(x0)

n!(x− x0)n + ω(x)(x− x0)n, ∀x ∈ I.

silimx→x0

ω(x) = 0

Notatie. In cele ce urmeaza vom folosi notatia o(f) pentru a desemna o functie g

(definita ıntr–o vecinatate a lui 0), care are proprietatea ca limx→0

g(x)

f(x)= 0.

43

Page 51: Teme si probleme pentru concursurile studentesti de matematica ...

44

In cazul particular x0 = 0 se obtine formula lui MacLaurin:

(∗) f(x) = f(0) +f ′(0)

1!x+

f ′′(0)

2!x2 + . . .+

f (n)(0)

n!xn + o(xn)

Formula lui Taylor cu restul lui Lagrange. Fie I interval deschis si f : I → R den+ 1 ori derivabila pe I. Atunci pentru x0 si x ∈ I exista c ıntre x si x0 astfel ıncat

f(x) = f(x0) +f ′(x0)

1!(x− x0) + . . .+

f (n)(x0)

n!(x− x0)n+

+f (n+1)(c)

(n+ 1)!(x− x0)n+1.

In cazul particular x0 = 0, se obtine formula lui MacLaurin :

f(x) = f(0) +f ′(0)

1!x+

f ′′(0)

2!x2 + . . .+

f (n)(0)

n!xn +

f (n+1)(c)

(n+ 1)!xn+1.

Probleme

Problema 8.1 Se considera functia f : R \ 0 → R f(x) := x

[1

x

]. Precizati disconti-

nuitatile functiei f si calculati limx→0

f(x).

Solutie. Se expliciteaza: pe fiecare interval(

1n+1 ,

1n

]avem f(x) = nx. Limitele laterale

ın 1n sunt 1 = f( 1

n) resp. n−1n . Analog, pe fiecare interval

(− 1n ,−

1n+1

]avem f(x) =

−(n+ 1)x. Pe (−∞,−1) functia este −x, iar pe (1,∞) este 0. Din cele de mai sus rezulta

ca limita ın 0 exista si este 1. Punctele de discontinuitate sunt de forma1

k, k ∈ Z

Problema 8.2 f : R → R o functie continua, periodica de perioada 1, adica f(x + 1) =f(x), ∀x ∈ R.

(i) Sa se arate ca f este marginita si ısi atinge marginile.(ii) Sa se arate ca exista x0 ∈ R pentru care f(x0 + π) = f(x0).

Solutie. (i) Se restrange la [0, 1], valorile ın rest fiind aceleasi.(ii) Fie g(x) := f(x + π) − f(x). Daca xm resp. xM denota puncte ın care f ısi

atinge minimul, resp, maximul, avem g(xm) = f(xm + π) − f(xm) ≥ 0 iar g(xM ) =f(xM+π)−f(xM ) ≤ 0, de unde existenta unui punct x0, ıntre xm si xM , ın care g(x0) = 0.

Problema 8.3 Functia F : (0,∞)→ R este continua si limn→∞

F (x+ n) = 0 pentru fiecare

x > 0. Rezulta ca limx→∞

F (x) = 0?

Solutie. Nu. Se construieste un contraexemplu astfel: fie f0(x) := 1 − 2|x − 12 |; apoi

fk este definita astfel: fk(x) := f0(kx) pentru x ∈ (0, 1k ) si fk(x) := 0 pentru x ≥ 1

k . Insfarsit: g(x) = fk(x − k) pentru x ∈ [k, k + 1]. Functia g este bine definita si continua.Deoarece g(k+ 1

2k ) = 1, pentru fiecare k, urmeaza ca limx→∞

g(x) nu este 0. Pe de alta parte,

pentru orice x ∈ N avem g(x + n) = 0. Daca x = [x] + x, atunci, pentru k suficient demare astfel ıncat x > 1

k avem g(x+ k) = 0.

Page 52: Teme si probleme pentru concursurile studentesti de matematica ...

Calcul diferential pentru functii de o variabila reala 45

Problema 8.4 Fie f : R → R o functie reala. Analizati daca urmatoarele afirmatii suntadevarate si justificati.

(a) Daca f este continua si Im(f) = y ∈ R| ∃x ∈ R, f(x) = y, atunci f estemonotona.

(b) Daca f este monotona si Im(f) = R, atunci f este continua.(c) Daca f este monotona si f este continua, atunci Im(f) = R.

Solutie. (a) Nu este adevarata. Consideram functia f(x) = x3−x, care este continua,Im(f) = R, dar de exemplu f(0) = 0, f(1

2) = −38 si f(1) = 0; de aceea f(0) > f(1

2), f(12) <

f(1) si f nu este monotona.(b) Este adevarata. Presupunem pentru ınceput ca f este nedescrescatoare. Pentru un

numar arbitrar a exista limitele laterale b := limxa

f(x) si c := limxa

f(x) si b ≤ c. Daca cele

doua limite sunt egale, atunci functia este continua ın a. In caz contrar, daca b < c, avemf(x) ≤ b, ∀x < a si and f(x) ≥ c, ∀x > a si de aceea Im(f) ⊂ (−∞, b)∪ (c,∞)∪f(a) nureprezinta toata multimea R. Pentru cazul functiei necrescatoare se aplica rationamentulfunctiei g(x) = −f(x).

(c) Fals. Functia f(x) = arctg x este monotona si continua, dar Im(f) = (−π2,π

2) 6= R.

Problema 8.5 Sa se determine toate functiile continue f : R → R cu proprietatea caoricare ar fi x, y ∈ R cu x− y ∈ Q, are loc f(x)− f(y) ∈ Q.

Solutie. Pentru fiecare r ∈ Q, functia x 7→ f(x + r) − f(x) este continua si arenumai valori rationale. Deci este constanta. Sa definim atunci g(r) := f(x + r) − f(x).Prin recurenta f(nr) = f(0) + ng(r), ∀n ∈ N. Deoarece f(0) − f(−r) = g(r), formulase extinde la numere ıntregi: f(kr) = f(0) + kg(r), ∀k ∈ Z. Luand r = 1

n , se gasestef(1) = f(0) + ng( 1

n). De fapt: g(r) = rg(1), ∀r ∈ Q. Adica f(r) = f(0) + rg(1), ∀r ∈ Q.Deoarece f este continua, urmeaza ca f(x) = ax + b, ∀x ∈ R, cu a ∈ Q. Reciproc, oriceasemenea functie convine.

Problema 8.6 Se da functia f : R → R. Sa se arate ca f este monotona daca si numaidaca pentru orice interval I ⊆ R, f−1(I) este interval.( f−1(A) = x ∈ R |f(x) ∈ A.)

Solutie. Presupunem ca f este crescatoare. Fie I un interval x, x′ ∈ f−1(I) cu x ≤ x′;exista deci y, y′ ∈ I cu y ≤ y′ astfel ca f(x) = y, f(x′) = y′. Pentru a arata ca f−1(I)este un interval, fie x′′ ∈ [x, x′′], din x ≤ x′′ ≤ x′ rezulta f(x) ≤ f(x′′) ≤ f(x′) si deoareceI este interval, avem f(x′′) ∈ I, deci x′′ ∈ f−1(I).

Reciproc. Presupunem ca f nu este monotona. Exista deci x1, x2, cu x1 < x2 astfelca f(x1) ≥ f(x2) si x3 < x4 astfel ca f(x3) ≤ f(x4). sau f(x1) ≥ f(x2), f(x3) ≥ f(x2).Comparand cele 4 valori se poate obtine urmatoarea conditie echivalenta. Exista x1 < x2 <x3 astfel ca f(x1) ≤ f(x2) si f(x3) ≤ f(x2). Fie a = min(f(x1), f(x3)) si b = f(x2). Atuncirezulta ca f−1[a, b) nu este interval, deoarece x1, x3 ∈ f−1[a, b), dar x2 /∈ f−1[a, b).

Problema 8.7 Fie C ⊂ R o multime nevida, ınchisa, marginita si f : C → C o functienedescrescatoare. Aratati ca exista un punct p ∈ C astfel ca f(p) = p.

Solutie. Presupunem ca f(x) 6= x, ∀x ∈ C. Fie [a, b] cel mai mic interval ınchiscare contine C. Deoarece C este ınchisa a, b ∈ C. Din ipoteza f(a) > a si f(b) < b. Fiep = supx ∈ C : f(x) > x. Deoarece C este ınchisa si f este continua f(p) ≥ p si astfelf(p) > p. Pentru orice x > p, x ∈ C avem f(x) < x. De aceea f(f(p)) < f(p), ceea cecontrazice faptul ca f este nedescrescatoare.

Page 53: Teme si probleme pentru concursurile studentesti de matematica ...

46

Problema 8.8 Fie f : [0, 1]→ [0, 1] cu proprietatea ca exista c ∈ [0, 1) astfel ca

|f(x)− f(y)| ≤ c|x− y|, ∀x, y ∈ [0, 1] (8.1)

Aratati ca sirul definit

x0 ∈ [0, 1], xn+1 = f(xn) (8.2)

este convergent la x ∈ [0, 1] si acesta este unicul punct din interval cu proprietatea f(x) = x(punct fix). Aratati ca are loc

|x− xn| ≤cn

1− c|x0 − x1|.

Solutie. Mai ıntai din conditia (8.1) deducem ca functia f este continua. Fixam x0 ∈[0, 1]. Pentru orice ε > 0, exista δ =

1

cε astfel ca daca |x− x0| < δ sa rezulte

|f(x)− f(x0)| ≤ c|x− x0| < ε.

Unicitatea. Presupunem ca ar exista doua puncte x1, x2, x1 6= x2. Atunci are loc

|x1 − x2| = |f(x1)− f(x2)| ≤ c|x1 − x2| < |x1 − x2|.

Pentru convergenta se arata ca xn este sir Cauchy. Evaluam pentru ınceput, folosind (8.1)

|xn+1 − xn| ≤ cn|x0 − x1|

Apoi

|xn+p − xn| ≤ |xn+p − xn+p−1|+ |xn+p−1 − xn+p−2|+ · · ·+ |xn+1 − xn| ≤

≤ (cn+p−1 + · · ·+ cn)|x0 − x1| =

= cn1− cp

1− c|x0 − x1| ≤

cn

1− c|x0 − x1|.

Folosind ultima relatie, rezulta ca este sir Cauchy. Trecand la limita pentru p → ∞ de-ducem

|xn − x0| ≤cn

1− c|x0 − x1|.

Problema 8.9 Fie f : R −→ R o functie continua, cu proprietatea ca exista C > 0 astfelincat

|f(x)− f(y)| ≥ C |x− y| ,∀x, y ∈ R

Sa se arate ca functia f este surjectiva.

Solutie. Evident functia este injectiva. Fiind si continua, este strict monotona. Astfel,imaginea este un interval. Daca ar fi majorat (minorat se discuta la fel) cu margineasuperioara M < ∞, atunci ar exista un sir xn cu f(xn) −→ M . Din ipoteza, ar urma casirul (xn) este sir Cauchy, deci convergent xn −→ x, adica M = f(x)

Problema 8.10 Fie f : [a, b]→ R o functie continua, atunci f este uniform continua.

Page 54: Teme si probleme pentru concursurile studentesti de matematica ...

Calcul diferential pentru functii de o variabila reala 47

Solutie. Prin reducere la absurd: faptul ca f nu ar fi uniform continua pe [a, b] setraduce prin∃ε0 > 0 astfel ıncat ∀n ∈ N, ∃x′n, x′′n ∈ [a, b] astfel ıncat

(∗) |x′n − x′′n| <1

ndar |f(x′n)− f(x′′n)| ≥ ε0.

Pe baza lemei lui Cesaro, putem extrage un subsir (x′nk)k, care sa fie convergent. Notamlimita cu x0 ∈ [a, b]. Datorita (∗), subsirul (x′′nk)k va avea aceeasi limita. Dar

0 < ε0 ≤ |f(x′nk)− f(x′′nk)| −→ 0

ceea ce constituie contradictia cautata.

Problema 8.11 Fie f : [0, 1] → R o functie continua, cu f(0) = f(1) = 0. Pentru cevalori λ ∈ (0, 1) exista x ∈ [0, 1] astfel ıncat f(x+ λ) = f(x)?

Solutie. Se arata ca orice λ = 1n , n ≥ 2 convine. Se considera functia g : [0, 1−λ]→ R,

g(x) := f(x+ λ)− f(x). Deoarece

g(0) + g(λ) + g(2λ) + . . .+ g((n− 1)λ) = 0

Rezulta ca se produce cel putin o schimbare de semn ın sirul g(0), g(λ), g(2λ), . . ., g((n−1)λ). Deci exista x pentru care g(x) = 0.

Daca λ nu este de forma 1n , atunci se construieste functia h(x) := 1 − cos

(2πxλ

).

Aceasta functie este continua iar h(0) = 0. Este evident periodica, de perioada λ. Mai multh(x) = 0 ⇔ x = k.λ, cu k ıntreg. Deci H := h(1) 6= 0. Fie atunci f(x) := h(x) −Hx =1− cos

(2πxλ

)−(1− cos

(2πλ

))x. Functia f are proprietatile cerute, dar:

f(x+ λ)− f(x) = (h(x+ λ)−H.(x+ λ))− (h(x)−H.x) = −H.λ 6= 0

Problema 8.12 Daca f este derivabila, atunci f este convexa daca si numai daca f ′ estecrescatoare.

Solutie. Fie x1 < x2 ∈ I. Daca f ′ este crescatoare, sa definim functia

g(x) := f(x)− f(x1)− (x− x1)f(x2)− f(x1)

x2 − x1

Deoarece g(x1) = g(x2) = 0 si g′ este crescatoare, urmeaza ca exista un punct c ∈ (x1, x2)astfel ıncait g′(x) ≤ 0, ∀ x ∈ (x1, c) si g′(x) ≥ 0, ∀ x ∈ (c, x2). Rezulta g(x) ≤ 0,∀x ∈ (x1, x2).

Reciproc, daca f este convexa, iar x1 < x2 ∈ I, atunci pentru orice x ∈ (x1, x2) avem:

f(x)− f(x1)

x− x1≤ f(x2)− f(x1)

x2 − x1≤ f(x)− f(x2)

x− x2

de unde, prin trecere la limita, se obtine f ′(x1) ≤ f ′(x2).

Problema 8.13 Gasiti numarul de extreme locale ale functiei g(x) = f(f(x)), dacaf(x) = x3 − 3x+ 1.

Page 55: Teme si probleme pentru concursurile studentesti de matematica ...

48

Solutie. Derivata functiei este

g′ = f ′(f(x))f ′(x) = 3(f2(x)− 1)3(x2 − 1) =

= 9(x3 − 3x)(x3 − 3x+ 2)(x2 − 1).

Radacinile derivatei, care verifica conditia de extrem sunt 0,−2,±1,±√

3. Exista deci 6puncte de extrem.

Problema 8.14 Fie f : R→ R o functie de doua ori derivabila care verifica

f ′′(x) + f ′(x) = f(x), ∀x ∈ [0, 1]

si astfel ca f(0) = f(1) = 0. Aratati ca f(x) = 0,∀x ∈ [0, 1]

Solutie. Din teorema lui Rolle, exista c ∈ (0, 1) astfel ca f ′(c) = 0. Daca presupunemca c este un punct de maxim ar urma ca f(c) > 0, iar din ipoteza f ′′(c) = f(c) > 0 ar fiimposibil, deoarece functia ar rezulta convexa ın jurul punctului de maxim. Daca f(c) < 0ar urma f ′′(c) < 0 situatie iar imposibila. Analog daca c ar fi punct de minim.

Problema 8.15 Calculand ın doua moduri derivatele de ordin p, 1 ≤ p ≤ n ale functiei

(1− ex)n, sa se deduca valoarea sumei

n∑k=0

(−1)kCknkp.

Solutie. Pe de o parte, deoarece (1 − ex)n =

n∑k=0

(−1)kCknekx, rezulta ca derivata de

ordin p este

n∑k=0

(−1)kCknkpekx. Sumele cerute se obtin deci pentru x = 0.

Pe de alta parte, se arata prin recurenta ca derivata de ordin p ( pentru 1 ≤ p ≤ n)

este:

p∑k=1

apk(ex)k(1− ex)n−k, unde app = (−1)pApn (aranjamente).

In adevar, cazul p = 1 este imediat; iar prin derivare ınca odata, se obtine:

p∑k=1

apkk(ex)k−1(1− ex)n−k +

p∑k=1

apk(ex)k(−1)kex(n− k)(1− ex)n−k−1

Regrupand termenii, gasim:

ap1(ex)(1− ex)n−1 +

p∑k=2

[apkk + apk−1(k − 1− n)

](ex)k(1− ex)n−k+

+app(p− n)(−ex)p+1(1− ex)n−p−1

ceea ce arata ca ap+1p+1 = app (p− n). Verificarea este astfel ıncheiata.

Pe baza acestei formule, valoarea derivatei de ordin p ın x = 0 este: 0, daca 1 ≤ p < n;iar pentru p = n ramane ann = (−1)nAnn = (−1)nn!.

Problema 8.16 Un segment de lungime 1 se misca astfel ıncat capetele raman pe axelede coordonate. In timp ce se misca ın plan, segmentul schimba culoarea partii marginitede el la stanga si axe. Gasiti ecuatia curbei care separa partea din plan ın care culoareas-a schimbat, de partea din plan ın care culoarea ra mane aceeasi.

Page 56: Teme si probleme pentru concursurile studentesti de matematica ...

Calcul diferential pentru functii de o variabila reala 49

Solutie. Raspunsul este o parte a curbei a x2/3 + y2/3 = 1 (astroida). Evident pentrux ∈ [0, 1] ordonata punctului corespunzator de pe curba va fi egala cu cea mai marevaloare a ordonatelor punctelor de pe segment, pentru x fixat. Sa examinam pozitia ıncare unul dintre capete este A(t, 0) si celalalt B(0,

√1− t2), t ∈ [0, 1]. Fie y(x, t) ordonata

punctelor de pe segmentul de extremitati aA si B, corespunzator abscisei x. y = y(x) =maxy(x, t), t ∈ [x, 1]. (Pozitia segmentului ın care t < x nu trebuie examinata). Astfel,

y(x; t) = (1− x/t)√

1− t2) si

y′ =x

t2

√1− t2 − (1− x

t)

t√1− t2

=x− t3

t2√

1− t2.

Derivata se anuleaza ın t = 3√x care este punct de maxim, de unde urmeaza

y(x, 3√x) =

(1− x2/3

)3/2.

Problema 8.17 Data functia

f(x) =√

(1 + tg(2x))(1 + tg(4x))(1 + tg(6x)) · · · (1 + tg(32x))

calculati f ′(0).

Solutie. Notam u(x) = (1 + tg(2x))(1 + tg(4x))(1 + tg(6x)) · · · (1 + tg(32x)) si avem

u(0) = 1. Observam ca f ′(x) =u′(x)

2√u(x)

, iar u′ este o suma de 16 termeni, fiecare fiind

produs de 15 factori care apar ın scrierea lui u si un factor de forma (1+tg2kx)′ =2k

cos2 2kx.

Fiecare termen are ın x = 0 valoarea 2k, k = 1, · · · , 16, iar

f ′(0) =u′(0)

2√u(0)

=1

2

16∑k=1

2k = 136.

Problema 8.18 Fie f : R→ R, f(x) = 1x2+1

. Gasiti f (6)(0) .

Solutie. Raspunsul este -720. Din formula lui MacLaurin:

f(x) = f(0) +f ′(0)

1!x+

f ′′(0)

2!x2 + . . .+

f (6)(0)

6!x6 + o(x6).

Pe de alta parte seria geometrica cu ratia x2 este

1

x2 + 1= 1− x2 + x4 − x6 + · · · .

Din unicitate rezulta afirmatia.

Problema 8.19 O particula se misca de-a lungul unei drepte. Directia miscarii se poateschimba, dar acceleratia ın orice moment nu depaseste 1 m/sec ın valoare absoluta. Dupao secunda de la ınceputul miscarii particula se ıntoarce ın punctul de pornire. Aratati caviteza la 0,5 sec de la ınceputul miscarii nu este mai mare ca 0,25 m/sec.

Page 57: Teme si probleme pentru concursurile studentesti de matematica ...

50

Solutie. In calculele urmatoare se subıntelege ca timpul se masoara ın secunde, iardistantele ın metri. Fie v(t) viteza, iar a(t) acceleratia particulei la momentul t. Din enunt

avem |a(t)| = |v′(t)| ≤ 1 si

∫ 1

0v(t)dt = 0. Sa estimam v(0, 5).

|v(0, 5)| = |v(0, 5)− 0| =∣∣∣∣v(0, 5)−

∫ 1

0v(t)dt

∣∣∣∣ =

∣∣∣∣v(0, 5)

∫ 1

0dt−

∫ 1

0v(t)dt

∣∣∣∣ =

∣∣∣∣∫ 1

0(v(0, 5)− v(t))dt

∣∣∣∣ .Folosind teorema lui Lagrange v(0, 5)− v(t) = v′(c)(0, 5− t), c ∈ (0, 5, t). Astfel

|v(0, 5)| =∣∣∣∣v(0, 5)−

∫ 1

0v(t)dt

∣∣∣∣ ≤ ∫ 1

0|v′(c)(0, 5− t)|dt =

∫ 1

0|v′(c)||(0, 5− t)|dt ≤

∫ 1

0|(0, 5− t)|dt = 1/4.

Problema 8.20 Fie Pn(x) un polinom de grad par n (n > 1), care are coeficientul domi-nant pozitiv si fie Pn(x) > P ′′n (x), ∀x ∈ R. Aratati ca Pn(x) > 0, ∀x ∈ R.

Solutie. Sa observam ca daca Pn are grad par, limx→±∞

Pn(x) = ∞. De aceea exista

x0 punct de minim absolut pentru polinom, ın care P ′n(x0) = 0 si P ′′n (x0) ≥ 0. Folosindipoteza avem

Pn(x) ≥ Pn(x0) > P ′′n (x0) ≥ 0, ∀x.

Problema 8.21 Aratati ca pentru orice polinom p(x) de grad n > 1 si orice punct Q,numarul tangentelor la graficul lui p(x) care trec prin Q nu depaseste n.

Solutie. Derivata functiei p(x) = a0 + a1x + · · · + anxn este p′(x) = a1 + 2a2x +

· · · + nanxn−1. Ecuatia tangentei ın punctul (x0, p(x0)) este y − y(x0) = p′(x)(x − x0).

Presupunem ca Q are coordonatele (a, b) si ınlocuind ın ecuatia tangentei obtinem

b− (a0 + a1x0 + · · ·+ anxn0 ) = (a1 + 2a2x0 + · · ·+ nanx

n−10 )(a− x0).

Daca n > 1, coeficientii lui xn0 din cei doi membrii ai relatiei nu pot fi egali si egalitateaconduce la o ecuatie algebrica de ordin n; urmeaza ca este imposibil sa gasim un numarde solutii mai mare ca n.

Problema 8.22 Aratati ca are loc identitatea:

2 arccosx = arccos(2x2 − 1), 0 ≤ x ≤ 1.

Solutie. Fie f(x) = 2 arccosx− arccos(2x2 − 1) 0 ≤ x ≤ 1. Functia este continua pe[0, 1], derivabila pe (0, 1) si se verifica imediat ca

f ′(x) = − 2√1− x2

+2x

|x|√

1− x2.

Deci f ′(x) = 0, x ∈ (0, 1). Deci f este o constanta, iar din faptul ca f(0) = 0 si continuitaterezulta afirmatia.

Page 58: Teme si probleme pentru concursurile studentesti de matematica ...

Calcul diferential pentru functii de o variabila reala 51

Problema 8.23 Fie f o functie derivabila pe (a, b) cu derivata continua, cu proprietatilelimxa

f(x) =∞, limxb

f(x) = −∞ si f ′(x) + f2(x) ≥ −1, ∀x ∈ (a, b). Aratati ca b− a ≥ πsi dati un exemplu pentru care b− a = π.

Solutie. Din inegalitate deducem

d

dx(arctg f(x) + x) =

f ′(x)

1 + f2(x)+ 1 ≥ 0 ∀x ∈ (a, b)

urmeaza ca arctg f(x) + x este nedescrescatoare pe interval si folosind limitele obtinemπ/2+a ≤ −π/2+b, de unde b−a ≥ π. Egalitatea are loc pentru f(x) = ctg x, a = 0, b = π.

Problema 8.24 Fie f o functie de doua ori diferentiabila , cu derivata a doua continua

pe (0,∞), astfel ca limx0

f ′(x) = −∞ si limx0

f ′′(x) =∞. Aratati ca limx0

f(x)

f ′(x)= 0.

Solutie. Deoarece f ′ tinde la −∞ si f ′′ tinde la ∞, cand x tinde la 0 prin valorimai mari, exista un interval (0, r) astfel ca f ′(x) < 0 si f ′′(x) > 0, ∀x ∈ (0, r). Decif este descrescatoare si f ′ este crescatoare pe (0, r). Din teorema de medie, pentru orice0 < x < x0 < r avem

f(x)− f(x0) = f ′(ξ)(x− x0) > 0

pentru un ξ ∈ (x, x0). Folosind faptul ca f ′ este crescatoare, f ′(x) < f ′(ξ) < 0, obtinem

x− x0 <f ′(ξ)

f ′(x)(x− x0) =

f(x)− f(x0)

f ′(x)< 0.

Luand limita cand x 0 obtinem

−x0 ≤ lim infx0

f(x)

f ′(x)≤ lim sup

x0

f(x)

f ′(x)≤ 0.

Deoarece acest lucru are loc pentru orice x0 ∈ (0, r) deducem ca exsita lims→0

f(x)

f ′(x)si

valoarea ei este 0.

Problema 8.25 Fie f : [0, 1] → R o functie derivabila, cu f(0) = 0. Daca exista M ≥ 0astfel ıncat |f ′(x)| ≤M |f(x)|, pentru orice x ∈ [0, 1], atunci f este identic nula.

Solutie. Definind g : [0, 1] → R prin g(x) := f2(x), obtinem o functie pozitiva,

derivabila pe [0, 1], care satisface g(0) = 0 si |g′(x)| ≤ M

2|g(x)|, pentru orice x ∈ [0, 1]. Fie

mai departe h : [0, 1]→ R, h(x) := e−M

2xg(x). Deoarece h′(x) = e

−M2x

[g′(x)− M

2g(x)],

ipoteza arata ca h′(x) ≤ 0. Deci functia h rezulta descrescatoare, pozitiva si h(0) = 0. Deaici concluzia.

Problema 8.26 Exista functii continuu diferentiabile f : R → R astfel ca pentru oricex ∈ R sa avem f(x) > 0 si f ′(x) = f(f(x))?

Solutie. Presupunem ca ar exista o astfel de functie. Deoarece f ′(x) = f(f(x)) > 0,functia este strict crescatoare. Din f(x) > 0 deducem f(f(x)) > f(0), ∀x. Deci f ′(x) >f(0) si pentru orice x < 0 avem f(x) < f(0) + xf(0) = (1 + x)f(0). Deci daca x ≤ −1rezulta f(x) ≤ 0, contrazicand proprietatea f(x) > 0. Concluzia este ca astfel de functiinu exista.

Page 59: Teme si probleme pentru concursurile studentesti de matematica ...

52

Problema 8.27 Functia f : R → R este de doua ori diferentiabila si satisface f(0) =2, f ′(0) = −2 si f(1) = 1. Aratati ca exista un numar real ξ ∈ (0, 1), astfel ca

f(ξ)f ′(ξ) + f ′′(ξ) = 0.

Solutie. Definim functia g(x) := 12f

2(x)+f ′(x). Deoarece g(0) = 0 si and f(x)f ′(x)+f ′′(x) = g′(x) este suficient sa aratam ca exista un numar real 0 < η ≤ 1 astfel ca g(η) = 0.

a) Daca f nu se anuleaza niciodata, fie h(x) :=x

2− 1

f(x). Deoarece h(0) = h(1) = −1

2 ,

exista un numar real 0 < η < 1 pentru care h′(η) = 0. Dar g = f2h′ si afirmatia rezulta.b) Daca f are cel putin un zero, fie z1 cel mai mic si z2 cel mai mare, care exista

deoarece multimea zerourilor este ınchisa. Are loc 0 < z1 ≤ z2 < 1. Functia f estepozitva pe intervalele [0, z1) si (z2, 1], ceea ce implica f ′(z1) ≤ 0 si f ′(z2) ≥ 0. Atuncig(z1) = f ′(z1) ≤ 0 si g(z2) = f ′(z2) ≥ 0, si exista numarul real η ∈ [z1, z2] pentru careg(η) = 0.

Observam ca pentru functia f(x) =2

x+ 1conditiile au loc si ff ′ + f ′′ este constant

egala cu 0.

Problema 8.28 Daca x, y > 0 aratati ca xy + yx > 1.

Solutie. Daca x sau y sunt ≥ 1 afirmatia este evidenta. Presupunem 0 < x ≤ 1, 0 <y ≤ 1. Putem nota y = kx, 0 < k < 1 si consideram functia

f(x) = xkx + (kx)x.

Se poate arata ca xx are valoarea minima a = e−1e si deoarece pentru x ∈ (0, 1] are loc

kx ≥ k functia de mai sus poate fi minorata

f(x) ≥ ak + ka, k ∈ (0, 1].

Derivata functiei h(k) = ak + ak este pozitiva, deci h este cresca toare si urmeaza cah(k) > 1, de unde rezulta afirmatia.

Problema 8.29 Fie f : (0, 1) → R o functie de doua ori derivabila cu derivata a douacontinua, astfel ca |f ′′(x) + 2xf ′(x) + (x2 + 1)f(x)| ≤ 1,∀x. Aratati ca lim

x→∞f(x) = 0.

Solutie. Fie g(x) = f ′(x)+xf(x); atunci f ′′(x)+2xf ′(x)+(x2+1)f(x) = g′(x)+xg(x).Aratam mai ıntai ca daca h este continuu diferentiabila si satisface faptul ca h′(x)+xh(x)este marginita atunci lim

x→∞h(x) = 0. Fie M marginea superioara pentru |h′(x) + xh(x)| si

fie p(x) = h(x)ex2/2. Atunci

|p′(x)| = |h′(x) + xh(x)|ex2/2 ≤Mex2/2

si

|h(x)| =∣∣∣∣ p(x)

ex2/2

∣∣∣∣ =

=

∣∣∣∣p(0) +∫ x

0 p′

ex2/2

∣∣∣∣ ≤ |p(0)|+M∫ x

0 ex2/2dx

ex2/2

Page 60: Teme si probleme pentru concursurile studentesti de matematica ...

Calcul diferential pentru functii de o variabila reala 53

Deoarece limx→∞

ex2/2 =∞ rezulta, daca folosim regula lui l’Hopital, ca lim

x→∞

∫ x0 e

t2/2dt

ex2/2= 0;

aceasta implica limx→∞

h(x) = 0.

Aplicand acest rezultat pentru h = g, apoi pentru h = f , rezulta afirmatia problemei.

Problema 8.30 Presupunem ca functiile diferentiabile a, b, f, g : R→ R satisfac

f(x) ≥ 0, f ′(x) ≥ 0, g(x) > 0, g′(x) > 0, ∀x ∈ R,

limx→∞

a(x) = A > 0, limx→∞

b(x) = B > 0, limx→∞

f(x) = limx→∞

g(x) =∞.

sif ′(x)

g′(x)+ a(x)

f(x)

g(x)= b(x).

Aratati ca limx→∞

f(x)

g(x)=

B

A+ 1.

Solutie. Fie 0 < ε < A un numar real oarecare. Pentru x suficient de mare f(x) >0, g(x) > 0, |a(x)−A| < ε, |b(x)−B| < ε

B − ε < b(x) =f ′(x)

g′(x)+ a(x)

f(x)

g(x)<f ′(x)

g′(x)+ (A+ ε)

f(x)

g(x)<

<(A+ ε)(A+ 1)

A

f ′(x)(g(x))A +Af(x)(g(x))A−1g′(x)

(A+ 1)(g(x))Ag′(x)=

=(A+ ε)(A+ 1)

A

(f(x)(g(x))A)′

((g(x))A+1)′.

Din inegalitatea de mai sus deducem

(f(x)(g(x))A)′

((g(x))A+1)′>

A(B − ε)(A+ ε)(A+ 1)

.

In mod similar se obtine, pentru x suficient de mare

(f(x)(g(x))A)′

((g(x))A+1)′<

A(B + ε)

(A− ε)(A+ 1).

Daca ε→ 0, avem

limx→∞

(f(x)(g(x))A)′

((g(x))A+1)′=

B

A+ 1.

Aplicand regula lui l’Hopital deducem

limx→∞

f(x)

g(x)= lim

x→∞

f(x)(g(x))A

(g(x))A+1=

B

A+ 1.

Problema 8.31 Presupunem ca f : R→ R este o functie de doua ori diferentiabila caresatisface f(0) = 1, f ′(0) = 0 si care pentru orice x ∈ [0,∞) are proprietatea

f ′′(x)− 5f ′(x) + 6f(x) ≥ 0.

Aratati ca pentru orice x ∈ [0,∞), are loc

f(x) ≥ 3e2x − 2e3x.

IMC, 2009

Page 61: Teme si probleme pentru concursurile studentesti de matematica ...

54

Solutie. Avem f ′′(x)− 2f ′(x)− 3(f ′(x)− 2f(x)) ≥ 0,∀x ∈ [0,∞).Fie g(x) = f ′(x)− 2f(x), x ∈ [0,∞). Rezulta ca

g′(x)− 3g(x) ≥ 0, ∀x ∈ [0,∞),

de unde urmeaza ca(g(x)e−3x)′ ≥ 0 ∀x ∈ [0,∞),

de undeg(x)e−3x ≥ g(0) = −2, ∀x ∈ [0,∞),

sau echivalentf ′(x)− 2f(x) ≥ −2e3x, ∀x ∈ [0,∞).

Analog obtinem(f(x)e−2x)′ ≥ −2ex, ∀x ∈ [0,∞),

sau echivalent(f(x)e−2x + 2ex)′ ≥ 0, ∀x ∈ [0,∞).

Rezulta caf(x)e−2x + 2ex ≥ f(0) + 2 = 3, ∀x ∈ [0,∞)

sau echivalentf(x) ≥ 3e2x − 2e3x,∀x ∈ [0,∞).

Problema 8.32 Comparati tg (sin x) si sin(tg x) pentru x ∈ (0, π/2).

Solutie. Fie f(x) = tg (sin x)− sin(tg x). Atunci

f ′(x) =cosx

cos2(sinx)− cos(tg x)

cos2 x=

cos3 x− cos(tg x) cos2(sin x)

cosx cos2(tg x).

Fie 0 < x < arctg π/2. Din concavitatea functiei cosinus pe (0, π/2), rezulta ca

3√

cos(tg x) cos2(sin x) <cos(tg x) + 2 cos(sin x)

3≤ cos

tg x + 2 sin x

3< cosx.

Ultima inegalitate rezulta din(tg x + 2 sin x

3

)′=

1

3

(1

cos2 x+ 2 cosx

)≥ 3

√1

cos2 xcosx cosx = 1.

De aici deducem cacos3 x− cos(tg x) cos2(sin x) > 0.

Rezulta ca f ′(x) > 0 si deci f creste pe intervalul [0, arctgπ/2].Urmeaza sa mai observam ca folosind si faptul ca 4 + π2 < 16 avem

tg[sin(arctg π/2)] = tgπ/2

1 + π2/4> tg π/4 = 1.

Aceasta implica faptul ca daca x ∈ [arctg π/2, π/2] atunci tg(sin x) > 1 de undeobtinem ca f(x) > 0.

Problema 8.33 Fie f : (−1, 1)→ [0,∞) o functie de trei ori derivabila, cu f(0) = 0. Sase discute derivabilitatea ın 0 a functiilor

√f(x) si 3

√f(x).

Page 62: Teme si probleme pentru concursurile studentesti de matematica ...

Calcul diferential pentru functii de o variabila reala 55

Solutie. Functia f fiind pozitiva si f(0) = 0, rezulta ca 0 este punct de minim, deci

f ′(0) = 0. Astfel: f(x) =f ′′(0)

2x2 + o(x2). Obtinem ca exista derivatele laterale ın 0

pentru√f(x):

limx→ 0x > 0

√f(x)

x= lim

x→ 0x > 0

√f ′′(0)

2+ o(1) =

√f ′′(0)

2

Analog, derivata la stanga este −√f ′′(0)

2. In concluzie, daca f ′′(0) = 0, atunci

√f(x)

este derivabila ın 0, cu derivata egala cu 0.

Pentru functia 3√f(x) se foloseste scrierea f(x) =

f ′′(0)

2x2 +

f ′′′(0)

3!x3 + o(x3) si se

cauta eventuala limita ın 0 pentru

3

√f ′′(0)

2x−1 +

f ′′′(0)

6+ o(1)

Pentru existenta derivatei, conditia f ′′(0) = 0 este necesara. Reciproc, daca f ′′(0) = 0,

atunci functia 3√f(x) este derivabila ın 0, iar valoarea derivatei este

3

√f ′′′(0)

6.

Problema 8.34 Sa se calculeze:

limx→0,x>0

xx −(x

2

)2x

xsinx −(x

2

)sin 2x.

Solutie. Folosind scrierea uv = ev lnu se obtin dezvoltarile cu rest Peano:

xx = 1 + x lnx+ o(x lnx)

(x2

)2x= 1 + 2x ln

x

2+ o(x lnx)

xsinx = 1 + x lnx+ o(x lnx)(x2

)sin 2x= 1 + 2x ln

x

2+ o(x lnx)

ceea ce arata ca limita cautata este 1.

Problema 8.35 Se considera functia f : (0,∞)→ R

f(x) =C

x− e−ax, a > 0.

Aratati ca nu exista nici un C ∈ R astfel ca

(−1)kf (k)(x) ≥ 0, ∀k ∈ N, x ∈ (0,∞).

Page 63: Teme si probleme pentru concursurile studentesti de matematica ...

56

Solutie. Derivata de ordin k a functiei este

f(x)(k) = (−1)k(Ck!

xk+1− ake−ax

).

Daca ınmultim cu (−1)k avem

(−1)kf(x)(k) =Ck!

xk+1− ake−ax

iar aceasta este pozitiva. Deducem

C ≥ ak

k!xk+1e−ax, x > 0. (8.3)

Studiem variatia functiei g(x) = xk+1e−ax. Din tabloul de variatie deducem ca functiag are un punct de maxim ın x = −k+1

a . Avem atunci

C ≥ ak

k!g(k + 1

a) =

ak

k!

(k + 1

a

)k+1

e−ak+1a =

=(k + 1)k+1

k!

e−(k+1)

a.

Pe baza formulei lui Stirling membrul din dreapta relatiei de mai sus poate fi minorat cu

(k + 1)k+1

√2πk(ke )kxkaek+1

=1

ae√

2π(1 +

1

k)kk + 1√k

xk → 1, daca k →∞

expresie care tinde la ∞, pentru k →∞. Deci inegalitatea (8.3) nu mai este posibila.

Problema 8.36 Fie f : [0,∞) −→ R o functie de doua ori derivabila pe [0,∞). Daca|f(x)| ≤ A si |f ′′(x)| ≤ B, pentru orice x ∈ [0,∞), atunci |f ′(x)| ≤ 2

√AB, pentru orice

x ∈ [0,∞).

Solutie. Pentru x ∈ (0,∞) fixat, pentru fiecare h > 0, exista θ ∈ (0, 1) astfel ıncat

f(x+ h) = f(x) + hf ′(x) +h2

2f ′′(x+ θh)

De aici:

f ′(x) =f(x+ h)− f(x)

h− h

2f ′′(x+ θh)

deci:

|f ′(x)| ≤ 2A

h+hB

2

Scriind aceasta inegalitate pentru h := 2

√A

B, se obtine rezultatul.

Problema 8.37 Fie f : (a, b) → R o functie de doua ori continuu derivabila. Aratati capentru orice x ∈ (a, b) are loc

limh→0

f(x+ h)− 2f(x) + f(x− h)

h2= f ′′(x).

Page 64: Teme si probleme pentru concursurile studentesti de matematica ...

Calcul diferential pentru functii de o variabila reala 57

Solutie. Folosim formula lui Taylor pentru h > 0. Exista z ∈ (x, x+h) si u ∈ (x−h, h)astfel ca

f(x+ h)− f(x) = f ′(x)h+f ′′(z)

2h2

f(x− h)− f(x) = −f ′(x)h+f ′′(u)

2h2.

Daca adunam cele doua relatii si ımpartim la h2 trecem la limita si obtinem

limh→0

f(x+ h)− 2f(x) + f(x− h)

h2= lim

h→0

f ′′(z) + f ′′(u)

2= f ′′(x).

Problema 8.38 Fie f : [a, b] → R de doua ori continuu derivabila pe (a, b). Aratati capentru orice x ∈ [a, b] exista ξ ∈ (a, b) astfel ca

f(x)− f(a)− f(b)− f(a)

b− a(x− a) =

1

2(x− a)(x− b)f ′′(ξ).

Solutie. Daca x = a sau x = b relatia este evidenta.Definim functia h : [a, b]→ R

h(x) =

f(x)−f(a)

(x−a) x 6= a

f ′(a) x = a.

Functia h este continuu pe [a, b], derivabila pentru x ∈ (a, b) iar

h′(x) =(x− a)f ′(x)− (f(x)− f(a))

(x− a)2, ∀x ∈ (a, b).

Deci pentru orice x ∈ (a, b) exista ξ1 ∈ (x, b) astfel ca

h(x)− h(b) = h′(ξ1)(x− b). (8.4)

Aplicam dezvoltarea functiei f ın y ∈ (a, b) cu rest Lgrange. Exista ξ2 ∈ (a, y) astfelca

f(a) = f(y) + (a− y)f ′(y) +(a− y)2

2f ′′(ξ2). (8.5)

Fie x ∈ (a, b) si ξ1 ∈ (x, b) care verifica (8.4). Fie ξ2 ∈ (a, ξ1) ales ın (8.5) pentruy = ξ1. Astfel avem

h′(ξ1) =h(x)− h(b)

x− b=

f(x)−f(a)x−a − f(b)−f(a)

b−ax− b

=

(f(x)− f(a))− f(b)−f(a)b−a (x− a)

(x− a)(x− b).

Dar, daca folosim (8.5)

h′(ξ1) =(ξ1 − a)f ′(ξ1)− (f(ξ1)− f(a))

(ξ1 − a)2=f ′′(ξ2)

2,

de unde deducem afirmatia problemei.

Page 65: Teme si probleme pentru concursurile studentesti de matematica ...

58

Problema 8.39 Fie I ⊆ IR un interval deschis cu 0 ∈ I si f : I → IR o functie continua,cu f(0) = 0, derivabila (la dreapta) ın 0. Atunci:

limn→∞

n∑k=1

f

(k

n2

)=

1

2f ′d(0)

Caz particular: Sa se calculeze

limn→∞

n∑k=1

(√1 +

k

n2− 1

)

Solutie. Din definitia derivatei la dreapta deducem ca: pentru orice ε > 0 exista nεastfel ıncat pentru orice n ≥ nε si 1 ≤ k ≤ n:

f ′d(0)− ε <f

(k

n2

)k

n2

< f ′d(0) + ε

Prin sumare rezulta:

f ′d(0)− εn2

n(n+ 1)

2<

n∑k=1

f

(k

n2

)<f ′d(0) + ε

n2

n(n+ 1)

2

deci limita exista si este egala cuf ′d(0)

2.

Observatie. Sirul dat nu este reductibil la o suma integrala.

Problema 8.40 Fie f : (0,∞)→ R o functie derivabila. Presupunem ca exista

limx→∞

(f(x) + f ′(x)

)= l ∈ R

Sa se arate ca limx→∞

f(x) = l si limx→∞

f ′(x) = 0.

Solutie. Considerand functia g(x) := f(x)ex, scriem f(x) =g(x)

exsi aplicam regula lui

l’Hopital (generalizata!). Deoareceg′(x)

ex= f(x) + f ′(x)→ l, deducem ca si f(x)→ l.

Problema 8.41 Fie f : R → R o functie cu derivata de ordin doi continua. Daca f(x +y)f(x − y) ≤ f2(x), oricare ar fi x, y ∈ R, sa se arate ca f(x)f ′′(x) ≤ f

′2(x) pentru oricex ∈ R

Solutie. Pentru x ∈ R definim g : R → R prin g(y) := f(x + y)f(x − y) − f2(x).Urmeaza ca

g′(y) = f ′(x+ y)f(x− y)− f(x+ y)f ′(x− y)

g′′(y) = f ′′(x+ y)f(x− y)− 2f ′(x+ y)f ′(x− y) + f(x+ y)f ′′(x− y)

In particular: g(0) = g′(0) = 0 si g′′(0) = 2[f(x)f ′′(x) − f ′2(x)]. Ipoteza arata acum cag(y) ≤ g(0) = 0, pentru orice y ∈ R. Deci y = 0 este un punct de maxim, ın care g′′(0) ≤ 0ın mod necesar, deci concluzia.

Page 66: Teme si probleme pentru concursurile studentesti de matematica ...

Calcul diferential pentru functii de o variabila reala 59

Problema 8.42 Fie I ⊆ R un interval. Fie f : I → R o functie oarecare. Sa se arate caf are o multime cel mult numarabila de extreme stricte.

Solutie. Pentru fiecare subinterval [a, b] ⊆ (0, 1) exista c ∈ (a, b) ın care f ısi atingemaximul absolut. Fiind si punct de minim local, urmeaza ca f este constanta pe ovecinatate a punctului c. Sa notam

α := infx ∈ [a, b] | f(x) = f(c)

Avem α < c; daca am presupune a < α prin acelasi rationament s–ar gasi puncte x < αcu f(x) = f(c). Considerand analog supx ∈ [a, b] | f(x) = f(c), rezulta f constanta pefiecare subinterval.

Problema 8.43 Fie I ⊆ R un interval. Fie f : I −→ R o functie cu derivate laterale ınfiecare punct. Sa se arate ca f are derivata pe I, cu exceptia eventuala a unei multimi celmult numarabile.

Solutie. Sa notam, pentru fiecare r ∈ Q:

Ar := x ∈ I | f ′s(x) < r < f ′d(x)

Este suficient sa aratam ca fiecare Ar este o multime cel mult numarabila. Consideramfunctia g(x) := f(x) − rx; deci g′s(x) < 0 < g′d(x) ın fiecare x ∈ Ar. Dar aceasta relatieimplica faptul ca x este punct de minim strict pentru g, de unde concluzia.

Problema 8.44 Fie f : [a, b] −→ R o functie continua pe [a, b], derivabila ın punctele asi b; f ′(a) < 0, f ′(b) > 0. Sa se arate ca exista c ∈ (a, b) punct de minim pentru f .

Solutie. f fiind functie continua pe [a, b], exista c ∈ [a, b] astfel ıncat f(c) = infx∈[a,b]

f(x).

Avem de aratat ca c 6= a si c 6= b. Ipoteza ca f ′(a) < 0 arata ca exista ε > 0 astfel ıncatf(x) < f(a), pentru orice x ∈ (a, a + ε). De aici urmeaza ca c 6= a. Analog se arata sic 6= b.

Problema 8.45 Fie f : R −→ R o functie de doua ori derivabila. Daca f(x) ≥ 0 sif ′′(x) ≤ 0, pentru orice x ∈ R, atunci f este constanta.

Solutie. f ′ este descrescatoare, deci exista A := limx→−∞

f ′(x) ∈ (−∞,∞] si

B := limx→∞

f ′(x) ∈ [−∞,∞) iar A ≥ B. Analizam pe rand cazurile:

I. A > 0; exista a ∈ R astfel ıncat f ′(x) ≥ A2 , pentru orice x ∈ (−∞, a]. Astfel

f(a)− f(x) = f ′(ξ)(a− x) ≥ A

2(a− x), de unde f(x) ≤ f(a)− Aa

2+Ax

2−→ −∞, ceea

ce contrazice f ≥ 0.

II. A ≤ 0. Daca B = 0, se obtine concluzia: daca B < 0, se repeta rationamentul:

exista b ∈ R astfel ıncat f ′(x) ≤ B

2, pentru orice x ∈ [b,∞), de unde:

f(x)− f(b) = f ′(ξ)(x− b) ≤ B

2(x− b)

deci f(x) ≤ f(b)− Bb

2+Bx

2−→ −∞, din nou contradictie.

Page 67: Teme si probleme pentru concursurile studentesti de matematica ...

60

Problema 8.46 Fie f : [0, 1] −→ R o functie de doua ori derivabila, care verifica f ′′(x) +2f ′(x) + f(x) ≥ 0, pentru orice x ∈ (0, 1) si f(0) = f(1) = 0. Atunci f(x) ≤ 0, pentruorice x ∈ [0, 1].

Solutie. Se considera functia g(x) := exf(x). Atunci g′′(x) ≥ 0 asigura concluzia.

Problema 8.47 Fie f : [a, b] −→ R o functie cu derivata de ordin doi continua pe [a, b],satisfacand f(a) = f(b) = 0. Notam M := sup

x∈[a,b]|f ′′(x)|.

(i) Sa se arate ca, oricare ar fi x ∈ [a, b], au loc inegalitatile:

(∗) |f ′(x)| ≤ M(b− a)

2

(∗∗) |f(x)| ≤ M(x− a)(b− x)

2

(ii) Daca exista x0 ∈ [a, b] (resp. (a, b)) astfel ıncat (∗) (resp. (∗∗)) sa devina egalitate

pentru x = x0, atunci f este una din functiile ±M(x− a)(b− x)

2.

Solutie. (i) Pentru fiecare x ∈ (a, b) fixat, consideram functia g : [a, b] −→ R,

g(t) := f(t)− A(t− a)(t− b)2

ın care A :=2f(x)

(x− a)(x− b). Avem g(a) = g(x) = g(b) = 0, deci exista c′, c′′ ∈ (a, b) astfel

ıncat g′(c′) = g′(c′′) = 0. Exista deci c ∈ (a, b) astfel ıncat g′′(c) = 0. Aceasta revine la

f(x) =(x− a)(x− b)

2f ′′(c), de unde rezulta (∗∗).

Prin trecere la limita, se obtine (∗), pentru x = a si x = b. Pentru a demon-stra (∗) ın restul cazurilor, consideram x ∈ (a, b] fixat si functia h : [a, b] −→ R,

h(t) := f(t)− f(x)(t− a)

x− a. Deoarece h(a) = h(x) = 0 si h′′ = f ′′, rezulta ca putem

aplica cele de mai sus functiei h, pe intervalul [a, x]. Deducem ca |h′(x)| ≤ M

2(x− a).

Insa h′(x) = f ′(x)− f(x)

x− a, de unde

|f ′(x)| ≤ |h′(x)|+ |f(x)|x− a

≤ M

2(x− a) +

M(x− a)(b− x)

2(x− a)=M(b− a)

2

(ii) Observam ca, daca se realizeaza egalitate ın (∗), pentru un x0 ∈ (a, b), atuncise realizeaza egalitate ın acelasi punct si ın (∗∗). Fie deci x ∈ (a, b), astfel ıncat (∗∗)

devine egalitate. Definim functia u : [a, b] −→ R, u(x) := f(x)− M(x− a)(x− b)2

. Deci

u(a) = u(b) = 0; u(x) ≥ 0, pentru orice x ∈ [a, b]; u′′(x) ≤ 0, pentru orice x ∈ (a, b). Inplus, x0 fiind punct de minim pentru u, deducem ca u′(x0) = 0. u′ fiind descrescatoare,rezulta ca u′(x) ≤ 0, pentru orice x ∈ [x0, b], deci u este descrescatoare pe acest interval.Dar u(x0) = 0 arata ca u ≡ 0 pe acest interval. Analog se arata ca u ≡ 0 pe [a, x0]. Insfarsit, daca (∗) devine egalitate pentru x0 = a sau b, atunci, cu aceleasi notatii ca maisus, rezulta direct u′(a) = 0 si deci concluzia.

Page 68: Teme si probleme pentru concursurile studentesti de matematica ...

Calcul diferential pentru functii de o variabila reala 61

Problema 8.48 (i) Sa se arate ca exista o unica functie f : [0,∞) −→ R astfel ıncat,pentru fiecare x ∈ [0,∞) sa avem f3(x) + xf(x) = 1. Sa se arate ca f este derivabila, iar

f ′(x) = − f(x)

x+ 3f2(x), pentru orice x ∈ [0,∞).

(ii) Sa se arate ca exista o unica functie f : R −→ R, care are proprietatile: x3 −f3(x) + xf(x) = 0, pentru orice x ∈ R si f(x) > 0, pentru orice x > 0. Sa se demonstrezeca aceasta functie este derivabila pe R \ 0.

Solutie. (i) Consideram functia g : (0, 1] −→ [0,∞), g(y) =1− y3

y. Deoarece

limy→0

g(y) =∞, g(1) = 0, g′(y) = − 1

y2− 2y, rezulta ca g este bijectie. Inversa ei f(x) = y

verifica exact relatia propusa. Conform unor rezultate cunoscute, f este continua, deriv-abila, iar derivata se calculeaza dupa formula:

f ′(x) =1

g′(f(x))=

1

− 1

f2(x)− 2f(x)

= − f(x)

x+ 3f2(x)

(ii) Fie functia g : R → R, g(y) = −y3 + xy + x3. Pentru x < 0, g este strict de-screscatoare pe R si lim

x→−∞g(y) =∞, lim

x→∞g(y) = −∞, deci exista y ∈ R unic astfel ıncat

g(y) = 0.Pentru x = 0, evident y = 0 este singura radacina a ecuatiei g(y) = 0.Pentru x > 0 ecuatia g(y) = 0 are o singura radacina y > 0 (si eventual altele negative).

Aceasta arata existenta si unicitatea functiei f , cu proprietatile din enunt.Fie x0 ∈ R. Scazand membru cu membru formulele pentru x si x0, gasim:

(x− x0)(x2 + xx0 + x20 + f(x0)) =

= (f(x)− f(x0))(f2(x)f(x)f(x0) + f2(x0)− x)

Pentru x0 < 0, aceasta relatie arata continuitatea functiei f ın x0. Pentru x0 > 0, estenecesar sa observam, pe baza sirului lui Rolle, ca f2(x) > x; deci si ın acest caz rezultacontinuitatea functiei f ın x0. Acum putem trece la limita ın egalitatea:

f(x)− f(x0)

x− x0=

x2 + xx0 + x20 + f(x0)

f2(x)f(x)f(x0) + f2(x0)− x

de unde deducem ca f este derivabila ın x0 6= 0 si:

f ′(x0) =3x2

0 + f(x0)

3f2(x0)− x0

Page 69: Teme si probleme pentru concursurile studentesti de matematica ...

Capitolul 9

Calcul integral pentru functii de ovariabila reala

Definitii si rezultate

1. Integrala ın sensul lui Riemann

Fie f : [a, b]→ R, a, b ∈ R, o functie si d = x0, x1, . . . , xn, a = x0 < x1 < x2 < . . . <xn = b o diviziune a intervalului [a, b].

Numarul real ν(d) := max1≤i≤n

(xi − xi−1) se numeste norma diviziunii d, iar numarul

real σd(f) definit prin

σd(f) :=

n∑i=1

f(ξi)(xi − xi−1), ξi ∈ [xi−1, xi]

se numeste suma Riemann asociata functiei f , diviziunii d si punctelor intermediareξ1, ξ2, . . . , ξn.

Functia f se numeste integrabila Riemann pe [a, b] daca exista un numar real I astfelca pentru orice ε > 0 exista un numar δ(ε) > 0 cu proprietatea ca pentru orice diviziuned a lui [a, b] cu ν(d) < δ(ε) si orice alegere a punctelor intermediare ξi, 1 ≤ i ≤ n, avem

|σd(f)− I| < ε.

Numarul I se noteaza

∫ b

af(x)dx si se numeste integrala lui f pe [a, b].

Numim lungimea unui interval deschis si marginit I = (a, b) numarul real

m(I) := b− a.

O multime A ⊆ R se numeste de masura Lebesgue nula sau neglijabila daca pentruorice ε > 0 exista un sir de intervale (In)n≥1 cu proprietatea ca

A ⊆∞⋃n=1

In si∞∑n=1

m(In) < ε.

O multime cel mult numarabila de numere reale este neglijabila.Teorema. (Lebesgue) O functie f : [a, b]→ R este integrabila Riemann pe [a, b] daca

si numai daca este marginita si multimea punctelor sale de discontinuitate este neglijabila.

62

Page 70: Teme si probleme pentru concursurile studentesti de matematica ...

Calcul integral pentru functii de o variabila reala 63

Din teorema lui Lebesgue rezulta imediat ca functiile f : [a, b]→ R care au o multimede discontinuitati cel mult numarabila si sunt marginite sunt integrabile. De asemeneafunctiile monotone f : [a, b]→ R sunt integrabile. Are loc de asemenea urmatorul rezultat.

Daca f : [a, b]→ [c, d] este functie integrabila Riemann si g : [c, d]→ R este o functiecontinua, atunci g f : [a, b]→ R este o functie integrabila Riemann.

Un rezultat util ın rezolvarea problemelor de calcul integral este urmatorul.Teorema. (Teorema de medie) Daca f : [a, b] → R este o functie continua si

g : [a, b]→ [0,∞) este o functie integrabila, atunci exista c ∈ [a, b] astfel ca∫ b

af(x)g(x)dx = f(c)

∫ b

ag(x)dx.

2. Integrale improprii

Fie I ⊆ R un interval. O functie f : I → R se numeste local integrabila (l.i.) pe Idaca este integrabila pe orice interval compact inclus ın I.

Fie f : [a, b)→ R, a ∈ R, b ∈ R o functie l.i. pe [a, b). Definim F : [a, b)→ R prin

F (x) =

∫ x

af(t)dt, ∀ x ∈ [a, b).

Functia f se numeste integrabila impropriu pe [a, b) daca limx→bx<b

F (x) exista si este

finita. In cest caz se spune ca

∫ b

af(x)dx este convergenta. Perechea de functii (f, F )

se numeste integrala improprie a lui f pe [a, b) si se noteaza

∫ b

af(x)dx. Notam de

asemenea limx→bx<b

F (x) =:∫ ba f(x)dx daca limita din membrul stang exista.

Analog se defineste integrala improprie pentru functii definite pe intervale de forma(a, b], a ∈ R, b ∈ R.

Mentionam cateva criterii de convergenta pentru integrale improprii.C1. Fie f, g : [a, b) → R local integrabile pe [a, b), a ∈ R, b ∈ R astfel ca g(x) > 0 pe

[a, b) si limxb

f(x)

g(x)= l, l ∈ R. Atunci:

i) Daca l 6= 0 ⇒∫ b

af(x)dx si

∫ b

ag(x)dx au aceeasi natura.

ii) Daca l = 0 si

∫ b

ag(x)dx este convergenta ⇒

∫ b

af(x)dx este absolut convergenta.

C2. Fie f : [a,∞)→ R local integrabila pe [a,∞) astfel ca

limx→∞

xαf(x) = l, l ∈ R.

i) Daca α > 1 ⇒∫ ∞a

f(x)dx este absolut convergenta.

ii) Daca α ≤ 1 si l 6= 0 ⇒∫ ∞a

f(x)dx este divergenta.

C3. Fie f : [a, b)→ R, a, b ∈ R, local integrabila pe [a, b) astfel ca

limxb

(b− x)αf(x) = l, l ∈ R.

Page 71: Teme si probleme pentru concursurile studentesti de matematica ...

64

i) Daca α < 1 ⇒∫ b

af(x)dx este absolut convergenta.

ii) Daca α ≥ 1 si l 6= 0 ⇒∫ b

af(x)dx este divergenta.

C4. Fie f : [0,+∞)→ R+ o functie descrescatoare. Atunci

∫ ∞0

f(x)dx si

∞∑n=0

f(x) au

aceeasi natura.C5. (Abel-Dirichlet) Fie f, g : [a, b) → R, a ∈ R, b ∈ R, f continua si g monotona

pe [a, b). Daca:i) limxb

g(x) = 0

ii) Functia F (x) =

∫ x

af(t)dt, x ∈ [a, b) este marginita, atunci

∫ b

af(x)g(x)dx

este convergenta.Fie f : (a, b)→ R, a, b ∈ R local integrabila pe (a, b). Definim

F (x, y) =

∫ y

xf(t)dt

pentru orice x, y ∈ (a, b). Functia f se numeste integrabila impropriu pe (a, b) daca

limx→ay→b

F (x, y) exista si este finita. Se arata ca

∫ b

af(x)dx este convergenta daca si numai

daca exista c ∈ R, a < c < b, astfel ca integralele

∫ c

af(x)dx si

∫ b

cf(x)dx sa fie conver-

gente. In acest caz avem ∫ b

af(x)dx =

∫ c

af(x)dx+

∫ b

cf(x)dx.

3. Integrale cu parametru

Fie I un interval cu capetele a, b ∈ R si f : I × [c, d] → R, c, d ∈ R, o functie cuproprietatea ca este integrabila pe I pentru orice y ∈ [c, d].

Functia F : [c, d]→ R

F (y) =

∫ b

af(x, y)dx

se numeste integrala cu parametru. Daca I = [a, b], a, b ∈ R, avem o integrala propriecu parametru, ın caz contrar ea numindu-se integrala improprie cu parametru.

Integralele proprii cu parametru au proprietatile urmatoare.1) Daca f este continua pe [a, b]× [c, d] ⇒ F continua pe [c, d].

2) Daca f este continua pe [a, b] × [c, d] si∂f

∂yexista si este continua pe [a, b] × [c, d],

atunci F este derivabila pe [c, d] si are loc relatia

F ′(y) =

∫ b

a

∂f

∂y(x, y)dx.

Page 72: Teme si probleme pentru concursurile studentesti de matematica ...

Calcul integral pentru functii de o variabila reala 65

3) Daca f este continua pe [a, b]× [c, d], atunci∫ b

a

(∫ d

cf(x, y)dy

)dx =

∫ d

c

(∫ b

af(x, y)dx

)dy.

Fie α, β : [c, d]→ [a, b] doua functii de clasa C1, f : [a, b]× [c, d]→ R o functie continua

cu proprietatea ca∂f

∂yeste continua pe [a, b]×[c, d]. Atunci integrala cu parametru cu limite

variabile I : [c, d]→ R, definita prin

I(y) =

∫ β(y)

α(y)f(x, y)dx

este derivabila pe [c, d] si are loc formula lui Leibniz:

I ′(y) =

∫ β(y)

α(y)

∂f

∂y(x, y)dx+ f(β(y), y)β′(y)− f(α(y), y)α′(y), ∀ y ∈ [c, d].

4. Functiile Beta si Gamma ale lui Euler

Functiile B : (0,∞)× (0,∞)→ R, Γ : (0,∞)→ R se definesc prin

B(a, b) =

∫ 1

0xa−1(1− x)b−1dx, a, b ∈ (0,∞)

Γ(a) =

∫ ∞0

xa−1e−xdx, a ∈ (0,∞)

Au loc relatiile:1) B(a, b) = B(a, b), a, b > 0;

2) B(a, b) =b− 1

a+ b− 1B(a, b− 1), a > 0, b > 1;

B(a, b) =

∫ ∞0

xa−1

(1 + x)a+bdx, a, b > 0;

3) Γ(a+ 1) = aΓ(a), a ∈ (0,∞);4) Γ(n+ 1) = n!, n ∈ N;

5) B(a, b) =Γ(a)Γ(b)

Γ(a+ b), a, b > 0;

6) Γ

(1

2

)=√π;

7) Γ(a)Γ(1− a) =π

sin(aπ), a ∈ (0, 1).

8) Γ(a) = limn→∞

na · n!

a(a+ 1) . . . (a+ n), a > 0.

Probleme

Problema 9.1 Fie In =

∫ π2

0sinn xdx, n ∈ N.

a) Sa se arate ca In =n− 1

nIn−2, n ≥ 2, si sa se calculeze In.

b) Sa se demonstreze formula lui Wallis

π = limn→∞

1

n

(2 · 4 . . . 2n

1 · 3 . . . (2n− 1)

)2

.

Page 73: Teme si probleme pentru concursurile studentesti de matematica ...

66

Solutie. a) In =

∫ π2

0sinn−1 x(− cosx)′dx

= − sinn−1 x cosx∣∣∣π20

+ (n− 1)

∫ π2

0sinn−2 x cos2 xdx

= (n− 1)

∫ π2

0sinn−2 x(1− sin2 x)dx = (n− 1)In−2 − (n− 1)In

de unde rezulta relatia din enunt. Din relatia de recurenta se obtine

In =

1 · 3 . . . (2k − 1)

2 · 4 . . . 2k· π

2, n = 2k

2 · 4 . . . (2k − 2)

1 · 3 . . . (2k − 1), n = 2k − 1.

b) Are loc relatia evidenta

sin2n+1 x ≤ sin2n x ≤ sin2n−1 x, x ∈[0,π

2

], n ∈ N∗,

care integrata pe[0,π

2

]conduce la inegalitatea

I2n+1 ≤ I2n ≤ I2n−1

de unde ımpartind cu I2n+1 obtinem

1 ≤(

1 · 3 . . . (2n− 1)

2 · 4 . . . 2n

)2

· 2n+ 1

2· π ≤ 2n+ 1

2n

sau

π ≤(

2 · 4 . . . 2n1 · 3 . . . (2n− 1)

)2

· 1

n≤ π2n+ 1

2n.

Trecand la limita ın relatia de mai sus obtinem formula lui Wallis.

Problema 9.2 (Formula lui Stirling) Sa se arate ca pentru orice n ∈ N∗, exista θn ∈(0, 1) astfel ca

n! =√

2πn(ne

)neθn12n .

Solutie. Consideram sirul (an)n≥1, an =n!en

nn+ 12

, n ≥ 1. Demonstram ca sirul (an)n≥1

este descrescator. Avem

anan+1

=1

e

(1 +

1

n

)n+ 12

, n ≥ 1.

Din dezvoltarile ın serie de puteri

ln(1 + x) = x− x2

2+x3

3− . . .

ln(1− x) = −x− x2

2− x3

3− . . .

, |x| < 1

Page 74: Teme si probleme pentru concursurile studentesti de matematica ...

Calcul integral pentru functii de o variabila reala 67

obtinem

ln1 + x

1− x= 2x

(1 +

1

3x2 +

1

5x4 + . . .

), |x| < 1.

Punand x =1

2n+ 1, n ∈ N∗, ın relatia de mai sus avem(

n+1

2

)ln

(1 +

1

n

)= 1 +

1

3· 1

(2n+ 1)2+

1

5· 1

(2n+ 1)4+ . . .

de unde deducem

1 <

(n+

1

2

)ln

(1 +

1

n

)< 1 +

1

3

(1

(2n+ 1)2+

1

(2n+ 1)4+ . . .

)⇔

1 <

(n+

1

2

)ln

(n+

1

n

)< 1 +

1

12n(n+ 1)⇔

e <

(1 +

1

n

)n+ 12

< e1+ 1

12n(n+1) ⇔

1 <anan+1

< e1

12n(n+1) (1)

Rezulta ca (an)n≥1 este descrescator si fiind marginit inferior este convergent. Fielimn→∞

an = a. Din relatia (1) obtinem

ane− 1

12n < an+1e− 1

12(n+1) , n ≥ 1,

deci sirul (ane− 1

12n )n≥1 este crescator si are limita a.Prin urmare are loc relatia

a < an < a1

12n , ∀ n ≥ 1

deci exista θn ∈ (0, 1) astfel ca

an = aeθn12n , n ≥ 1. (2)

Demonstram ın continuare ca a =√

2π.Din formula lui Wallis rezulta ca sirul (bn)n≥1

bn =1√n· 2 · 4 . . . 2n

1 · 3 . . . (2n− 1)=

1√n· 22n(n!)2

(2n)!(3)

este convergent si are limita√π.

Din relatia (2) obtinem

n! = a(ne

)n√n · e

θn12n , θn ∈ (0, 1) (4)

(2n)! = a

(2n

e

)2n√2n · e

θ2n24n , θ2n ∈ (0, 1)

care ınlocuite ın (3) conduc la

bn =a√2eθn6n− θ2n

24n .

Rezulta ca a = limn→∞

√2bn =

√2π. Inlocuind ın (4) obtinem

n! =√

2nπ(ne

)n θn12n

.

Page 75: Teme si probleme pentru concursurile studentesti de matematica ...

68

Problema 9.3 (Formula lui Taylor) Fie f : [a, b]→ R o functie de (n+1) ori derivabilape [a, b] cu f (n+1) integrabila pe [a, b]. Atunci pentru orice x ∈ [a, b] avem:

f(x) = f(a) +f ′(a)

1!(x− a) +

f ′′(a)

2!(x− a)2 + · · ·+ f (n)(a)

n!(x− a)n +Rn(x)

unde

Rn(x) =

∫ x

af (n+1)(t)

(x− t)n

n!dt.

Solutie. f(x)− f(a) =

∫ x

af ′(t)dt =

∫ x

a(t− x)′f ′(t)dt

= (t− x)f ′(t)∣∣∣xa−∫ x

af ′′(t)(t− x)dt

= f ′(a)(x− a)−∫ x

af ′′(t)

((t− x)2

2

)′dt

= f ′(a)(x− a)− f ′′(t)(t− x)2

2

∣∣∣xa

+

∫ x

af ′′′(t)

(t− x)2

2dt

= f ′(a)(x− a) +f ′′(a)

2!(x− a)2 +

∫ x

af ′′′(t)

(t− x)2

2dt

Continuand sa integram prin parti obtinem

f(x) = f(a) +f ′(a)

1!(x− a) + · · ·+ f (n)(a)

n!(x− a)n +

∫ x

af (n+1)(t)

(x− t)n

n!dt.

Problema 9.4 (A doua teorema de medie) Fie f : [a, b] → R o functie de clasa C1,pozitiva, descrescatoare, si g : [a, b]→ R o functie continua. Atunci exista c ∈ [a, b] astfelca ∫ b

af(x)g(x)dx = f(a)

∫ c

ag(x)dx.

Solutie. Fie G(x) =

∫ x

ag(t)dt, x ∈ [a, b]. Avem conform formulei de integrare prin

parti ∫ b

af(x)g(x)dx = f(b)G(b)−

∫ b

af ′(x)G(x)dx

Functia G fiind continua pe [a, b] este marginita si ısi atinge marginile. Fie m =mina≤x≤b

G(x), M = maxa≤x≤b

G(x). Cum f este descrescatoare rezulta ca f ′ este negativa pe

[a, b] si avem ∫ b

af(x)g(x)dx ≤ f(b)G(b) +M

∫ b

a(−f ′(x))dx

= f(b)G(b) +M(f(a)− f(b))

≤Mf(a)

Analog se demonstreaza ca ∫ b

af(x)g(x)dx ≥ mf(a)

Page 76: Teme si probleme pentru concursurile studentesti de matematica ...

Calcul integral pentru functii de o variabila reala 69

Intrucat G este continua pe [a, b], are proprietatea lui Darboux, prin urmare existac ∈ [a, b] astfel ca ∫ b

af(x)g(x)dx = f(a)

∫ c

ag(x)dx.

Problema 9.5 (Formula lui Bonnet-Weierstrass) Fie f : [a, b]→ R o functie de clasaC1, descrescatoare, si g : [a, b]→ R o functie continua. Atunci exista c ∈ [a, b] astfel ca∫ b

af(x)g(x)dx = f(a)

∫ c

ag(x)dx+ f(b)

∫ b

cg(x)dx.

Solutie. Fie h(x) = f(x) − f(b), x ∈ [a, b]. Functia h este monoton descrescatoare sih(x) ≥ 0, ∀ x ∈ [a, b]. Atunci, pe baza problemei 9.4 rezulta ca exista c ∈ [a, b] astfel ca∫ b

ag(x)h(x)dx = h(a)

∫ c

ag(x)dx

sau ∫ b

ag(x)(f(x)− f(b))dx = (f(a)− f(b))

∫ c

ag(x)dx.

Prin urmare ∫ b

af(x)g(x)dx = f(b)

∫ b

ag(x)dx+ (f(a)− f(b))

∫ c

ag(x)dx

= f(a)

∫ c

ag(x)dx+ f(b)

∫ b

cg(x)dx.

Problema 9.6 (Lema lui Gronwall) Fie f, g : [a, b] → R functii continue astfel cag(x) ≥ 0 pentru orice x ∈ [a, b]. Daca y : [a, b]→ R este o functie continua cu proprietatea

y(x) ≤ f(x) +

∫ x

ag(t)y(t)dt, ∀ x ∈ [a, b]

atunci

y(x) ≤ f(x) +

∫ x

af(t)g(t)e

∫ xa g(s)dsdt.

Solutie. Fie u(x) =

∫ x

ag(t)y(t)dt, x ∈ [a, b]. Inmultind relatia din ipoteza cu g(x) se

obtineu′(x)− g(x)u(x) ≤ f(x)g(x), x ∈ [a, b]

Inmultind ultima relatie cu −∫ xa g(t)dt obtinem(

u(x)e−∫ xa g(t)dt

)′≤ f(x)g(x)e−

∫ xa g(t)dt

care integrata pe intervalul [a, x] devine

u(x)e−∫ xa g(t)dt ≤

∫ x

af(t)g(t)e−

∫ ta g(s)dsdt

de unde obtinem

u(x) ≤∫ x

af(t)g(t)e

∫ ta g(s)dsdt, x ∈ [a, b]

Rezulta

y(x) ≤ f(x) + u(x) ≤ f(x) +

∫ x

af(t)g(t)e

∫ tx g(s)dsdt.

Page 77: Teme si probleme pentru concursurile studentesti de matematica ...

70

Problema 9.7 (Integrala Poisson) Utilizand relatia

X2n − 1 = (X2 − 1)n−1∏k=1

(X2 − 2X cos

n+ 1

)

sa se demonstreze ca valoarea integralei

I(a) =

∫ π

0ln(a2 − 2a cosx+ 1)dx, a ∈ R \ −1, 1

este 0 daca |a| < 1 si 2π ln |a| daca |a| > 1.

Solutie. Fie f(x) = ln(x2 − 2a cosx+ 1), x ∈ [0, π] si fie

In(a) =π

n

n−1∑k=0

f

(kπ

n

).

Avem evident limn→∞

In(a) = I(a).

Pe de alta parte avem

In(a) =π

n

n−1∑k=0

ln

(a2 − 2a cos

n+ 1

)=π

nln(a− 1)2 +

π

nlna2n − 1

a2 − 1

Daca |a| < 1 avem lim a2n = 0, deci I(a) = 0. Daca |a| > 1 avem

In(a) =π

nln(a− 1)2 +

π

nln

1− a−2n

1− a−2+

2n− 2

nπ ln |a|

de unde obtinem limn→∞

In(a) = 2π ln |a|.

Problema 9.8 (Formula lui Gauss) Fie a > b > 0 si

G(a, b) =

∫ π/2

0

dx√a2 cos2 x+ b2 sin2 x

Definim sirurile (an)n≥0, (bn)n≥0 prin relatiile de recurenta

an =an−1 + bn−1

2, bn =

√an−1bn−1, a0 = a, b0 = b.

a) Sa se demonstreze ca sirurile (an)n≥0, (bn)n≥0 sunt convergente si limn→∞

an =

limn→∞

bn =: µ(a, b).

b) G(a, b) =

∫ π/2

0

dx√a2n cos2 x+ b2n sin2 x

, ∀ n ≥ 0.

c) G(a, b) =π

2µ(a, b).

Page 78: Teme si probleme pentru concursurile studentesti de matematica ...

Calcul integral pentru functii de o variabila reala 71

Solutie. a) Avem a1 =a0 + b0

2, b1 =

√a0b0 de unde obtinem

b0 < b1 < a1 < a0

Prin inductie se demonstreaza ca

b0 < b1 < · · · < bn < an < an−1 < · · · < a0

de unde rezulta ca (an)n≥0, (bn)n≥0 sunt monotone si marginite.

Fie limn→∞

an = l1, limn→∞

bn = l2, l1, l2 ∈ R. Din relatia an =an−1 + bn−1

2, prin trecere la

limita, obtinem l1 = l2 =: µ(a, b).b) Facem schimbarea de variabila

sinx =2a sin t

a+ b+ (a− b) sin2 t, t ∈

[0,π

2

](1)

obtinem

cosxdx = 2aa+ b− (a− b) sin2 t

[a+ b+ (a− b) sin2 t]2cos tdt.

Din relatia (1) se obtine

cosx =

√(a+ b)2 − (a− b)2 sin2 t

a+ b+ (a− b) sin2 tcos t

de unde rezulta

dx = 2a(a+ b)− (a− b) sin2 t

a+ b+ (a− b) sin2 t· dt√

(a+ b)2 − (a− b)2 sin2 t

Avem de asemenea√a2 cos2 x+ b2 sin2 x = a

a+ b− (a− b) sin2 t

a+ b+ (a− b) sin2 t

si ın continuare

dx√a2 cos2 x+ b2 sin2 x

==dt√(

a+ b

2

)2

cos2 t+ ab sin2 t

Tinand seama ca a1 =a+ b

2si b1 =

√ab rezulta

G(a, b) =

∫ π/2

0

dt√a2

1 cos2 t+ b21 sin2 t

Aplicand ın mod repetat rationamentul anterior, obtinem:

G(a, b) =

∫ π/2

0

dt√a2n cos2 t+ b2n sin2 t

, ∀ n ≥ 0.

c) Au loc relatiileπ

2an≤ G(a, b) ≤ π

2bn, n ≥ 1

de unde facand n→∞ obtinem

G(a, b) =π

2µ(a, b).

Page 79: Teme si probleme pentru concursurile studentesti de matematica ...

72

Problema 9.9 Fie f ∈ C1[a, b]. Fie sirul (un)n≥1 definit prin

un =

∫ b

af(x)dx− b− a

n

n∑k=1

f

(a+ k

b− an

)

Demonstrati ca limn→∞

nun =a− b

2(f(b)− f(a)).

Solutie. Fie xn = a+ kb− an

, 0 ≤ k ≤ n. Avem

un =

k−1∑k=0

∫ xk+1

xk

f(x)dx−n∑k=1

(xk+1 − xk)f(xk)

Fie F o primitiva a functiei f . Atunci

un =n−1∑k=0

(F (xk+1)− F (xk))−n∑k=1

(xk+1 − xk)f(xk)

= F (x1)− F (x0) +n−1∑k=1

(F (xk+1)− F (xk)− (xk+1 − xk)F ′(xk))−b− an

f(b).

Aplicand formula lui Taylor functiei F pe intervalul [xk, xk+1] rezulta ca exista ξk ∈(xk, xk+1) astfel ca

un = F (x1)− F (x0) +

n∑k=1

(xk+1 − xk)2

2f ′(ξk)−

b− an

f(b)

Tinand seama ca n =b− a

xk+1 − xkavem

nun = (b− a)F (x1)− F (x0)

x1 − x0+b− a

2

n−1∑k=1

(xk+1 − xk)f ′(ξk)− (b− a)f(b).

De asemenea

limn→∞

F (x1)− F (x0)

x1 − x0= F ′(x0) = f(x0) = f(a)

si

limn→∞

n−1∑k=1

(xk+1 − xk)f ′(ξk) =

∫ b

af ′(t)dt = f(b)− f(a).

Prin urmare

limn→∞

nun = (b− a)f(a) +b− a

2(f(b)− f(a))− (b− a)f(b)

=a− b

2(f(b)− f(a)).

Problema 9.10 Sa se determine a, b ∈ R astfel ca pentru orice n ∈ N∗ sa aiba locegalitatea ∫ π

0(ax+ bx2) cosnxdx =

1

n2.

Sa se deduca de aici ca

∞∑n=1

1

n2=π2

6.

Page 80: Teme si probleme pentru concursurile studentesti de matematica ...

Calcul integral pentru functii de o variabila reala 73

Solutie. Integrand prin parti se gaseste a = −1, b =1

2π. Rezulta ca

sn =n∑k=1

1

k2=

∫ π

0

(1

2πx2 − x

) n∑k=1

cos kxdx.

Avem

n∑k=1

cos kx =sin

(2n+ 1)x

2− sin

x

2

2 sinx

2

=sin(nx+

x

2

)− sin

x

2

2 sinx

2

=1

2

(sinnxctg

x

2+ cosnx− 1

)Fie functiile f, g : [0, π]→ R,

f(x) =1

2πx2 − x

g(x) =

f(x)ctg

x

2, x ∈ (0, π]

−2, x = 0

Se arata usor ca g este de clasa C1[0, π]. Se obtine

sn =1

2

(∫ π

0g(x) sinnxdx+

∫ π

0f(x) cosnxdx−

∫ π

0

(1

2πx2 − x

)dx

)(1)

Pentru functia h ∈ C1[a, b] avem∫ b

ah(x) sinnxdx = lim

n→∞

∫ b

ah(x) cosnxdx = 0.

Intr-adevar, integrand prin parti obtinem∫ b

ah(x) cosnxdx =

h(b) sinnb− h(a) sinna

n− 1

n

∫ b

ah′(x) sinnxdx

Cum h si h′ sunt marginite pe [a, b] rezulta ca

limn→∞

∫ b

ah(x) cosnxdx = 0

Trecand acum la limita ın (1) obtinem limn→∞

sn =π2

6.

Problema 9.11 Fie f : [0, 1]→ R o functie continua cu proprietatea∫ 1

0f(x)dx =

∫ 1

0xf(x)dx = 1.

Sa se arate ca

∫ 1

0f2(x)dx ≥ 4.

Page 81: Teme si probleme pentru concursurile studentesti de matematica ...

74

Solutie. Se cauta o functie de forma f(x) = ax+ b, a, b ∈ R, care verifica relatiile dinenunt. Prin identificare se obtine f(x) = 6x− 2. Avem∫ 1

0(f(x)− (6x− 2))2dx =

∫ 1

0f2(x)dx− 4 ≥ 0.

Egalitatea se obtine pentru f(x) = 6x− 2.

Problema 9.12 Fie In =

∫ π2

0cosn x sinnx, Jn =

∫ π2

0cosn x cosnxdx, n ∈ N.

a) Sa se arate ca:

2In =1

n+ In−1,

In =1

2In−1,

n ≥ 1.

b) Sa se deduca de aici ca

1

2n+1

(2

1+

22

2+ · · ·+ 2n

n

)

In =π

2n+1.

Solutie. a) Integrand prin parti avem

In = − 1

n

∫ π2

0cosn x(cosnx)′dx

= − 1

ncosn x cosnx

∣∣∣π20− 1

n

∫ π2

0n cosn−1 x sinx cosnxdx

=1

n−∫ π

2

0cosn−1 x sinx cosnxdx.

Adunand aceasta relatie cu cea initiala obtinem

2In =1

n+

∫ π/2

0cosn x sinnxdx−

∫ π/2

0cosn−1 x sinx cosnxdx

=1

n+

∫ π/2

0cosn−1 x(sinnx cosx− cosnx sinx)dx

=1

n+

∫ π/2

0cosn−1 x sin(n− 1)xdx =

1

n+ In−1.

Analog se demonstreaza cealalta recurenta.

b) Relatia 2In =1

n+ In−1 se scrie sub forma echivalenta

2nIn = 2n−1In−1 +2n−1

n, n ≥ 1.

Insumand relatiile anterioare de la 1 la n se obtine relatia ceruta. Din In =1

2In−1

rezulta ca (In)n≥1 este o progresie geometrica cu ratia1

2, deci

In =1

2nI0 =

π

2n+1.

Page 82: Teme si probleme pentru concursurile studentesti de matematica ...

Calcul integral pentru functii de o variabila reala 75

Problema 9.13 Sa se calculeze

∫ 1

0

arctg x

1 + xdx.

Solutie. Avem

I =

∫ 1

0

arctg x

1 + xdx =

∫ 1

0arctg x(ln(1 + x))′dx

= arctg x ln(1 + x)∣∣∣10−∫ 1

0

ln(1 + x)

1 + x2dx =

π

4ln 2−

∫ 1

0

ln(1 + x)

1 + x2dx.

In ultima integrala facem substitutia x = tg t obtinand

J =

∫ 1

0

ln(1 + x)

1 + x2dx =

∫ π/4

0ln(1 + tg t)dt

ın care se face substitutiaπ

4− t = u. Se obtine J =

π

8ln 2 = I.

Problema 9.14 Fie f : R→ R o functie continua si periodica de perioada T > 0.Sa se demonstreze relatiile:

a)

∫ a+T

af(x)dx =

∫ T

0f(x)dx, ∀ a ∈ R;

b)

∫ a+nT

af(x)dx = n

∫ T

0f(x)dx, ∀ a ∈ R, ∀ a ∈ N.

Aplicatie. Sa se calculeze

∫ 2003π

0arcsin(sinx)dx.

Solutie. a) Fie g : R → R, g(a) =

∫ a+T

af(x)dx. Avem g′(a) = 0 de unde rezulta

g(a) = g(0).

b)

∫ a+nT

af(x)dx =

∫ a+T

af(x)dx+

∫ a+2T

a+Tf(x)dx+ . . .

+

∫ a+nT

a+(n−1)Tf(x)dx = n

∫ a+T

af(x)dx = n

∫ T

0f(x)dx.

Aplicatie.

∫ 2003π

0arcsin(sinx)dx

=

∫ π

0arcsin(sinx)dx+

∫ 2003π

πarcsin(sinx)dx

=

∫ π

0arcsin(sinx)dx+

∫ π+1001·2π

πarcsin(sinx)dx

=

∫ π

0arcsin(sinx)dx+ 1001

∫ 2π

0arcsin(sinx)dx

=

∫ π

0arcsin(sinx)dx+ 1001

∫ π

−πarcsin(sinx)dx

=

∫ π

0arcsin(sinx)dx =

∫ π/2

0xdx+

∫ π

π/2(π − x)dx =

π2

4.

Page 83: Teme si probleme pentru concursurile studentesti de matematica ...

76

Problema 9.15 Fie f : R→ R o functie continua si periodica de perioada T > 0.Sa se arate ca

a) limx→∞

1

x

∫ x

0f(t)dt =

1

T

∫ T

0f(t)dt.

b) limt→∞

∫ b

af(xt)dx =

b− aT

∫ T

0f(t)dt.

Solutie. a) Fie x = knT + an, kn ∈ N, an ∈ [0, T ), x > 0. Avem

1

x

∫ x

0f(t)dt =

1

knT + an

∫ knT+an

0f(t)dt

=1

knT + an

(∫ an

0f(t)dt+

∫ knT+an

an

f(t)dt

)

=1

knT + an

(∫ an

0f(t)dt+ kn

∫ T

0f(t)dt

)

=1

knT + an

∫ an

0f(t)dt+

knknT + an

∫ T

0f(t)dt.

Facand n→∞ se obtine relatia ceruta.

b) In

∫ b

af(xt)dx se face substitutia xt = u.

Problema 9.16 Fie f ∈ C3[−1, 1] cu proprietatea f(−1) = f(0) = f(1) = 0.Sa se arate ca ∫ 1

−1|f(x)|dx ≤ 1

12max−1≤x≤1

|f ′′′(x)|.

Lema. Fie f ∈ C2[a, b] astfel ıncat f(a) = f(b) = 0. Demonstram ca pentru oricex ∈ [a, b], exista cx ∈ (a, b) astfel ıncat

f(x) =(x− a)(x− b)

2f ′′(cx).

Generalizare. Fie f ∈ Cp[a, b], p ≥ 2 astfel ıncat exista α1 = a < α2 < . . . < αp = bcu f(αj) = 0, j = 1, p. Atunci pentru orice x ∈ [a, b], exista cx ∈ (a, b) astfel ıncat

f(x) =

p∏j=1

(x− αj)f (p)(cx)

p!.

Demonstratie. fie x ∈ (a, b) fixat si A astfel ıncat ϕ : [a, b]→ R,

ϕ(t) = f(t)− (t− a)(t− b)2

A

sa se anuleze ın x.Avem ϕ(a) = ϕ(x) = ϕ(b) = 0. Din teorema lui Rolle rezulta ca exista x1, x2,

a < x1 < x < x2 < b astfel ıncat ϕ′(x1) = ϕ′(x2) = 0 ⇒ ∃ cx ∈ (x1, x2) astfel ıncatϕ′′(cx) = 0 ⇒ A = f ′′(cx). Daca x = a sau x = b atunci cx este arbitrar.

Page 84: Teme si probleme pentru concursurile studentesti de matematica ...

Calcul integral pentru functii de o variabila reala 77

Solutie. Folosind generalizarea lemei anterioare se arata ca pentru orice x ∈ [a, b]exista cx ıntre a si b astfel ca

f(x) =1

6x(x− 1)(x+ 1)f ′′′(cx).

∫ 1

−1|f(x)|dx ≤ 1

6

∫ 1

−1|x(1− x2)|dx · sup

|x|≤1|f ′′′(x)|

=1

3

∫ 1

0x(1− x3)dx · sup

|x|≤1|f ′′′(x)| = 1

12sup|x|≤1

|f ′′′(x)|.

Problema 9.17 Fie f ∈ C1[0, 1] astfel ıncat

∫ 1

0f(x)dx =

∫ 1

0xf(x)dx = 1.

Sa se arate ca ∫ 1

0(f ′(x))2dx ≥ 30.

Solutie. Integrand prin parti obtinem:

1 =

∫ 1

0f(x)dx = xf(x)

∣∣∣10−∫ 1

0xf ′(x)dx = f(1)−

∫ 1

0xf ′(x)dx

1=

∫ 1

0

(x2

2

)′f(x)dx =

x2

2f(x)

∣∣∣10− 1

2

∫ 1

0x2f ′(x)dx=

1

2f(1)− 1

2

∫ 1

0x2f ′(x)dx.

Eliminand f(1) din relatiile anterioare rezulta

1 =

∫ 1

0(x− x2)f ′(x)dx.

Inegalitatea Cauchy-Schwartz conduce la

1 =

(∫ 1

0(x− x2)f ′(x)dx

)2

≤∫ 1

0(x− x2)2dx

∫ 1

0(f ′(x))2dx.

Cum

∫ 1

0(x− x2)2dx =

1

30rezulta

∫ 1

0(f ′(x))2dx ≥ 30.

Egalitatea are loc pentru f ′(x) = λ(x− x2), de unde obtinem

f(x) = λ

(x2

2− x3

3

)+ µ, λ, µ ∈ R.

Inlocuind ın relatiile din enunt obtinem λ = 30, µ = −3

2.

Problema 9.18 Fie f : [0, 1]→ R o functie avand derivata de ordinul doi integrabila. Sase demonstreze ca

limn→∞

n2

(∫ 1

0f(x)dx− 1

n

n∑k=1

f

(2k − 1

2n

))=f ′(1)− f ′(0)

24.

Page 85: Teme si probleme pentru concursurile studentesti de matematica ...

78

Solutie. Notam

rn =

∫ 1

0f(x)dx− 1

n

n∑k=1

f

(2k − 1

2n

).

Avem

rn =

n∑k=1

∫ k/n

(k−1)/n

(f(x)− f

(2k − 1

2n

))dx.

Fiemk = inf

x∈[(k−1)/n,k/n]f ′′(x), Mk = sup

x∈[(k−1)/n,k/n]f ′′(x),

k ∈ 0, 1, . . . , n− 1. Conform formulei lui Taylor, avem:

f(x)− f(

2k − 1

2n

)=

(x− 2k − 1

2n

)f ′(

2k − 1

2n

)+

1

2

(x− 2k − 1

2n

)2

f ′′(ξk),

x, ξk ∈ [(k − 1)/n, k/n]. Folosind∫ k/n

(k−1)/n

(x− 2k − 1

2n

)dx = 0,

∫ k/n

(k−1)/n

(x− 2k − 1

2n

)2

dx =1

12n3

obtinem

1

24n

1

n

n∑k=1

mk ≤ nrn ≤1

24n

1

n

n∑k=1

Mk,

de unde

limn→∞

n2rn =1

24

∫ 1

0f ′′(x)dx =

f ′(1)− f ′(0)

24.

Problema 9.19 Fie g : R→ R de perioada T > 0 si f : [0, T ]→ R functii integrabile. Sase demonstreze ca

limn→∞

∫ T

0f(x)g(nx)dx =

1

T

∫ T

0f(x)dx

∫ T

0g(x)dx.

Solutie. Pentru ınceput presupunem g ≥ 0. Notam:

mk = infx∈[k Tn ,(k+1)T

n ]f(x), Mk = sup

x∈[k Tn ,(k+1)Tn ]f(x),

k ∈ 0, 1, . . . , n− 1. Avem:∫ T

0f(x)g(nx)dx =

1

n

∫ nT

0f

(t

n

)g(t)dt

=1

n

n−1∑k=0

∫ (k+1)T

kTf

(t

n

)g(t)dt =

1

n

n−1∑k=0

fk

∫ (k+1)T

kTg(t)dt

=1

n

n−1∑k=0

fk

∫ T

0g(t)dt =

1

T

∫ T

0g(t)dt · T

n

n−1∑k=0

fk

Page 86: Teme si probleme pentru concursurile studentesti de matematica ...

Calcul integral pentru functii de o variabila reala 79

→ 1

T

∫ T

0g(t)dt

∫ T

0f(t)dt,

deoarece fk ∈ [mk,Mk], k ∈ 0, 1, . . . , n− 1.In cazul ın care functia g nu este pozitiva, fiind integrabila, exista o constanta M > 0

astfel ıncat g +M > 0.Conform celor deja demonstrate, avem:

limn→∞

∫ T

0f(t)(g(nt) +M)dx =

1

T

∫ T

0(g(t) +M)dt

∫ T

0f(t)dt,

de unde

limn→∞

∫ T

0f(t)g(nt)dx =

1

T

∫ T

0g(t)dt

∫ T

0f(t)dt.

Problema 9.20 Fie f : [0, 1] → R o functie cu derivata continua si pozitiva. Fie cn ∈(0, 1

n

), n ∈ N∗, astfel ca ∫ 1/n

0f(x)dx =

1

nf(cn).

Sa se arate ca

limn→∞

ncn =1

2.

Solutie.

limn→∞

ncn = limn→∞

ncn − 0

f(cn)− f(0)(f(cn)− f(0))

=1

f ′(0)limn→∞

n2

∫ 1/n

0(f(x)− f(0))dx =

1

f ′(0)limt→0

1

t2

∫ t

0(f(x)− f(0))dx

=1

f ′(0)limt→0

f(t)− f(0)

2t=

f ′(0)

2f ′(0)=

1

2.

Problema 9.21 Daca f : [0, 1]→ [0, 1] este o functie continua atunci(∫ 1

0f(x)dx

)2

≤ 2

∫ 1

0xf(x)dx.

Solutie. Consideram functia

F (t) =

(∫ t

0f(x)dx

)2

− 2

∫ t

0xf(x)dx.

Avem

F ′(t) = 2f(t)

∫ t

0f(x)dx− 2tf(t) = 2f(t)

∫ t

0(f(x)− 1)︸ ︷︷ ︸

≤0

dx ≤ 0.

Rezulta ca F este descrescatoare, deci

F (1) ≤ F (0) = 0,

adica (∫ 1

0f(x)dx

)2

≤ 2

∫ 1

0xf(x)dx.

Page 87: Teme si probleme pentru concursurile studentesti de matematica ...

80

Problema 9.22 Fie f : [a, b] → R continua pe [a, b] si derivabila pe (a, b), astfel caf(a) = f(b) = 0. Sa se arate ca exista c ∈ (a, b) astfel ca

|f ′(c)| ≥ 4

(b− a)2

∫ b

af(x)dx.

Solutie. Admitem ca

|f ′(x)| < 4

(b− a)2

∫ b

af(x)dx, ∀ x ∈ (a, b).

Avem ∣∣∣∣f(x)− f(a)

x− a

∣∣∣∣ = |f ′(ξ)| < 4

(b− a)2

∫ b

af(x)dx, ∀ x ∈ (a, b),

de unde

|f(x)| < 4(x− a)

(b− a)2

∫ b

af(x)dx, ∀ x ∈ (a, b).

Analog

|f(x)| < 4(b− x)

(b− a)2

∫ b

af(x)dx, ∀ x ∈ (a, b).

Avem: ∫ b

a|f(x)|dx =

∫ (a+b)/2

a|f(x)|dx+

∫ b

(a+b)/2|f(x)|dx

<4

(b− a)2

(∫ (a+b)/2

a(x− a)dx+

∫ b

(a+b)/2(b− x)dx

)∫ b

af(x)dx,

deci ∫ b

a|f(x)|dx <

∫ b

af(x)dx.

Rezulta ca exista c ∈ (a, b) astfel ca

|f ′(c)| ≥ 4

(b− a)2

∫ b

af(x)dx.

Problema 9.23 Fie f : (1, 2)→ R continua. Sa se arate ca

limx→1+

∫ x2

xf(t)dt = lim

x→1+(x− 1)f(x) ln 2.

Solutie. Aplicand teorema de medie pe intervalul [x, x2] putem scrie:

limx→1+

∫ x2

xf(t)dt = lim

x→1+

∫ x2

xf(t)(t− 1) · 1

t− 1dt

= limx→1+

(c− 1)f(c)

∫ x2

x

1

t− 1dt

(vezi x < c < x2),

= limx→1+

(c− 1)f(c) ln 2 = ln 2 limx→1+

(x− 1)f(x).

Page 88: Teme si probleme pentru concursurile studentesti de matematica ...

Calcul integral pentru functii de o variabila reala 81

Problema 9.24 Sa se demonstreze relatia∫ 1

0x−xdx =

∞∑n=1

n−n.

Solutie. Pentru x ∈ (0, 1] are loc relatia

x−x = e−x lnx = 1 +∞∑n=1

(−1)nxn lnn x

n!.

Seria converge uniform pe (0, 1] deoarece sup|x lnx| : x ∈ (0, 1] =1

esi seria

∞∑n=1

1

n!

(1

e

)neste convergenta (criteriul de convergenta uniforma a lui Weierstrass). Avem:

∫ 1

0x−xdx = 1 +

∞∑n=1

(−1)n

n!

∫ 1

0xn lnn xdx.

Fie In =

∫ 1

0xn lnn xdx. Substitutia x = e−t conduce la

In = (−1)n∫ ∞

0e−(n+1)ttndt.

Notand (n+ 1)t = u obtinem

In = (−1)n∫ ∞

0e−u · un

(n+ 1)n· du

n+ 1=

(−1)n

(n+ 1)n+1

∫ ∞0

e−uundu

=(−1)n

(n+ 1)n+1· Γ(n+ 1) =

(−1)n

(n+ 1)n+1· n!.

Rezulta ∫ 1

0x−xdx =

∞∑n=0

(n+ 1)−(n+1) =

∞∑n=1

n−n.

Problema 9.25 Fie M = f ∈ C1[0, 1] | f(1) = 1, f(0) = 0 si functia

J : M → R, J(f) =

∫ 1

0(1 + x2)(f ′(x))2dx.

Sa se determine minf∈M

J(f).

Vojtech Jarnik, 2009

Solutie. 1 = |f(1)− f(0)| =∣∣∣∣∫ 1

0f ′(x)dx

∣∣∣∣1 =

(∫ 1

0f ′(x)dx

)2

=

(∫ 1

0(√

1 + x2f ′(x))1√

1 + x2dx

)2

C−B−S≤

∫ 1

0(1 + x2)(f ′(x))2dx

∫ 1

0

1

1 + x2dx =

∫ 1

0(1 + x2)(f ′(x))2dx · π

4

Page 89: Teme si probleme pentru concursurile studentesti de matematica ...

82

⇒∫ 1

0(1 + x2)(f ′(x))2dx ≥ 4

π

Luand f0(x) =4

πarctg x avem

f0(0) = 0, f0(1) = 1 si f ′0(x) =4

π· 1

1 + x2,

J(f0) =

∫ 1

0(1 + x2)

(4

π

)2

· 1

(1 + x2)2dx =

(4

π

)2∫ 1

0

1

1 + x2dx =

(4

π

)2

· π4

=4

π,

deci

minf∈M

J(f) = J(f0) =4

π.

Problema 9.26 Sa se determine functiile f : [0, 1] → (0,∞) de clasa C1 care verificarelatiile:

f(1) = ef(0) si

∫ 1

0(f ′(x))2dx+

∫ 1

0

1

(f(x))2dx ≤ 2.

Putnam

Solutie. 0 ≤∫ 1

0

(f ′(x)− 1

f(x)

)2

dx

=

∫ 1

0(f ′(x))2dx− 2

∫ 1

0

f ′(x)

f(x)dx+

∫ 1

0

dx

(f(x))2

≤ 2− 2 ln f(x)∣∣∣10

= 2− 2 lnf(1)

f(0)= 0

Rezulta

f ′(x)− 1

f(x)= 0, ∀ x ∈ [0, 1] ⇔

(f2(x))′ = 2 ⇒ f2(x) = 2x+ c ⇒ f(x) =√

2x+ c, c > 0

si revenind la f(1) = f(0) · e rezulta c =2

e2 − 1si

f(x) =

√2x+

2

e2 − 1, x ∈ [0, 1].

Problema 9.27 Fie f : [0, 1]→ [0, 1] o functie continua. Sa se arate ca ecuatia

2x−∫ x

0f(t)dt = 1

are o singura solutie ın [0, 1].

Putnam

Solutie. Fie g(x) = 2x−∫ x

0f(t)dt−1, functie care este derivabila si g′(x) = 2−f(x) ≥

2− 1 = 1, deci g este strict crescatoare si atunci ecuatia data are cel mult o solutie. Avemg(0) = −1 < 0 si

g(1) = 2−∫ 1

0f(t)dt− 1 = 1−

∫ 1

0f(t)dt ≥ 0

deci ecuatia g(x) = 0 are o solutie unica.

Page 90: Teme si probleme pentru concursurile studentesti de matematica ...

Calcul integral pentru functii de o variabila reala 83

Problema 9.28 Stiind ca

∫ 1

0

ln(1 + x)

xdx =

π2

12sa se calculeze

∫ 1

0

ln(1− x3)

xdx.

Putnam

Solutie. Fie Ik =

∫ 1

0

ln(1− xk)x

dx =1

k

∫ 1

0

ln(1− t)t

dt =1

kI1.

Apoi

I =

∫ 1

0

ln(1 + x)

xdx = I2 − I1 = −1

2I1

I3 =1

3I1 = −2

3I = −π

2

18

Problema 9.29 Fie f ∈ C[0, 1] astfel ca xf(y) + yf(x) ≤ 1, pentru orice x, y ∈ [0, 1].

Sa se arate ca

∫ 1

0f(x)dx ≤ π

4.

Putnam

Solutie. Avem

I =

∫ 1

0f(x)dx =

∫ π2

0f(sin t) cos tdt =

∫ π2

0f(cos t) sin tdt ⇒

2I =

∫ π2

0(f(sin t) cos t+ f(cos t) sin t)dt ≤

∫ π2

01 · dt =

π

2.

Problema 9.30 Fie M

f ∈ C[0, π] |

∫ π

0f(x) sinxdx =

∫ π

0f(x) cosxdx = 1

.

Sa se determine minf∈M

∫ π

0(f(x))2dx.

Putnam

Solutie. Cautam o functie f0(x) = a sinx+ b cosx, x ∈ [0, π] care verifica relatiile dinenunt. Se obtine:

f0(x) =2

π(sinx+ cosx) ∈M.

Avem:

0 ≤∫ π

0(f(x)− f0(x))2dx

=

∫ π

0(f(x))2dx− 2

∫ π

0f0(x)f(x)dx+

∫ π

0(f0(x))2dx ⇒∫ π

0(f(x))2dx ≥ 2

∫ π

0f0(x)f(x)dx−

∫ π

0(f0(x))2dx

=8

π− 4

π=

4

π=

∫ π

0(f0(x))2dx.

Minimul este4

πsi se atinge pentru f = f0.

Page 91: Teme si probleme pentru concursurile studentesti de matematica ...

84

Problema 9.31 Fie f : [0, 1]→ R o functie de clasa C1. Sa se arate ca

f

(1

2

)≤∫ 1

0|f(x)|dx+

1

2

∫ 1

0|f ′(x)|dx.

Putnam

Solutie. Avem:∫ 1

0xf ′(x)dx = xf(x)

∣∣∣10−∫ 1

0f(x)dx = f(1)−

∫ 1

0f(x)dx

si ∫ 1

12

f ′(x)dx = f(1)− f(

1

2

).

Astfel

f

(1

2

)=

∫ 1

0xf ′(x)dx−

∫ 1

12

f ′(x)dx+

∫ 1

0f(x)dx

=

∫ 12

0xf ′(x)dx+

∫ 1

12

(x− 1)f ′(x)dx+

∫ 1

0f(x)dx

si ∣∣∣∣f (1

2

)∣∣∣∣ ≤ ∫ 12

0|xf ′(x)|dx+

∫ 1

12

|(x− 1)f ′(x)|dx+

∫ 1

0|f(x)|dx

≤∫ 1

0|f(x)|dx+

1

2

∫ 12

0|f ′(x)|dx+

1

2

∫ 1

12

|f ′(x)|dx

=

∫ 1

0|f(x)|dx+

1

2

∫ 1

0|f ′(x)|dx.

Problema 9.32 Sa se arate ca nu exista functia derivabila f : R→ R astfel ca

|f(x)| < 2x si f(x)f ′(x) ≥ sinx, pentru orice x ∈ R.

Solutie. Avem:

(f(x))2 − (f(0))2 =

∫ x

02f(t)f ′(t)dt ≥ 2

∫ x

0sin tdt = 2(1− cosx).

Pentru x = π rezulta (f(π))2 ≥ 4 ın contradictie cu |f(π)| < 2.

Problema 9.33 Sa se arate ca limn→∞

∫ 3

0

x2(1− x)xn

1 + x2ndx = 0.

Solutie.

∣∣∣∣∫ 3

0

x2(1− x)xn

1 + x2ndx

∣∣∣∣ ≤ ∫ 3

0

x2|1− x|xn

1 + x2ndx

=

∫ 1

0

x2(1− x)xn

1 + x2ndx+

∫ 3

1

x2(x− 1)xn

1 + x2ndx ≤

∫ 1

0xndx+ 18

∫ 3

1

xn

x2ndx

=1

n+ 1+

18

n− 1− 18

3n−1(n− 1)→ 0.

Page 92: Teme si probleme pentru concursurile studentesti de matematica ...

Calcul integral pentru functii de o variabila reala 85

Problema 9.34 Fie f1, f2, f3, f4 ∈ R[x]. Atunci

F (x) =

∫ x

1f1(t)f3(t)dt

∫ x

1f2(t)f4(t)dt−

∫ x

1f1(t)f4(t)dt

∫ x

1f2(t)f3(t)dt

este un polinom divizibil cu (x− 1)4.

Putnam, 1946

Solutie. F (1) = 0,

F ′(x) = f1(x)f3(x)

∫ x

1f2f4dt+ f2(x)f4(x)

∫ x

1f1f3

−f1(x)f4(x)

∫ x

1f2f3 − f2(x)f4(x)

∫ x

1f1f4

F ′(1) = 0

F ′′(1) = (f1f3)′∫ x

1+f1f2f3f4 + (f2f4)′

∫ x

1+f2f4f1f3 − . . .

= (f1f3)′∫ x

1f2f4 + (f2f4)′

∫ x

1f1f3 − (f1f4)′

∫ x

1f2f3 − (f2f3)′

∫ x

1f1f4

F ′′′(x) = (f1f2)′′∫ x

1f2f4 + (f1f2)′(f2f2) + . . . =

(f1f3)′f2f4 + (f2f4)′(f1f3)− (f1f4)′(f2f3)− (f1f4)(f2f3)′

⇒ [(f1f2)(f3f4)]′ − [(f1f4)(f2f3)]′ = 0

Problema 9.35 Fie f : [0, π]→ R continua astfel ca:∫ π

0f(x) cosxdx =

∫ π

0f(x) sinxdx = 0.

Sa se arate ca ecuatia f(x) = 0 are cel putin doua radacini ın intervalul (0, π).

Putnam, 1963

Solutie. Din

∫ π

0f(x) sinxdx = 0 si sinx ≥ 0, ∀ x ∈ [0, π] rezulta ca functia f ısi

schimba semnul pe [0, π], deci exista x1 ∈ [0, π] astfel ca f(x1) = 0. Daca ın x1 ar fisingura schimbare de semn, la fel ca functia sin(x− x1), atunci∫ π

0f(x) sin(x− x1)dx 6= 0 ⇔(∫ π

0f(x) sinxdx

)︸ ︷︷ ︸

=0

cosx1 −(∫ π

0f(x) cosxdx

)︸ ︷︷ ︸

=0

sinx1 6= 0 fals

rezulta ca exista x1 6= x2 ın care f schimba semnul, deci f(x1) = f(x2) = 0.

Problema 9.36 Fie f : [0, 1]→ [0,∞) crescatoare. Atunci∫ 1

0xf2(x)dx

∫ 1

0f(x)dx ≥

∫ 1

0f2(x)dx

∫ 1

0xf(x)dx.

Page 93: Teme si probleme pentru concursurile studentesti de matematica ...

86

Putnam, 1957

Solutie.

∫ 1

0xf2(x)dx

∫ 1

0f(y)dy −

∫ 1

0xf(x)dx

∫ 1

0f2(y)dy ≥ 0 ⇔

I =:

∫ 1

0

∫ 1

0f(x)f(y)x(f(x)− f(y))dxdy ≥ 0.

Schimband x cu y ın relatia anterioara obtinem

I =

∫ 1

0

∫ 1

0f(x)f(y)y(f(y)− f(x))dxdy,

deci

2I =

∫ 1

0

∫ 1

0f(x)f(y)(x− y)(f(x)− f(y))dxdy ≥ 0.

Cum f este crescatoare rezulta ca (x− y)|f(x)− f(y)| ≥ 0 pentru orice x, y ∈ [0, 1]. Prinurmare I ≥ 0.

Problema 9.37 Fie f : [−1, 1]→ R de doua ori derivabila astfel ıncat

|f(x)| ≤ 1, ∀ x ∈ [0, 1], |f ′′(x)| ≤ 1, ∀ x ∈ [0, 1].

Sa se arate ca |f ′(x)| ≤ 2, ∀ x ∈ [0, 1].

Putnam, 1962

Solutie. Conform formulei lui Taylor avem:

f(1) = f(x) + (1− x)f ′(x) +1

2(1− x)2f ′′(ξ), ξ ∈ (x, 1)

f(−1) = f(x) + (−1− x)f ′(x) +1

2(−1− x)2f ′′(η), η ∈ (−1, x)

⇒ f(1)− f(−1) = 2f ′(x) +1

2(1− x)2f ′′(ξ)− 1

2(1 + x)2f ′′(η)

⇒ 2(f ′(x)) ≤ |f(1)|+ |f(−1)|+ 1

2(1− x)2|f ′′(ξ)|+ 1

2(1 + x)2|f ′′(η)|

≤ 2 +1

2(1− x)2 +

1

2(1 + x)2 = 3 + x2 ≤ 4 ⇒ |f ′(x)| ≤ 2.

Problema 9.38 Sa se determine maximul expresiei∫ 1

0x2f(x)dx−

∫ 1

0x(f(x))2dx pentru f ∈ C[0, 1].

Putnam, 2006

Solutie.

∫ 1

0x2f(x)dx−

∫ 1

0x(f(x))2dx

=

∫ 1

0

(x3

4− x

(f(x)− x

2

)2)dx ≤

∫ 1

0

x3

4dx =

1

16

cu egalitate pentru f(x) =x

2, x ∈ [0, 1].

Page 94: Teme si probleme pentru concursurile studentesti de matematica ...

Calcul integral pentru functii de o variabila reala 87

Problema 9.39

∫ 1

0

(∫ 1

0f(x, y)dx

)2

dy +

∫ 1

0

(∫ 1

0f(x, y)dy

)2

dx

≤(∫ 1

0

∫ 1

0f(x, y)dxdy

)2

+

∫ 1

0

∫ 1

0(f(x, y))2dxdy.

Putnam, 2004

Solutie. Inegalitatea este echivalenta cu∫ 1

0

∫ 1

0

∫ 1

0

∫ 1

0(F (x, y, z, t))2dxdydzdt ≥ 0

undeF (x, y, z, t) = f(x, y) + f(z, t)− f(x, t)− f(z, y), x, y, z, t ∈ [0, 1].

Problema 9.40 Sa se determine f : (0,∞)→ (0,∞) pentru care

f ′(

1

x

)=

x

f(x), x ∈ (0,∞).

Putnam, 2005

Solutie. Avem

f ′(x) =1

xf

(1

x

) , x ∈ (0,∞)

f ′′(x) = −f

(1

x

)− 1

xf ′(

1

x

)x2f2

(1

x

) = − 1

x2f

(1

x

) +

f ′(

1

x

)x3f2

(1

x

)

= −f′(x)

x+

x

f(x)

x(f ′(x))2 = −f

′(x)

x+

(f ′(x))2

f(x)⇒

xf(x)f ′′(x) + f(x)f ′(x) = x(f ′(x))2 |: (f(x))2

⇒ f ′(x)

f(x)+xf ′′(x)

f(x)− x(f ′(x))2

(f(x))2⇒(

xf ′(x)

f(x)

)′= 0 ⇒ xf ′(x)

f(x)= c ⇒

f ′(x)

f(x)=c

x⇒ f(x) = dxc

Revenind: d2c = 1, deci f(x) = dx1d2 , d ∈ (0,∞).

Problema 9.41 Fie f ∈ C1[0, 1] cu f(0) = 0, 0 ≤ f ′(x) ≤ 1, ∀ x ∈ (0, 1). Sa se arate ca(∫ 1

0f(x)dx

)2

≥∫ 1

0(f(x))3dx.

Cand are loc egalitatea?

Page 95: Teme si probleme pentru concursurile studentesti de matematica ...

88

Putnam, 1973

Solutie. Fie G(t) = 2

∫ t

0f(x)dx− (f(t))2

G(0) = 0, G′(t) = 2f(t)(1− f ′(t)) ≥ 0

⇒ G(t) ≥ 0 si f(t)G(t) ≥ 0.

Fie H(t) =

(∫ t

0f(x)dx

)2

−∫ t

0(f(x))3dx, t ∈ [0, 1]. Avem:

H(0) = 0, H ′(t) = f(t)G(t) ≥ 0.

Rezulta H(t) ≥ 0, deci H(1) ≥ 0, q.e.d.Egalitatea are loc numai pentru f(t)G(t) = H ′(t) = 0, ∀ t ∈ [0, 1].Obtinem f(x) = x.

Problema 9.42 Fie f : [0, 1]→ R continua astfel ca∫ 1

0f(x)dx =

∫ 1

0xf(x)dx = . . . =

∫ 1

0xn−1f(x)dx = 0

si ∫ 1

0xnf(x)dx = 1.

Sa se arate ca exista a ∈ [0, 1] astfel ca |f(a)| ≥ 2n(n+ 1).

Putnam, 1972

Solutie. Din conditiile date rezulta∫ 1

0

(x− 1

2

)nf(x)dx = 1.

Daca prin absurd |f(x)| < 2n(n+ 1), ∀ x ∈ [0, 1] rezulta

1 ≤∫ 1

0

∣∣∣∣x− 1

2

∣∣∣∣n |f(x)|dx < 2n(n+ 1)

∫ 1

0

∣∣∣∣x− 1

2

∣∣∣∣n dx

= 2n(n+ 1) · 2∫ 1

12

(x− 1

2

)ndx = 2n(n+ 1) · 2 ·

(x− 1

2

)n+1

n+ 1

∣∣∣112

= 2n+1 · 1

2n+1= 1, contradictie.

Problema 9.43 Fie f : R → R o functie aditiva (f(x + y) = f(x) + f(y), ∀ x, y ∈ R)si integrabila pe orice interval compact din R. Sa se arate ca exista a ∈ R astfel ıncatf(x) = ax, ∀ x ∈ R.

Page 96: Teme si probleme pentru concursurile studentesti de matematica ...

Calcul integral pentru functii de o variabila reala 89

Solutie.

∫ x

0f(t+ y)dt =

∫ x

0f(t)dt+

∫ x

0f(y)dt ⇔

xf(y) =

∫ x

0f(t+ y)dt−

∫ x

0f(t)dt

=

∫ x+y

yf(t)dt−

∫ x

0f(t)dt

=

∫ x+y

0f(t)dt−

∫ x

0f(t)dt−

∫ y

0f(t)dt︸ ︷︷ ︸

ϕ(x,y)

ϕ(x, y) = ϕ(y, x) ⇒ xf(y) = yf(x), ∀ x, y ∈ R

⇒ f(x)

x=f(1)

1= f(1), ∀ x ∈ R ⇒ f(x) = xf(1), ∀ x ∈ R.

Problema 9.44 Fie f : [0, 1]→ R continua. Sa se arate ca∫ 1

0

(∫ 1

xf(t)dt

)dx =

∫ 1

0tf(t)dt.

Iran

Solutie. In prima integrala integram prin parti∫ 1

0

(∫ 1

xf(t)dt

)dx =

∫ 1

0x′(F (1)− f(x))dx

x =

∫ 1

xf(t)dt

∣∣∣10−∫ 1

0x(−f(x))dx =

∫ 1

0xf(x)dx.

Problema 9.45 Fie f : [0,∞)→ R o functie continua cu limx→∞

f(x) = 1.

Sa se calculeze limn→∞

∫ 1

0f(nx)dx.

Iran

Solutie. Vom calcula

limt→∞

∫ 1

0f(tx)dx = lim

t→∞

1

t

∫ t

0f(u)du

= limt→∞

F (t)− F (0)

t= lim

t→∞f(t) = 1,

unde

F (t) =

∫ t

0f(u)du, t ∈ [0, 1].

Problema 9.46 Fie f : [0, 1]→ [0,∞) astfel ıncat

∫ 1

0f(x)dx = 1.

Sa se arate ca ∫ 1

0

(x−

∫ 1

0tf(t)dt

)2

f(x)dx ≤ 1

4.

Page 97: Teme si probleme pentru concursurile studentesti de matematica ...

90

Iran

Solutie. Vom arata ca pentru orice α ∈ R avem:∫ 1

0(x− α)2f(x)dx ≥

∫ 1

0(x− β)2f(x)dx

unde β =

∫ 1

0xf(x)dx.

Avem: ∫ 1

0(x− α)2f(x)dx =

∫ 1

0((x− β) + (β − α))2f(x)dx

=

∫ 1

0(x− β)2f(x)dx+ (β − α)2

∫ 1

0f(x)dx+ 2(β − α)

∫ 1

0(x− β)f(x)dx

=

∫ 1

0(x− β)2f(x)dx+ (β − α)2 ≥

∫ 1

0(x− β)2f(x)dx.

Pentru α =1

2obtinem:∫ 1

0(x− β)2f(x)dx ≤

∫ 1

0

(x− 1

2

)2

f(x)dx ≤∫ 1

0

1

4f(x)dx =

1

4.

Problema 9.47 Sa se calculeze

∫ ∞0

x

1 + exdx.

Solutie. Fie f(x) =x

1 + ex, x ∈ [0,∞). Cum lim

x→∞x2f(x) = 0 rezulta ca integrala

este convergenta.

I : =

∫ ∞0

x

1 + exdx =

∫ ∞0

xe−x

1 + e−xdx = −

∫ ∞0

x(ln(1 + e−x))′dx

= −x ln(1 + e−x)∣∣∣∞0

+

∫ ∞0

ln(1 + e−x)dx

=

∫ ∞0

ln(1 + e−x)dx.

Punand e−x = t obtinem

I =

∫ 1

0

ln(1 + t)

tdt =

∫ 1

0

(1− t

2+t2

3− . . .

)dt

= 1− 1

22+

1

32− . . .

Din relatia 1 +1

22+

1

32+ . . . =

π2

6obtinem

1− 1

22+

1

32− . . . =

π2

12.

Problema 9.48 Fie f : [0,∞) → R o functie continua cu proprietatea ca

∫ ∞0

f(x)dx

este convergenta. Sa se arate ca

limn→∞

1

n

∫ n

0xf(x)dx = 0.

Page 98: Teme si probleme pentru concursurile studentesti de matematica ...

Calcul integral pentru functii de o variabila reala 91

Solutie. Fie

∫ ∞0

f(x)dx = I, I ∈ R. Avem:

I = limx→∞

∫ x

0f(t)dt = lim

x→∞

x

∫ x

0f(t)dt

x= lim

x→∞

(x

∫ x

0f(t)dt

)′(x)′

= limx→∞

(∫ x

0f(t)dt+ xf(x)

)= I + lim

x→∞xf(x).

Rezulta ca limx→∞

xf(x) = 0. Evident

limn→∞

1

n

∫ n

0xf(x)dx = lim

n→∞nf(n) = 0.

Problema 9.49 Fie M = f | f ∈ C2[0, 1], f(0) = f(1) = 0, f ′(0) = 1.Sa se determine

minf∈M

∫ 1

0(f ′′(x))2dx

si functiile pentru care se atinge minimul.

Solutie. Fie f ∈M . Are loc relatia∫ 1

0(1− x)f ′′(x)dx = (1− x)f ′(x)

∣∣∣10

+

∫ 1

0f ′(x)dx = −1.

Din inegalitatea lui Cauchy-Schwartz obtinem:(∫ 1

0(1− x)f ′′(x)dx

)2

≤∫ 1

0(1− x)2dx ·

∫ 1

0(f ′′(x))2dx (1)

de unde rezulta ca ∫ 1

0(f ′′(x))2dx ≥ 3.

In (1) egalitatea are loc pentru f ′′(x) = λ(1 − x), λ ∈ R. Punand conditia ca f ∈ Mrezulta

f(x) =1

2(x3 − 3x2 + 2x), x ∈ [0, 1].

Problema 9.50 Fie f ∈ C[a, b] astfel ca∫ b

axnf(x)dx = 0

pentru orice n ∈ N. Sa se arate ca f este identic nula.

Solutie. Din relatia

∫ b

axnf(x)dx = 0 rezulta ca pentru orice functie polinomiala P

are loc relatia ∫ b

af(x)P (x)dx = 0.

Page 99: Teme si probleme pentru concursurile studentesti de matematica ...

92

Din teorema lui Weierstrass rezulta ca exista un sir de functii polinomiale (Pn)n≥1

care este uniform convergent la f pe intervalul [a, b]. Cum f este marginita rezulta caPn · f ⇒ f2 pe [a, b]. Avem:∫ b

af2(x)dx = lim

n→∞

∫ b

aPn(x)f(x)dx = 0,

de unde rezulta ca f(x) = 0 pentru orice x ∈ [a, b].

Problema 9.51 Fie f ∈ C1[a, b], f ′(a) 6= 0. Pentru orice x ∈ (a, b] fie θ(x) ∈ [a, x] astfelca ∫ x

af(t)dt = (x− a)f(θ(x)).

Sa se calculeze limx→a

θ(x)− ax− a

.

Solutie. Existenta lui θ(x) rezulta din teorema de medie. Avem evident limx→a

θ(x) = a.

limx→a

θ(x)− ax− a

= limx→a

θ(x)− af(θ(x))− f(a)

· f(θ(x))− f(a)

x− a

= limx→a

θ(x)− af(θ(x))− f(a)

· limx→a

1

x− a

∫ x

af(t)dt− f(a)

x− a

=1

f ′(a)· limx→a

∫ x

af(t)dt− f(a)(x− a)

(x− a)2

=1

f ′(a)· limx→a

f(x)− f(a)

2(x− a)=

1

f ′(a)· 1

2· f ′(a) =

1

2.

Observatie. Din f ′(a) 6= 0 rezulta ca exista o vecinatate a lui a unde f este strictmonotona, prin urmare f(θ(x))− f(a) 6= 0 pentru x suficient de apropiat de a.

Page 100: Teme si probleme pentru concursurile studentesti de matematica ...

Capitolul 10

Functii de mai multe variabilereale

Definitii si rezultate

Fie Rn = R × R × · · · × R, x = (x1, . . . , xn) ∈ Rn, y = (y1, . . . , yn) ∈ Rn si λ ∈ R.Operatiile + : Rn × Rn → Rn, · : R× Rn → Rn date de

x+ y = (x1 + y1, . . . , xn + yn)

λx = (λx1, . . . , λxn)(1)

determina pe Rn o structura de spatiu vectorial peste R.Aplicatia 〈·, ·〉 : Rn × Rn → R,

〈x, y〉 =

n∑i=1

xiyi (2)

este un produs scalar pe Rn. Aceasta determina pe Rn aplicatiile ‖ · ‖ : Rn → R, d :Rn × Rn → R definite prin

‖x‖ =√〈x, x〉 =

√√√√ n∑i=1

x2i (3)

d(x, y) = ‖x− y‖ =

√√√√ n∑i=1

(xi − yi)2 (4)

numite norma, respectiv distanta euclidiana pe Rn.Spatiul vectorial Rn ınzestrat cu produsul scalar definit prin relatia (1) se numeste

spatiul euclidian Rn.Fie (xp)p≥1, xp = (x1

p, x2p, . . . , x

np ), un sir din Rn si a = (a1, a2, . . . , an) ∈ Rn. Sirul

(xp)p≥1 se numeste convergent cu limita a daca pentru orice ε > 0 exista pε ∈ N astfel capentru orice p ≥ pε sa avem

‖xp − a‖ < ε. (5)

In acest caz notam limp→∞

xp = a sau xp → a ın Rn. Are loc relatia

xp → a ın Rn ⇔ x1p → a1, . . . , x

np → an ın R.

93

Page 101: Teme si probleme pentru concursurile studentesti de matematica ...

94

Sirul (xp)p≥1 se numeste fundamental sau sir Cauchy daca pentru orice ε > 0 existapε ∈ N astfel ca pentru orice p, q ≥ pε sa aiba loc relatia ‖xp − xq‖ < ε.

Spatiul metric (Rp, d) este complet, i.e. orice sir din Rp este convergent daca si numaidaca este fundamental.

Multimi remarcabile din Rn

Fie a ∈ Rn, r ∈ R, r ≥ 0.B(a, r) = x ∈ Rn | ‖x− a‖ < r se numeste bila deschisa de centru a si raza rB(a, r) = x ∈ Rn | ‖x− a‖ ≤ r se numeste bila ınchisa de centru a si raza rS(a, r) = x ∈ Rn | ‖x− a‖ = r se numeste sfera de centru a si raza r.O multime V ⊆ Rn se numeste vecinatate a lui a daca exista B(a, r) ⊆ V . Notam

cu V(a) multimea vecinatatilor lui a. O multime G ⊆ Rn se numeste deschisa daca estevecinatate pentru orice punct al sau. Multimea vida se considera deschisa. O multimeF ⊆ Rn se numeste ınchisa daca CRnF = Rn \ F este multime deschisa.

Fie A ⊆ Rn. Punctul a ∈ Rn se numeste:- punct interior al lui A daca exista B(a, r) ⊆ A- punct aderent al lui A daca pentru orice V ∈ V(a) ⇒ V ∩A 6= ∅- punct de acumulare al lui A daca pentru orice V ∈ V(a) ⇒

V ∩ (A \ a) 6= ∅- punct izolat al lui A daca exista V ∈ V(a) astfel ca V ∩A = a- punct frontiera al lui A daca V ∩A 6= ∅ si V ∩ (Rn \A) 6= ∅.Notam prin intA, A, A′, izA, frA multimea punctelor interioare, aderente, de acumu-

lare, izolate respectiv frontiera ale lui A.Multimea A se numeste marginita daca exista M > 0 astfel ca ‖x‖ ≤M pentru orice

x ∈ A.Multimea A se numeste densa ın Rn daca A = Rn.Au loc relatiile:

a ∈ A ⇔ ∃ (xp)p≥1 ın A cu limp→∞

xp = a;

a ∈ A′ ⇔ ∃ (xp)p≥1 ın A \ a cu limp→∞

xp = a;

A este ınchisa ⇔ orice sir convergent din A are limita ın A.

Multimea A se numeste compacta daca din orice sir de puncte din A se poate extrageun subsir convergent la un element din A. Are loc caracterizarea:

A este compacta ⇔ A este ınchisa si marginita.

Multimea A se numeste conexa daca nu exista doua submultimi deschise si nevide alelui Rn astfel ca A ⊆ U ∪ V , A ∩ U 6= ∅, A ∩ V 6= ∅ si A ∩ U ∩ V = ∅.

Intuitiv o multime conexa e formata dintr-o singura ”bucata”.Multimea A se numeste conexa prin arce daca pentru orice puncte a, b ∈ A exista

un arc continuu cu capetele a si b continut ın A. Orice multime conexa prin arce e conexa.Are loc urmatorul rezultat:

O multime deschisa A ⊆ Rn este conexa daca si numai daca pentru orice a, b ∈ Aexista o linie poligonala cu capetele ın a si b continuta ın A.

Multimea A se numeste convexa daca pentru orice a, b ∈ A segmentul cu capetele asi b este continut ın A.

Page 102: Teme si probleme pentru concursurile studentesti de matematica ...

Functii de mai multe variabile reale 95

Functii continue

Fie f : A→ Rm, A ⊆ Rn si a ∈ A.Functia f se numeste continua ın a daca pentru orice V ∈ V(f(a)) exista U ∈ V(a)

astfel ca f(x) ∈ V pentru orice x ∈ U ∩A.Teorema. Urmatoarele relatii sunt echivalente:1) f este continua ın a.2) Pentru orice ε > 0, exista δε > 0 astfel ca ‖f(x)− f(a)‖ < ε pentru orice x ∈ A cu

‖x− a‖ < δε.3) Pentru orice sir (xp)p≥1 din A cu lim

p→∞xp = a ⇒ lim

p→∞f(x) = f(a).

Functia f se numeste continua pe B ⊆ A daca este continua ın fiecare punct al lui B.Definitie. Functia f se numeste uniform continua pe A daca pentru orice ε > 0

exista δε > 0 astfel ca pentru orice x, y ∈ A cu ‖x− y‖ < δε sa avem ‖f(x)− f(y)‖ < ε.Daca f este functie Lipschitz, i.e. ∃ L ≥ 0 astfel ca

‖f(x)− f(y)‖ ≤ L‖x− y‖, ∀ x, y ∈ A

atunci f este uniform continua pe A.Teorema. (Weiertrass) Fie A ⊆ Rn o multime compacta si f : A → R o functie

continua pe A. Atunci f este marginita si ısi atinge marginile.Teorema. Fie A ⊆ Rn o multime compacta si f : A→ Rn o functie continua. Atunci

f este uniform continua.Teorema. (Darboux) Fie A ⊆ Rn o multime conexa si f : A→ R o functie continua

cu proprietatea ca exista a, b ∈ A astfel ca f(a) < 0 si f(b) > 0. Atunci exista c ∈ A astfelca f(c) = 0.

Derivate partiale

Fie A ⊆ Rn o multime nevida, a = (a1, . . . , an) ∈ intA si f : A → R o functie caredepinde de x = (x1, . . . , xn) ∈ A. Spunem ca f este derivabila partial ın raport cuvariabila xk, 1 ≤ k ≤ n, ın punctul a daca

limxk→ak

f(a1, . . . , ak−1, xk, ak+1, . . . , an)− f(a1, . . . , an)

xk − ak

exista si este finita. Valoarea limitei anterioare se noteaza cu f ′xk(a) sau∂f

∂xk(a) si se

numeste derivata partiala a lui f ın raport cu xk ın punctul a. Functia f se numestederivabila ın raport cu xk, 1 ≤ k ≤ n, pe o multime B ⊆ A, daca f ′xk(x) exista si estefinita pentru orice x ∈ B.

Derivatele de ordin superior se definesc prin

f ′′xkxi = (f ′xk)′xi sau∂2f

∂xk∂xi=

∂xk

(∂

∂xi

)

1 ≤ i, k ≤ n. Se noteaza f ′′xixi = f ′′x2i

sau∂2f

∂xi∂xi=∂2f

∂x2i

.

In general pentru α = (α1, . . . , αn) ∈ Nn se definesc analog

f|α|xα11 ...xαnn

sau∂|α|f

∂xα11 . . . ∂xαnn

Page 103: Teme si probleme pentru concursurile studentesti de matematica ...

96

unde |α| = α1 + α2 + · · ·+ αn.Teorema. (Schwarz) Daca f : A ⊆ Rn → R admite derivatele partiale f ′′xkxi si f ′′xixk

pe o vecinatate V a punctului a ∈ intA si acestea sunt continue ın a, atunci f ′′xkxi(a) =f ′′xixk(a).

Teorema. (Derivata functiilor compuse) Fie D ⊆ Rm si A ⊆ Rn multimi deschisesi u = (u1, . . . , un) : D → A cu proprietatea ca u1, . . . , un admit derivate partiale ın raportcu variabila xk, 1 ≤ k ≤ m pe D. Daca f ∈ C1(A), atunci functia compusa F : D → R

F (x) = f(u1(x), . . . , un(x))

admite derivate partiale ın raport cu xk pe D si

∂F

∂xk=

∂f

∂u1· ∂u1

∂xk+

∂f

∂u2· ∂u2

∂xk+ · · ·+ ∂f

∂um· ∂um∂xk

.

Fie A ⊆ Rn, a ∈ intA, s ∈ Rn, ‖s‖ = 1. Spunem ca f este derivabila pe directia sın punctul a daca

limt→0

f(a+ ts)− f(a)

t

exista si este finita. Ea se noteaza cudf

ds(a) si se numeste derivata lui f pe directia s

ın punctul a.Teorema. Daca f : A→ R, A ⊆ Rn are derivate partiale de ordinul unu continue pe o

vecinatate a lui a ∈ intA, atunci ea este derivabila pe orice directie s = (s1, . . . , sn) ∈ Rn,‖s‖ = 1 si are loc relatia

df

ds(a) = f ′x1(a)s1 + f ′x2(a)s2 + · · ·+ f ′xn(a)sn.

Prin introducerea operatorului gradient notat prin grad sau ∇ si definit prin

∇ : C1(A)→ C(A,Rn), ∇f =

(∂f

∂x1,∂f

∂x2, . . . ,

∂f

∂xn

),

unde A ⊆ Rn este o multime deschisa, derivata pe directia s se poate exprima prin

df

ds(a) = 〈∇f(a), s〉.

Diferentiala unei functii

Fie f : I → R, I ⊆ R, o functie derivabila ın punctul a ∈ intI. Relatia

limx→a

f(x)− f(a)

x− a= f ′(a)

se poate scrie sub forma echivalenta

limx→a

f(x)− f(a)− f ′(a)(x− a)

x− a= 0. (1)

Fie T : R→ R functia liniara definita prin T (h) = f ′(a)h. Relatia (1) se scrie ın modechivalent

limx→a

f(x)− f(a)− T (x− a)

x− a= 0 (2)

Page 104: Teme si probleme pentru concursurile studentesti de matematica ...

Functii de mai multe variabile reale 97

prin urmare derivabilitatea lui f ın a implica existenta unei aplicatii liniare T astfel ca (2)sa aiba loc. De aici se poate deduce relatia

f(x)− f(a) ∼= T (x− a)

pe o vecinatate a lui a, deci variatia lui f ın jurul lui a poate fi aproximata printr-o functieliniara.

Functia f se numeste diferentiabila ın a daca exista T : R → R, liniara astfel carelatia (2) sa aiba loc. Se arata ca aplicatia T este unica. Ea se numeste diferentiala lui fın punctul a si se noteaza cu df(a). Functia f este diferentiabila ın a daca si numai dacaeste derivabila ın a. Avem

T (h) = df(a)(h) = f ′(a)h.

Cum diferentiala aplicatiei identice 1R : R → R, 1R(x) = x, pentru orice x ∈ R ested1R(x)(h) = h, se noteaza ın mod traditional h = dx. Deci:

df(x)(dx) = f ′(x)dx.

Fie f : A→ Rm, A ⊆ Rn, a ∈ intA. Functia f se numeste diferentiabila ın a ∈ intA,daca exista T : Rn → Rm liniara astfel ca

limx→a

f(x)− f(a)− T (x− a)

‖x− a‖= 0. (3)

Aplicatia T din (3) se numeste diferentiala lui f ın a si se noteaza cu df(a). Dacaf = (f1, . . . , fm), atunci f este diferentiabila ın a daca si numai daca f1, . . . , fm suntdiferentiabile ın a si ın acest caz

df(a) = (df1(a), df2(a), . . . , dfm(a)).

Teorema. Fie f : A→ R, A ⊆ Rn, diferentiabila ın a ∈ intA. Atunci:i) f este continua ın a;ii) f este derivabila pe orice directie s ∈ Rn, ‖s‖ = 1 si are loc relatia

df

ds(a) = df(a)(s).

Teorema. Fie f : A→ R, A ⊆ Rn, diferentiabila ın a ∈ intA. Atunci

df(a)(h) = f ′x1(a)h1 + f ′x2(a)h2 + · · ·+ f ′xn(a)hn.

Notand hk = dxk, 1 ≤ k ≤ n, avem

df(a)(dx) = f ′x1(a)dx1 + f ′x2(a)dx2 + · · ·+ f ′xn(a)dxn.

Teorema. Daca A ⊆ Rn este o multime deschisa si f ∈ C1(A), atunci f estediferentiabila pe A.

Teorema. (Diferentiala functiilor compuse) Fie f : A→ B, g : B → Rp, A ⊆ Rn,B ⊆ Rm. Daca f este diferentiabila ın a ∈ intA si g este diferentiabila ın b = f(a) ∈ intB,atunci g f : A→ Rp este diferentiabila ın a si are loc relatia

d(g f)(a) = dg(b) df(a).

Page 105: Teme si probleme pentru concursurile studentesti de matematica ...

98

Teorema. (Teorema de medie) Fie A ⊆ Rn o multime deschisa si convexa si f :A → R o functie diferentiabila pe A si a, b ∈ A. Atunci exista c pe segmentul [a, b] astfelca

f(b)− f(a) = df(c)(b− a).

Teorema. Fie A ⊆ Rn o multime deschisa si conexa. Daca f : A → R estediferentiabila pe A si df(x) = 0 pentru orice x ∈ A, atunci f este constanta pe A.

Calculul diferentialei se poate face si prin utilizarea regulilor de diferentiere. Dacaf, g : A→ R sunt diferentiabile ın a ∈ intA, atunci

d(f + g)(a) = df(a) + dg(a)

d(λf)(a) = λdf(a), λ ∈ R

d(fg)(a) = f(a)dg(a) + g(a)df(a)

d

(f

g

)(a) =

g(a)df(a)− f(a)dg(a)

g2(a), g(a) 6= 0.

Pentru o functie f ∈ Cp(A),A ⊆ Rm fiind o multime deschisa, se defineste diferentialade ordinul n, 1 ≤ n ≤ p, a lui f prin

dnf(x) =

(∂

∂x1dx1 + · · ·+ ∂

∂xmdxm

)nf(x) =

=∑

k1+···+km=n

n!

k1!k2! . . . km!

∂k1+···+kmf

∂xk11 . . . ∂xkmm(x)dxk11 . . . dxkmm .

Pentru n = 2 se obtine formula

d2f(x) =

n∑k=1

∂2f(x)

∂x2k

dx2k + 2

∑1≤j<k≤n

∂2f(x)

∂xk∂xjdxkdxj .

Teorema. (Formula lui Taylor) Fie A ⊆ Rn o multime deschisa si convexa, f ∈Cm+1(A) si a ∈ A. Atunci pentru orice x ∈ A exista ξ pe segmentul [a, x] astfel ca

f(x) = f(a) +df(a)(x− a)

1!+ · · ·+ dmf(a)(x− a)

m!+dm+1f(ξ)(x− a)

(m+ 1)!.

Teorema. (Teorema functiilor implicite) Fie A ⊆ Rn×Rm o multime deschisa sif : A→ Rm, f = (f1, f2, . . . , fm) si (x0, y0) ∈ A, x0 ∈ Rn, y0 ∈ Rm. Daca sunt ındepliniteconditiile:

1) f(x0, y0) = 0;2) f ∈ C1(A);

3)D(f1, . . . , fm)

D(y1, . . . , ym)(x0, y0) 6= 0

atunci exista o vecinatate deschisa U a lui x0 ın Rn, o vecinatate deschisa V a lui y0 ınRm si o functie ϕ : U → V astfel ca

a) ϕ(x0) = y0;b) f(x, ϕ(x)) = 0, ∀ x ∈ U ;c) ϕ este diferentiabila pe U .

Page 106: Teme si probleme pentru concursurile studentesti de matematica ...

Functii de mai multe variabile reale 99

Extremele functiilor de mai multe variabile

Extreme localeFie f : D → R, D ⊆ Rn. Punctul a ∈ D se numeste punct de minim (maxim) local al

functiei f daca exista o vecinatate V a lui a astfel ca

f(a) ≤ f(x) (f(a) ≥ f(x)), pentru orice x ∈ V ∩D.

Punctul a ∈ D se numeste punct de minim (maxim) global al lui f daca f(a) ≤ f(x)(f(a) ≥ f(x)) pentru orice x ∈ D.

Punctele de minim sau maxim local (global) se numesc puncte de extrem local (global)ale lui f .

Teorema. (Fermat) Daca a ∈ intD este punct de extrem local al lui f si f estediferentiabila ın a, atunci df(a) = 0.

Punctele a ∈ D ın care df(a) = 0 se numesc puncte stationare ale lui f . Rezulta ca unpunct a ∈ D este stationar daca si numai daca

f ′x1(a) = 0, f ′x2(a) = 0, . . . , f ′xn(a) = 0.

Pentru a decide care din punctele stationare sunt puncte de extrem local se poate folosiurmatorul rezultat:

Teorema. Fie D ⊆ Rn o multime deschisa, f ∈ C2(D) si a ∈ D un punct stationaral lui f . Au loc:

a) Daca diferentiala de ordinul al doilea d2f(a) este pozitiv definita, atunci a este punctde minim local al lui f ;

b) Daca d2f(a) este negativ definita, atunci a este punct de maxim local al lui f ;c) Daca d2f(a) este nedefinita, atunci a nu este punct de extrem local al lui f .Prezentam mai ın detaliu cazul functiilor de doua variabile reale. Fie D ⊆ R2 o multime

deschisa, f ∈ C2(D) si (x0, y0) un punct stationar al lui f . Atunci

d2f(x0, y0) = pdx2 + 2qdxdy + dy2,

unde p = f ′′x2(x0, y0), q = f ′′xy(x0, y0), r = f ′′y2(x0, y0). Notam δ := q2 − pr. Au loc:1) Daca δ > 0 atunci (x0, y0) nu este punct de extrem local al lui f ,2) Daca δ < 0 si p > 0 atunci (x0, y0) este punct de minim local al functiei f ,3) Daca δ < 0 si p < 0 atunci (x0, y0) este punct de maxim local al functiei f .Revenind la cazul general, reamintim un rezultat din algebra. Fie ϕ : Rn → R o

forma patratica si A matricea sa. Notam cu λ1, λ2, . . . , λn valorile proprii ale lui A si cu∆1,∆2, . . . ,∆n minorii sai principali.

Teorema. (Sylvester) Au loc echivalentele:1) ϕ este pozitiv definita ⇔ λ1 > 0, λ2 > 0, . . . , λn > 0 ⇔

∆1 > 0,∆2 > 0, . . . ,∆n > 02) ϕ este negativ definita ⇔ λ1 < 0, λ2 < 0, . . . , λn < 0 ⇔

∆1 < 0,∆2 > 0,∆3 < 0, . . . , (−1)n∆n > 03) ϕ este nedefinita ⇔ A are doua valori proprii de semne contrare.

Extreme conditionateFie D ⊆ Rn o multime nevida, f : D → R si g1, g2, . . . , gm : D → R m functii date

(cu m < n). Fie

C = x ∈ D | g1(x) = 0, . . . , gm(x) = 0, x = (x1, . . . , xn).

Page 107: Teme si probleme pentru concursurile studentesti de matematica ...

100

Un punct a ∈ C se numeste punct de extrem local conditionat daca a este punct de extremlocal pentru functia f |C (restrictia lui f la C). Relatiile

g1(x) = 0, g2(x) = 0, . . . , gm(x) = 0

se mai numesc legaturi, din aceasta cauza extremele conditionate se mai numesc extremecu legaturi.

Fie G = (g1, . . . , gn) : D → Rm si L : D ×Rm → R, definita prin

L(x, λ) = f(x) + λ1g1(x) + · · ·+ λmgm(x), λ = (λ1, . . . , λm) ∈ Rm.

Teorema. Presupunem ca D este multime deschisa si f, g1, . . . , gm ∈ C1(D). Dacaa ∈ C este punct de extrem local conditionat al lui f si matricea jacobiana JG verificarelatia rang JG(a) = m, atunci exista λ0 = (λ0

1, λ02, . . . , λ

0m) ∈ Rm astfel ca

L′xk(a, λ0) = 0, 1 ≤ k ≤ ngj(a) = 0, 1 ≤ j ≤ m

Functia L se numeste Lagrangeanul lui f iar λ01, λ

02, . . . , λ

0m se numesc multiplicatorii

lui Lagrange.Observatie. Daca a este punct de extrem local conditionat, atunci (a, λ0) este punct

stationar al lui L.Are loc urmatorul rezultat:Teorema. Presupunem ca D este multime deschisa, f, g1, . . . , gm ∈ C1(D) si (a, λ0) ∈

C ×Rm este un punct stationar al lui L. Daca pentru orice h ∈ Rn \ 0 cu proprietateadG(a)(h) = 0 avem

d2L(a, λ0)(h) > 0 (d2L(a, λ0)(h) < 0),

atunci a este punct de minim (maxim) local conditionat al lui f .

Probleme

Problema 10.1 Fie m,n ∈ N∗ si f : R2 → R definita prin

f(x, y) =

xmyn

x2 − xy + y2, (x, y) 6= (0, 0)

0, (x, y) = (0, 0)

Sa se studieze continuitatea, existenta derivatelor partiale de ordinul I, derivabilitateape o directie si diferentiabilitatea lui f ın punctul (0, 0).

Solutie.Continuitatea: daca m = n = 1, considerand dreapta y = λx, λ ∈ R, avem

limx→0

f(x, λx) =λ

1− λ+ λ2 ,

care depinde de λ, deci lim(x,y)→(0,0)

f(x, y) nu exista. Daca m+ n > 2 avem

|f(x, y)| = |xmyn|x2 − xy + y2

≤ |xmyn|xy

= |x|m−1|y|n−1

Page 108: Teme si probleme pentru concursurile studentesti de matematica ...

Functii de mai multe variabile reale 101

pentru (x, y) 6= (0, 0), de unde rezulta

lim(x,y)→(0,0)

f(x, y) = 0 = f(0, 0).

Deci f este continua ın (0, 0) daca si numai daca m+ n > 2.Derivabilitatea ın raport cu x si y:

limx→0

f(x, 0)− f(0, 0)

x− 0= lim

x→0

0

x= 0

si

limy→0

f(0, y)− f(0, 0)

y − 0= lim

y→0

0

y= 0,

deci f ′x(0, 0) = f ′y(0, 0) = 0.Derivabilitatea pe o directie: fie s = (u, v) ∈ R2 cu u2 + v2 = 1, uv 6= 0. Avem

limt→0

f((0, 0) + t(u, v))− f(0, 0)

t= lim

t→0

tm+numvn

t3(u2 − uv + v2)=

= limt→0

tm+n−3 umvn

u2 − uv + v2= lim

t→0tm+n−3f(u, v).

Daca m+ n > 3 rezulta cadf

ds(0, 0) = 0 pentru orice s.

Daca m+ n = 3 rezulta cadf

ds(0, 0) = f(u, v).

Daca m+ n < 3, rezulta m = n = 1 si f nu este derivabila pe nici o directie s.Diferentiabilitatea ın (0, 0): daca m = n = 1, f nu este diferentiabila ın (0, 0), deoarecenu este continua ın (0, 0). Pentru m + n ≥ 3, cum f ′x(0, 0) = f ′y(0, 0) = 0, daca f ar fidiferentiabila ın (0, 0) ar trebui ca

df(0, 0)(h, k) = f ′x(0, 0)h+ f ′y(0, 0)k = 0.

Relatia

lim(x,y)→(0,0)

f(x, y)− f(0, 0)− T (x− 0, y − 0)√x2 + y2

= 0 ⇔

lim(x,y)→(0,0)

xmyn

(x2 − xy + y2)√x2 + y2

= 0

este verificata daca si numai daca m + n > 3 (pentru m + n = 3 se considera limita pedreapta y = λx). Deci f este diferentiabila ın (0, 0) daca si numai daca m+ n > 3.

Problema 10.2 Fie f : R2 → R definita prin

f(x, y) =

(x2 + y2) sin1√

x2 + y2, (x, y) 6= (0, 0)

0, (x, y) = (0, 0).

Demonstrati ca f este diferentiabila ın (0, 0), dar f ′x si f ′y nu sunt continue ın (0, 0).

Solutie. Avem

f ′x(0, 0) = limx→0

f(x, 0)− f(0, 0)

x= lim

x→0|x| sin 1

|x|= 0

Page 109: Teme si probleme pentru concursurile studentesti de matematica ...

102

si analog

f ′y(0, 0) = limy→0

f(0, y)− f(0, 0)

y= lim

y→0|y| sin 1

|y|= 0.

Vom arata ca df(0, 0) = 0. Intr-adevar, avem

lim(x,y)→(0,0)

f(x, y)− f(0, 0)− df(0, 0)(x− 0, y − 0)√x2 + y2

=

= lim(x,y)→(0,0)

√x2 + y2 sin

1√x2 + y2

= 0.

Pentru (x, y) 6= (0, 0) avem

f ′x(x, y) = 2x sin1√

x2 + y2− x√

x2 + y2cos

1√x2 + y2

f ′y(x, y) = 2y sin1√

x2 + y2− y√

x2 + y2cos

1√x2 + y2

.

Cum

limn→∞

f ′x

(1

2nπ, 0

)= lim

n→∞(

1

nπsin 2nπ − cos 2nπ) = −1

si

limn→∞

f ′x

(0,

1

2nπ

)= 0

rezulta ca lim(x,y)→(0,0)

f ′x(x, y) nu exista, deci f ′x este discontinua ın (0, 0). Prin simetrie

rezulta ca si f ′y este discontinua ın (0, 0).Observatie. Continuitatea derivatelor partiale ıntr-un punct nu este o conditie nece-

sara pentru diferentiabilitatea functiei ın acel punct.

Problema 10.3 Sa se arate ca functia f : R2 → R

f(x, y) =

|x|a sin

y

x, x 6= 0

0, x = 0

este diferentiabila pentru orice a > 1.

Solutie. Functia f este diferentiabila ın (x0, y0) daca si numai daca exista derivatele

partiale∂f

∂x(x0, y0),

∂f

∂y(x0, y0) si ın plus

lim(x,y)→(x0,y0)

f(x, y)− f(x0, y0)− ∂f

∂x(x0, y0)(x− x0)− ∂f

∂y(x0, y0)(y − y0)√

(x− x0)2 + (y − y0)2= 0.

Functia f are derivate partiale continue pe multimea R2 − (0, b)| b ∈ R, deci estediferentiabila ın aceste puncte. Ramane de studiat diferentiabilitatea ın punctele de forma(0, b), b ∈ R.

In (0, 0) avem:∂f

∂x(0, 0) = lim

x→0

f(x, 0)− f(0, 0)

x= 0,

Page 110: Teme si probleme pentru concursurile studentesti de matematica ...

Functii de mai multe variabile reale 103

∂f

∂y(0, 0) = lim

y→0

f(0, y)− f(0, 0)

y= 0

si |f(x, y)| ≤ |x|a, deci

lim(x,y)→(0,0)

|f(x, y)|√x2 + y2

= lim(x,y)→(0,0)

|x|√x2 + y2

|x|a−1 ≤

≤ lim(x,y)→(0,0)

|x|a−1 = 0,

deci exista diferentiala ın (0, 0) si este egala cu zero.In (0, b) cu b 6= 0,

∂f

∂x(0, b) = lim

x→0

f(x, b)− f(0, b)

x= lim

x→0xa−1 sin

b

x= 0

∂f

∂y(0, b) = 0,

iar ∣∣∣∣f(x, y)− f(0, b)− ∂f

∂x(0, b)x− ∂f

∂y(0, b)(y − b)

∣∣∣∣ = |f(x, y)| ≤ |x|a

si

lim(x,y)→(0,0)

|f(x, y)|√x2 + (y − b)2

≤ lim(x,y)→(0,0)

|x|√x2 + (y − b)2

|x|a−1 ≤

≤ lim(x,y)→(0,0)

|x|a−1 = 0, deci df(0, b) = 0.

Problema 10.4 Fie f : D → R, D ⊆ R2 si (x0, y0) ∈ intD. Sa se arate ca daca f arederivate partiale ıntr-o vecinatate V a lui (x0, y0) si daca una din ele este continua ın(x0, y0), atunci f este diferentiabila ın (x0, y0).

Solutie. Sa presupunem ca f ′x este continua ın (x0, y0). Pentru orice (x, y) ∈ V ∩ Davem:

f(x, y)− f(x0, y0) = f(x, y)− f(x0, y) + f(x0, y)− f(x0, y0).

Conform teoremei lui Lagrange exista c1 ıntre x0 si x astfel ca

f(x, y)− f(x0, y) = (x− x0)f ′x(c1, y).

Cum f ′y(x0, y0) exista rezulta ca

limy→y0

f(x0, y)− f(x0, y0)

y − y0− f ′y(x0, y0) = 0

decif(x0, y)− f(x0, y0) = (y − y0)(f ′y(x0, y0) + ω2(x0, y))

cu limy→y0

ω2(x0, y) = 0.

Din continuitatea lui f ′x ın (x0, y0) rezulta ca

f ′x(c1, y) = f ′x(x0, y0) + ω1(x, y)

cu lim(x,y)→(x0,y0)

ω1(x, y) = 0.

Page 111: Teme si probleme pentru concursurile studentesti de matematica ...

104

Rezulta ca are loc relatia

f(x, y)− f(x0, y0) = (x− x0)f ′x(x0, y0) + (y − y0)f ′y(x0, y0)+

+ω1(x, y)(x− x0) + ω2(x0, y)(y − y0)

pentru (x, y) ∈ V ∩D, ceea ce arata ca f este diferentiabila ın (x0, y0).

Problema 10.5 Fie g : R → R o functie derivabila pe R, satisfacand conditiile g(0) = 0si g′(0) 6= 0. Definim functia f : R2 → R prin relatia

f(x, y) =

g(xy)x2 − y2

x2 + y2, (x, y) 6= (0, 0)

0, (x, y) = (0, 0)

a) Sa se studieze existenta derivatelor partiale de ordinul I si diferentiabilitatea lui fın (0, 0).

b) Sa se arate ca f ′′xy(0, 0) 6= f ′′yx(0, 0).

Solutie. a) f ′x(0, 0) = limx→0

f(x, 0)− f(0, 0)

x= lim

x→0

0

x= 0 si analog f ′y(0, 0) = 0. Demon-

stram ca f este diferentiabila ın (0, 0) si T = df(0, 0) = 0. Avem

lim(x,y)→(0,0)

f(x, y)− f(0, 0)− T (x− 0, y − 0)√x2 + y2

=

= lim(x,y)→(0,0)

g(xy)√x2 + y2

· x2 − y2

x2 + y2=

= lim(x,y)→(0,0)

g(xy)− g(0)

xy· xy√

x2 + y2· x

2 − y2

x2 + y2=

= g′(0) lim(x,y)→(0,0)

xy√x2 + y2

· x2 − y2

x2 + y2= 0,

deoarece ∣∣∣∣∣ xy√x2 + y2

· x2 − y2

x2 + y2

∣∣∣∣∣ =|x|√x2 + y2︸ ︷︷ ︸≤1

·∣∣∣∣x2 − y2

x2 + y2

∣∣∣∣︸ ︷︷ ︸≤1

|y| ≤ |y|,

pentru (x, y) 6= (0, 0).b) Avem

f ′x(x, y) = yg′(xy)x2 − y2

x2 + y2+ g(xy)

4xy2

(x2 + y2)2

si

f ′y(x, y) = xg′(xy)x2 − y2

x2 + y2− g(xy)

4x2y

(x2 + y2)2.

Obtinem

f ′′xy(0, 0) = (f ′x)′y(0, 0) = limy→0

f ′x(0, y)− f ′(0, 0)

y=

= limy→0

−yg′(0)

y= −g′(0)

si

f ′′yx(0, 0) = (f ′y)′x(0, 0) = lim

x→0

f ′y(x, 0)− f ′y(0, 0)

x= lim

x→0

xg′(0)

x= g′(0).

Page 112: Teme si probleme pentru concursurile studentesti de matematica ...

Functii de mai multe variabile reale 105

Problema 10.6 Se da functia f : R2 → R definita prin

f(x, y) =

y2 ln

(1 +

x2

y2

), y 6= 0

0, y = 0

Sa se arate ca derivatele mixte de ordinul doi ale lui f nu sunt continue ın origine sitotusi f ′′xy(0, 0) = f ′′yx(0, 0).

Solutie. In punctele de forma (x, y) cu y 6= 0 avem

f ′x(x, y) =2xy2

x2 + y2si f ′y(x, y) = 2y ln

(1 +

x2

y2

)− 2x2y

x2 + y2

iar ın punctele de forma (x0, 0) avem

f ′x(x0, 0) = limx→x0

f(x, 0)− f(x0, 0)

x− x0= lim

x→x0

0

x− x0= 0

f ′y(x0, 0) = limy→0

f(x0, y)− f(x0, 0)

y= lim

y→0y ln

(1 +

x20

y2

)= 0.

Se obtine de asemenea

f ′′xy(x, y) =

4x3y

(x2 + y2)2, y 6= 0

0, y = 0

si

f ′′yx(x, y) =

4x3y

(x2 + y2)2, y 6= 0

0, y = 0

deci f ′′xy = f ′′yx. Calculand limitele pe dreapta y = λx se obtin rezultatele

limx→0

f ′′xy(x, λx) = limx→0

f ′′yx(x, λx) =4λ

(1 + λ2)2,

de unde rezulta ca functiile f ′′xy si f ′′yx nu au limita ın (0, 0), deci sunt discontinue ın (0, 0).Observatie. Continuitatea derivatelor partiale mixte de ordinul II din teorema lui

Schwarz nu este o conditie necesara pentru egalitatea lor.

Problema 10.7 Fie g : [0,+∞)→ R si f : Rn → R, f(x) = g(‖x‖) pentru orice x ∈ Rn.Sa se arate ca urmatoarele afirmatii sunt echivalente:

i) f este o functie de clasa C1;ii) g este o functie de clasa C1 si g′(0) = 0.

Solutie. ”i) ⇒ ii)” Presupunem ca f este de clasa C1 si fie u ∈ Rn, ‖u‖ = 1. Pentruorice t ≥ 0 avem

f(tu) = g(‖tu‖) = g(t)

si tinand seama ca f ∈ C1(Rn) rezulta ca g ∈ C1([0,∞)). Avem de asemenea relatia

f(t(−u)) = g(‖ − tu‖) = g(| − t| · ‖u‖) = g(t)

Page 113: Teme si probleme pentru concursurile studentesti de matematica ...

106

pentru orice t ≥ 0. Deci pentru orice t ≥ 0 avem:

g(t) = f(tu) = f(−tu).

Notand u = (u1, u2, . . . , un) si derivand relatia anterioara avem

g′(t) = u1f′x1(tu) + · · ·+ unf

′xn(tu) = −u1f

′x1(−tu)− · · · − unf ′xn(−tu)

de unde rezulta

g′(0) = u1f′x1(0) + · · ·+ unf

′xn(0) = −u1f

′x1(0)− · · · − unf ′xn(0)

ceea ce este echivalent cu g′(0) = 0.”ii) ⇒ i)” Cum norma euclidiana a lui Rn este de clasa C1 pe Rn \ 0 rezulta ca f

este de clasa C1 pe Rn \ 0. Demonstram ca f ′xk(0) = 0 pentru 1 ≤ k ≤ n. Avem:

limxk→0

∣∣∣∣f(0, . . . , 0, xk, 0, . . . , 0)− f(0, . . . , 0)

xk

∣∣∣∣ = limxk→0

|g(|xk|)− g(0)||xk|

=

= |g′(0)| = 0 pentru 1 ≤ k ≤ n.

De asemenea pentru x ∈ Rn, x 6= 0, avem

f ′xk(x) = g′(‖x‖) xk‖x‖

, 1 ≤ k ≤ n.

Rezulta ca

|f ′xk(x)| = |g′(‖x‖)| |xk|‖x‖≤ |g′(‖x‖)|

si tinand seama de faptul ca limx→0

g′(‖x‖) = g′(0) = 0 obtinem

limx→0

f ′xk(x) = 0 = f ′xk(0), 1 ≤ k ≤ n.

Deci f ∈ C1(Rn).

Problema 10.8 Fie f : R3 → R o functie definita prin relatia

f(x) =

sin(a‖x‖)‖x‖

, x 6= 0

a, x = 0

unde a ∈ R. Sa se arate ca f ∈ C2(R3) si ∆f + a2f = 0.

Solutie. Fie g : R→ R, g(t) =sin(at)

tpentru t 6= 0 si g(0) = a. Se verifica imediat ca

g este o functie de clasa C∞(R), dezvoltand functia sin(at) ın serie de puteri ale lui t, sica g′(0) = 0. Cum f(x) = g(‖x‖) pentru orice x ∈ R3 rezulta din problema 10.7 ca f estede clasa C1(R3) si f ′xk(0) = 0, 1 ≤ k ≤ 3.

Printr-un rationament analog celui din problema 10.7 se arata ın continuare ca f ∈C2(R3). Avem

f ′′x2k(x) = g′′(‖x‖)

x2k

‖x‖2+ g′(‖x‖)

‖x‖2 − x2k

‖x‖3

pentru 1 ≤ k ≤ 3 de unde rezulta

∆f = g′′(‖x‖) + g′(‖x‖) 2

‖x‖= −a2f.

Page 114: Teme si probleme pentru concursurile studentesti de matematica ...

Functii de mai multe variabile reale 107

Problema 10.9 Fie K ⊆ Rn o multime nevida cu proprietatea ca pentru orice x ∈ Ksi orice t > 0 avem tx ∈ K. Fie de asemenea f : K → R o functie omogena de gradul p,p ∈ R, adica avand proprietatea

f(tx) = tpf(x)

pentru orice x ∈ K si orice t > 0.a) Sa se arate ca daca f ∈ C1(K), atunci

x1∂f

∂x1(x) + · · ·+ xn

∂f

∂xn(x) = pf(x), ∀ x ∈ K.

b) Sa se arate ca daca f ∈ C2(K), atunci(x1

∂x1+ · · ·+ xn

∂xn

)(2)

f(x) = p(p− 1)f(x), ∀ x ∈ K.

(Identitatile lui Euler pentru functii omogene.)

Solutie. a) Derivand relatia

f( tx1︸︷︷︸u1

, tx2︸︷︷︸u2

, . . . , txn︸︷︷︸un

) = tpf(x), x = (x1, . . . , xn) ∈ K,

ın raport cu t obtinem

∂f

∂u1(tx)

du1

dt+ · · ·+ ∂f

∂un(tx)

dundt

= ptp−1f(x)

sau

x1∂f

∂u1(tx) + · · ·+ xn

∂f

∂un(tx) = ptp−1f(x).

Punand ın ultima relatie t = 1 obtinem

x1∂f

∂x1(x) + · · ·+ xn

∂f

∂xn(x) = pf(x).

b) Derivam relatia de la punctul a) ın raport cu x1, x2, . . . , xn obtinem succesiv

∂f

∂x1+ x1

∂2f

∂x21

+ x2∂2f

∂x1∂x2+ · · ·+ xn

∂2f

∂x1∂xn= p

∂f

∂x1

x1∂2f

∂x2∂x1+

∂f

∂x2+ x2

∂2f

∂x22

+ · · ·+ xn∂2f

∂x2∂xn= p

∂f

∂x2. . . . . .

x1∂2f

∂xn∂x1+ · · ·+ ∂f

∂xn+ xn

∂2f

∂x2n

= p∂f

∂xn.

Inmultind relatiile anterioare cu x1, x2, . . . , xn si adunandu-le obtinem

n∑k=1

x2k

∂2f

∂x2k

+ 2∑

1≤k<j≤n

∂2f

∂xk∂xjxkxj +

n∑k=1

xk∂f

∂xk= p

n∑k=1

xk∂f

∂xk

si ınlocuindn∑k=1

xk∂f

∂xk= pf obtinem

(x1

∂x1+ · · ·+ xn

∂xn

)(2)

f = p(p− 1)f.

Page 115: Teme si probleme pentru concursurile studentesti de matematica ...

108

Problema 10.10 Sa se calculeze∂m+nf

∂xm∂ynsi diferentiala de ordinul n pentru functia:

f(x, y) = (x2 + y2)ex+y, (x, y) ∈ R2.

Solutie. Aplicand formula lui Leibniz avem

∂mf

∂xm= C0

mex+y(x2 + y2) + C1

mex+y · 2x+ C2

mex+y · 2 =

= ex+y(x2 + y2 + 2mx+m2 −m)

∂m+nf

∂xm∂yn= C0

nex+y(x2 + y2 + 2mx+m2 −m) + C1

nex+y · 2y + C2

nex+y · 2 =

= ex+y(x2 + y2 + 2mx+ 2ny +m2 + n2 −m− n).

Inlocuind derivatele partiale ın formula

dnf =

(∂

∂xdx+

∂ydy

)nf

se obtine diferentiala de ordinul n a lui f .

Problema 10.11 a) Sa se arate ca derivata functiei f : R∗ × R → R, f(x, y) =y2

xın

orice punct al elipsei 2x2 + y2 = 1 pe directia normalei la elipsa este egala cu zero.

b) Sa se calculeze derivata functiei f(x, y, z) =1√

x2 + y2 + z2, pe directia gradientului

sau.

Solutie. a) Fie (x0, y0) un punct al elipsei 2x2 + y2 = 1. Tangenta la elipsa ın (x0, y0)are ecuatia 2xx0 + yy0 = 1, deci versorul normalei la elipsa este s = (s1, s2) unde

s1 =2x0√

4x20 + y2

0

, s2 =y0√

4x20 + y2

0

.

Aplicand formula derivatei pe directie avem

df

ds(x0, y0) = f ′x(x0, y0)s1 + f ′y(x0, y0)s2 =

= −y20

x20

· 2x0√4x2

0 + y20

+2y0

x0· y0√

4x20 + y2

0

= 0.

b) Fie punctul M(x, y, z), diferit de originea O.Notam prin r =

√x2 + y2 + z2. Avem

gradf =(− xr3,− y

r3,− z

r3

)iar versorul sau este s =

(−xr,−y

r,−z

r

). Obtinem

df

ds(M) = f ′x(M)s1 + f ′y(M)s2 + f ′z(M)s3 =

1

r2.

Page 116: Teme si probleme pentru concursurile studentesti de matematica ...

Functii de mai multe variabile reale 109

Problema 10.12 Sa se arate ca ecuatia xey + yex = 1 defineste o functie implicita

y = f(x)

ıntr-o vecinatate a punctului (0, 1). Sa se determine primii trei termeni din dezvoltarea luif dupa formula lui Taylor ın punctul 0.

Solutie. Fie F : R2 → R, F (x, y) = xey+yex−1. Cum F este de clasa C∞, F (0, 1) = 0si F ′y(0, 1) = 1 6= 0 rezulta ca ecuatia F (x, y) = 0 defineste o functie y = f(x) de clasa C∞

ıntr-o vecinatate a lui (0, 1). Derivand ecuatia F (x, y) = 0 ın care y = f(x) obtinem:

ey + xy′ey + y′ex + yex = 0

si punand x = 0, y = 1 rezulta y′(0) = −1− e. Derivand din nou obtinem

2y′ey + xy′′ey + x(y′)2ey + y′′ex + 2y′ex + yex = 0

si punand din nou x = 0, y = 1 rezulta

y′′(0) = 2e2 + 4e+ 1.

In sfarsit derivand din nou ultima relatie si punand x = 0, y = 1 se obtine

y′′′(0) = −9e3 − 24e2 − 15e− 1.

Dezvoltarea ceruta este

f(x) = f(0) +f ′(0)

1!x+

f ′′(0)

2!x2 +

f ′′′(0)

3!x3 +R3(x)

f(x) = 1− (1 + e)x+

(e2 + 2e+

1

2

)x2 −

(3

2e3 + 4e2 +

5

2e+

1

6

)x3 +R3(x).

Problema 10.13 Consideram sistemul de ecuatii3x+ y − z + u4 = 0x− y + 2z + u = 02x+ 2y − 3z + 2u = 0

a) Demonstrati ca sistemul dat defineste pe x, y, u ca functii de z satisfacand conditiilex(0) = y(0) = u(0) = 0 pe un interval de forma ]− ε, ε[ cu ε > 0.

b) Demonstrati ca nu exista nici un interval de forma ]−δ, δ[ cu δ > 0 pe care sistemuldat sa defineasca pe x, y, z ca functii de u.

Solutie. a) Fie F1, F2, F3 : R4 → R,

F1(x, y, z, u) = 3x+ y − z + u4,

F2(x, y, z, u) = x− y + 2z + u,

F3(x, y, z, u) = 2x+ 2y − 3z + 2u.

Avem

D(F1, F2, F3)

D(x, y, u)(0, 0, 0, 0) =

∣∣∣∣∣∣3 1 4u3

1 −1 12 2 2

∣∣∣∣∣∣ (0, 0, 0, 0) =

Page 117: Teme si probleme pentru concursurile studentesti de matematica ...

110

=

∣∣∣∣∣∣3 1 01 −1 12 2 2

∣∣∣∣∣∣ = −12 6= 0

de unde rezulta pe baza teoremei functiilor implicite ca exista o vecinatate U a lui 0 si ovecinatate V a punctului (0, 0, 0) si o unica functie vectoriala F = (f1, f2, f3) : U → Vdefinita de sistemul dat si care satisface conditia F (0) = (0, 0, 0). Din faptul ca U ∈ VR(0)rezulta ca exista ε > 0 astfel ca ]− ε, ε[⊂ U . Cum F1, F2, F3 sunt de clasa C∞ pe ]− ε, ε[rezulta ca si functia F este de clasa C∞ pe ]− ε, ε[.

b) Presupunem ca exista δ > 0 astfel ca sistemul dat sa defineasca pe x, y, z ca functiide u pe intervalul ] − δ, δ[. Scazand ecuatiile 2 si 3 ale sistemului din prima ecuatie seobtine u4 − 3u = 0, de unde rezulta ca u ∈ 0, 3

√3, deci ]− δ, δ[⊆ 0, 3

√3, contradictie.

Problema 10.14 Sa se transforme ecuatia (1 − x2)y′′ − xy′ + ω2y = 0 prin schimbareade variabila x = cos t.

Solutie. Fie z(t) = y(cos t). Avem z′(t) = −y′(cos t) sin t de unde y′(cos t) = −z′(t)

sin t.

Derivand din nou aceasta relatie se obtine

−y′′(cos t) sin t = −z′′(t) sin t− z′(t) cos t

sin2 t

y′′(cos t) =z′′(t) sin t− z′(t) cos t

sin3 t.

Inlocuind ın ecuatie se obtine z′′(t) + ω2z(t) = 0.

Problema 10.15 Sa se transforme ecuatia y′′ − y′2 + 2xy′3 = 0 schimband rolul vari-abilelor.

Solutie. Avem din formula de derivare a functiei inverse

y′(x) =1

x′(y), y = y(x).

Derivand din nou aceasta relatie ın raport cu x se obtine

y′′(x) = − x′′(y)y′

(x′(y))2= − x′′(y)

(x′(y))3.

Inlocuind ın ecuatia data, obtinem ecuatia:

x′′ + x′ − 2x = 0.

Problema 10.16 Se da ecuatia cu derivate partiale

az′′x2 + 2bz′′xy + cz′′y2 = 0

unde a, b, c ∈ R si ac− b2 < 0. Sa se afle α, β ∈ R astfel ca prin schimbarea de variabileu = x+ αyv = x+ βz

ecuatia sa devina w′′uv = 0. Sa se rezolve ecuatia data.

Page 118: Teme si probleme pentru concursurile studentesti de matematica ...

Functii de mai multe variabile reale 111

Solutie. Fie z(x, y) = w(x+ αy, x+ βy). Avem

z′x = w′uu′x + w′vv

′x = w′u + w′v

z′y = w′uu′y + w′vv

′y = αw′u + βw′v

z′′x2 = w′′u2u′x + w′′uvv

′x + w′′uvu

′x + w′′v2v

′x = w′′u2 + 2w′′uv + w′′v2

z′′xy = w′′u2u′y + w′′uvv

′y + w′′uvu

′y + w′′v2v

′y = αw′′u2 + (α+ β)w′′uv + βw′′v2

z′′y2 = α(w′′u2u′y + w′′uvv

′y) + β(w′′uvu

′y + w′′v2v

′y) = α2w′′u2 + 2αβw′′uv + β2w′′v2 .

Inlocuind ın ecuatia data se obtine

(a+ 2bα+ cα2)w′′u2 + (2a+ 2b(α+ β) + cα2)w′′uv + (a+ 2bβ + cβ2)w′′v2 = 0.

Rezulta ca α si β trebuie sa fie radacini ale ecuatiei

cγ2 + 2bγ + a = 0.

Pentru aceste valori ecuatia devine w′′uv = 0 cu solutia

w(u, v) = ϕ(u) + ψ(v)

unde ϕ,ψ sunt functii arbitrare de clasa C2. Solutie ecuatiei date este

z(x, y) = ϕ(x+ γ1y) + ψ(x+ γ2y).

Problema 10.17 Sa se transforme ecuatia lui Laplace

∆z = z′′x2 + z′′y2 = 0

trecand la coordonate polare.

Solutie. Fie x = r cos ty = r sin t

si fie functia w definita prin

w(r, t) = z(r cos t, r sin t).

Din formula de derivare a functiilor compuse avem

w′r = z′x cos t+ z′y sin t

w′t = −z′xr sin t+ z′yr cos t.

Derivand din nou aceste relatii obtinem:

w′′r2 = z′′x2 cos2 t+ 2z′′xy sin t cos t+ z′′y2 sin2 t

w′′t2 = z′′x2 cos2 t− 2z′′xyr2 sin t cos t+ z′′y2r

2 cos2 t− z′xr cos t− z′yr sin t

de unde se obtine

w′′r2 +1

r2w′′t2 +

1

rw′r = 0.

Page 119: Teme si probleme pentru concursurile studentesti de matematica ...

112

Problema 10.18 Sa se rezolve ecuatia z′′x2 + 2z′′xy + z′′y2 = 0 cu schimbarea de variabile side functie

u = x+ yv = x− yw = xy − z.

Solutie. Fie

w(x+ y, x− y) = xy − z(x, y)

sau echivalent

w(u, v) =u2 − v2

2− z

(u+ v

2,u− v

2

).

Avem:

z(x, y) = xy − w(x+ y, x− y)

de unde obtinem

z′x = y − w′uu′x − w′vv′x = y − w′u − w′vz′y = x− w′uu′y − w′vv′y = x− w′u + w′v

si ın continuare

z′′x2 = −w′′u2 − 2w′′uv − w′′v2

z′′xy = 1− w′′u2 − w′′v2

z′′y2 = −w′′u2 + 2w′′uv − w′′v2 .

Inlocuind ın ecuatia initiala obtinem

−w′′u2 − 2w′′uv − w′′v2 + 2− 2w′′u2 + 2w′′v2 − w′′u2 + 2w′′uv − w′′v2 = 0

ceea ce este echivalent cu

w′′u2 =1

2.

Integrand succesiv ın raport cu u se obtine

w′u =1

2u+ ϕ(v)

w =1

4u2 + ϕ(v)u+ ψ(v).

Revenind la variabilele x si y se obtine solutia

z(x, y) = −(x− y)2

4− (x+ y)ϕ(x− y)− ψ(x− y)

unde ϕ si ψ sunt functii arbitrare de clasa C2.

Problema 10.19 Sa se arate ca functia f : R2 → R definita prin

f(x, y) =

xy sin1

x2 + y2, (x, y) 6= (0, 0)

0, (x, y) = (0, 0)

este diferentiabila pe R2, dar nu este de clasa C1.

Page 120: Teme si probleme pentru concursurile studentesti de matematica ...

Functii de mai multe variabile reale 113

Solutie. Avem f ′x(0, 0) = f ′y(0, 0) = 0, iar pentru (x, y) 6= (0, 0)

f ′x(x, y) = y sin1

x2 + y2− 2x2y

(x2 + y2)2cos

1

x2 + y2

f ′y(x, y) = x sin1

x2 + y2− 2xy2

(x2 + y2)2cos

1

x2 + y2.

Se arata ca lim(x,y)→(0,0)

f ′x(x, y) nu exista si ca df(0, 0) = 0.

Problema 10.20 Sa se determine functiile f de clasa C2 de forma:a) f(x, y) = ϕ(x2 + y2);b) f(x, y) = ϕ(y2 − x2);

c) f(x, y) = ϕ(yx

)unde ϕ ∈ C2(R), care verifica ecuatia lui Laplace.

Solutie. a) Se obtine ∆f = 4uϕ′′(u) + 4ϕ′(u), u(x, y) = x2 + y2. Notand ϕ′(u) = ψ(u)se obtine uψ′(u) + ψ(u) = 0 ceea ce se scrie (uψ(u))′ = 0 de unde rezulta ca uψ(u) = C1,

C1 ∈ R. Deci ϕ′(u) =C1

ucu solutia

ϕ(u) = C1 ln |u|+ C2, C2 ∈ R.

Prin urmaref(x, y) = C1 ln(x2 + y2) + C2, (x, y) ∈ R2 \ (0, 0).

b) Procedeu analog. Se obtine f(x, y) = C1(y2 − x2) + C2.

c) ∆f =x2 + y2

x4ϕ′′(u) + 2

y

x3ϕ′(u) = 0, u(x, y) =

y

x.

Se obtine ecuatia

(1 + u2)ψ′(u) + 2uψ(u) = 0, ψ(u) = ϕ′(u) ⇒

((1 + u2)ψ(u))′ = 0 ⇔ ψ(u) =C1

1 + u2⇒

ϕ(u) = C1arctg u+ C2, C1, C2 ∈ R.

Deci f(x, y) = C1arctgy

x+ C2, x 6= 0.

Problema 10.21 Sa se determine functia f ∈ C2(R2), care satisface relatia

a2f ′′x2 − b2f ′′y2 = 0, ab 6= 0,

efectuand schimbarea de variabile u = bx+ ayv = bx− ay

Solutie. Punand g(bx + ay, bx − ay) = f(x, y) se obtine g′′uv = 0, de unde rezultag(u, v) = ϕ(u) + ψ(v), adica

f(x, y) = ϕ(bx+ ay) + ψ(bx− ay),

cu ϕ,ψ functii de clasa C2(R).

Page 121: Teme si probleme pentru concursurile studentesti de matematica ...

114

Problema 10.22 Sa se arate ca functia f : Rn → R, definita prin

f(x) =

‖x‖2 sin1

‖x‖, x 6= 0

0, x = 0

unde ‖ · ‖ este norma euclidiana din Rn, este diferentiabila pe Rn, dar nu este de clasa C1.

Solutie. Se arata ca f ′xk(0) = 0, 1 ≤ k ≤ n si df(0) = 0.

Problema 10.23 Fie D o submultime nevida, deschisa si conexa a lui Rm si f : D → Rno functie cu proprietatea ca exista α > 1 si L > 0 astfel ca

‖f(x)− f(y)‖ ≤ L‖x− y‖α

pentru orice x, y ∈ D. Sa se arate ca f este constanta.

Solutie. Vom demonstra ca f este diferentiabila pe D si ca df(x) = 0 pentru oricex ∈ D. Fie x0 ∈ D. Avem

‖f(x)− f(x0)‖‖x− x0‖

≤ L‖x− x0‖α−1, ∀ x ∈ D \ x0.

De aici rezulta ca limx→x0

f(x)− f(x0)

‖x− x0‖= 0, deci df(x0) = 0. Cum D este conexa rezulta

ca f este constanta pe D.

Problema 10.24 Fie D ⊆ Rm o multime nevida, deschisa si convexa. Sa se arate ca dacaf : D → R este diferentiabila pe D si are derivate partiale marginite pe D, atunci f esteuniform continua pe D.

Solutie. Fie M > 0 astfel ca |f ′xk(x)| ≤ M pentru orice x ∈ D si orice k, 1 ≤ k ≤ m.Fie x, y ∈ D. Din teorema de medie rezulta ca

f(x)− f(y) = df(c)(x− y), c = x+ θ(y − x), 0 < θ < 1.

Avem

|f(x)− f(y)| =

∣∣∣∣∣n∑k=1

f ′xk(c)(xk − yk)

∣∣∣∣∣ ≤≤

n∑k=1

|f ′xk(c)| · |xk − yk| ≤Mn∑k=1

|xk − yk| ≤

≤ mM‖x− y‖, x = (x1, . . . , xm), y = (y1, . . . , ym).

Rezulta ca f este lipschitziana, deci este uniform continua.

Problema 10.25 Fie f : D → R, D ⊆ R2 si (x0, y0) ∈ intD. Presupunem ca existao vecinatate V ⊆ D a lui (x0, y0) astfel ıncat f sa fie continua ın (x0, y0) si sa admitaderivate partiale de ordinul unu pe V \ (x0, y0) cu proprietatea

lim(x,y)→(x0,y0)

f ′x(x, y) = a, lim(x,y)→(x0,y0)

f ′y(x, y) = b, a, b ∈ R.

Sa se arate ca f este diferentiabila ın (x0, y0).

Page 122: Teme si probleme pentru concursurile studentesti de matematica ...

Functii de mai multe variabile reale 115

Solutie. Exista o bila deschisa B cu centrul ın (x0, y0) astfel ca B ⊆ V ∩D. Pentru(x, y) ∈ B avem

|f(x, y)− f(x0, y0)− a(x− x0)− b(y − y0)| = |f(x, y)− f(x, y0)+

+f(x, y0)− f(x0, y0)− a(x− x0)− b(y − y0)| = |(y − y0)f ′y(x, c2)+

+(x− x0)f ′x(c1, y0)− a(x− x0)− b(y − y0)| ≤

≤ |x− x0| · |f ′x(c1, y0)− a|+ |y − y0| · |f ′x(x, c2)− b|

unde c1, c2 sunt punctele intermediare din teorema lui Lagrange aplicata functiilor f(·, y0)si f(x, ·).

Rezulta, notand ρ =√

(x− x0)2 + (y − y0)2, ca

|f(x, y)− f(x0, y0)− a(x− x0)− b(y − y0)|ρ

≤ |x− x0|ρ

|f ′x(c1, y0)− a|+

+|y − y0|

ρ|f ′x(x, c2)− b|, pentru (x, y) 6= (x0, y0).

Cum|x− x0|

ρ≤ 1,

|y − y0|ρ

≤ 1 rezulta ca membrul drept tinde la 0 cand (x, y) →

(x0, y0), deci f este diferentiabila ın (x0, y0) si

df(x0, y0)(h, k) = ah+ bk.

Problema 10.26 Sa se arate ca functia f : Rn → B(0, 1), unde B(0, 1) este bila deschisadin Rn definita prin relatia

f(x) =x√

1 + ‖x‖2,

pentru orice x ∈ Rn este o bijectie de clasa C1 cu inversa de clasa C1 (difeomorfism sautransformare regulata).

Solutie. Evident f ∈ C1(Rn,Rn). Se gaseste f−1 : B(0, 1)→ Rn,

f−1(x) =x√

1− ‖x‖2, x ∈ B(0, 1)

care este de clasa C1.

Problema 10.27 Fie f : R→ R o functie de clasa C2 si F : R2 → R

F (x, y) =

f(x)− f(y)

x− y, x 6= y

f ′(x), x = y.

Sa se arate ca F este functie de clasa C1.

Solutie. Pentru x 6= y avem

F ′x(x, y) =f ′(x)(x− y)− f(x) + f(y)

(x− y)2.

Fie a ∈ R. Avem

F ′x(a, a) = limx→a

F (x, a)− F (a, a)

x− a

Page 123: Teme si probleme pentru concursurile studentesti de matematica ...

116

= limx→a

f(x)− f(a)− (x− a)f ′(a)

(x− a)2

H= lim

x→a

f ′(x)− f ′(a)

2(x− a)=

1

2f ′′(a).

Aratam ca F ′x este continua ın (a, a) ceea ce revine la a arata ca

lim(x,y)→(a,a)

F ′x(x, y) =1

2f ′′(a).

Din formula lui Taylor aplicata functiei f obtinem

f(y)− f(x) = (y − x)f ′(x) +(y − x)2

2f ′′(x+ θ(y − x))

cu 0 < θ < 1, prin urmare

F ′x(x, y) =(y − x)2f ′′(x+ θ(y − x))

2(x− y)2=

1

2f ′′(x+ θ(y − x))

si de aici rezulta imediat ca

lim(x,y)→(a,a)

F ′x(x, y) =1

2f ′′(a).

Demonstratia este analoaga pentru existenta si continuitatea lui F ′y.

Problema 10.28 Se considera functia f : R2 → R de clasa C1 care satisface relatia

∂f

∂x(x, y) +

∂f

∂y(x, y) = f(x, y), ∀ (x, y) ∈ R.

1. Sa se arate ca functia g : R2 → R definita prin g(x, y) = f(x, x+ y) satisface relatia

∂g

∂x(x, y) = g(x, y), ∀ (x, y) ∈ R2.

2. Sa se arate ca exista o functie ϕ : R→ R de clasa C1 astfel ca

f(x, y) = ϕ(y − x)ex, ∀ (x, y) ∈ R2.

Solutie. 1. Fie x = u, x+ y = v. Avem

∂g

∂x(x, y) =

∂f

∂u(u, v) +

∂f

∂v(u, v) = f(u, v) = g(x, y).

2. Relatia∂g

∂x(x, y)− g(x, y) = 0, (x, y) ∈ R2 este echivalenta cu

∂x(g(x, y)e−x) = 0

de unde rezulta cag(x, y)e−x = ϕ(y) ⇔ g(x, y) = ϕ(y)ex

sau, tinand seama de definitia lui g,

f(x, y) = ϕ(y − x)ex, ∀ (x, y) ∈ R2

unde ϕ ∈ C1(R).

Page 124: Teme si probleme pentru concursurile studentesti de matematica ...

Functii de mai multe variabile reale 117

Problema 10.29 Fie D ⊆ Rn o multime deschisa si f : D → Rm o functie diferentiabilacu proprietatea ca exista M > 0 astfel ca

‖f(x)− f(y)‖ ≤M‖x− y‖, ∀ x, y ∈ D.

Sa se arate ca ‖df(x)‖ ≤M, ∀ x ∈ A.

Solutie. Fie a ∈ D arbitrar si T = df(a). Demonstram ca ‖T‖ ≤ M . Dindiferentiabilitatea lui f ın a rezulta ca exista o functie ω : D → Rn astfel ca

f(x)− f(a) = T (x− a) + ‖x− a‖ω(x)

cu limx→a

ω(x) = 0. Fie s ∈ Rm, ‖s‖ = 1. Cum D este deschisa rezulta ca exista δ > 0 astfelca

a+ ts ∈ D, ∀ t ∈ (0, δ).

Avemf(a+ ts)− f(a) = tT (s) + tω(a+ ts), t ∈ (0, δ).

Obtinem:

t‖T (s)‖ = ‖f(a+ ts)− f(a)− t(a+ ts)‖≤ ‖f(a+ ts)− f(a)‖+ t‖ω(a+ ts)‖≤ t(M + ‖ω(a+ ts)‖).

Facand acum t→ 0 ın relatia

‖T (s)‖ ≤M + ‖ω(a+ ts)‖

rezulta ca ‖T‖ ≤M .

Problema 10.30 Fie B = x ∈ Rn | ‖x‖ < 1 bila unitate din Rn si fie f : B → B ofunctie continua cu proprietatea ‖f(x)‖ < ‖x‖ pentru orice x ∈ B, x 6= 0.

Sa se demonstreze ca sirul (xm)m≥0 definit prin relatia

xm+1 = f(xm), m ≥ 0, x0 ∈ B, x0 6= 0n,

are limita 0n. (‖ · ‖ noteaza norma euclidiana din Rn, iar 0n vectorul nul din Rn).

Berkeley, 1991

Solutie. Din continuitata lui f si din relatia ‖f(x)‖ < ‖x‖, x ∈ B, x 6= 0n, rezulta caf(0n) = 0n. Daca exista k ∈ N astfel ca xk = 0n rezulta ca xm = 0n pentru orice m ≥ k,deci lim

m→∞xm = 0n. Sa presupunem ca xk 6= 0n pentru orice k ∈ N.

Fie (tm)m≥0 sirul de numere reale dat prin tm = ‖xm‖, m ≥ 0. Din relatia ‖f(xm)‖ <‖xm‖ rezulta ca (tm)m≥0 este strict descrescator si fiind marginit inferior de 0 este con-vergent.

Fie limm→∞

tm = t. Vom arata ca t = 0. Sa presupunem ca t > 0. Sirul (xm)m≥0 fiind

marginit are un subsir convergent (xmj )j≥0,

limj→∞

xmj = x, x ∈ B.

Avem ‖x‖ = limj→∞

‖xmj‖ = t, deci ‖f(x)‖ < t. Din continuitatea lui f obtinem

f(x) = limj→∞

f(xmj ) = limj→∞

xmj+1

si ‖xmj+1‖ ≥ t pentru orice j ∈ N, contradictie.

Page 125: Teme si probleme pentru concursurile studentesti de matematica ...

118

Problema 10.31 Sa se demonstreze inegalitatea

|(x+ y)e−x2−y2 | ≤ 1√

e, ∀ (x, y) ∈ R2.

Solutia 1. Trecand la coordonate polare

x = r cos t, y = r sin t, r ≥ 0, t ∈ [0, 2π],

inegalitatea devine:

|r(cos t+ sin t)e−r2 | ≤ 1√

e

sau echivalent ∣∣∣r√2 cos(t− π

4

)e−r

2∣∣∣ ≤ 1√

e.

Consideram functia f(r) = re−r2, r ∈ [0,∞). Avem

f ′(r) = e−r2(1− 2r2).

Radacina ecuatiei f ′(r) = 0 este r =

√2

2. Cum lim

r→∞f(r) = f(0) = 0 rezulta ca

0 ≤ f(r) ≤ f

(√2

2

)=

√2

2e−

12 .

Tinand seama ca ∣∣∣√2 cos(t− π

4

)∣∣∣ ≤ √2, t ∈ [0, 2π]

rezulta inegalitatea din enunt.

Egalitatea are loc pentru x = y =1

2si x = y = −1

2.

Solutia 2. Determinam extremele globale ale functiei

f : R2 → R, f(x, y) = (x+ y)e−x2−y2 .

Consideram compactul D(0, r) = (x, y) ∈ R2 | x2 + y2 ≤ r2, r > 1 si determinamextremele lui f pe D(0, r). Avem

f ′x(x, y) = e−x2−y2(1− 2x(x+ y)), f ′y(x, y) = e−x

2−y2(1− 2y(x+ y)).

Punctele stationare ale lui f sunt

(1

2,1

2

),

(−1

2,−1

2

)care sunt puncte de extrem

local

f

(1

2,1

2

)=

1√e, f

(−1

2,−1

2

)= − 1√

e.

Determinam extremele pe frontiera lui D(0, r) deci pentru x2 + y2 = r2. Avem

f(x, y) = (x+ y)e−r2.

Fie

L(x, y;λ) = (x+ y)e−r2 − λ(x2 + y2 − r2), λ ∈ R.

Page 126: Teme si probleme pentru concursurile studentesti de matematica ...

Functii de mai multe variabile reale 119

Punctele stationare conditionate se obtin din sistemul

L′x = 0, L′y = 0, x2 + y2 = 0.

Rezulta

x = y = ±r√

2

2.

Cum

f

(r√

2

2,r√

2

2

)= r√

2e−r2 ≤ 1√

e

rezulta ca

maxR2|f(x, y)| = 1√

e.

Problema 10.32 Sa se arate ca pentru orice x, y ∈ [0,∞) are loc inegalitatea

x2 + y2

4≤ ex+y−2.

Berkeley, 1993

Solutie. Fie x = r cos t, y = r sin t, r ≥ 0, t ∈[0,π

2

]. Inegalitatea este echivalenta cu

r2

4≤ er(cos t+sin t)−2.

Fie ın continuare r fixat. Avem

er(cos t+sin t)−2 = er√

2 sin(t+π4 )−2 ≥ er−2.

Inegalitatea are loc daca aratam ca er−2 ≥ r2

4.

Fie f : [0,∞)→ R, f(r) = er−2 − r2

4. Avem

f ′(r) = er−2 − r

2si f ′′(r) = er−2 − 1

2.

Ecuatia f ′′(r) = 0 are solutia unica r0 = 2− ln 2. Cum f ′ este strict descrescatoare pe[0, r0] si strict crescatoare pe [r0,+∞),

f ′(0) = e−2 > 0, f ′(r0) =ln 2− 1

2< 0, lim

r→∞f ′(r) = +∞

rezulta ca ecuatia f ′(r) = 0 are doua radacini r1 ∈ [0, r0) si r2 = 2 ∈ (r0,+∞). Tinandseama de semnul lui f ′ rezulta ca min f(r) = 0 = f(2). In inegalitatea initiala egalitateaare loc ın punctele (0, 2), (2, 0).

Problema 10.33 Fie f : R2 → R o functie de clasa C2 cu proprietatea ca

f ′′x2(x, y) + f ′′y2(x, y) > 0, ∀ (x, y) ∈ R2.

Sa se arate ca f nu admite maxime locale.

Berkeley, 1998

Page 127: Teme si probleme pentru concursurile studentesti de matematica ...

120

Solutie. Sa presupunem ca f admite un maxim local ın punctul (x0, y0) ∈ R2. Atuncid2f(x0, y0) este negativ semidefinita, adica

f ′′x2(x0, y0)u2 + 2f ′′xy(x0, y0)uv + f ′′y2(x0, y0)v2 ≤ 0

pentru orice (u, v) ∈ R2. Pentru (u, v) = (1, 0) obtinem f ′′x2(x0, y0) ≤ 0, iar pentru (u, v) =(0, 1) obtinem f ′′y2(x0, y0) ≤ 0, deci

f ′′x2(x0, y0) + f ′′y2(x0, y0) ≤ 0

contradictie.

Problema 10.34 Fie a, b, c, d, e, f, g ∈ R cu proprietatea b2 − 4ac < 0 si fie M multimeaperechilor (x, y) ∈ R2 cu proprietatea

ax2 + bxy + cy2 + dx3 + ex2y + fxy2 + gy3 = 0.

Demonstrati ca exista un numar r > 0 cu proprietatea ca ın multimea

D = (x, y) ∈ R2 | 0 < x2 + y2 < r2

nu exista nici un punct din M .

Putnam, 1970

Solutie. Evident M 6= ∅ deoarece (0, 0) ∈M .Fie F (x, y) = ax2 + bxy+ cy2 +dx3 + ex2y+fxy2 + gy3 pentru (x, y) ∈ R2. Vom arata

ca F admite un extrem local ın punctul (0, 0). Avem

F ′x(x, y) = 2ax+ by + 3dx2 + 2exy + fy2

F ′y(x, y) = bx+ 2cy + ex2 + 2fxy + 3gy2.

Evident (0, 0) este punct stationar al lui F .Pe de alta parte f ′′x2(0, 0) = 2a, f ′′xy(0, 0) = b si f ′′y2(0, 0) = 2c, deci:

d2f(0, 0)(h, k) = 2ah2 + 2bhk + 2ck2, (h, k) ∈ R2.

Conditia b2 − 4ac < 0 implica si a 6= 0, deci (0, 0) este punct de extrem local al lui F .Atunci exista o bila B(0, r) astfel ca

F (x, y) > 0 sau F (x, y) < 0 ın B(a, r) \ (0, 0).

In concluzie D = B(a, r) \ (0, 0).

Problema 10.35 Fie B = (x, y) ∈ R2 | x2 + y2 ≤ 1 si f : B → R o functie de clasaC1 cu proprietatea |f(x, y)| ≤ 1 pentru orice (x, y) ∈ B. Sa se arate ca exista un punct(x0, y0) interior lui B astfel ca(

∂f

∂x(x0, y0)

)2

+

(∂f

∂y(x0, y0)

)2

≤ 16.

Putnam, 1967

Page 128: Teme si probleme pentru concursurile studentesti de matematica ...

Functii de mai multe variabile reale 121

Solutie. Fie g : B → R, g(x, y) = f(x, y) + 2(x2 + y2) pentru orice (x, y) ∈ B. Pentru(x, y) cu proprietatea x2 + y2 = 1 avem g(x, y) ≥ 1 iar g(0, 0) = f(0, 0) ≤ 1. Cum geste continua pe compactul B ea este marginita si ısi atinge marginile pe B. Atunci geste constanta sau ısi atinge minimul ıntr-un punct interior (x0, y0) a lui B. Daca g esteconstanta atunci evident

f(x, y) = 1− 2(x2 + y2)

si (∂f

∂x(x, y)

)2

+

(∂f

∂y(x, y)

)2

= 16(x2 + y2) ≤ 16.

Daca g ısi atinge minimul ın (x0, y0) atunci

∂g

∂x(x0, y0) =

∂g

∂y(x0, y0) = 0

iar∂f

∂x(x0, y0) =

∂g

∂x(x0, y0)− 4x0 = −4x0

∂f

∂y(x0, y0) =

∂g

∂y(x0, y0)− 4y0 = −4y0.

Obtinem (∂f

∂x(x0, y0)

)2

+

(∂f

∂y(x0, y0)

)2

= 4(x20 + y2

0) < 16.

Page 129: Teme si probleme pentru concursurile studentesti de matematica ...

Capitolul 11

Siruri si serii de functii: seriiTaylor, serii Fourier

Definitii si rezultate

Siruri de functii

In cele ce urmeaza (X, d) va desemna un spatiu metric.

• Definitie. Fie A ⊂ (X, d) si F(A) := f | f : A → R. Se numeste sir de functii dinF(A) o aplicatie g : N→ F(A), g(n) = fn.Notatie: (fn)n∈N sau (fn).

Daca (fn) este un sir de functii si x ∈ A, atunci (fn(x))n∈N formeaza un sir nu-meric. Daca sirul (fn(x)) este convergent vom spune ca x este un punct de convergenta alsirului (fn). Totalitatea punctelor de convergenta ale sirului (fn) formeaza multimea deconvergenta a sirului (fn).

• Definitie. Fie A ⊂ (X, d) si fn, f : A→ R. Vom spune ca sirul de functii (fn) convergepunctual sau simplu la f pe multimea A daca sirul numeric (fn(x)) converge la f(x)pentru orice x ∈ A. Vom nota acest lucru prin fn

s→Af sau fn

s→ f.

Folosind scrierea analitica a convergentei unui sir numeric, putem da o formulareechivalenta. Asadar, sirul de functii (fn) converge punctual la f pe multimea A daca

∀x ∈ A,∀ε > 0,∃nx,ε ∈ N, ∀n ≥ nx,ε : |fn(x)− f(x)| < ε.

In general, convergenta punctuala nu conserva proprietatile sirului de functii. Acestaeste principalul motiv pentru introducerea conceptului de convergenta uniforma.

• Definitie. Fie A ⊂ (X, d) si fn, f : A→ R. Vom spune ca sirul de functii (fn) convergeuniform la f pe multimea A daca

∀ε > 0,∃nε ∈ N, ∀n ≥ nε,∀x ∈ A : |fn(x)− f(x)| < ε.

Vom nota acest lucru prin fnu→Af sau fn

u→ f.

Se observa cu usurinta faptul ca daca un sir de functii (fn) converge uniform la f peo multime A, atunci (fn) converge punctual la f pe A.

Prezentam ın continuare criterii care asigura convergenta uniforma.

Teorema. (criteriul cu supremum) Fie A ⊂ (X, d) si fn, f : A→ R. Sirul (fn) convergeuniform la f daca si numai daca

limn→∞

[supx∈A|fn(x)− f(x)|

]= 0.

122

Page 130: Teme si probleme pentru concursurile studentesti de matematica ...

Siruri si serii de functii: serii Taylor, serii Fourier 123

Un alt criteriu, de tip Cauchy, este prezentat ın urmatoarea teorema.

Teorema. (criteriul Cauchy) Fie A ⊂ (X, d) si fn, f : A → R. Sirul (fn) convergeuniform la f daca si numai daca

∀ε > 0,∃nε ∈ N, ∀n,m ≥ nε,∀x ∈ A : |fn(x)− fm(x)| < ε. (11.1)

Daca un sir (fn) satisface conditia (11.1), se va numi sir uniform fundamental sauuniform Cauchy.

Un alt criteriu ıl reprezinta cel al majorarii.

Teorema. (criteriul majorarii) Fie A ⊂ (X, d) si fn, f : A→ R. Daca exista un sir denumere pozitive (an) convergent la 0, astfel ıncat

|fn(x)− f(x)| < an,∀n ∈ N, ∀x ∈ A,

atunci fnu→Af.

Desi, dupa cum se poate observa din exemple simple, convergenta punctuala nu antre-neaza convergenta uniforma, ın anumite conditii speciale acest lucru are loc.

Teorema. (prima teorema a lui Dini) Fie A ⊂ (X, d) o multime compacta. Daca(fn) este un sir de functii continue pe A astfel ıncat, pentru orice x ∈ A, sirul numeric(fn(x)) este (des)crescator, iar (fn) converge punctual pe A la o functie continua f, atuncifn

u→Af.

Teorema. (a doua teorema a lui Dini) Fie fn : [a, b] → R un sir de functii(des)crescatoare pe [a, b] astfel ıncat (fn) converge punctual pe [a, b] la o functie continuaf, atunci fn

u→[a,b]

f.

In continuare vom prezenta principalele proprietati ale sirurilor uniform convergente,legate, dupa cum spuneam mai sus, de transferul unor proprietati ale termenilor siruluicatre functia limita.

Teorema. (transfer de marginire) Fie A ⊂ (X, d) si fn : A → R. Daca fn estemarginita pe A pentru orice n ∈ N si fn

u→Af, atunci f este marginita pe A.

Teorema. (transfer de existenta a limitei ıntr-un punct) Fie A ⊂ (X, d) si fn : A→ Rastfel ıncat fn

u→Af. Daca x este punct de acumulare pentru A si ∃ lim

x→xfn(x) pentru

fiecare n ∈ N, atunci ∃ limx→x

f(x) si, ın plus,

limx→x

f(x) = limn→∞

(limx→x

fn(x)).

Teorema. (transfer de continuitate) Fie A ⊂ (X, d) si fn : A→ R astfel ıncat fnu→Af .

Daca functiile fn sunt continue ın x ∈ A (respectiv pe A), atunci f este continua ın x(respectiv pe A).

Teorema. (transfer de integrabilitate) Fie fn : [a, b]→ R integrabile Riemann pe [a, b]astfel ıncat fn

u→[a,b]

f. Atunci f este integrabila Riemann pe [a, b] si

limn→∞

b∫a

fn(x)dx =

b∫a

f(x)dx.

Page 131: Teme si probleme pentru concursurile studentesti de matematica ...

124

Teorema. (transfer de derivabilitate) Fie I un interval marginit din R si fn : I → R unsir de functii derivabile pe I. Daca exista x ∈ I astfel ıncat sirul (fn(x)) sa fie convergentsi exista g : I → R astfel ıncat f ′n

u→Ig, atunci:

(i) exista o functie f : I → R astfel ıncat fnu→If ,

(ii) f este derivabila pe I, iar derivata sa este g, adica(limn→∞

fn

)′= lim

n→∞f ′n.

Observatie. Remarcam faptul ca uniforma convergenta ofera conditii suficientepentru transferul unor proprietati, nu ınsa si necesare. Exemplu: fn : [0, 1] → R,fn(x) =

nx

1 + n2x2. Atunci fn

s→[0,1]

0, ınsa fnu6→[0,1]

0. Totusi, atat fn, cat si f, sunt

continue pe [0, 1] (deci marginite, integrabile). De asemenea, fie sirul gn : [0, 1] → R,

gn(x) =ln(1 + n4x2)

2n. Se arata ca gn

s→[0,1]

0, g′ns→

[0,1]0, adica (gn) se poate deriva termen cu

termen. Totusi, g′nu6→[0,1]

0.

Serii de functii

• Definitie. Fie A ⊂ (X, d) si fn : A → R un sir de functii. Spunem ca seria∞∑n=1

fn este

convergenta punctual pe multimea A daca sirul sumelor partiale (Sn), cu Sn :=n∑k=1

fk,

este convergent punctual pe multimea A. Daca Sns→Af, atunci f se numeste suma seriei

de functii∞∑n=1

fn ın sensul convergentei punctuale si vom nota acest lucru prin

∞∑n=1

fns=Af sau

∞∑n=1

fns= f.

Daca seria∞∑n=1|fn| este convergenta punctual pe multimea A, spunem ca seria

∞∑n=1

fn

este absolut convergenta pe A.

Multimea tuturor punctelor din A ın care seria∞∑n=1

fn converge punctual se numeste

multimea de convergenta a seriei∞∑n=1

fn.

• Definitie. Fie A ⊂ (X, d) s i fn : A → R un sir de functii. Spunem ca seria∞∑n=1

fn este

convergent a uniform pe multimea A daca sirul sumelor partiale (Sn) este convergent

uniform pe multimea A. Daca Snu→Af, atunci f se numeste suma seriei de functii

∞∑n=1

fn

ın sensul convergentei uniforme si vom nota acest lucru prin

∞∑n=1

fnu=Af sau

∞∑n=1

fnu= f.

Prezentam ın continuare criterii care asigura convergenta uniforma a unei serii defunctii.

Page 132: Teme si probleme pentru concursurile studentesti de matematica ...

Siruri si serii de functii: serii Taylor, serii Fourier 125

Teorema. (criteriul lui Cauchy) Fie A ⊂ (X, d) si fn : A→ R. Seria de functii∞∑n=1

fn

converge uniform pe A daca si numai daca

∀ε > 0,∃nε ∈ N, ∀n ≥ nε,∀p ∈ N, ∀x ∈ A : |fn+1(x) + · · ·+ fn+p(x)| < ε.

Teorema. (criteriul lui Weierstrass) Fie A ⊂ (X, d) si fn : A→ R. Daca exista o serie

numerica convergenta cu termeni pozitivi∞∑n=1

an astfel ıncat

|fn(x)| ≤ an, ∀n ∈ N∗,∀x ∈ A,

atunci seria∞∑n=1

fn este uniform si absolut convergenta pe A.

Vom da ın continuare doua criterii de convergenta uniforma neabsoluta. Prezentammai ıntai definitia uniformei marginiri a unui sir de functii.

• Definitie. Fie A ⊂ (X, d) si fn : A→ R. Sirul (fn) se numeste uniform marginit (peA) daca exista M ∈ R astfel ıncat

|fn(x)| ≤M, ∀n ∈ N∗,∀x ∈ A.

Teorema. (criteriul lui Abel) Fie A ⊂ (X, d) si fn, gn : A→ R. Daca seria∞∑n=1

fn este

uniform convergenta pe A iar sirul (gn) este uniform marginit si monoton pentru fiecare

x ∈ A, atunci seria∞∑n=1

fngn este uniform convergenta pe A.

Teorema. (criteriul lui Dirichlet) Fie A ⊂ (X, d) si fn, gn : A→ R. Daca seria∞∑n=1

fn

are sirul sumelor partiale uniform marginit iar sirul (gn) este uniform descrescator (pentru

orice x ∈ A) si convergent uniform la 0, atunci seria∞∑n=1

fngn este uniform convergenta

pe A.

O aplicatie directa a criteriului lui Dirichlet o reprezinta criteriul lui Leibniz.

Teorema. (criteriul lui Leibniz) Fie A ⊂ (X, d) si fn : A → R. Daca sirul (fn)este uniform descrescator (pentru orice x ∈ A) si convergent uniform la 0, atunci seria∞∑n=1

(−1)nfn este uniform convergenta pe A.

De asemenea, drept consecinta a primei teoreme a lui Dini de la siruri de functii,obtinem urmatorul rezultat:

Teorema. (criteriul lui Dini) Fie A ⊂ (X, d) o multime compacta si fn : A→ R un sir

de functii continue cu proprietatea ca fn ≥ 0 pe A. Daca∞∑n=1

fn converge punctual pe A

la functia continua f, atunci∞∑n=1

fnu=Af.

In continuare, vom prezenta cele mai importante proprietati ale seriilor uniform con-vergente.

Teorema. (transfer de marginire) Fie A ⊂ (X, d) si fn : A → R. Daca∞∑n=1

fnu=Af si

fn sunt functii marginite pe A, atunci f este marginita pe A.

Page 133: Teme si probleme pentru concursurile studentesti de matematica ...

126

Teorema. (transfer de existenta a limitei ıntr-un punct) Fie A ⊂ (X, d) si fn, f : A→ R

astfel ıncat∞∑n=1

fnu=Af. Daca x este un punct de acumulare al multimii A si ∃ lim

x→xfn(x)

pentru orice n ∈ N∗, atunci ∃ limx→x

f(x) si, ın plus,

limx→x

f(x) =

∞∑n=1

(limx→x

fn(x)).

Teorema. (transfer de continuitate) Fie A ⊂ (X, d) si fn, f : A → R astfel ıncat∞∑n=1

fnu=Af. Daca functiile fn sunt continue ıntr-un punct x ∈ A (respectiv pe A), atunci

f este continua ın x (respectiv pe A).

Teorema. (transfer de integrabilitate) Fie fn : [a, b]→ R integrabile Riemann pe [a, b]

astfel ıncat∞∑n=1

fnu=

[a,b]f. Atunci f este integrabila Riemann pe [a, b] si

∞∑n=1

b∫a

fn(x)dx

=

b∫a

f(x)dx.

Teorema. (transfer de derivabilitate) Fie I un interval marginit din R si fn : I → R

un sir de functii derivabile pe I. Daca exista x ∈ I astfel ıncat seria∞∑n=1

fn(x) sa fie

convergenta si exista g : I → R astfel ıncat∞∑n=1

f ′nu=Ig, atunci:

(i) exista o functie f : I → R astfel ıncat∞∑n=1

fnu=If ,

(ii) f este derivabila pe I, iar derivata sa este g, adica( ∞∑n=1

fn

)′=∞∑n=1

f ′n.

Serii de puteri

• Definitie. Numim serie de puteri o serie de forma

a0 + a1x+ a2x2 + · · ·+ anx

n + · · · ,

unde (an) este un sir numeric. Numerele an se numesc coeficientii seriei.

Seriile de puteri sunt de fapt serii de functii, functiile avand o forma particulara:fn(x) = anx

n. Este de asteptat ca seriile de puteri sa aiba proprietati speciale, avand ınvedere proprietatile functiilor polinomiale. Observam mai ıntai ca orice serie de puteri areca punct de convergenta originea.

Teorema. (prima teorema a lui Abel) Daca o serie de puteri converge ıntr-un punctx0 6= 0, atunci seria este absolut convergenta ın orice punct x cu |x| < |x0|. Daca o seriede puteri diverge ıntr-un punct x1, atunci seria diverge ın orice punct x cu |x| ≥ |x1|.

Observatie. Analizand teorema anterioara observam ca exista:

• Serii convergente doar ın origine:∞∑n=0

n!xn;

Page 134: Teme si probleme pentru concursurile studentesti de matematica ...

Siruri si serii de functii: serii Taylor, serii Fourier 127

• Serii convergente pentru orice x real:∞∑n=0

1

n!xn;

• Serii de puteri pentru care exista r > 0 astfel ıncat seria converge ın orice x cu|x| < r si diverge pentru orice x cu |x| > r. Acest r este de fapt

r = sup

|x| :

∞∑n=0

anxn <∞

si se numeste raza de convergenta a seriei.

Teorema. Fie seria∞∑n=0

anxn si ρ = lim sup

n→∞n√|an|. Atunci raza de convergenta a seriei

este r =1

ρ(cu conventiile

1

∞= 0,

1

0=∞).

Corolar. Fie seria∞∑n=0

anxn. Daca exista lim

n→∞n√|an| = ρ, atunci r =

1

ρeste raza de

convergenta a seriei. Daca exista limn→∞

∣∣∣∣an+1

an

∣∣∣∣ = ρ, atunci r =1

ρeste raza de convergenta

a seriei.

Prezentam ın continuare rezultate privind convergenta uniforma a seriilor de puteri.

Teorema. Fie seria de puteri∞∑n=0

anxn avand raza de convergenta r > 0. Atunci seria

este uniform convergenta pe orice interval [−ρ, ρ] ⊂ (−r, r). Teorema. Functia suma a unei serii de puteri avand raza de convergenta r > 0 estecontinua pe (−r, r).

Teorema. (a doua teorema a lui Abel) Fie seria de puteri∞∑n=0

anxn avand raza de

convergenta r > 0. Daca seria converge ın r sau −r, atunci suma sa este continua ın r,respectiv −r.

Teorema. Seriile de puteri∞∑n=0

anxn,∞∑n=0

(n+ 1)an+1xn,∞∑n=0

ann+ 1

xn+1 au aceeasi raza

de convergenta.

Teorema. Daca seria∞∑n=0

anxn are raza de convergenta r > 0 si suma f pe (−r, r),

atuncix∫

0

f(t)dt =∞∑n=0

ann+ 1

xn+1, pentru |x| < r,

adica orice serie de puteri poate fi integrata termen cu termen pe orice interval [0, x], undex ∈ (−r, r).

Teorema. Daca seria de puteri∞∑n=0

anxn are raza de convergenta r > 0 si suma f pe

(−r, r), atunci f este derivabila pe (−r, r) si

f ′(x) = a1 + 2a2x+ 3a3x2 + · · ·+ nanx

n−1 + . . . pentru orice x ∈ (−r, r),

adica seria de puteri poate fi derivata termen cu termen pe intervalul deschis deconvergenta.

Page 135: Teme si probleme pentru concursurile studentesti de matematica ...

128

Mai mult, f este de clasa C∞ pe (−r, r),

f (k)(x) =∞∑n=k

n(n− 1) . . . (n− k + 1)anxn−k, pentru orice x ∈ (−r, r)

si f (k)(0) = k!ak, k = 0, 1, 2 . . .

Cu acest rezultat se poate demonstra unicitatea seriilor de puteri:

Teorema. Daca seriile de puteri∞∑n=0

anxn si

∞∑n=0

bnxn au aceeasi raza de convergenta

si aceeasi suma pe intervalul de convergenta, atunci an = bn, ∀n ∈ N.

Prezentam ın continuare un rezultat legat de operatii cu serii de puteri. Consideram

doua serii de puteri∞∑n=0

anxn si

∞∑n=0

bnxn si numarul real λ ∈ R. Se pot construi urmatoarele

serii:

• seria suma: (a0 + b0) + (a1 + b1)x+ · · ·+ (an + bn)xn + · · · ;

• seria produs a unei serii cu un scalar: λa0 + λa1x+ · · ·+ λanxn + · · · ;

• produsul Cauchy a doua serii:∞∑n=0

cnxn, unde cn =

n∑k=0

akbn−k, ∀n ∈ N.

Teorema. Fie doua serii de puteri∞∑n=0

anxn,∞∑n=0

bnxn avand razele de convergenta r1,

respectiv r2 si numarul real λ ∈ R. Atunci:(i) Raza de convergenta a seriei suma este cel putin egala cu minr1, r2 si

∞∑n=0

(an + bn)xn =

∞∑n=0

anxn +

∞∑n=0

bnxn.

(ii) Raza de convergenta a seriei∞∑n=0

(λan)xn este egala cu cea a seriei∞∑n=0

anxn pentru

orice λ 6= 0 si∞∑n=0

(λan)xn = λ

∞∑n=0

anxn.

(iii) Seria produs Cauchy are raza de convergenta r ≥ minr1, r2. In plus, sumaseriei produs este egala cu produsul sumelor celor doua serii pe intersectia multimilor lorde convergenta.

S-a observat mai sus ca, avand data o serie de puteri, se pot obtine unele informatiiprivind continuitatea, derivabilitatea sau integrabilitatea sumei sale. In practica problemase pune de multe ori invers: avand o functie pe un interval, ın ce conditii poate fi scrisaaceasta ca suma unei serii de puteri pe acel interval, sau pe o submultime a sa? Conformcelor aratate, suma f a unei serii de puteri poate fi scrisa sub forma

f(x) =∞∑n=0

f (n)(0)

n!xn.

Vom prezenta ın continuare un exemplu de serie de puteri cu importanta ın practica.Fie α ∈ R. Seria de puteri

1 +α

1!x+

α(α− 1)

2!x2 + · · ·+ α(α− 1) . . . (α− n+ 1)

n!xn + · · ·

Page 136: Teme si probleme pentru concursurile studentesti de matematica ...

Siruri si serii de functii: serii Taylor, serii Fourier 129

se numeste serie binomiala.Suma acestei serii este f(x) = (1 + x)α. Atribuind diferite valori particulare pentru α

se obtin sumele unor importante serii de puteri.

1. Pentru α = −1 se obtin sumele seriilor geometrice

1

1 + x= 1− x+ x2 − x3 + · · ·+ (−1)nxn + · · · ∀x ∈ (−1, 1)

1

1− x= 1 + x+ x2 + x3 + · · ·+ xn + · · · ∀x ∈ (−1, 1)

2. Pentru α = 12 avem, ∀x ∈ (−1, 1)

√1 + x = 1 +

1

2 · 1!x− 1

22 · 2!x2 + · · ·+ (−1)n−1 (2n− 3)!!

2n · n!xn + · · ·

3. Pentru α = −12 obtinem, ∀x ∈ (−1, 1)

1√1 + x

= 1− 1

2 · 1!x+

1 · 322 · 2!

x2 + · · ·+ (−1)n−1 (2n− 1)!!

2n · n!xn + · · ·

4. Trecand ın prima serie pe x ın x2 obtinem, ∀x ∈ (−1, 1)

1

1 + x2= 1− x2 + x4 − x6 + · · ·+ (−1)nx2n + · · ·

5. Prin integrare termen cu termen a seriei de mai sus avem, ∀x ∈ (−1, 1)

arctg x = x− x3

3+x5

5− x7

7+ · · ·+ (−1)n

x2n+1

2n+ 1+ · · ·

6. Trecand pe x ın x2 ın seria 3 si integrand obtinem, ∀x ∈ (−1, 1)

ln(x+√

1 + x2) = x− x3

3 · 2 · 1!+ · · ·+ (−1)n

(2n− 1)!!

(2n)!!x2n+1 + · · ·

7. In aceeasi serie 3, trecand pe x ın −x2 si integrand apoi termen cu termen obtinem,∀x ∈ (−1, 1)

arcsin x = x+x3

3 · 2+

1 · 35 · 4 · 2

x5 + · · ·+ (2n− 1)!!

(2n+ 1)(2n)!!x2n+1 + · · ·

Probleme

Problema 11.1 Sa se studieze convergenta simpla si uniforma a seriilor de functii pemultimile indicate:

(i)∞∑n=1

(π2− arctg(n2(1 + x2))

), x ∈ R.

(ii)∞∑n=1

ln(1 + nx)

nxn, x ≥ 2.

(iii)∞∑n=1

n2x2e−n2|x|, x ∈ R.

Page 137: Teme si probleme pentru concursurile studentesti de matematica ...

130

Solutie. (i) Din identitatea arctg x+ arctg1

x=π

2, rezulta:

∣∣∣π2− arctg(n2(1 + x2))

∣∣∣ = arctg1

n2(1 + x2)≤

≤ 1

n2(1 + x2)≤ 1

n2, ∀x ∈ R.

Seria∞∑n=1

1n2 este convergenta, deci ın baza criteriului lui Weierstrass, seria data este uniform

convergenta pe R.(ii) si (iii) Analog, cu acelasi criteriu:∣∣∣∣ ln(1 + nx)

nxn

∣∣∣∣ ≤ 1

xn−1≤ 1

2n−1,∀x ≥ 2

supx∈R

∣∣∣n2x2e−n2|x|∣∣∣ =

4

n2e2.

Problema 11.2 Sa se studieze convergenta simpla si uniforma pe R a seriilor de functii:

∞∑n=1

(−1)n−1

x2 + n2si

∞∑n=1

(−1)n−1x2

(1 + x2)n.

Ce se poate spune despre absolut convergenta lor?

Solutie. Ambele serii converg simplu (pentru x ∈ R fixat se aplica criteriul lui Leib-niz). Pentru a demonstra convergenta uniforma, demonstram ca resturile celor doua seriiconverg uniform la 0. Fie Rn(x)si respectiv Tn(x) resturile de ordin n ale celor doua serii.Atunci:

|Rn(x)| ≤ 1

x2 + n+ 1≤ 1

n+ 1,∀x ∈ R,

|Tn(x)| ≤ x2

(1 + x2)n+1=

x2

1 + C1n+1x

2 + C2n+1x

4 + · · ·+ x2n+2≤ 1

n+ 1, ∀x 6= 0.

Evident Tn(0) = 0, deci seriile sunt uniform convergente.In ceea ce priveste convergenta absoluta, prima serie nu converge nici simplu: ın x = 0

se obtine seria armonica. Seria∞∑n=1

x2

(1 + x2)neste punctual convergenta (serie geometrica

cu ratie subunitara), dar nu este uniform convergenta; daca notam restul de ordinul n cuPn, pentru x 6= 0, avem:

Pn(x) =x2

(1 + x2)n+1+

x2

(1 + x2)n+2+ · · ·

=x2

(1 + x2)n+1· 1 + x2

x2=

1

(1 + x2)n→ 1 pentru x→ 0.

Problema 11.3 Fie sirul un : [0,∞) → R, un(x) =ne−x + xe−n

x+ n, n ∈ N si fie An =

1∫0

un(x)dx,∀n ∈ N∗. Sa se studieze convergenta sirului un si sa se calculeze limn→∞

An.

Page 138: Teme si probleme pentru concursurile studentesti de matematica ...

Siruri si serii de functii: serii Taylor, serii Fourier 131

Solutie. Fie x ≥ 0, fixat.

limn→∞

un(x) = limn→∞

ne−x + xe−n

x+ n= e−x,

deci un converge punctual la f(x) = e−x, ∀x ≥ 0.Evaluam ın continuare

gn(x) = |un(x)− f(x)| =(

1− n

n+ x

)|e−n − e−x|, ∀x > 0,∀n ∈ N∗.

Daca x ∈ (n,∞) atunci:

e−x < e−n ⇒ |e−n − e−x| < e−n ⇒ gn(x) < e−n → 0.

Daca x ∈ [0, n] atunci

e−x ≥ e−n ⇒ |e−n − e−x| ≤ |e−n|+ |e−x| == e−n + e−x ≤ 2e−x,

deci

g(x) ≤ 2xe−x

x+ n,∀x ∈ [0, n].

Studiem acum variatia functiei [0, 1] 3 x 7→ 2xe−x

x+ n.

(2xe−x

x+ n

)′= 2

e−x

(x+ n)2· (−x2 − nx+ n),

deci functia2xe−x

x+ neste crescatoare pe

(0,√n2+4n−n

2

)si descrescatoare pe

(√n2+4n−n

2 , n)

,

deci

gn(x) ≤ 8n

(√n2 + 4n+ n)2

· e− 2n√

n2+4n+n → 0.

Deci un converge uniform la f .Sirul se poate integra termen cu termen:

limn→∞

An =

1∫0

f(x)dx =

1∫0

e−xdx =e− 1

e.

Problema 11.4 Fie fn(x) =n−1∑i=0

1

nf(x+ i

n

), unde f este o functie continua pe R. Sa se

arate ca (fn) converge uniform pe orice interval compact din R.

Solutie. Fie [a, b] ⊂ R si x ∈ [a, b]. Evaluam diferenta

fn(x)−x+1∫x

f(t)dt =

n−1∑i=0

1

nf

(x+

i

n

)−n−1∑i=0

x+ i+1n∫

x+ in

f(t)dt.

Page 139: Teme si probleme pentru concursurile studentesti de matematica ...

132

Cum functia f este continua pe R, aplicam o teorema de medie pe fiecare subintervalde forma

[x+ i

n , x+ i+1n

], i = 0, n− 1 pentru a gasi c = cx,i,n ∈

(x+ i

n , x+ i+1n

)astfel

ıncatx+ i+1

n∫x+ i

n

f(t)dt =1

nf(cx,i,n). Atunci

∣∣∣∣∣∣fn(x)−x+1∫x

f(t)dt

∣∣∣∣∣∣ ≤n−1∑i=0

1

n

∣∣∣∣f (x+i

n

)− f(cx,i,n)

∣∣∣∣ .Cum f este continua pe [a, b], este uniform continua pe acest interval (ın baza teoremei luiCantor). Atunci, pentru orice ε > 0 oarecare fixat,∃δ = δε > 0 astfel ıncat |f(x)− f(x′)| <ε pentru orice x, x′ ∈ [a, b] cu proprietatea ca |x− x′| < δ. Fie acum nε ∈ N astfel ıncat1n < δ pentru toti n ≥ nε. Atunci, deoarece x+ i

n , cx,i,n ∈ [a, b] si∣∣x+ i

n − cx,i,n∣∣ < 1

n < δ,obtinem ca

∣∣f (x+ in

)− f(cx,i,n)

∣∣ < ε, de unde∣∣∣∣∣∣fn(x)−x+1∫x

f(t)dt

∣∣∣∣∣∣ ≤n−1∑i=0

1

nε = ε.

In concluzie, fnu→

[a,b]g, unde g(x) =

x+1∫xf(t)dt.

Problema 11.5 Fie sirul fn :[0, π2

]→ R, definit prin relatia de recurenta:

f1(x) = x, fn+1(x) = sin (fn−1(x)) .

Sa se studieze convergenta punctuala si uniforma.

Solutie. Fie x ∈[0, π2

]fixat. Sirul fn(x) este descrescator:

fn+1(x) = sin (fn(x)) ≤ fn(x), ∀n ∈ N

si marginit:0 ≤ fn(x) ≤ x, ∀n ∈ N.

Deci sirul fn este punctual convergent.Fie f :

[0, π2

]→ R, f(x) = lim

n→∞fn(x). Trecand la limita ın relatia de recurenta, se obtine:

f(x) = sin (f(x)) , ∀x ∈[0, π2

]. Se demonstreaza simplu ca singura solutie a ecuatiei

t = sin t este t = 0, deci functia f este constanta zero.Studiem acum convergenta uniforma; pentru aceasta, trebuie calculat sup

x∈[0,π2 ]|fn(x)|.

Demonstram ın continuare ca funtia fn este crescatoare pentru orice n ∈ N si deci supre-mumul de mai sus este fn(π2 ). Sirul fn(π2 ) converge la zero (deoarece s-a demonstrat maisus ca lim

n→∞fn(x) = 0,∀x ∈

[0, π2

]) si deci fn este uniform convergent.

Demonstratia faptului ca fn sunt functii crescatoare se face prin inductie: f1 estecrescatoare; presupunem ca fk sunt crescatoare pentru orice 1 ≤ k ≤ n si demonstram cafn+1 este crescatoare. Fie 0 ≤ x < y ≤ π

2 ; atunci:

fn+1(x) = sin (fn(x)) ≤ sin (fn(y)) = fn+1(y),

deci fn+1 este functie crescatoare, ceea ce ıncheie demonstratia.

Page 140: Teme si probleme pentru concursurile studentesti de matematica ...

Siruri si serii de functii: serii Taylor, serii Fourier 133

Problema 11.6 Consideram sirul de functii Pn : [0, 1]→ R definit prin:

P0(x) = 0,

Pn+1(x) = Pn(x) +1

2(x− P 2

n(x)), daca n ∈ N∗.

Studiati convergenta uniforma a sirului (Pn) pe [0, 1].

Solutie. Se verifica imediat, prin inductie, ca aplicatiile Pn sunt polinomiale, decicontinue. In continuare, vom arata inductiv ca

(An) : 0 ≤ Pn(x) ≤√x, ∀n ∈ N, ∀x ∈ [0, 1].

Afirmatia este evident adevarata pentru n = 0. Presupunem acum ca (Ak) este adevarata.Atunci, deoarece x ≥ P 2

k (x) si Pk(x) ≥ 0, obtinem Pk+1(x) ≥ 0,∀x ∈ [0, 1]. De asemenea,observam ca

√x− Pk+1(x) =

(√x− Pk(x)

)(1− 1

2(√x+ Pk(x))

).

Cum√x+ Pk(x) ≤ 2

√x ≤ 2, deducem ca Pk+1(x) ≤

√x, ∀x ∈ [0, 1].

Folosind acum (An), se observa ca pentru orice x ∈ [0, 1] fixat, sirul (Pn(x)) estecrescator. Cum, pentru orice x ∈ [0, 1], (Pn(x)) este majorat de

√x, obtinem ca (Pn(x))

este convergent. Notam aceasta limita cu f(x). Atunci, f(x) va satisface pentru oricex ∈ [0, 1] relatia

f(x) = f(x) +1

2(x− f2(x)). (11.2)

Cum 0 ≤ f(x) ≤√x, avem din (11.2) ca f(x) =

√x, ∀x ∈ [0, 1]. Evident, f este continua.

Aplicand acum prima teorema a lui Dini se obtine ca Pnu→

[0,1]f.

Problema 11.7 1) Fie sirul de functii fn : R+ → R+ dat prin:

fn(x) =

(1− x

n

)n, daca x ∈ [0, n]

0, daca x > n.

Aratati ca fnu→R+

f, unde f este functia x 7→ e−x.

2) Aratati ca

I =

∞∫0

e−t2dt = lim

n→∞

√n∫

0

(1− t2

n

)ndt.

Deduceti de aici valoarea lui I.

Solutie. 1) Se observa ca, pentru orice n ∈ N∗, functia fn este continua, descrescatoaresi cu valori pozitive. Fixam acum x ∈ R+. Gasim atunci n0 ∈ N∗ astfel ıncat n0 ≥ x. Seobserva ca, pentru orice n ≥ n0,

fn(x) =(

1− x

n

)n,

deci sirul (fn(x))n≥n0 este convergent la e−x. Cum sirurile (fn(x))n≥1 si (fn(x))n≥n0 auaceeasi natura, eventual aceeasi limita ın caz de convergenta, deducem ca (fn) converge

Page 141: Teme si probleme pentru concursurile studentesti de matematica ...

134

punctual pe R+ la f. Fie A ∈ R∗+ fixat. Folosind a doua teorema a lui Dini obtinem ca

fnu→

[0,A]f. Utilizand inegalitatea

ln(1 + x) ≤ x, ∀x > −1, (11.3)

deducem ca

0 ≤ fn(x) ≤ f(x), ∀n ∈ N∗,∀x ∈ R+.

Fixam acum ε > 0. Exista atunci A > 0 astfel ıncat e−x < ε,∀x ≥ A. Asadar,

supx∈[A,∞)

|f(x)− fn(x)| = supx∈[A,∞)

(f(x)− fn(x))

≤ supx∈[A,∞)

f(x) ≤ ε. (11.4)

De asemenea, folosind convergenta uniforma a lui (fn) pe [0, A] (criteriul cu supremum),avem ca

limn→∞

(sup

x∈[0,A]|f(x)− fn(x)|

)= 0,

deci exista n1 ∈ N astfel ıncat, pentru orice n ≥ n1,

supx∈[0,A]

|f(x)− fn(x)| < ε. (11.5)

In concluzie, folosind (11.4) si (11.5), pentru orice ε > 0, exista n1 ∈ N astfel ıncat, pentruorice n ≥ n1, sup

x∈R+

|f(x)− fn(x)| < 2ε. Deci,

limn→∞

(supx∈R+

|f(x)− fn(x)|

)= 0.

Folosind din nou criteriul cu supremum, fnu→R+

f.

2) Folosind punctul 1), g(x) = e−x2

este limita uniforma pe R+ a sirului (gn)n∈N∗ deaplicatii continue de la R+ ın R+ definit prin:

gn(x) =

(1− x2

n

)n, daca x ∈ [0,

√n]

0, daca x >√n.

De asemenea, stim ca 0 ≤ gn(x) ≤ g(x), ∀n ∈ N∗,∀x ∈ R+.Se verifica usor convergenta integralelor improprii

I =

+∞∫0

g(x)dx si In =

+∞∫0

gn(x)dx =

√n∫

0

(1− x2

n

)ndx.

Pentru orice A ∈ R∗+, putem scrie

0 ≤ I − In ≤A∫0

(g(x)− gn(x)) dx+

+∞∫A

g(x)dx. (11.6)

Page 142: Teme si probleme pentru concursurile studentesti de matematica ...

Siruri si serii de functii: serii Taylor, serii Fourier 135

Fixam acum ε > 0. Cum+∞∫

0

g(x)dx este convergenta, exista A ∈ R∗+ astfel ıncat

+∞∫A

g(x)dx < ε. De asemenea, folosind convergenta uniforma a sirului (gn) pe [0, A],

aplicand transferul de integrabilitate, avem ca

limn→∞

A∫0

(g(x)− gn(x)) dx = 0.

Asadar, exista n2 ∈ N astfel ıncat, pentru orice n ≥ n2,A∫0

(g(x)− gn(x)) dx ≤ ε. In

concluzie, deducem din (11.6) ca In → I.

Facand schimbarea de variabilat√n7→ u, observam ca In =

√n

π2∫0

sin2n+1 udu. Folosind

faptul caπ2∫0

sin2n+1 udu =2 · 4 · · · · · (2n)

1 · 3 · · · · · (2n+ 1),

avem din formula lui Wallis ca I = limn→∞

In =

√π

2.

Problema 11.8 Studiati existenta limitei sirului xn =

(1

n

)n+

(2

n

)n+ · · ·+

(n− 1

n

)n.

Solutie. Fie uk : N∗ → R+ dat prin

uk(n) =

(1− k

n

)n, daca k ∈ 1, n− 1

0, daca k ≥ n.

Remarcam ca

xn =

∞∑k=1

uk(n).

De asemenea, pentru orice k ∈ N∗ fixat, limn→∞

uk(n) = e−k. Folosind acum (11.3), avem si

ca n ln

(1− k

n

)≤ −k, ∀k ∈ 1, n− 1, de unde

0 ≤ uk(n) ≤ e−n, ∀k ∈ N∗,∀n ∈ N∗.

Cum∞∑n=1

e−n este evident convergenta, aplicam criteriul lui Weierstrass si obtinem ca

∞∑k=1

uk converge uniform pe N∗. Cum +∞ este (singurul) punct de acumulare al multimii

N∗, aplicam acum transferul de existenta al limitei si deducem ca ∃ limn→∞

xn si

limn→∞

xn = limn→∞

∞∑k=1

uk(n) =

∞∑k=1

limn→∞

uk(n) =

∞∑k=1

e−k =1

e− 1.

Page 143: Teme si probleme pentru concursurile studentesti de matematica ...

136

Problema 11.9 Fie seria de functii∞∑n=1

(−1)nx2 + n

n2, x ∈R.

(a) Sa se studieze convergenta punctuala pentru orice x ∈ R si convergenta uniformape orice interval [a, b]. Este seria uniform convergenta pe R?

(b) Sa se studieze absolut convergenta pentru orice x ∈ R.(c) Sa se studieze continuitatea sumei seriei (acolo unde ea exista).(d) Se poate deriva seria termen cu termen ?

Solutie. (a) Fie x ∈ R; seriile numerice∞∑n=1

(−1)nx2

n2si∞∑n=1

(−1)n1

nsunt ambele con-

vergente, deci seria data este punctual convergenta pentru orice x ∈ R.

Studiem acum convergenta uniforma pe intervalul [a, b]. Seria∞∑n=1

(−1)nx2

n2este uni-

form convergenta pe [a, b]; pentru aceasta, aplicam criteriul lui Weierstrass de convergentauniforma: ∣∣∣∣(−1)n

x2

n2

∣∣∣∣ ≤ b2

n2, ∀x ∈ [a, b],

iar seria numerica∞∑n=1

1

n2este convergenta.

Seria nu este uniform convergenta pe R; pentru aceasta, fie A suma seriei∞∑n=1

(−1)n

n2si B

suma seriei∞∑n=1

(−1)n

n. Evident, seria data converge punctual la functia f(x) = Ax2 + B.

Fie sn(x) =n∑k=1

(−1)kx2 + k

k2. Calculam:

limn→∞

supx∈R|f(x)− sn(x)| = lim

n→∞supx∈R

x2

∣∣∣∣∣n∑k=1

(−1)k

k2−A

∣∣∣∣∣ =∞,

deci seria nu converge uniform pe R la f .

(b) Seria nu converge absolut pentru niciun x ∈ R deoarece seria∞∑n=1

x2 + n

n2este

divergenta (se poate compara cu seria armonica).(c) Evident, functia f (suma seriei) este continua pe R (desi seria nu converge uniform

pe R).(d) Seria nu verifica ipotezele teoremei de derivare termen pe R; seria derivatelor este

∞∑n=1

(−1)n2x

n2, serie care nu este uniform convergenta pe R:

limn→∞

supx∈R

∣∣∣∣∣ 2x

n∑k=1

(−1)k

k2− 2Ax

∣∣∣∣∣ =∞.

Totusi, seria derivatelor converge uniform pe orice compact din R. Seria data se poatederiva termen cu termen, egalitateax2

∑n≥1

(−1)n

n2+∑n≥1

(−1)n

n

′ = 2x∑n≥1

(−1)n

n2,∀x ∈ R

fiind adevarata.

Page 144: Teme si probleme pentru concursurile studentesti de matematica ...

Siruri si serii de functii: serii Taylor, serii Fourier 137

Problema 11.10 Fie seria∞∑n=1

nxn + x

n2 + 1.

(a) Pentru ce valori ale lui x ∈ R seria converge ?(b) Sa se studieze convergenta uniforma.(c) Se poate deriva seria termen cu termen ?

Solutie. (a) Fie x ∈ (−1, 1), fixat. Descompunem seria:

∞∑n=1

nxn + x

n2 + 1= x

∞∑n=1

1

n2 + 1+∞∑n=1

n

n2 + 1xn.

Prima serie este convergenta pentru orice x ∈ R. A doua serie converge absolut dacax ∈ (−1, 1); pentru demonstratie se poate aplica criteriul raportului:

limn→∞

∣∣∣∣∣n+1

(n+1)2+1n

n2+1

· xn+1

xn

∣∣∣∣∣ = |x| < 1.

Daca x = −1, seria converge (cele doua serii de mai sus sunt convergente) dar nu convergeabsolut.Daca x = 1, seria este divergenta (se poate compara cu seria armonica).Daca |x| > 1, seria diverge (se poate aplica criteriul necesar).In concluzie, seria converge daca si numai daca x ∈ [−1, 1).

(b) Aplicand criteriul lui Weierstrass, seria converge absolut si uniform pe orice com-pact [−r, r] ⊂ (−1, 1):

∞∑n=1

∣∣∣∣nxn + x

n2 + 1

∣∣∣∣ ≤ ∞∑n=1

nrn + r

n2 + 1, ∀ |x| ≤ r,

ultima serie (numerica) fiind convergenta.(c) Seria derivatelor este:

∞∑n=1

(nxn + x

n2 + 1

)′=∞∑n=1

(n2

n2 + 1xn−1 +

1

n2 + 1

)=

=∞∑n=1

1

n2 + 1+

∞∑n=1

n2

n2 + 1xn−1.

Seria derivatelor converge uniform pe orice interval [−r, r] ⊂ (−1, 1), deci seria se poatederiva termen cu termen.

Problema 11.11 Fie seria∞∑n=1

sin(2nx)

2n, x ∈ R.

(i) Sa se demostreze ca seria converge absolut si uniform.(ii) Sa se studieze derivabilitatea sumei seriei ın 0.

Solutie. (i) Evident.(ii) Fie f suma seriei si fie xm = π

2m+1 ; deoarece sin(2kxm) = 0, ∀k ≥ m+ 1, aplicandinegalitatea sinx ≥ 2

πx, ∀x ∈(0, π2

), rezulta:

f(xm) =

m∑k=1

sin(2kxm)

2k≥

m∑k=1

2

π· 2kxm

2k=

2

πmxm.

Page 145: Teme si probleme pentru concursurile studentesti de matematica ...

138

Functia nu este derivabila ın 0:

f(xm)− f(0)

xm≥ 2

πm −→∞, pentru m→∞.

Problema 11.12 Sa se calculeze suma seriei∞∑n=0

x4n

(4n)!si sa se generalizeze rezultatul la

seria∞∑n=0

1(pn)! , p ∈ N∗ fixat.

Solutie. Seria de puteri∞∑n=0

1(4n)!x

4n are raza de convergenta∞. Notand cu f suma se-

riei, problema revine la a calcula f(1). Pentru aceasta, vom obtine (prin derivari succesive)o ecuatie diferentiala satisfacuta de functia f . Avem:

f (4)(x) =∞∑n=1

(4n)(4n− 1)(4n− 2)(4n− 3)1

(4n)!x4n−4

=

∞∑n=1

1

(4n− 4)!x4n−4 = f(x).

Deci f este solutia problemei Cauchy:

f (4) = f, f(0) = 1, f ′(0) = f ′′(0) = f ′′′(0) = 0.

Polinomul caracteristic este λ4 − 1, iar radacinile sunt radacinile de ordinul 4 ale unitatii,

ζk = eikπ2 , k = 0, 1, 2, 3. Rezulta

f(x) = c1eζ1x + c2e

ζ2x + c3eζ3x + c4e

ζ4x,

constantele c1, c2, c3, c4 satisfacand sistemul:

c1ζ1 + c2ζ2 + c3ζ3 + c4ζ4 = 1,

c1ζm1 + c2ζ

m2 + c3ζ

m3 + c4ζ

m4 = 0, ∀m = 2, 3, 4.

Rezulta imediat c1 = c2 = c3 = c4 = 14 si deci

f(x) =1

4

(eζ1x + eζ2x + eζ3x + eζ4x

)=

1

4

(ex + e−x + 2 cosx

).

In concluzie, suma seriei∞∑n=0

x4n

(4n)!este 1

4(e+ e−1 + 2 cos 1).

Printr-un rationament analog (cu ζ1, ζ2, . . ., ζp radacinile de ordinul p ale unitatii), seobtine:

∞∑n=0

1

(pn)!xpn =

1

p

p∑k=1

eζkx si

∞∑n=0

1

(pn)!=

1

p

p∑k=1

eζk .

Problema 11.13 Fie a si b doua numere reale astfel ıncat a < b si fie f0 : [a, b] → R ofunctie continua. Fie sirul de functii fn : [a, b]→ R definit prin:

fn+1(x) =

∫ x

afn(t)dt.

Sa se studieze convergenta si sa se calculeze suma seriei∞∑n=1

fn.

Page 146: Teme si probleme pentru concursurile studentesti de matematica ...

Siruri si serii de functii: serii Taylor, serii Fourier 139

Solutie. Vom demonstra mai ıntai ca seria converge absolut si uniform pe [a, b].Fie M ≥ 0 astfel ıncat sup

x∈[a,b]|f0(x)| ≤ M . Se demonstreaza simplu prin inductie inegali-

tatea:

|f(x)| ≤M (x− a)n

n!, ∀x ∈ [a, b].

Rezulta deci:

supx∈[a,b]

|f(x)| ≤M (b− a)n

n!.

Seria∞∑n=1

M (b−a)n

n! este convergenta, deci seria∞∑n=1

fn converge absolut si uniform pe [a, b].

Rezulta ca functia suma S este continua.

Demonstram acum ca seria derivatelor∞∑n=1

f ′n este uniform convergenta. Este suficient sa

observam ca f ′n+1 = fn, deci se poate repeta rationamentul anterior. Rezulta ca seria∞∑n=1

fn se poate deriva termen cu termen:

S′ =∞∑n=1

f ′n =∞∑n=0

fn = f0 + S.

Problema Cauchy S′ = f0 + S, S(a) = 0, are solutia

S(x) = ex(∫ x

af0(t)e−t dt

), x ∈ [a, b],

ceea ce ıncheie demonstratia.

Problema 11.14 (Teorema lui Abel) Fie (an)n un sir de numere reale astfel ıncat seria

de puteri∞∑n=0

anxn are raza de convergenta R ∈ (0,∞). Presupunem ca seria

∞∑n=0

anRn

este convergenta. Sa se demonstreze ca seria∞∑n=0

anxn este uniform convergenta pe [0, R]

si

limn→∞

∞∑n=0

anxn =

∞∑n=0

anRn.

Solutie. Vom demonstra ca restul seriei∞∑n=0

anxn tinde uniform la 0 pe [0, R].

Fie, pentru orice K natural, restul de ordinul K al seriei∞∑n=0

anRn:

QK =

∞∑n=K

anRn −→ 0 pentru K →∞.

Page 147: Teme si probleme pentru concursurile studentesti de matematica ...

140

Pentru orice x ∈ [0, R) avem:

QK =∞∑n=K

anRn( xR

)n=

∞∑n=K

(Qn −Qn+1)( xR

)n=

=

∞∑n=K

Qn

( xR

)n−∞∑n=K

Qn+1

( xR

)n=

cele doua serii de mai sus converg

=

∞∑n=K

Qn

( xR

)n−

∞∑n=K+1

Qn

( xR

)n−1=

= QK

( xR

)K+

∞∑n=K+1

Qn

(( xR

)n−( xR

)n−1).

Fie ε > 0; atunci exista un rang m0 astfel ıncat |Qn| ≤ ε, ∀n ≥ m0. Rezulta ca pentru

orice K ≥ m0 si x ∈ [0, R), avem pentru restul de ordinul K al seriei∞∑n=0

anxn evaluarea:∣∣∣∣∣∣

∑n≥K

anxn

∣∣∣∣∣∣ ≤ ε+∑

n≥K+1

ε

∣∣∣∣( xR)n − ( xR)n−1∣∣∣∣ ≤ ε+ ε

( xR

)K≤ 2ε.

Evaluarea de mai sus este adevarata si pentru x = R, (prin alegerea lui m0) si deci seria∞∑n=0

anxn converge uniform pe [0, R]. Fie f suma seriei; atunci f este functie continua pe

[0, R] si deci limx→R

f(x) = f(R), ceea ce ıncheie demonstratia.

Problema 11.15 a) Fie (an)n∈N un sir crescator de numere reale strict pozitive, cu limita∞. Aratati ca functia f : R∗+ → R, data prin

f(x) =

∞∑n=0

(−1)ne−anx

este bine definita si continua.

b) Aratati ca∞∫0

f(x)dx este convergenta, avand valoarea∞∑n=0

(−1)n

an. Cazuri particulare:

i) an = n+ 1; ii) an = 2n+ 1.

Solutie. a) Fie un : R∗+ → R, un(x) = e−anx. Consideram asadar seria∞∑n=0

(−1)nun.

Fie x ∈ R∗+ arbitrar. Sirul (un(x)) este descrescator la 0, deci seria numerica∞∑n=0

(−1)nun(x) este convergenta conform criteriului Leibniz pentru serii numerice. Asadar,

∞∑n=0

(−1)nun converge punctual pe R∗+. Notam suma sa cu f. De asemenea, folosind mono-

tonia sirului (un(x)), se arata ca f(x) ≥ 0 si ca pentru orice n ∈ N∣∣∣∣∣∞∑k=n

(−1)kuk(x)

∣∣∣∣∣ ≤ un(x). (11.7)

Fie acum α ∈ R∗+ fixat. Cum∣∣e−anx − 0∣∣ = e−anx ≤ e−anα, ∀x ∈ [α,∞)

Page 148: Teme si probleme pentru concursurile studentesti de matematica ...

Siruri si serii de functii: serii Taylor, serii Fourier 141

si cum limn→∞

e−anα = 0, avem aplicand criteriul majorarii ca (un)u−→

[α,∞)0. Remarcand si ca

(un) este uniform descrescator la 0, putem folosi criteriul lui Leibniz pentru serii de functii

pentru a deduce ca∞∑n=0

(−1)nun converge uniform pe [α,∞) la f . Cum functiile (−1)nun

sunt continue, avem ca f e continua pe [α,∞). Folosind acum ca α este arbitrar, avem caf e continua pe R∗+.

b) Fie n ∈ N. Pentru orice x ∈ R+ avem ca

x∫0

e−antdt =1

an

(1− e−anx

).

Cum an > 0, rezulta ca integrala∞∫0

e−antdt este convergenta la1

an. Folosind (11.7) cu

n = 0, avem ca0 ≤ f(x) = |f(x)| ≤ e−a0x, ∀x ∈ R∗+.

Deducem de aici ca f, prelugita prin continuitate cu f(0) = 0, admite pe [0,∞) o integralaimproprie convergenta. De asemenea, folosind iarasi (11.7), avem ca pentru orice n ∈ N∗∣∣∣∣∣f(x)−

n−1∑k=0

(−1)nuk(x)

∣∣∣∣∣ ≤ e−anx.De aici, cum toate integralele care apar sunt convergente,∣∣∣∣∣∣

∞∫0

f(x)dx−n−1∑k=0

(−1)k

ak

∣∣∣∣∣∣ =

∣∣∣∣∣∣∞∫0

f(x)dx−n−1∑k=0

(−1)k∞∫0

uk(x)dx

∣∣∣∣∣∣=

∣∣∣∣∣∣∞∫0

(f(x)−

n−1∑k=0

(−1)kuk(x)

)dx

∣∣∣∣∣∣≤∞∫0

∣∣∣∣∣f(x)−n−1∑k=0

(−1)kuk(x)

∣∣∣∣∣ dx≤∞∫0

e−anxdx =1

an.

Trecand la limita pentru n→∞ ın relatia de mai sus, obtinem ca∞∫0

f(x)dx =∞∑n=0

(−1)n

an.

Ne ocupam acum de cele doua cazuri particulare specificate.i) Daca an = n+ 1, atunci

f(x) =∞∑n=0

(−1)ne−(n+1)x =e−x

1 + e−x.

Cum∞∫0

e−x

1 + e−xdx = ln 2, regasim egalitatea cunoscuta: ln 2 =

∞∑n=0

(−1)n

n+ 1.

ii) Daca an = 2n+ 1,

f(x) =∞∑n=0

(−1)ne−(2n+1)x =e−x

1 + e−2x.

Page 149: Teme si probleme pentru concursurile studentesti de matematica ...

142

Cum∞∫0

e−x

1 + e−2xdx =

π

4, regasim o alta egalitate cunoscuta:

π

4=∞∑n=0

(−1)n

2n+ 1.

Problema 11.16 Fie sirul de functii fn : R2 → R dat prin fn(x, y) =xn

1 + y2n.

1) Determinati Ω =

(x, y) ∈ R2 |

∞∑n=0

fn(x, y) este convergenta

.

2) Aratati ca suma seriei∞∑n=0

fn(x, y) =: f(x, y) este de clasa C1 pe Ω.

Solutie. Se observa cu usurinta faptul ca functiile fn sunt de clasa C∞ pe R2.

Fixam y ∈ R. Atunci∞∑n=0

xn

1 + y2neste o serie de puteri, avand raza de convergenta

ry = max1, y2. Cum pentru x = ±ry, termenul general al seriei numerice∞∑n=0

fn(x, y) nu

converge la 0, rezulta ca

Ω =

(x, y) ∈ R2 | |x| < max1, y2.

Remarcam ca Ω este o multime deschisa.Consideram pe R2 norma maxim ‖(x, y)‖∞ = max|x| , |y| si bila ınchisa asociata de

centru (x0, y0) si raza r > 0 : Br(x0, y0) = (x, y) ∈ R2 : max|x− x0| , |y − y0| ≤ r.Vom nota cu α := sup

(x,y)∈Br(x0,y0)|x| si β := inf

(x,y)∈Br(x0,y0)|y| . Atunci

α = max|x0 − r| , |x0 + r| si (11.8)

β =

0, daca |y0| ≤ rmin|y0 − r| , |y0 + r|, daca |y0| > r.

Daca (x0, y0) ∈ Ω, gasim r > 0 astfel ıncat Br(x0, y0) ⊂ Ω, deoarece Ω este deschisa. Inacest caz, avem ın plus ca |α| < 1 daca |y0| ≤ 1.

a) Continuitatea lui f : Ω→ R. Fie (x0, y0) ∈ Ω. Exista atunci Br(x0, y0) ⊂ Ω. Folosind(11.8), deducem ca pentru orice n ∈ N

sup(x,y)∈Br(x0,y0)

|fn(x, y)| ≤ αn

1 + β2n .

Cum (α, β) ∈ Br(x0, y0) ⊂ Ω, avem ca seria∞∑n=0

αn

1 + β2n converge, de unde aplicand cri-

teriul lui Weierstrass obtinem ca∞∑n=0

fn(x, y) converge uniform pe Br(x0, y0) la f. Deducem

ca f este continua pe Br(x0, y0), deci ın (x0, y0). De aici, f este continua pe Ω.

b) Existenta si continuitatea lui∂f

∂x: Ω → R. Pentru y ∈ R fixat, seria de puteri

∞∑n=0

xn

1 + y2npoate fi derivata termen cu termen pe intervalul (−ry, ry). Exista asadar

∂f

∂x:

Ω→ R,∂f

∂x(x, y) =

∞∑n=1

nxn−1

1 + y2n. Rationand ca la punctul a), se arata ca

∂f

∂xeste continua

pe Ω.

c) Existenta si continuitatea lui∂f

∂y: Ω→ R. Pentru orice n ∈ N, consideram aplicatiile

∂fn∂y

: R2 → R,∂fn∂y

(x, y) =−2nxny2n−1

(1 + y2n)2. Pentru y ∈ R fixat,

∞∑n=1

∂fn∂y

(x, y) este o serie

Page 150: Teme si probleme pentru concursurile studentesti de matematica ...

Siruri si serii de functii: serii Taylor, serii Fourier 143

de puteri avand raza de convergenta r′y = max

y2,

1

y2

. Deducem ca

∞∑n=1

∂fn∂y

converge

punctual pe

Ω′ =

(x, y) ∈ R2 : |x| < max

y2,

1

y2

.

Vom studia comportarea acestei serii pe Ω ⊂ Ω′.Fie (x0, y0) ∈ Ω si r > 0 astfel ıncat Br(x0, y0) ⊂ Ω. Consideram urmatoarele cazuri:(i) |y0| < 1. Notam B1 := (x, y) ∈ Br(x0, y0) : |y| ≤ 1. Atunci, pentru orice n ∈ N∗,

sup(x,y)∈B1

∣∣∣∣∂fn∂y (x, y)

∣∣∣∣ ≤ 2nαn.

Cum ın acest caz α < 1, avem ca∞∑n=1

2nαn este convergenta, de unde avem folosind criteriul

lui Weierstrass ca∞∑n=1

∂fn∂y

converge uniform pe B1.

(ii) |y0| = 1. Notam B2 :=

(x, y) ∈ Br(x0, y0) : y ∈[

12 ,

32

]. Atunci, folosind variatia

functiei u 7→ u

(1 + u2)de la R+ la R, deducem ca pentru orice (x, y) ∈ B2

|y|2n−1

(1 + y2n)2≤ 1

4 |y|≤ 1

2.

De aici, pentru orice n ∈ N∗,

sup(x,y)∈B2

∣∣∣∣∂fn∂y (x, y)

∣∣∣∣ ≤ nαn.Din nou, α < 1, deci

∞∑n=1

∂fn∂y

converge uniform pe B2.

(iii) |y0| > 1. Alegem r ∈ (0, 1) astfel ıncat Br(x0, y0) ⊂ Ω. Notam B3 :=(x, y) ∈ Br(x0, y0) : |y| ≥ 1 . Atunci, cum |y0| > 1 > r, avem ca β > 0. De aseme-

nea, folosind faptul ca functia u 7→ u

(1 + u2)este descrescatoare pentru u ≥ 1, obtinem ca

pentru orice (x, y) ∈ B3,

|y|2n−1

(1 + y2n)2=

1

|y|· y2n

(1 + y2n)2≤ 1

β· β2n

(1 + β2n)2.

De aici, pentru orice n ∈ N∗,

sup(x,y)∈B3

∣∣∣∣∂fn∂y (x, y)

∣∣∣∣ ≤ 2nαnβ2n−1

(1 + β2n)2.

Cum (α, β) ∈ Ω ⊂ Ω′, seria∞∑n=1

∂fn∂y

converge uniform pe B3.

Revenind, pentru orice (x0, y0) ∈ Ω, ın toate cele trei cazuri considerate, exista s ∈

(0, r) astfel ıncat∞∑n=1

∂fn∂y

(x0, ·) converge uniform pe Is := (y0 − s, y0 + s). Cum seria

∞∑n=0

fn(x0, ·) converge punctual pe Is, deducem existenta lui∂f

∂y(x0, y0) =

∞∑n=1

∂fn∂y

(x0, y0).

Page 151: Teme si probleme pentru concursurile studentesti de matematica ...

144

De asemenea, analizand cazurile de mai sus, pentru orice (x0, y0) ∈ Ω, exista r > 0 si

B ∈ B1, B2, B3 astfel ıncat (x0, y0) ∈ B ⊂ Br(x0, y0) ⊂ Ω si∞∑n=1

∂fn∂y

converge uniform

pe B. Cum toate aplicatiile∂fn∂y

sunt continue ın (x0, y0), deducem ca∂f

∂yeste continua

ın (x0, y0).

Serii Fourier

Serii Fourier ın spatii Hilbert

Fie X un spatiu liniar real si 〈·, ·〉 : X × X → R un produs scalar. Atunci aplicatia‖·‖ : X → R data prin ‖x‖ :=

√〈x, x〉, ∀x ∈ X defineste o norma pe X. De asemenea,

aplicatia d : X×X → R data prin d(x, y) := ‖x− y‖ , ∀x, y ∈ X defineste o metrica pe X.Spatiul cu produs scalar (X, 〈·, ·〉) se numeste spatiu Hilbert daca este complet ın raportcu metrica indusa de norma, adica daca orice sir Cauchy de puncte din X este convergentla un element din X. Spatiile Hilbert reprezinta generalizari (posibil infinit dimensionale)ale spatiilor euclidiene. Un exemplu de spatiu Hilbert infinit dimensional ıl reprezinta

`2 :=

(xn)n∈N ⊂ R |

∞∑n=0

x2n <∞

,

spatiul sirurilor de patrat sumabil, ın raport cu produsul scalar 〈·, ·〉 : `2× `2 → R dat prin

〈x, y〉 :=∞∑n=0

xnyn, x = (xn), y = (yn) ∈ `2.

Fie (X, 〈·, ·〉) un spatiu cu produs scalar. Doi vectori x, y ∈ X se numesc ortogonalidaca 〈x, y〉 = 0, iar acest lucru se mai noteaza si x ⊥ y. De asemenea, pentru x ∈ X,A ⊂ X, vom nota x ⊥ A daca x ⊥ y, ∀y ∈ A.Definitie. Un sistem ortonormat ın X este un sir de vectori (en)n∈N ⊂ X cu propri-

etatea ca 〈en, em〉 = δnm =

1, daca n = m0, daca n 6= m

.

Cu alte cuvinte, sistemul (en)n∈N este si ortogonal, adica en ⊥ em, ∀n 6= m. Inparticular, acesta este si liniar independent. In continuare, vom nota cu (en)n∈N un sistemortonormat ın spatiul cu produs scalar (X, 〈·, ·〉).Definitie. Pentru orice x ∈ X si n ∈ N, notam cu cn := 〈x, en〉 si numim numerele cn

coeficientii Fourier asociati lui x, iar seria∞∑n=0

cnen o numim seria Fourier asociata lui

x ın raport cu sistemul ortonormat (en)n∈N.O prima problema care apare este aceea a convergentei seriei Fourier asociate unui

vector x ∈ X. Are loc urmatoarea teorema.

Teorema. Fie (X, 〈·, ·〉) un spatiu cu produs scalar, x ∈ X, (cn)n∈N sirul coeficientilorFourier asociat lui x ın raport cu sistemul ortonormat (en)n∈N si (αn)n∈N ⊂ R un sirarbitrar de numere reale. Atunci:

(i)

∥∥∥∥x− n∑k=0

ckek

∥∥∥∥ ≤ ∥∥∥∥x− n∑k=0

αkek

∥∥∥∥ , ∀n ∈ N;

(ii)∞∑n=0|cn|2 ≤ ‖x‖2 (inegalitatea lui Bessel);

Page 152: Teme si probleme pentru concursurile studentesti de matematica ...

Siruri si serii de functii: serii Taylor, serii Fourier 145

(iii) limn→∞

cn = 0.

Definitie. Sistemul ortonormat (en)n∈N se numeste total daca ∀x ∈ X\0, ∃n ∈ N astfelıncat 〈x, en〉 6= 0. Sistemele ortonormate totale se mai numesc si baze ortonormate.

Cu alte cuvinte, (en) este total daca din x ⊥ en,∀n ∈ N rezulta x = 0. Urmatorulrezultat face legatura ıntre sistemele ortonormate totale si convergenta seriilor Fourier ınspatii Hilbert.

Teorema. Fie X un spatiu Hilbert si (en)n∈N un sistem ortonormat ın X. Urmatoareleafirmatii sunt echivalente:

(i) Sistemul ortonormat (en) este total;(ii) ∀x ∈ X, seria Fourier asociata lui x converge la x;

(iii) ∀x ∈ X, ‖x‖2 =∞∑n=0|cn|2 (egalitatea lui Parseval-Liapunov);

(iv) Aplicatia ϕ : X → `2, definita prin ϕ(x) := (cn), unde (cn) este sirul coeficientilorFourier asociat lui x, este un izomorfism de spatii normate.

Un alt exemplu important de spatiu Hilbert este L2[a, b]. Deoarece nu putem intra ıntoate detaliile de ordin tehnic ce implica notiuni legate de integrabilitatea ın sens Lebesgue,precizam doar faptul ca orice functie integrabila Riemann pe intervalul compact [a, b]este integrabila ın sens Lebesgue pe [a, b], iar valorile celor doua integrale coincid. Maimult, L2[a, b] este format din clasele de echivalenta de functii de patrat integrabil ın sensLebesgue, egale aproape peste tot (adica egale cu exceptia unor multimi de puncte demasura Lebesgue nula; de exemplu, doua multimi care difera printr-o multime cel multnumarabila de puncte sunt egale aproape peste tot). Teoria seriilor Fourier clasice admiteo dezvoltare foarte naturala ın L2[−π, π], spatiu Hilbert ın care sistemul trigonometric cla-

sic(

1√2π, 1√

πcosx, 1√

πsinx, . . . , 1√

πcosnx, 1√

πsinnx, . . .

)este un sistem ortonormat total,

deci toate rezultatele din teorema de mai sus sunt adevarate.

Serii Fourier clasice

O serie de formaa0

2+

∞∑n=1

(an cos

nπx

l+ bn sin

nπx

l

), (11.9)

unde a0, an, bn ∈ R (n = 1, 2, . . . ) si l > 0 se numeste serie trigonometrica de perioada2l.

Teorema. Daca presupunem ca seria (11.9) converge uniform pe intervalul [−l, l] catres, atunci s este continua pe [−l, l], iar coeficientii seriei (11.9) sunt dati de formulele

an =1

l

l∫−l

s(x) cosnπx

ldx, pentru n ∈ N, (11.10)

bn =1

l

l∫−l

s(x) sinnπx

ldx, pentru n ∈ N∗. (11.11)

In mod evident, ın locul intervalului [−l, l] putem considera orice alt interval de lungime2l.

Fie acum f : R → R o functie periodica de perioada 2l, integrabila ın sens propriusau absolut integrabila ın sens impropriu pe [−l, l]. Numerele an si bn date de formulele

Page 153: Teme si probleme pentru concursurile studentesti de matematica ...

146

(11.10) si (11.11), ın care ıl ınlocuim pe s cu f, se numesc coeficientii Fourier ai functieif, iar seria trigonometrica de perioada 2l formata cu acesti coeficienti se numeste seriaFourier asociata functiei f. Notam uneori seria Fourier asociata functiei f prin

f(x) ∼ a0

2+∞∑n=1

(an cos

nπx

l+ bn sin

nπx

l

).

Functia este dezvoltabila ın serie Fourier pe multimea A ⊂ R daca seria Fourierasociata converge pe aceasta multime catre f. Datorita periodicitatii lui f, coeficientii saiFourier nu depind de intervalul de lungime 2l pe care se calculeaza formulele ce ne dauacesti coeficienti. Mai remarcam faptul ca, pentru a studia posibilitatea dezvoltarii ın serieFourier pe R, este suficient sa facem acest studiu pe [−l, l].

Dam ın continuare cateva criterii utile pentru a studia posibilitatea dezvoltarii ın serieFourier.

Teorema. (Criteriul lui Dirichlet) Daca functia f este monotona pe portiuni ın in-tervalul [−l, l] si are ın acest interval cel mult un numar finit de puncte de discontinui-tate de speta I, atunci seria Fourier asociata converge ın fiecare punct x0 ∈ [−l, l] catref(x0 + 0) + f(x0 − 0)

2.

Teorema Daca functia f este derivabila sau derivabila pe portiuni ın intervalul[−l, l], atunci seria Fourier asociata converge ın fiecare punct x0 ∈ [−l, l] catref(x0 + 0) + f(x0 − 0)

2.

In cazul functiilor neperiodice, definite de exemplu pe un interval [−l, l], se considera

o functie ajutatoare f : R→ R, data prin f(x) :=

f(x), daca x ∈ (−l, l]f(l), daca x = −l pe intervalul

[−l, l] si prelungita prin periodicitate. Seria Fourier atasata lui f se va numi seria Fourieratasata lui f pe [−l, l]. Convergenta acestei serii revine la ındeplinirea de catre f a unui

criteriu ın acest sens pe [−l, l]. In punctele ±l, suma acestei serii va fif(−l + 0) + f(l − 0)

2.

Daca functia f : [−l, l]→ R este para, atunci coeficientii Fourier au valorile:

an =2

l

l∫0

f(x) cosnπx

ldx, pentru n ∈ N, bn = 0, pentru n ∈ N∗, (11.12)

adica seria Fourier asociata ei pe intervalul [−l, l] contine numai cosinusuri. Daca functiaf : [−l, l]→ R este impara, atunci coeficientii Fourier au valorile:

an = 0, pentru n ∈ N, bn = bn =2

l

l∫0

f(x) sinnπx

ldx, pentru n ∈ N∗, (11.13)

adica seria Fourier asociata ei pe intervalul [−l, l] contine numai sinusuri.Daca o functie f este definita numai pe intervalul [0, l] si ındeplineste conditiile de

dezvoltare ın serie Fourier ın interiorul acestui interval, cerandu-se dezvoltarea ei numaiın serie de cosinusuri sau sinusuri, se folosesc urmatoarele functii ajutatoare:

f1(x) =

f(x), daca x ∈ [0, l]f(−x), daca x ∈ [−l, 0)

si

f2(x) =

f(x), daca x ∈ [0, l]−f(−x), daca x ∈ [−l, 0)

.

Page 154: Teme si probleme pentru concursurile studentesti de matematica ...

Siruri si serii de functii: serii Taylor, serii Fourier 147

Egalitatea lui Parseval-Liapunov, valabila, dupa cum am vazut mai sus, ın spatiiHilbert generale, are urmatoarea forma particulara.

Teorema. (Egalitatea Parseval-Liapunov) Daca functia f : [−l, l]→ R este integrabilaın sens propriu sau de patrat integrabila ın sens impropriu pe [−l, l], atunci coeficientiiFourier asociati verifica egalitatea

a20

2+∞∑n=1

(a2n + b2n) =

1

l

l∫−l

f2(x)dx. (11.14)

Are loc de asemenea urmatorul rezultat.

Teorema. Daca sirurile (an) si (bn) formate cu coeficientii seriei (11.9) sunt monotonesi converg la 0, atunci seria este convergenta pentru orice x 6= 2kl, k ∈ Z si uniformconvergenta ın orice interval compact care nu contine puncte de acesta forma.

Prezentam mai jos cateva teoreme legate de derivarea si integrarea termen cu termena seriilor Fourier.

Teorema. Fie f o functie continua de perioada 2l admitand o derivata absolut inte-grabila (care poate sa nu existe ıntr-un numar finit de puncte dintr-un interval de lungimeegala cu perioada). Atunci seria Fourier a lui f ′ poate fi obtinuta din seria Fourier a luif prin derivare termen cu termen.

Teorema. Fie f o functie continua definita pe [−l, l] si admitand o derivata absolutintegrabila (care poate sa nu existe ıntr-un numar finit de puncte dintr-un interval delungime 2l). Atunci

f ′(x) ∼ c

2+∞∑n=1

[(nbn + (−1)nc) cos

nπx

l− nan sin

nπx

l

],

unde an si bn sunt coeficientii Fourier ai functiei f, iar constanta c este data de una dinegalitatile:

c =f(l)− f(−l)

lsau

c = limn→∞

[(−1)n+1nbn

]daca aceasta limita exista.

Teorema anterioara presupune continuitatea lui f si existenta unei derivate absolutintegrabile. In aplicatii se pot ıntalni cazuri ın care cunoastem numai seria Fourier a lui f.Atunci problema devine mai dificila: trebuie sa deducem dupa seria Fourier daca functiaeste derivabila si derivata integrabila si, daca raspunsul este afirmativ, sa formam seriaFourier a acestei derivate. Teorema urmatoare contribuie la rezolvarea acestei probleme.

Teorema. Fie seria (11.9). Daca seria

c

2+∞∑n=1

[(nbn + (−1)nc) cos

nπx

l− nan sin

nπx

l

](11.15)

unde c = limn→∞[(−1)n+1nbn

]este seria Fourier a unei anumite functii absolut integra-

bile ϕ, atunci seria (11.9) este seria Fourier a functiei

f(x) =

x∫0

ϕ(x)dx+a0

2+

∞∑n=1

an

Page 155: Teme si probleme pentru concursurile studentesti de matematica ...

148

continua pentru x ∈ (−l, l). In plus, (11.15) este convergenta catre f si avem f ′(x) = ϕ(x)ın orice punct de continuitate a lui ϕ.

Pentru functiile pare, respectiv impare, posibilitatea derivarii termen cu termen iaurmatoarele forme particulare.

Teorema. Daca functia f este continua pe [0, l], admite o derivata absolut integrabila sieste dezvoltabila ın serie Fourier de cosinusuri sau de sinusuri, atunci seria cosinusurilorpoate fi derivata ıntotdeauna termen cu termen, iar acest lucru este valabil si pentru seriade sinusuri daca f(0) = f(l) = 0.

Teorema. Fie f o functie continua pe [0, l], cu derivata absolut integrabila (care poatesa nu existe ın anumite puncte) si dezvoltabila ın serie Fourier de sinusuri

f(x) =

∞∑n=1

bn sinnπx

l, x ∈ (0, l).

Atunci

f ′(x) ∼ c

2+

∞∑n=1

[nbn − d+ (c+ d)(−1)n] cosnπx

l,

unde:

c =2(f(l)− f(0))

l, d =

2

lf(0) sau

c = − limn→∞

[2nb2n] , d = limn→∞

[(2n+ 1)b2n+1 − c] daca aceste limite exista.

Teorema. (Integrarea termen cu termen) Daca functia f : [−l, l]→ R este integrabilaın sens propriu sau absolut integrabila ın sens impropriu pe intervalul [−l, l] iar seriatrigonometrica (11.9) este seria Fourier asociata ei, atunci pentru orice interval [x′, x′′] ⊂[−l, l] avem

x′′∫x′

f(t)dt =

x′′∫x′

a0

2+∞∑n=1

x′′∫x′

[an cos

nπt

l+ bn sin

nπt

l

]dt.

Pentru a studia convergenta uniforma a seriilor Fourier, se aplica de obicei urmatoarelecriterii.

Teorema. (Criteriul Dirichlet-Jordan) Daca functia f : [−l, l] → R este continua sicu variatie marginita pe [−l, l] si satisface conditia f(−l) = f(l), atunci seria Fourierasociata ei converge uniform catre f pe acest interval.

Corolar. Daca functia f : [−l, l] → R este derivabila cu derivata integrabila pe [−l, l]si satisface conditia f(−l) = f(l), atunci seria Fourier asociata ei converge absolut siuniform catre f pe acest interval.

Probleme

Problema 11.17 Sa se determine:(a) Seria Fourier asociata functiei f : R→ R, f(x) = x sinx pe intervalul [0, π];(b) Seria Fourier numai de cosinusuri si seria Fourier numai de sinusuri asociate functiei

f pe acelasi interval;(c) Multimea pe care fiecare din aceste trei serii converge catre f.

Page 156: Teme si probleme pentru concursurile studentesti de matematica ...

Siruri si serii de functii: serii Taylor, serii Fourier 149

Solutie. Deoarece functia este continua pe R, ea este integrabila pe orice intervalcompact, deci are sens problema determinarii seriei Fourier asociate ei pe un interval.

(a) Avem

an =2

π

π∫0

x sinx cos 2nx dx =

=1

π

π∫0

x[sin(2n+ 1)x− sin(2n− 1)x] dx = − 2

4n2 − 1, n ∈ N

si

bn =2

π

π∫0

x sinx sin 2nx dx =

=1

π

π∫0

x[cos(2n− 1)x− cos(2n+ 1)x] dx = − 16n

π(4n2 − 1)2, n ∈ N∗.

Atunci seria Fourier asociata functiei f pe [0, π] este

1− 2

π

∞∑n=1

4n2 − 1cos 2nx+

8n

(4n2 − 1)2sin 2nx

]. (11.16)

(b) Pentru a determina coeficientii Fourier ai seriei numai de cosinusuri asociate functieif pe [0, π] vom aplica formulele coeficientilor Fourier, ın care luam l = π. Avem

an =2

π

π∫0

x sinx cosnx dx =1

π

π∫0

x[sin(n+ 1)x− sin(n− 1)x] dx.

Integrand prin parti, obtinem an = (−1)n+1 2

n2 − 1, pentru n = 0, 2, 3, . . . , si a1 = −1

2.

Prin urmare, seria ceruta este

1− 1

2cosx+ 2

∞∑n=2

(−1)n+1

n2 − 1cosnx. (11.17)

Asemanator, pentru seria numai de sinusuri corespunzatoare functiei f pe [0, π] obtinem

bn = − 16k

π(4k2 − 1), daca n = 2k si bn = 0, daca n = 2k + 1. Atunci seria ceruta este

π

2sinx− 16

π

∞∑n=1

k

(4k2 − 1)sin 2kx. (11.18)

(c) Functia considerata verifica conditiile Dirichlet-Jordan pe [−π, π] si este para. Rezultaatunci ca seria (11.17) converge uniform catre f pe acest interval. Dar functia f verificaconditiile Corolarului de mai sus pe [0, π], deci seriile (11.16) si (11.18) converg uniformpe acest interval catre f .

Deoarece functiile care apar ın seria (11.16) sunt periodice de perioada π, rezulta caaceasta serie defineste pe R o functie periodica de perioada π, catre care seria converge

Page 157: Teme si probleme pentru concursurile studentesti de matematica ...

150

uniform pe R. In virtutea paritatii lui f deducem ca si seria (11.16) converge uniform catref pe intervalul [−π, π]. Aceste trei serii mai converg catre f si ın punctele de forma kπ cuk ∈ Z, deoarece ın aceste puncte valoarea 0 a functiei se repeta periodic. Asadar, seriile(11.16) si (11.17) converg catre f pe multimea [−π, π] ∪ kπ | k ∈ Z, iar seria (11.18) pemultimea [0, π] ∪ kπ | k ∈ Z.

Problema 11.18 1) Verificati:

x2 =π2

3+ 4

∞∑n=1

(−1)ncosnx

n2, ∀x ∈ [−π, π], (11.19)

x = 2∞∑n=1

(−1)n+1 sinnx

n, ∀x ∈ (−π, π). (11.20)

2) Fie a ∈ R∗+. Aratati ca exista o aplicatie f : [−π, π] → R, de clasa C2, astfel

ıncat: f(x) =∞∑n=1

(−1)2 cosnx

n2 + a2. Formati o ecuatie diferentiala satisfacuta de f, si deduceti

expresia lui f(x).

3) Existenta si calculul lui∞∑n=1

(−1)nn sinnx

n2 + a2, x ∈ (−π, π).

Solutie. 1) Fie g : R→ R, de perioada 2π, definita prin g(x) = x2 pentru x ∈ [−π, π].Conform Criteriului Dirichlet-Jordan, seria Fourier asociata converge uniform catre g peR. Cum g este para, seria Fourier va fi numai de cosinusuri. Folosind formulele pentru

coeficientii Fourier an si bn, obtinem a0 = 2π3 si an =

4(−1)n

n2, n = 1, 2, . . . , de unde se

deduce (11.19).Fie acum h : R→ R, de perioada 2π, data prin h(x) = x daca x ∈ (−π, π) si h(π) = 0.

Cum h este local integrabila pe R, iar pentru orice x 6= (2k+ 1)π, k ∈ Z, f este derivabila,seria sa Fourier converge si are drept suma h. Seria Fourier va fi numai de sinusuri, iar

bn =2(−1)n

n2, n = 1, 2, . . . , de unde deducem (11.20).

2) Avem ∣∣∣∣(−1)ncosnx

n2 + a2

∣∣∣∣ ≤ 1

n2 + a2,∀x ∈ R.

Cum∞∑n=1

1

n2 + a2converge, deducem ca f exista ca fiind suma unei serii de functii uniform

convergente pe [−π, π], si ca f este continua pe [−π, π].

Fie u : [−π, π]→ R definita prin f(x) =x2

4− π2

12+ a2u(x), adica

u(x) =∞∑n=1

un(x), unde u(x) = (−1)n+1 cosnx

n2(n2 + a2).

Se verifica faptul ca seriile∞∑n=1

un,∞∑n=1

u′n,∞∑n=1

u′′n converg uniform pe [−π, π], ceea ce

antreneaza faptul ca u este de clasa C2, precum si relatiile:

u′(x) =∞∑n=1

(−1)nsinnx

n(n2 + a2);

u′′(x) =

∞∑n=1

(−1)ncos(nx)

n2 + a2= f(x).

Page 158: Teme si probleme pentru concursurile studentesti de matematica ...

Siruri si serii de functii: serii Taylor, serii Fourier 151

Rezulta ca f este de clasa C2, si ca f este solutia pe [−π, π] a ecuatiei diferentiale

y′′ − a2y =1

2. (11.21)

O solutie particulara a ecuatiei (11.21) este x 7→ − 1

2a2. Tinand cont de paritatea

lui f, deducem ca exista o constanta A astfel ıncat, pentru orice x ∈ [−π, π], f(x) =

A ch ax − 1

2a2. Cum u′(π) = 0, deducem ca f ′(π) =

π

2. Totodata, f ′(π) = Aa sh aπ. De

aici, A =π

2a sh aπsi

∞∑n=1

(−1)ncosnx

n2 + a2=π

a

ch ax

sh aπ− 1

2a2,∀x ∈ (−π, π).

3) Rezulta ca f ′(x) = π2

sh axsh aπ . Pe de alta parte, f ′(x) =

x

2+ a2u′(x). Restrangandu-ne

la (−π, π) si folosind 2), avem

f ′(x) =

∞∑n=1

(−1)n+1 sinnx

n

(1− 1

n2 + a2

).

In final,∞∑n=1

(−1)n+1n sinnx

n2 + a2=π

2

sh ax

sh aπ.

Problema 11.19 Sa se demonstreze ca, pentru orice x ∈[−π

4 ,π4

], are loc egalitatea:

secx =4

πln(1 +

√2)

+8

π

∞∑n=1

[ln(1 +

√2) + 2

2n−1∑k=0

(−1)k+1

2k + 1sin(2k + 1)

π

4

]cos 4nx.

Solutie. Functia f :[−π

4 ,π4

]→ R, f(x) = secx este para si verifica toate conditiile

din criteriul lui Dirichlet, decieste dezvoltabila ın serie Fourier numai de cosinusuri peacest interval. Avem

a0 =8

π

π4∫0

secx dx, an =8

π

π4∫0

secx cos 4nx dx, ∀n ∈ N.

Folosind schimbarea de variabila tg x = t, deducem a0 =8

πln(1 +

√2). Pentru calculul lui

an, folosim identitatea

cos 4nx

cosx= 2 cos(4n− 1)x− 2 cos(4n− 3)x+

cos 4(n− 1)x

cosx,

de unde deducem ca

an =16

π

[1

4n− 1sin(4n− 1)

π

4− 1

4n− 3sin(4n− 3)

π

4

]+ an−1.

Adunand aceste relatii, obtinem ca an =16

π

2n−1∑k=0

(−1)k+1

2k + 1sin(2k + 1)

π

4+ a0. De aici, con-

cluzia.

Page 159: Teme si probleme pentru concursurile studentesti de matematica ...

152

Problema 11.20 1) Sa se arate ca functia f(x) =1

2 + cosxeste dezvoltabila ın serie

Fourier convergenta pe toata axa reala si sa se deduca aceasta dezvoltare.

2) Deduceti valoarea integraleiπ∫0

cosnx

2 + cosxdx.

Solutie. 1) Functia f verifica conditiile criteriului Dirichlet. Se observa ca f este ofunctie para, de perioada 2π. In consecinta, functia f este dezvoltabila ın serie Fourierdecosinusuri pe toata axa reala.

Fie

f(x) =a0

2+

∞∑n=1

an cosnx. (11.22)

Avem

a0 =2

π

π∫0

dx

2 + cosx=

2√

3

3.

Inmultind ambii membri ai relatiei (11.22) cu 2(2 + cosx) obtinem

2 = 2a0 + a0 cosx+ 4

∞∑n=1

an cosnx+

∞∑n=1

2an cosx cosnx

= 2a0 + a0 cosx+ 4

∞∑n=1

an cosnx+

∞∑n=1

an[cos(n+ 1)x+ cos(n− 1)x.

Cum functia g(x) = 2 este para, este dezvoltabila ın serie Fourier de cosinusuri pe toataaxa reala. Tinand seama de egalitatea a doua serii Fourier, avem

2 = 2a0 + a1

0 = a0 + 4a1 + a2

0 = 4ak + ak+1 + ak−1, k = 1, 2, . . .

Sirul (an) verifica deci recurenta liniara ak = −4ak−1−ak−2, cu a0 =2√

3

3si a1 =

6− 4√

3

3.

Se deduce ca ak =2√

3

3(√

3− 2)k, k = 1, 2, . . .

Deci

f(x) =

√3

3+

2√

3

3

∞∑n=1

(√

3− 2)n cosnx.

2) Deoarece an =2

π

π∫0

cosnx

2 + cosxdx, rezulta ca

π∫0

cosnx

2 + cosxdx =

√3

3π(√

3− 2)n.

Problema 11.21 (Formula lui Poisson) Sa se arate ca functia f : R → R, f(x) = 1 −2α cosx+ α2, unde |α| 6= 1, este strict pozitiva pentru orice x ∈ R, iar functia

g : R→ R, g(x) =1− α2

1− 2α cosx+ α2, |α| < 1

este dezvoltabila ın serie Fourier pe R; sa se determine apoi dezvoltarea functiei g.

Page 160: Teme si probleme pentru concursurile studentesti de matematica ...

Siruri si serii de functii: serii Taylor, serii Fourier 153

Solutie. Folosind inegalitatea

1− 2α cosx+ α2 ≥ (1− |α|2), ∀x ∈ R,

si faptul ca |α| 6= 1, obtinem ca f > 0 pe R. Functia g este periodica de perioada 2π,derivabila cu derivata continua pe R. In consecinta, verifica conditiile din criteriul luiDirichlet, deci este dezvoltabila ın serie Fourier pe R. Seria Fourier converge chiar uniformpe R catre g. Cum g este para, seria Fourier asociata ei va fi numai de cosinusuri:

g(x) =a0

2+∞∑n=1

an cosnx. (11.23)

Intrucat integralele care apar ın calculul coeficientilor Fourier implica unele calcule destulde complicate, vom evita acest calcul, dupa cum se va vedea ın continuare. Amplificamrelatia (11.23) cu 2(1− 2α cosx+ α2), si obtinem

2(1− α2) = a0(1− 2α cosx+ α2) + 2(1 + α2)

∞∑n=1

an cosnx

−2α∞∑n=1

an[cos(n+ 1)x− cos(n− 1)x].

De aici deducem

2(1− α2) = a0(1 + α2)− 2αa1 + 2∞∑n=1

[(1 + α2)an − αan−1 − αan+1] cosnx. (11.24)

Functia constanta h(x) = 2(1 − α2) poate fi considerata ca fiind periodica de perioada2π, para, deci dezvoltabila ın serie de cosinusuri pe R ın mod unic. Deci, (11.24) e chiardezvoltarea functiei h ın serie Fourier pe R. Dar aceasta functie are toti coeficientii Fouriernuli, ın afara de primul care este 4(1− α2). Prin urmare,

a0(1 + α2)− 2αa1 = 4(1− α2) si (11.25)

(1 + α2)an − αan−1 − αan+1 = 0, ∀n ∈ N∗.

Coeficientul a0 al functiei g se calculeaza direct, fiind

a0 =2

π

π∫0

1− α2

1− 2α cosx+ α2dx = 2.

Folosind (11.25), deducem ca a1 = 2α si, prin recurenta, an = 2αn. In concluzie,

1− α2

1− 2α cosx+ α2= 1 + 2

∞∑n=1

αn cosnx, ∀x ∈ R. (11.26)

Ultima formula se mai numeste formula lui Poisson.

Problema 11.22 (Integrala Poisson) Fie α ∈ (0, 1), f o functie continua de perioada 2π,an si bn coeficientii sai Fourier. Notam prin

f(x, r) =a0

2+

∞∑n=1

αn (an cosnx+ bn sinnx) .

Page 161: Teme si probleme pentru concursurile studentesti de matematica ...

154

Sa se arate ca seriaa0

2+∞∑n=1

αn (an cosnx+ bn sinnx) (11.27)

este absolut convergenta si ca are loc relatia

f(x, r) =1

π∫−π

f(t)1− α2

1− 2α cos(t− x) + α2dt.

(f(x, r) poarta numele de integrala lui Poisson).

Solutie. Deoarece an, bn → 0, sirurile (an) si (bn) sunt marginite. Exista atunci M > 0astfel ıncat, pentru orice n ∈ N, |an| ≤M si |bn| ≤M. Cum

|αn (an cosnx+ bn sinnx)| ≤ 2Mαn

si cum α ∈ (0, 1), conform criteriului Weierstrass, rezulta convergenta uniforma a seriei(11.27).

Tinand acum cont de formulele lui an si bn, obtinem

f(x, r) =1

π∫−π

f(t)dt+1

π

∞∑n=1

αnπ∫−π

f(t) cosn(t− x)dt. (11.28)

Dar cum seria1

2+∞∑n=1

αn cosn(t−x) este absolut si uniform convergenta, putem integra

termen cu termen relatia (11.28) si obtinem

f(x, r) =1

π

∫ π

−πf(t)

[1

2+

∞∑n=1

αn cosn(t− x)

]dt.

Folosind si identitatea (11.26) de la problema anterioara, deducem concluzia.

Problema 11.23 Fie (gn) un sir de functii reale definit pe R prin:

gn(x) =

n, daca x = 2kπ, k ∈ Z(−1)n−1n, daca x = (2k + 1)π, k ∈ Zsinnx

sinx, daca x 6∈ πZ.

Sa se arate ca seria∞∑n=1

αngn, unde |α| < 1, converge uniform pe R catre o functie continua

si sa se determine aceasta functie.

Solutie. Se observa ca functiile gn sunt continue pe R. Cum

gn(x) = 2 cos(n− 1)x+ gn−2(x),∀x ∈ R, ∀n ≥ 3, (11.29)

rezulta ca |gn(x)| ≤ n, ∀x ∈ R, ∀n ∈ N. Atunci |αngn| ≤ n |α|n ,∀x ∈ R, ∀n ∈ N. Cum

seria∞∑n=1

n |α|n este evident convergenta, conform criteriului lui Weierstrass, seria∞∑n=1

αngn

Page 162: Teme si probleme pentru concursurile studentesti de matematica ...

Siruri si serii de functii: serii Taylor, serii Fourier 155

converge uniform pe R. In baza transferului de continuitate, suma ei este continua. Invirtutea relatiei (11.29), avem

∞∑n=3

αngn(x) = 2

∞∑n=3

αn cos(n− 1)x+

∞∑n=3

αngn−2(x), ∀x ∈ R, (11.30)

iar seriile din membrul drept sunt de asemenea uniform convergente pe R. Insa αg1 = αsi α2g2 = 2α2 cosx. Folosind aceste relatii si (11.30), obtinem

∞∑n=1

αngn(x) = α+ 2α

∞∑n=1

αn cosnx+ α2∞∑n=1

αngn(x), ∀x ∈ R,

de unde∞∑n=1

αngn(x) =α

1− α2+

1− α2

∞∑n=1

αn cosnx,∀x ∈ R.

Folosind si relatia (11.26), obtinem ca functia suma ceruta esteα

1− 2α cosx+ α2.

Problema 11.24 Fie f si F doua functii la patrat integrabile definite pe [−l, l] si

f(x) ∼ a0

2+

∞∑n=1

(an cosnπx

l+ bn sin

nπx

l),

F (x) ∼ A0

2+

∞∑n=1

(An cosnπx

l+Bn sin

nπx

l)

seriile Fourier atasate lor. Sa se arate ca

1

l

l∫−l

f(x)F (x)dx =a0A0

2+∞∑n=1

(anAn + bnBn).

Solutie. Seriile Fourier atasate functiilor f + F si f − F sunt

f(x) + F (x) ∼ a0 +A0

2+∞∑n=1

[(an +An) cosnπx

l+ (bn +Bn) sin

nπx

l],

f(x)− F (x) ∼ a0 −A0

2+

∞∑n=1

[(an −An) cosnπx

l+ (bn −Bn) sin

nπx

l].

Deoarece f si F sunt functii la patrat integrabile, f+F si f−F sunt la patrat integrabile.Egalitatea lui Parseval ne conduce la

1

l

l∫−l

[f(x) + F (x)]2dx =(a0 +A0)2

2+∞∑n=1

[(an +An)2 + (bn +Bn)2],

1

l

l∫−l

[f(x)− F (x)]2dx =(a0 −A0)2

2+

∞∑n=1

[(an −An)2 + (bn −Bn)2].

Scazand ultimele egalitati, obtinem egalitatea ceruta.

Page 163: Teme si probleme pentru concursurile studentesti de matematica ...

156

Problema 11.25 Fie α ∈ (0, 1) si f : R→ R periodica de perioada 2π, cu

f(x) =shx

|x|αdaca x ∈ (−π, π) \ 0, f(0) = f(π) = 0.

1) Aratati ca seria Fourier asociata lui f este de forma∞∑n=1

bn sinnx, si ca bn =

O

(1

n1−α

).

2) Studiati convergenta seriei Fourier de mai sus.

Solutie. 1) Deoarece f este periodica de perioada 2π este local integrabila pe R, deciseria sa Fourier exista. In plus, deoarece f este impara, seria Fourier va fi o serie numaide sinusuri. Avem

bn =2

π

π∫0

sh tsinnt

tαdt, ∀n ∈ N∗.

Putem scrie:π

2bn = lim

x→0+

π∫x

sh t ·ϕ′(t)dt, unde ϕ este aplicatia de clasa C1 : t 7→t∫π

sinnu

uαdu.

Integrand prin parti obtinem, pentru orice x ∈ (0, π] :

π∫x

sh t · ϕ′(t)dt = sh t · ϕ(t)

∣∣∣∣πx−

π∫x

ch t · ϕ(t)dt.

Cum0∫π

sinnu

uαdu, α < 1 converge, deducem ca

bn =2

π

π∫0

ch t · (−ϕ(t))dt.

Facand schimbarea de variabila v = nu, avem: −ϕ(t) =1

n1−α

nπ∫nt

sin v

vαdv. Aplicatia con-

tinua ψ : x 7→x∫1

sin v

vαdv din R∗+ ın R se poate prelungi prin continuitate la R+.

Folosind convergenta integralei∞∫1

sin v

vαdv, ea admite o limita finita la ∞. Exista deci

M = supx∈R+

|ψ(x)| . Deducem atunci ca

|bn| =2

π· 1

n1−α

π∫0

ch t · (ψ(nπ)− ψ(nt))dt

≤ 2

π· 2M · shπ · 1

n1−α .

2) Pentru orice x0 6∈ πZ, f este derivabila ın x0. Atunci seria Fourier asociata luif converge ın x0, avand drept suma f(x0). Pentru x0 ∈ πZ, seria Fourier este vizibilconvergenta ın x0, cu suma 0, iar ın punctele de acest tip f(x0) = 0. In concluzie, seriaFourier asociata lui f converge simplu pe R, avand drept suma f.

Page 164: Teme si probleme pentru concursurile studentesti de matematica ...

Siruri si serii de functii: serii Taylor, serii Fourier 157

Problema 11.26 Fie (bn)n∈N∗ un sir descrescator de numere reale, cu limita 0; acestui

sir ıi asociem seria trigonometrica∞∑n=1

fn, unde fn(x) = bn sinnx.

1) Aratati ca seria converge simplu pe R, si uniform pe orice interval compact din Rcare nu contine multipli de 2π.

2) Stabiliti echivalenta urmatoarelor afirmatii:

(i)∞∑n=1

fn converge uniform pe R;

(ii) bn = O(

1n

)ın vecinatatea lui ∞.

3) Presupunem ca bn = O(

1n

)ın vecinatatea lui ∞. Aratati ca suma seriei

∞∑n=1

fn este

marginita si local integrabila pe R; gasiti seria Fourier asociata functiei suma.

Solutie. 1) (a) Pentru toti x ∈ πZ, seria numerica∞∑n=1

fn(x), nula, este convergenta.

Notam Sn(x) :=n∑k=1

sin kx, n ∈ N∗, x ∈ R. Atunci (folosind transformarea lui Abel) putem

scrie pentru orice n, p ∈ Ns∑

k=1

fn+k(x) =

s∑k=1

(bn+k − bn+k+1)Sn+k(x)− bn+1Sn(x) + bn+p+1Sn+p(x).

Folosind formula clasica Sn(x) =sin nx

2 sin (n+1)x2

sin x2

, daca x 6∈ 2πZ, obtinem ca |Sn(x)| ≤

1∣∣sin x2

∣∣ ,∀n ∈ N, ∀x 6∈ 2πZ. Tinand cont si de faptul ca bn+k − bn+k+1 ≥ 0, obtinem ca

pentru orice x ∈ R \ 2πZ∣∣∣∣∣s∑

k=1

fn+k(x)

∣∣∣∣∣ ≤s∑

k=1

(bn+k − bn+k+1) |Sn+k(x)|+ bn+1 |Sn(x)|+ bn+p+1 |Sn+p(x)|

≤ 2bn+1∣∣sin x2

∣∣ , ∀n, p ∈ N. (11.31)

Deci, seria numerica∞∑n=1

fn(x) satisface criteriul lui Cauchy, deci este convergenta.

In concluzie,∞∑n=1

fn este convergenta punctual pe R. Fie f suma sa. Se observa ca f

este periodica de perioada 2π si impara.(b) Periodicitatea lui f permite reducerea problemei la a arata convergenta uniforma a

lui∞∑n=1

fn pe un interval de forma [α, 2π − α], cu α ∈ (0, π). Avem, ın virtutea monotoniei

functiei sin pe intervalul[α2 , π −

α2

],

supx∈[α,2π−α]

∣∣∣∣∣p∑

k=1

fn+k(x)

∣∣∣∣∣ ≤ 2bn+1

sin α2

.

Cum limn→∞

bn = 0,∞∑n=1

fn verifica criteriul lui Cauchy de convergenta uniforma pe [α, 2π−α],

deci converge uniform pe acest interval.Mai remarcam, ın virtutea transferului de continuitate, ca f este continua pe R \ 2πZ.

Page 165: Teme si probleme pentru concursurile studentesti de matematica ...

158

De asemenea, folosind inegalitatea sin x2 ≥

xπ pentru x ∈ [0, π], majorarea (11.31) arata

ca ∣∣∣∣∣∞∑

k=n+1

fk(x)

∣∣∣∣∣ ≤ 2πbn+1

x, ∀n ∈ N, ∀x ∈ (0, π]. (11.32)

2) (i)⇒ (ii) : Prin ipoteza,∞∑n=1

fn converge uniform pe R. Avem, ın particular,

limn→∞

(supx∈R

∣∣∣∣∣2n∑

k=n+1

bk sin kx

∣∣∣∣∣)

= 0,

de unde

limn→∞

(2n∑

k=n+1

bk sinkπ

4n

)= 0.

Cum, pentru orice numar natural din intervalul [n+ 1, 2n], avem ca

0 ≤ b2n sinπ

4≤ bk sin

4n,

rezulta ca limn→∞

(nb2n) = 0. Cum 0 ≤ b2n+1 ≤ b2n, avem si ca limn→∞

(nb2n+1) = 0. Deducem

de aici ca limn→∞

(nbn) = 0.

(ii)⇒ (i) : Prin ipoteza, limn→∞

(nbn) = 0.

Fie ε > 0. Exista atunci n0 ∈ N∗ astfel ıncat 0 ≤ nbn ≤ ε, pentru orice n ≥ n0.

Consideram x ∈ (0, π] si n ≥ n0. Notand cu p partea ıntreaga a luiπ

x, putem scrie:

∣∣∣∣∣∞∑k=n

fk(x)

∣∣∣∣∣ ≤n+p−1∑k=n

|fk(x)|+

∣∣∣∣∣∣∞∑

k=n+p

fk(x)

∣∣∣∣∣∣ .Majorand |sin kx| prin kx, x prin π

p , si kbk prin ε, obtinem

n+p−1∑k=n

|fk(x)| =n+p−1∑k=n

bk |sin kx| ≤n+p−1∑k=n

bkkx ≤n+p−1∑k=n

επ

p= πε.

Folosind acum (11.32), si majorand πx prin n+ p = n+

[πx

], obtinem ca∣∣∣∣∣∣

∞∑k=n+p

fk(x)

∣∣∣∣∣∣ ≤ 2πbn+p

x≤ 2(n+ p)bn+p ≤ 2ε.

In concluzie, pentru orice x ∈ (0, π], avem∣∣∣∣∣∞∑k=n

fk(x)

∣∣∣∣∣ ≤ (π + 2)ε.

Cum aceasta inegalitate este satisfacuta ın mod trivial ın punctul 0, folosind de asemenea

paritatea si periodicitatea, este satisfacuta ın orice x ∈ R. Rezulta cu seria∞∑n=1

fn converge

uniform pe R.

Page 166: Teme si probleme pentru concursurile studentesti de matematica ...

Siruri si serii de functii: serii Taylor, serii Fourier 159

3) Aplicam concluzia de la punctul 2). Stim ca ∃M > 0 astfel ıncat, pentru orice

n ∈ N∗, nbn ≤M. Fie x ∈ (0, π]. Notand iarasi cu p partea ıntreaga a luiπ

x, avem ca

|f(x)| ≤s∑

k=1

|fk(x)|+

∣∣∣∣∣∣∞∑

k=p+1

fk(x)

∣∣∣∣∣∣ .Majorand din nou |sin kx| prin kx, x prin π

p , si kbk prin M, obtinem ca

s∑k=1

|fk(x)| ≤ πM.

Folosind din nou (11.32), si majorand πx prin p+ 1, avem∣∣∣∣∣∣

∞∑k=p+1

fk(x)

∣∣∣∣∣∣ ≤ 2πbp+1

x≤ 2(p+ 1)bp+1 ≤ 2M.

Inegalitatea |f(x)| ≤ (π + 2)M este deci adevarata pentru orice x ∈ (0, π]; cum esteevident satisfacuta ın x = 0, este adevarata pe R ın virtutea paritatii si a periodicitatii luif. Aplicatia f : R→ R este deci marginita.

Restrictia lui f la intevalul [0, 2π] este marginita si are cel mult doua puncte de dis-continuitate (0 si 2π); ea este deci integrabila; rezulta ca f este local integrabila si ca estedezvoltabila ın serie Fourier. Seria Fourier asociata va fi o serie numai de sinusuri, pe care

o vom nota∞∑n=1

cn sinnx. Calculam cm pentru fiecare m ∈ N∗ fixat. Avem

π

2cm =

π∫0

f(t) sinmt dt =

π∫0

( ∞∑n=1

gn(t)

)dt,

unde gn(t) = bn sinnt sinmt. Este evident ca∞∑n=1

gn este convergenta simplu pe [0, π] si are

ca suma functia x 7→ f(x) sinmx. Vom arata ca aceasta convergenta este uniforma.

Studiem Tn(x) :=∞∑

k=n+1

gk(x) = sinmx ·∞∑

k=n+1

fk(x). Tn(0) = 0 e trivial. Pentru x ∈

(0, π] folosim (11.32) si obtinem

|Tn(x)| = |sinmx|

∣∣∣∣∣∞∑

k=n+1

fk(x)

∣∣∣∣∣ ≤ mx · 2πbn+1

x= 2πmbn+1.

Rezulta supx∈[0,π]

|Tn(x)| ≤ 2πmbn+1. Dar bn+1 → 0.

Folosind acum convergenta uniforma, putem scrie

π

2cm =

∞∑n=1

π∫0

bn sinnx sinmx dx

.

Cumπ∫0

sinnx sinmx dx = δmnπ2 , rezulta ca bm = cm.

Seria Fourier asocita lui f este deci∞∑n=1

fn.

Page 167: Teme si probleme pentru concursurile studentesti de matematica ...

160

Problema 11.27 Fie Γp := 1π2p

∞∑n=1

1n2p , p ∈ N∗. Aratati ca Γp ∈ Q∗+.

Solutie. Fie f2k−1, f2k, p ∈ N∗, periodice de perioada 2π de la R la R, definite prin

f2k−1(x) = x2k−1, daca x ∈ (−π, π), f2k−1(π) = 0,

f2k(x) = x2k, daca x ∈ (−π, π].

Aceste functii sunt local integrabile si admit (respectiv) dezvoltarile ın serii Fourier

∞∑n=1

bkn sinnx siak02

+∞∑n=1

akn cosnx.

Folosind formulele lui bkn si akn, obtinem (prin integrare prin parti) ca, pentru orice n ∈ N∗,

bk+1n = (−1)n+1 2π2k

n− 2k(2k + 1)

n2bkn,

akn = −2k

nbkn.

Prin calcul direct, gasim ca b1n = (−1)n+1 2

n. Prin recurenta dupa k ∈ N∗, se deduce ca

exista βk,l ∈ Z si αk,l ∈ Z, independente de n, astfel ıncat

(−1)nbkn =

k−1∑l=0

βk,lπ2l

n2k−1−2l; (−1)nakn =

k−1∑l=0

αk,lπ2l

n2k−2l,

pentru orice n, k ∈ N∗. De asemenea, obtinem direct ca ak0 =2π2k

2k + 1.

Pentru k ∈ N∗ fixat, avem, din formula lui Parseval:

1

2

∞∑n=1

(bkn)2 =1

π

π∫0

t4k−2dt =π4k−2

4k − 1, (11.33)

1

2

∞∑n=1

(akn)2 =π4k

4k + 1− 1

4(ak0)2.

Pentru k = 1, deducem ca

Γ1 =1

6si Γ2 =

1

90. (11.34)

Pentru k ≥ 2, avem, notand Λk = 0, 1, . . . , k − 12 :

∞∑n=1

(bkn)2 =∑

(l,l′)∈Λk

βk,lβk,l′π2(l+l′)

∞∑n=1

1

n4k−2−2(l+l′).

Din (11.33) si din definitia lui Γp, avem ca∑(l,l′)∈Λk

βk,lβk,l′Γ2k−1−(l+l′) ∈ Q.

In acelasi mod ∑(l,l′)∈Λk

αk,lαk,l′Γ2k−(l+l′) ∈ Q.

Folosind (11.34), deducem prin recurenta ca Γp ∈ Q∗+,∀p ∈ N∗.

Page 168: Teme si probleme pentru concursurile studentesti de matematica ...

Siruri si serii de functii: serii Taylor, serii Fourier 161

Problema 11.28 (Teorema lui Riemann) 1) Fie f : [a, b] → R o functie integrabila ınsens propriu sau absolut integrabila ın sens impropriu pe intervalul compact [a, b]. Sa searate ca:

limt→∞

b∫a

f(x) sin tx dx = 0 si limt→∞

b∫a

f(x) cos tx dx = 0.

2) Fie g : [−l, l] → R, integrabila ın sens propriu sau absolut integrabila ın sensimpropriu pe [−l, l]. Aratati ca an si bn, coeficientii Fourier corespunzatori lui g, satisfaclimn→∞

an = limn→∞

bn = 0. Deduceti un criteriu necesar pentru ca o serie trigonometrica sa

fie serie Fourier.

Solutie. 1) Observam ca, pentru orice interval compact [c, d], avem∣∣∣∣∣∣d∫c

sin tx dx

∣∣∣∣∣∣ =

∣∣∣∣cos tc− cos td

t

∣∣∣∣ ≤ 2

|t|. (11.35)

Presupunem mai ıntai ca functia f este integrabila ın sens propriu pe [a, b]. Fie a = x0 <x1 < · · · < xn−1 < xn = b o diviziune a intervalului [a, b]; notam mi = inf

x∈[a,b]f(x),

Mi = supx∈[a,b]

f(x) si ωi = Mi −mi. Atunci, tinand seama si de (11.35), avem

∣∣∣∣∣∣b∫a

f(x) sin tx dx

∣∣∣∣∣∣ =

∣∣∣∣∣∣n−1∑i=0

xi+1∫xi

f(x) sin tx dx

∣∣∣∣∣∣=

∣∣∣∣∣∣n−1∑i=0

xi+1∫xi

[f(x)−mi] sin tx dx+

n−1∑i=0

mi

xi+1∫xi

sin tx dx

∣∣∣∣∣∣≤

n−1∑i=0

ωi(xi − xi−1) +2

|t|

n−1∑i=0

|mi| .

Cum f este integrabila pe [a, b], obtinem din criteriul lui Darboux ca pentru orice ε > 0,exista δε > 0 astfel ıncat, pentru orice diviziune cu norma mai mica decat δε, diferentasumelor Darboux este mai mica decat ε

2 . Presupunem ca norma diviziunii alese de noi este

mai mica decat δε, si obtinem deci can−1∑i=0

ωi(xi−xi−1) < ε2 . Notam tε =

4

ε

n−1∑i=0|mi| . Atunci

2

t

n−1∑i=0

|mi| <ε

2,∀t > tε.

Asadar, ∀ε > 0,∃tε ∈ R, ∀t > tε :

∣∣∣∣∣ b∫af(x) sin tx dx

∣∣∣∣∣ < ε. Deci, limt→∞

b∫af(x) sin tx dx = 0.

Sa presupunem acum ca f este absolut integrabila ın sens impropriu pe [a, b]. Cum|f(x) sin tx| ≤ |f(x)| , ∀x ∈ [a, b],∀t ∈ R, folosind criteriul comparatiei, deducem ca in-

tegralab∫af(x) sin tx dx este absolut convergenta ın sens impropriu pe [a, b]. Pentru a re-

zolva problema, este suficient sa consideram cazul ın care aceasta este de tipulb−0∫af(x)dx,

Page 169: Teme si probleme pentru concursurile studentesti de matematica ...

162

celelalte cazuri rezolvandu-se analog. Intrucat integrala considerata este convergenta,∀ε > 0,∃η = ηε ∈ (a, b) astfel ıncat∣∣∣∣∣∣

b∫η

f(x) sin tx dx

∣∣∣∣∣∣ < ε

2,∀t ∈ R.

Dar cum pe intervalul compact [a, η] stim din demonstratia anterioara ca ∃tε ∈ R, ∀t > tε :∣∣∣∣ η∫af(x) sin tx dx

∣∣∣∣ < ε2 , rezulta ca ∀ε > 0,∃η = ηε ∈ (a, b),∃tε ∈ R, ∀t > tε∣∣∣∣∣∣

b∫a

f(x) sin tx dx

∣∣∣∣∣∣ ≤∣∣∣∣∣∣η∫a

f(x) sin tx dx

∣∣∣∣∣∣+

∣∣∣∣∣∣b∫η

f(x) sin tx dx

∣∣∣∣∣∣ < ε.

De aici, limt→∞

b∫af(x) sin tx dx = 0. Pentru cealalta integrala, demonstratiile sunt complet

analoage.2) Folosind punctul 1), deducem imediat ca an → 0, bn → 0. Sa remarcam faptul ca

acest lucru reiese si din inegalitatea lui Bessel sau din egalitatea lui Parseval, dar numaipentru functiile de patrat integrabil ın sens impropriu sau integrabile ın sens propriu. Dupa

cum se poate vedea examinand functia g : [0, 1]→ R, g(x) =1√x, daca x ∈ (0, 1], si g(0) =

0, nu orice functie de patrat integrabil ın sens impropriu este integrabila ın sens impropriu.

Reciproca este adevarata, dupa cum ne arata inegalitatea |h(x)| ≤ 1 + h2(x)

2,∀x ∈ [a, b].

In concluzie, un criteriu necesar ca o serie trigonometrica sa fie serie Fourier este ca sirurile(an) si (bn) formate din coeficientii ei sa convearga la 0.

Problema 11.29 1) (Nucleele Dirichlet si Fejer) Fie seria divergenta

1

2+

∞∑n=1

cosnx. (11.36)

Calculati sumele

sn+1(x) =1

2+

n∑k=1

cos kx si σn+1(x) =s0(x) + s1(x) + · · ·+ sn(x)

n+ 1.

Functia sn+1(x) se numeste nucleul lui Dirichlet, iar functia σn+1(x) se numeste nucleullui Fejer.Aratati ca pentru orice x 6= 2kπ, seria (11.36) este (C, 1)− sumabila catre 0. Verificati ca

1

π

π∫−π

sn+1(t)dt =1

π

π∫−π

σn+1(t)dt = 1.

(O serie se numeste (C, 1)−sumabila daca sirul mediilor aritmetice ale sumelor salepartiale este convergent.)

2) Fie seria∞∑n=1

sinnx. (11.37)

Aratati ca este (C, 1)−sumabila, cu (C, 1)−suma =

12 , daca x 6= 2kπ0, daca x = 2kπ.

.

Page 170: Teme si probleme pentru concursurile studentesti de matematica ...

Siruri si serii de functii: serii Taylor, serii Fourier 163

Solutie. 1) Se obtine sn+1(x) =sin(n+ 1

2)x

2 sin x2

si

σn+1(x) =1

n+ 1· 1

2 sin x2

[sin

x

2+ sin

3x

2+ · · ·+ sin(n+

1

2)x

]=

1

n+ 1

sin2(n+ 12)x

2 sin2 x2

.

Pentru orice x 6= 2kπ avem limn→∞

σn+1(x) = 0. Verificarea integralelor rezulta direct.

2) Se obtine

Sn(x) =n∑k=1

sin kx =cos x2 − cos

(n+ 1

2

)x

2 sin x2

,

S1(x) + S2(x) + · · ·+ Sn(x)

n=

1

2ctg

x

2− 1

n

sin(n+ 1)x− sinx

4 sin2 x2

,

de unde concluzia.

Problema 11.30 (Integralele Dirichlet si Fejer) Fie f : [−π, π] → R o functie periodicade perioada 2π. Notam cu s suma seriei Fourier asociate lui f, iar cu sn suma partiala

sn(x) =a0

2+

n∑k=1

(ak cos kx+ bk sin kx) .

Aratati ca

sn(x) =1

π∫0

sin(n+ 1

2

)u

sin u2

[f(x+ u) + f(x− u)]du, (11.38)

σn(x) =s0(x) + s1(x) + · · ·+ sn−1(x)

n

=1

2nπ

π∫0

sin2 nu2

sin2 u2

[f(x+ u) + f(x− u)]du. (11.39)

Calculati apoi diferentele sn(x)− s(x) si σn(x)− s(x).Integralele (11.38) si (11.39) se numesc, respectiv, integralele Dirichlet si Fejer.

Solutie. Folosind expresiile integrale ale coeficientilor ak si bk, precum si problemaanterioara, obtinem

sn(x) =1

π

π∫−π

f(u) ·

[1

2+

n∑k=1

cos k(u− x)

]du

=1

π

π∫−π

f(u) ·sin(2n+ 1)u−x2

2 sin u−x2

du.

Cum ın integrala precedenta functiile de u care apar sunt periodice de perioada 2π, valoareaintegralei nu se modifica daca schimbam intervalul de integrare ın [x− π, x+ π]. Atunci,

Page 171: Teme si probleme pentru concursurile studentesti de matematica ...

164

folosind si schimbarea de variabila t = u− x, obtinem

sn(x) =1

π

x+π∫x−π

f(u) ·sin(2n+ 1)u−x2

2 sin u−x2

du =1

π

π∫−π

f(x+ t) ·sin(n+ 1

2)t

2 sin t2

dt

=1

π

0∫−π

f(x+ t) ·sin(n+ 1

2)t

2 sin t2

dt+

π∫0

f(x+ t) ·sin(n+ 1

2)t

2 sin t2

dt

=

1

π

π∫0

[f(x+ t) + f(x− t)] ·sin(n+ 1

2)t

2 sin t2

dt,

adica (11.38) este aratata.Pentru calculul lui σn(t), tinem seama de (11.38), de proprietatea de aditivitate a

integralei si de formula (folosita mai sus)

sinx

2+ sin

3x

2+ · · ·+ sin(n+

1

2)x =

sin2(n+ 12)x

2 sin2 x2

.

Daca f ≡ 1 ın (11.38), atunci toti coeficientii Fourier asociati ei sunt 0, ın afara de a0,care este 2. Prin urmare, suma partiala de rang n asociata ei este identic egala cu 1. Din

relatia (11.38) deducem atunci ca2

π

π∫0

sin(n+ 12)t

2 sin t2

dt = 1. De aici, folosind aceeasi relatie,

obtinem

sn(x)− s(x) =1

π

π∫0

[f(x+ t) + f(x− t)− 2s(x)] ·sin(n+ 1

2)t

2 sin t2

dt. (11.40)

Cu un calcul similar care porneste de la formula (11.39), si folosind identitatea

12πn

π∫0

sin2 nt2

sin2 t2

dt = 1, obtinem

σn(x)− s(x) =1

2nπ

π∫0

[f(x+ t) + f(x− t)− 2s(x)] ·sin2 nt

2

2 sin2 t2

dt. (11.41)

Problema 11.31 Fie f ca ın problema anterioara. Folosind formula integralei Dirichlet,sa se arate ca daca f este marginita, exista A,M > 0 astfel ıncat are loc evaluarea Lebesgue∣∣∣∣∣a0

2+

n∑k=1

(ak cos kx+ bk sin kx)

∣∣∣∣∣ < AM lnn.

Solutie. Deoarece f este marginita, existaM > 0 astfel ıncat |f(x)| ≤M,∀x ∈ [−π, π].

In formula (11.38) folosim aceasta inegalitate, precum si faptul ca sin x2 ≥

x

π,∀x ∈ [−π, π].

Obtinem succesiv

|sn(x)| ≤ M

π

π∫0

∣∣sin (n+ 12

)u∣∣

sin u2

du ≤Mπ∫0

∣∣sin (n+ 12

)u∣∣

udu

=M

n+ 12

(n+ 12)π∫

0

|sin t|t

dt.

Page 172: Teme si probleme pentru concursurile studentesti de matematica ...

Siruri si serii de functii: serii Taylor, serii Fourier 165

Cum|sin t|t≤ 1, daca x ∈ (0, 1], lim

t→0+

|sin t|t

= 1, si cum|sin t|t≤ 1

t, daca x > 1, avem

|sn(x)| ≤M1∫0

dt+M

(n+ 12)π∫

1

dt

t= M

[1 + ln

(n+

1

2

)t

]≤M(1 + lnn+ ln 2π).

Fie A > 1 +1 + ln 2π

ln 2. Atunci 1 + lnn+ ln 2π < A lnn. De aici, concluzia.

Problema 11.32 (Teorema localizarii) Sa se arate ca, data o functie absolut integra-bila f, comportarea seriei Fourier asociata lui f, ıntr-un punct x0, depinde exclusiv decomportarea functiei f ıntr-o vecinatate a acestui punct.

Solutie. Fie δ ∈ (0, π). Consideram functia g : [x0 − π, x0 + π]→ R, data prin

g(x) =

f(x), daca x ∈ (x0 − δ, x0 + δ)0, daca x ∈ [x0 − π, x0 − δ) ∪ (x0 + δ, x0 + π].

Folosind (11.38), obtinem succesiv sumele partiale pentru f si pentru g

sn(x0) =1

π∫0

sin(n+ 1

2

)u

sin u2

[f(x0 + u) + f(x0 − u)]du

Sn(x0) =1

π∫0

sin(n+ 1

2

)u

sin u2

[g(x0 + u) + g(x0 − u)]du

=1

δ∫0

sin(n+ 1

2

)u

sin u2

[f(x0 + u) + f(x0 − u)]du.

Atunci, diferenta acestora este

sn(x0)− Sn(x0) =1

π∫δ

sin(n+ 1

2

)u

sin u2

[f(x0 + u) + f(x0 − u)]du.

Cum functia1

sin u2

[f(x0 + u) + f(x0 − u)] este absolut integrabila pe [δ, π], rezulta ın

baza teoremei lui Riemann (vezi Problema 11.28) ca

limn→∞

(sn(x0)− Sn(x0)) = 0.

De aici, se deduce faptul ca pentru δ > 0 arbitrar de mic, comportarea sumelor sn(x0)depinde de comportarea functiei f ın intervalul (x0 − δ, x0 + δ), si nu de valorile dinexteriorul acestui interval.

Problema 11.33 (Criteriul lui Dini) Fie f : R→ R o functie periodica de perioada 2π siintegrabila ın sens propriu sau absolut integrabila ın sens impropriu pe intervalul [−π, π],x0 un punct din acest interval unde f are limite laterale finite si

s0 =f(x0 + 0) + f(x0 − 0)

2.

Page 173: Teme si probleme pentru concursurile studentesti de matematica ...

166

Sa se arate ca daca exista un numar a ∈ (0, π] astfel ıncat integrala

a∫0

|f(x0 + t) + f(x0 − t)− 2s0|t

dt

sa fie convergenta, atunci seria Fourier asociata functiei f converge ın punctul x0 catre s0.

Solutie. Fie functia h : [0, π]→ R, h(t) =f(x0 + t) + f(x0 − t)− 2s0

2 sin t2

, daca t ∈ (0, π],

h(0) = 0. Atunci, cum limt→0

t2 sin t

2

= 1, rezulta ca pentru ε = 1,∃δ > 0,∀t a.ı. |t| < δ :∣∣∣ t2 sin t

2

− 1∣∣∣ < 1, sau

∣∣∣ t2 sin t

2

∣∣∣ < 2. Fie acum t ∈ (0,minδ, π) . Avem

|h(t)| =

∣∣∣∣f(x0 + t) + f(x0 − t)− 2s0

t· t

2 sin t2

∣∣∣∣<

2 |f(x0 + t) + f(x0 − t)− 2s0|t

.

Tinand seama de ipoteza, deducem caπ∫0

h(x)dx este absolut convergenta. Atunci, ın vir-

tutea Teoremei lui Riemann si relatiei (11.40), avem

limn→∞

(sn(x0)− s0) = limn→∞

1

π

π∫0

h(t) sin(n+1

2)tdt = 0,

de unde concluzia.Sa mai observam doar ca daca f este continua ın x0, atunci s0 = f(x0).

Problema 11.34 (Criteriul lui Lipschitz) Fie f : R→ R o functie periodica de perioada2π, integrabila ın sens propriu sau absolut integrabila ın sens impropriu pe intervalul[−π, π] si x0 ∈ R. Daca ın punctul x0 functia f este continua si daca exista constantelepozitive M,α si δ astfel ıncat, pentru orice t ∈ (0, δ),

|f(x0 ± t)− f(x0)| ≤Mtα,

atunci seria Fourier asociata lui f converge ın punctul x0 catre f(x0).

Solutie. Notam g(t) := f(x0 + t) + f(x0 − t)− 2f(x0). Atunci

δ∫0

|g(t)|t

dt ≤δ∫0

|f(x0 + t)− f(x0)|t

dt+

δ∫0

|f(x0 − t)− f(x0)|t

dt

≤δ∫0

2Mtα−1dt =2Mδα

α.

Rezulta caδ∫0

|g(t)|t dt este convergenta si deci, ın baza criteriului lui Dini (problema 11.33),

concluzia.

Page 174: Teme si probleme pentru concursurile studentesti de matematica ...

Siruri si serii de functii: serii Taylor, serii Fourier 167

Problema 11.35 (Teorema lui Fejer) Fie f ca ın Problema 11.30. Presupunem ca

limn→∞

1

2nπ

π∫0

sin2 nu2

sin2 u2

[f(x+ u) + f(x− u)− 2s(x)]du = 0.

Sa se arate ca seria Fourier asociata lui f este (C, 1)−sumabila catre s(x).Daca, ın plus, f este marginita, atunci seria Fourier asociata lui f este (C, 1)−sumabila

catref(x+ 0) + f(x− 0)

2ın punctele x ın care limitele laterale ale functiei sunt finite, si

(C, 1)−sumabila catre f(x) ın punctele x ın care f este continua.

Solutie. Prima parte rezulta direct din formula (11.41) si din definitia(C, 1)−sumabilitatii.

Pentru partea a doua, demonstratia este asemanatoare cu cea a Criteriului Dini. Notamϕ(u) := f(x+ u) + f(x− u)− 2s(x). Fie x un punct ın care f are limitele laterale finite.

Punand s(x) =f(x+ 0) + f(x− 0)

2, constatam ca lim

u→0ϕ(u) = 0. Atunci, ∀ε > 0,∃δ =

δε ∈ (0, π),∀u a.ı. u ≤ δ : |ϕ(u)| < ε. Fixand δ, avem (din (11.41))

σn(x)− s(x) =1

2nπ·δ∫0

ϕ(u) ·sin2 nu

2

2 sin2 u2

du+1

2nπ·π∫δ

ϕ(u) ·sin2 nu

2

2 sin2 u2

du.

Notand cu I1 si I2 cele doua integrale de mai sus, folosind faptul ca |ϕ(u)| ≤M pe (δ, π),

si totodata egalitatea1

2πn

π∫0

sin2 nu2

sin2 u2

du = 1, avem

|I1| ≤1

2πn

δ∫0

sin2 nu2

sin2 u2

|ϕ(u)| du ≤ ε

2πn

π∫0

sin2 nu2

sin2 u2

du =ε

2,

|I2| ≤1

2πn

π∫δ

sin2 nu2

sin2 u2

|ϕ(u)| du ≤ M

2πn sin2 δ2

.

π∫0

du =M

2n sin2 δ2

.

Din aceste doua relatii, deducem ca, pentru n suficient de mare,

|σn(x)− s(x)| ≤ ε.

De aici, concluzia.

Problema 11.36 Sa se arate ca functia f : R→ R data prin

f(x) =

ln∣∣cos x2

∣∣ , daca x 6= (2k + 1)π, k ∈ Z0, daca x = (2k + 1)π, k ∈ Z

este dezvoltabila ın serie Fourier pe R \ (2k + 1)π | k ∈ Z si sa se determine aceastadezvoltare.

Solutie. Functia f este periodica de perioada 2π si para, deci este suficient sa studiemproblema pe [0, π]. Cum f este nemarginita pe acest interval, trebuie aratat mai ıntai ca feste absolut integrabila ın sens impropriu pe [0, π]. Cum f este continua pe orice intervalde forma [0, x], cu x ∈ [0, π), ea este integrabila Riemann pe acest interval. Mai observam,

Page 175: Teme si probleme pentru concursurile studentesti de matematica ...

168

aplicand regula lui l’Hospital, ca limx→π−

√π − x |f(x)| = 0, de unde, ın virtutea criteriului

ın α, f este absolut integrabila pe [0, π], deci ıi putem determina coeficientii Fourier. Fiex0 ∈ [0, π) si fie un interval compact [x0 − α, x0 + α] ⊂ (−π, π). Cum f este derivabilacu derivata continua pe (−π, π), rezulta ca f este lipschitziana pe [x0 − α, x0 + α]. Prinurmare, f verifica toate conditiile din Criteriul lui Lipschitz (Problema 11.34), deci seriaFourier asociata ei converge ın punctul x0 catre f(x0). De aici, cum x0 a fost ales arbitrardin [0, π), folosind paritatea si periodicitatea functiei f, rezulta ca este dezvoltabila ın serieFourier numai de cosinusuri pe R \ (2k + 1)π | k ∈ Z. In plus,

an =2

π

π∫0

f(x) cosnx dx =2

π

sinnx

nln cos

x

2

∣∣∣∣π0

+1

2n

π∫0

sinnx tgx

2dx

.Folosind faptul ca lim

x→πsinnx ln cos x2 = 0, facand schimbarea de variabila x = π − y si

tinınd cont de formulele

sinny cosy

2=

1

2

[sin

(n+

1

2

)y + sin

(n− 1

2

)y

],

n∑k=1

cos kx =sin(n+ 1

2

)x− sin x

2

2 sin x2

,

avem

an =(−1)n+1

π∫0

(1

2+

n∑k=1

cos ky

)dy +

π∫0

(1

2+n−1∑k=1

cos ky

)dy

=

(−1)n+1

n.

De aici,

f(x) =a0

2+∞∑n=1

(−1)n+1

ncosnx, ∀x 6= (2k + 1)π, k ∈ Z.

Pentru x = 0 ın relatia de mai sus, obtinem a0 = −2∞∑n=1

(−1)n+1

n= −2 ln 2. In punctele

de forma (2k + 1)π, k ∈ Z seria din membrul drept este divergenta, deci egalitatea nu areloc.

Problema 11.37 (Hurwitz) Fie Γ multimea tuturor curbelor simple si ınchise care auaceeasi lungime l. Presupunand ca ecuatiile lor parametrice pot fi scrise sub forma seriilorFourier

x = x(s) =a0

2+

∞∑n=1

(an cos

2nπ

ls+ bn sin

2nπ

ls

),

y = y(s) =α0

2+∞∑n=1

(αn cos

2nπ

ls+ βn sin

2nπ

ls

),

unde s este chiar arcul de curba (0 ≤ s < l), sa se arate ca dintre toate curbele familiei Γcercul este curba care ınchide aria maxima.

Page 176: Teme si probleme pentru concursurile studentesti de matematica ...

Siruri si serii de functii: serii Taylor, serii Fourier 169

Solutie. Fie γ ∈ Γ. Lungimea sa l si aria A a multimii pe care o margineste sunt datede formulele

l =

∫γ

ds =

l∫0

√[x′(t)]2 + [y′(t)]2dt,

A =

∮γ

xdy − ydx =1

2

l∫0

(x(t) · y′(t)− y(t) · x′(t)

)dt.

Derivatele x′(t) si y′(t) sunt date de formulele

x′(t) =2π

l

∞∑n=1

(−nan sin

2nπ

lt+ nbn cos

2nπ

lt

),

y′(t) =2π

l

∞∑n=1

(−nαn sin

2nπ

lt+ nβn cos

2nπ

lt

).

Dar, (folosind eventual faptul ca aplicatia 〈·, ·〉 : L2[0, l] × L2[0, l] → R, 〈f, g〉 :=

1l

l∫0

f(t)g(t)dt este un produs scalar pe spatiul liniar al claselor de functii de patrat integrabil

ın sens Lebesgue pe [0, l]), se poate deduce ca

l =

l∫0

[x′(t)]2 + [y′(t)]2dt.

Folosind acum formula lui Parseval si cea data de Problema 11.24, obtinem din for-mulele lungimii si ariei de mai sus ca

l =2π2

l

∞∑n=1

n2(a2n + b2n + α2

n + β2n

),

A = π∞∑n=1

n (anβn − αnbn) .

Calculam diferenta dintre aria cercului de lungime l si aria A marginita de o curba deaceeasi lungime cu cercul. Avem

l2

4π−A =

π

2

∞∑n=1

n2(a2n + b2n + α2

n + β2n

)− π

∞∑n=1

n (anβn − αnbn)

2

∞∑n=1

[(nan − βn)2 + (nbn + αn)2 + (n2 − 1)(α2n + β2

n)] ≥ 0.

Diferental2

4π− A se anuleaza cand a1 = β1, b1 = −α1, an = bn = αn = βn = 0, n =

2, 3, . . . . In acest caz aria marginita de curba γ devine maxima, iar ecuatiile parametriceale acestei curbe sunt

x = x(s) =a0

2+ a1 cos

ls+ b1 sin

ls,

y = y(s) =α0

2− b1 cos

ls+ a1 sin

ls,

ceea ce reprezinta cercul de ecuatie implicita(x− a0

2

)2+(y − α0

2

)2= 2(a2

1 + b21).

Page 177: Teme si probleme pentru concursurile studentesti de matematica ...

Capitolul 12

Functii complexe

Definitii si rezultate

Numere complexe

Se noteaza C multimea numerelor complexe z = x + iy, unde x, y ∈ R iar i2 = −1.x = Re z si y = Im z se numesc, respectiv partea reala si partea imaginara a numaruluicomplex z.

Definitii. Fie z = x+ iy un numar complex. Se noteaza

z = x− iy

si se numeste conjugatul numarului complex z. Au loc relatiile:

Re z =z + z

2; Im z =

z − z2i

Se noteaza |z| =√

(x2 + y2 si se numeste modulul numarului complex z.Definitie si teorema. Fie z ∈ C, z 6= 0. Exista si este unic θ ∈ (−π, π], numit

argumentul numarului complex z si notat arg z, astfel ıncat:

cos θ =Re z

|z|; sin θ =

Im z

|z|

Intervalul (−π, π] se poate ınlocui cu [0, 2π) sau cu orice alt interval de lungime 2π.O expresie explicita a argumentului este:

arg z =

2arctg

y

x+ |z|, x+ |z| 6= 0

π, x+ |z| = 0

Scrierea z = |z| (cos θ + i sin θ) poarta numele de forma trigonometrica a numaruluiz ∈ C \ 0. Orice alegere θ = arg z + 2kπ, k ∈ Z convine). Deoarece

(cos θ + i sin θ)(cosϕ+ i sinϕ) = cos(θ + ϕ) + i sin(θ + ϕ)

deducem formula lui Moivre:

(cos θ + i sin θ)n = cosnθ + i sinnθ, ∀n ∈ Z

1.3. Observatie. Identificand R cu o dreapta (raportata la un reper), urmeaza caC = R× R se identifica cu un plan (raportat la un reper).

170

Page 178: Teme si probleme pentru concursurile studentesti de matematica ...

Functii complexe 171

Asadar, z 7→ (−z) reprezinta simetria fata de punctul O; z 7→ z reprezinta simetria fatade axa Ox; z 7→ z + a reprezinta translatia de vector a (a ∈ C); z 7→ ρz (ρ > 0) reprezintaomotetia de centru O si de raport ρ; z 7→ az (a ∈ C, |a| = 1) reprezinta rotatia de centru

O si de unghi arg a. In sfarsit, deoarece1

z=

z

|z|2urmeaza ca z 7→ 1

zreprezinta inversiunea

de pol O si de putere 1, urmata de simetria fata de axa Ox.In unele aplicatii, este comod sa introducem un punct, notat ∞ 6∈ C. Nu vom defini

operatii algebrice pe multimea C ∪ ∞ (care va fi numita planul complex extins). Sirul

(zn)n din C are limita ∞ daca si numai daca |zn| → ∞. Evident: zn →∞ ⇔1

zn→ 0.

Transformari omografice

Definitie. Cu fiecare numere a, b, c, d ∈ C, verificand ad − bc 6= 0, asociem functiaf : C ∪ ∞ → C ∪ ∞, definita astfel: daca c 6= 0, atunci

f(z) =

az + b

cz + d, z ∈ C \ −d/c

∞, z = −d/ca/c, z =∞

respectiv, daca c = 0

f(z) =

az + b

d, z ∈ C

∞, z =∞

Functiile de aceasta forma sunt numite transformari omografice, iar z = −dc

se

numeste polul transformarii.Observatii. Este evident ca transformarile omografice sunt functii continue. Cerinta

ca ad− bc 6= 0 este echivalenta cu faptul ca f este neconstanta.Fiecare transformare omografica este functie olomorfa ın C \ −d/c (respectiv ın C,

daca c = 0), iar:

f ′(z) =ad− bc

(cz + d)26= 0

In particular, transformarile omografice pastreaza unghiurile, ın afara polului.Propozitie. Multimea transformarilor omografice formeaza grup fata de compunerea

functiilor.Propozitie. Fiecare transformare omografica apare ca o compunere de urmatoarele

tipuri: (i) z 7→ z + a (translatie); (ii) z 7→ az (a 6= 0, rotatie ın jurul originii, de unghiarg a si omotetie de centru 0 si de raport |a|); (iii) z 7→ z−1 (inversiune de pol 0 si deputere 1, urmata de simetrie fata de axa Ox).

Propozitie. Fiind date doua triplete (z1, z2, z3), (w1, w2, w3) din C ∪ ∞, formatedin puncte distincte doua cate doua, exista si este unica o transformare omografica f ,astfel ıncat f(zk) = wk, k = 1, 2, 3.

Lema. Fiecare transformare omografica, diferita de identitate, are cel mult douapuncte fixe.

Propozitie. Transformarile omografice aplica: cercurile si dreptele care trec prin polultransformarii ın drepte; iar cercurile si dreptele care nu trec prin pol ın cercuri.

Propozitie. Fie γ1, γ2 doua cercuri sau drepte. Exista transformari omografice caresa aplice γ1 pe γ2.

Page 179: Teme si probleme pentru concursurile studentesti de matematica ...

172

Probleme

Problema 12.1 Fie f : C → C definita prin f(z) =z

1 + |z|. Sa se arate ca f stabileste

o bijectie ıntre C si o multime ∆, care se va determina. Sa se scrie explicit inversa sa.

Solutie. Din ecuatia f(z) = w, adicaz

1 + |z|= w se deduce: |z| = |w|

1− |w|daca

si numai daca |w| < 1. Revenind ın ecuatia initiala, se obtine z =w

1− |w|. Asadar

∆ = w ∈ C| |w| < 1 iar f−1(w) =w

1− |w|.

Problema 12.2 Fie z1, z2, . . . , zn ∈ C. Sa se arate ca ca exista J ⊆ 1, 2, . . . , n, astfelıncat ∣∣∣∣∣∣

∑j∈J

zj

∣∣∣∣∣∣ ≥ 1

4√

2

n∑k=1

|zk|

Berkeley, 1990

Solutie. Se partitioneaza multimea 1, 2, . . . , n ın patru submultimi, dupa cum zj seafla ın unul din cele patru cadrane determinate de cele doua bisectoare. Va exista deci celputin o parte, din cele patru, pe care o notam cu J , pentru care:

∑j∈J|zj | ≥

1

4

n∑j=1

|zj |

Din faptul ca toate numerele zj apartin unui acelasi cadran are loc una din ine-galitatile: |Re zj | ≥ 1√

2|zj | sau |Im zj | ≥ 1√

2|zj |. Mai departe, avem de asemenea∣∣∣∣∣∣

∑j∈J

Re zj

∣∣∣∣∣∣ =∑j∈J|Re zj | sau

∣∣∣∣∣∣∑j∈J

Im zj

∣∣∣∣∣∣ =∑j∈J|Im zj |. Inegalitatea ceruta se deduce acum,

observand ca: ∣∣∣∣∣∣∑j∈J

zj

∣∣∣∣∣∣ ≥∣∣∣∣∣∣Re

∑j∈J

zj

∣∣∣∣∣∣ =

∣∣∣∣∣∣∑j∈J

Re zj

∣∣∣∣∣∣sau ∣∣∣∣∣∣

∑j∈J

zj

∣∣∣∣∣∣ ≥∣∣∣∣∣∣Im

∑j∈J

zj

∣∣∣∣∣∣ =

∣∣∣∣∣∣∑j∈J

Im zj

∣∣∣∣∣∣Problema 12.3 Fie a, b, c ∈ C necoliniare. Daca (zn)n este un sir de numere complexe,avand proprietatea ca fiecare din sirurile: (|zn − a|)n; (|zn − b|)n; (|zn − c|)n converg,atunci (zn)n este convergent.

Solutia 1. Sirul (zn)n rezulta marginit. Daca nu ar fi convergent, ar avea cel putindoua subsiruri convergente, cu limite distincte, fie acestea z 6= z′. Ar urma: |z−a| = |z′−a|si analoagele. Adica a, b, c s-ar afla pe mediatoarea segmentului determinat de z si z′, ıncontradictie cu ipoteza de necoliniaritate.

Solutia 2. Se obtine prin calcul efectiv. Sa notam:

an = |zn − a|; bn = |zn − b|; cn = |zn − c|

Page 180: Teme si probleme pentru concursurile studentesti de matematica ...

Functii complexe 173

si:A = lim

n→∞an; B = lim

n→∞bn; C = lim

n→∞cn

Avem: znzn − azn − azn = a2

n − |a|2znzn − bzn − bzn = b2n − |b|2znzn − czn − czn = c2n − |c|2

Considerat ca sistem liniar de trei ecuatii cu necunoscutele: znzn, zn, zn, solutia este unica,deoarece ipoteza de necoliniaritate asigura exact ca determinantul sistemului este nenul.Se obtine:

zn =

∣∣∣∣∣∣1 a2

n − |a|2 −a1 b2n − |b|2 −b1 c2

n − |c|2 −c

∣∣∣∣∣∣∣∣∣∣∣∣1 −a −a1 −b −b1 −c −c

∣∣∣∣∣∣de unde rezulta ca sirul (zn)n este convergent si are limita:∣∣∣∣∣∣

1 A2 − |a|2 −a1 B2 − |b|2 −b1 C2 − |c|2 −c

∣∣∣∣∣∣∣∣∣∣∣∣1 −a −a1 −b −b1 −c −c

∣∣∣∣∣∣Observatii. Intre A,B,C exista o relatie, care se obtine scriind ca solutia znzn a

sistemului este produsul solutiilor zn si zn.Geometric, |zn − a| → A ınseamna ca sirul (zn)n are toate punctele de acumulare pe

cercul de centru a si de raza A. Cercurile de centre a, b, c nu pot avea mai mult decat unpunct comun, tocmai datorita ipotezei de necoliniaritate.

Problema 12.4 Sa se arate ca proiectia stereografica, definita prin:

(x1, x2, x3) 7→ x1 + ix2

1− x3daca x3 6= 1

stabileste o bijectie bicontinua ıntre planul complex extins C∪∞ si sfera unitate din R3.Sa se arate ca acesta corespondenta transforma orice cerc de pe sfera ıntr–un cerc sau

ıntr-o dreapta din plan.

Solutie. Notam cu S sfera unitate din R3, de ecuatie x21 + x2

2 + x23 = 1. Fie P ∈ S

punctul de coordonate (0, 0, 1) (“polul nord”)Pentru un punct oarecare M ∈ S\P, de coordonate (x1, x2, x3), dreapta determinata

de punctele P si M are ecuatia

X

x1=Y

x2=Z − 1

x3 − 1

deci intersecteaza planul x3 = 0 ın punctul

(x1

1− x3,

x2

1− x3, 0

). Vom defini deci aplicatia

φ : S → C ∪ ∞ prin

φ(x1, x2, x3) =

x1 + ix2

1− x3, daca x3 6= 1

∞ , daca x3 = 1

Page 181: Teme si probleme pentru concursurile studentesti de matematica ...

174

Prin calcul sau geometric, se constata ca φ este o bijectie. Inversa este ψ : C ∪ ∞ → Sdefinita astfel:

ψ(z) =

(z + z

1 + |z|2,

z − zi(1 + |z|2)

,|z|2 − 1

|z|2 + 1

);ψ(∞) = (0, 0, 1)

Deoarece

limx3 → 1

(x1, x2, x3) ∈ S

∣∣∣∣x1 + i.x2

1− x3

∣∣∣∣2 = limx3 → 1

(x1, x2, x3) ∈ S

1 + x3

1− x3=∞

iar

limz→∞

(z + z

1 + |z|2,

z − zi(1 + |z|2)

,|z|2 − 1

|z|2 + 1

)= (0, 0, 1)

rezulta ca φ este continua.Geometric este evident ca orice cerc de pe sfera, ce trece prin P , este transformat

ıntr–o dreapta din plan si reciproc. Scriind ecuatia unui cerc de pe sfera ca intersectia cuplanul

α1x1 + α2x2 + α3x3 = α0, unde α21 + α2

2 + α23 = 1 si 0 ≤ α0 < 1

urmeaza ca imaginea ın plan are ecuatia:

α1(z + z)− i.α2(z − z) + α3(|z|2 − 1) = α0(|z|2 + 1)

ceea ce reprezinta un cerc sau o dreapta, dupa cum α0 6= α3 sau nu. Reciproc, dat fiindun cerc sau o dreapta din plan, de ecuatie Azz + Bz + Bz + C = 0, coeficientii αk sedetermina unic, cu proprietatile

α21 + α2

2 + α23 = 1 si 0 ≤ α0 < 1

Problema 12.5 Fie z1, z2, z3, z4 ∈ C ∪ ∞ patru puncte distincte. Notam:

(z1, z2, z3, z4) =z1 − z2

z1 − z4:z3 − z2

z3 − z4

(numit biraportul celor patru puncte).(i) Sa se verifice ca, pentru orice transformare omografica f , are loc:

(f(z1), f(z2), f(z3), f(z4)) = (z1, z2, z3, z4)

(ii) Fie z1, z2, z3, z4 ∈ C∪∞ respectiv w1, w2, w3, w4 ∈ C∪∞ puncte distincte. Sase arate ca exista o transformare omografica f astfel ıncat f(zk) = wk, ∀k = 1, 4 daca sinumai daca : (z1, z2, z3, z4) = (w1, w2, w3, w4).

(iii) Fie f o transformare omografica cu f3 6= I. Sa se arate ca biraportul(z, f(z), f2(z), f3(z)) nu depinde de z (daca z nu este punct fix pentru f).

Solutie. (i) Fie g (unica) transformare omografica, care duce (f(z2), f(z3), f(z4)) ın(0, 1,∞). Se stie ca:

g(z) = (z, f(z2), f(z3), f(z4))

deci:g(f(z)) = (f(z), f(z2), f(z3), f(z4))

Pe de alta parte, g f este transformarea omografica, care duce (z2, z3, z4) ın (0, 1,∞).Asadar:

Page 182: Teme si probleme pentru concursurile studentesti de matematica ...

Functii complexe 175

(g f)(z) = (z, z2, z3, z4)

adica tocmai concluzia.(ii) O implicatie a fost demonstrata mai sus. Reciproc, daca birapoartele sunt egale,

sa notam f1, f2 transformarile omografice care duc, (z2, z3, z4), respectiv (w2, w3, w4) ın(0, 1,∞). Atunci:

f1(z1) = (f1(z1), 0, 1,∞) = (f1(z1), f1(z2), f1(z3), f1(z4)) =

= (z1, z2, z3, z4) = (w1, w2, w3, w4) = (f2(w1), f2(w2), f2(w3), f2(w4)) =

= (f2(w1), 0, 1,∞) = f2(w1)

Rezulta ca f = f−12 f1 convine.

(iii) Sa presupunem ca f are doua puncte fixe distincte z1, z2 ∈ C ∪ ∞. Fie z, w ∈C∪∞, diferite de z1 si de z2. Exista transformarea omografica g, care are z1 si z2 punctefixe, iar g(z) = w. g comuta cu f , deci:

(z, f(z), f2(z), f3(z)) = (g(z), g(f(z)), g(f2(z)), g(f3(z))) =

= (w, f(w), f2(w), f3(w))

Cazul cand punctele fixe sunt confundate se justifica analog.Observatie. Se observa ca functia considerata este continua de z. Daca sirul (fn(z))n

are limita, deducem concluzia. Valoarea constanta este:

1

1 +bc− ad(a+ d)2

si se obtine pentru z =∞.

Problema 12.6 Punctele distincte z1, z2, z3, z4 ∈ C ∪ ∞ sunt coliniare sau conciclicedaca si numai daca (z1, z2, z3, z4) ∈ R.

Solutie. Fie f transformarea omografica avand proprietatea ca duce (z2, z3, z4) ın(0, 1,∞), adica f(z) = (z, z2, z3, z4) = (f(z), 0, 1,∞). Urmeaza ca z1, z2, z3, z4 sunt col-iniare sau conciclice ⇐⇒ f(z1), 0, 1,∞ sunt coliniare sau conciclice ⇐⇒ f(z1) ∈ R ⇐⇒(z1, z2, z3, z4) ∈ R.

Observatii. Conditia de coliniaritate se obtine luand z4 = ∞. Deci z1, z2, z3 sunt

coliniare daca si numai dacaz1 − z2

z1 − z3∈ R.

Conditia (z1, z2, z3, z4) > 0 revine la faptul ca z1, z2 se afla pe acelasi arc determinatde z3, z4.

Problema 12.7 Fie b ∈ R si P un polinom cu toate radacinile reale. Se defineste polino-mul Q prin:

Q(z) = P (z + ib) + P (z − ib)

Sa se arate ca si Q are toate radacinile reale

Page 183: Teme si probleme pentru concursurile studentesti de matematica ...

176

Solutie. Cazul b = 0 fiind banal, putem presupune b > 0. Notand xk ∈ R radacinilelui P , se scrie P (z) = A(z − x1) . . . (z − xn) de unde

Q(z) = A [(z + ib− x1) . . . (z + ib− xn) + (z − ib− x1) . . . (z − ib− xn)]

Daca z = xk + ib, atunci z + ib − xj = 2ib + xk − xj 6= 0, deci ın acest caz Q(z) 6= 0. Inrest:

Q(z) = A(z − ib− x1) . . . (z − ib− xn)

[z + ib− x1

z − ib− x1· · · z + ib− xn

z − ib− xn+ 1

]Notand ak = xk − ib, constatam ca Im ak = −b < 0.Un calcul direct arata ca:∣∣∣∣z − az − a

∣∣∣∣2 − 1 =−4(Im z)(Im a)

|z − a|2

Deci, pentru fiecare k = 1, n si pentru Im z > 0:∣∣∣∣z + ib− xkz − ib− xk

∣∣∣∣ > 1

ceea ce arata ca si ın acest caz Q(z) 6= 0. Analog se rationeaza daca Im z < 0. Ramanedeci singura posibilitate: Q(z) = 0 =⇒ z ∈ R.

Problema 12.8 Fie D ⊆ C deschis, nevid, f : D → C o functie olomorfa cu proprietateaca f ′(z) 6= 0, ∀z ∈ D si care verifica relatia:

f ′′′(z)

f ′(z)− 3

2

[f ′′(z)

f ′(z)

]2

= 0, ∀z ∈ D

Sa arate ca f este o transformare omografica.

Solutie. Punand φ = f ′, ecuatia devine:

φ′′

φ− 3

2

(φ′

φ

)2

= 0

Punandφ′

φ= g (corect definita, datorita ipotezei), se obtine φ′′ = g′.φ+ g.φ′, deci ecuatia

devine 2g′ = g2. Aceasta ecuatie admite evident solutia g ≡ 0, care conduce la f(z) =

az + b. In rest, notand g =1

h(cu exceptia unor puncte izolate), ecuatia devine h′ = −1

2 .

Acest caz conduce laφ′(z)

φ(z)=

1

−12z + a

de unde se obtine usor concluzia.Observatie. Expresia:

Sf (z) =f ′′′(z)

f ′(z)− 3

2

[f ′′(z)

f ′(z)

]2

se numeste derivata Schwarziana a functiei f . Se verifica imediat ca orice transformareomografica f satisface Sf ≡ 0.

Page 184: Teme si probleme pentru concursurile studentesti de matematica ...

Functii complexe 177

Problema 12.9 Fie f o transformare omografica. Sa se arate ca, daca f admite ın Cdoua puncte fixe distincte (notate z1, z2), atunci exista α ∈ C, α 6= 0 astfel ıncat:

f(z)− z1

f(z)− z2= α

z − z1

z − z2

Daca f admite ın C puncte fixe confundate (notate z0), atunci exista β ∈ C astfel ıncat

1

f(z)− z0=

1

z − z0+ β

Sa se formuleze si sa se justifice rezultatele corespunzatoare pentru restul cazurilor.Utilizand acest rezultat, sa se arate ca, daca f admite ın C doua puncte fixe distincte,

atunci sirul (zn)n, definit prin recurenta astfel: z0 ∈ C, zn+1 = f(zn) are limita, ın afara

cazului ın caread− bc(a+ d)2

∈ (1/4,∞].

Solutie. Punctele fixe ale transformarii omografice f(z) =az + b

cz + dfiind radacinile

ecuatiei cz2 + (d− a)z − b = 0 distingem cazurile:1) c = d− a = −b = 0; ın acest caz f(z) = z, ∀z ∈ C ∪ ∞ si orice punct este fix.2) c = d − a = 0; b 6= 0. Se obtin translatiile f(z) = z + β. Se poate admite, prin

conventie, ca ∞ este punct fix dublu.3) c = 0, d − a 6= 0. Exista un singur punct fix z0 ∈ C, al doilea fiind ∞. Scriind

f(z) = αz + β, f(z0) = z0 = αz0 + β, obtinem scrierea:

f(z)− z0 = α(z − z0)

( cu α 6= 0). Este vorba deci de rotatii si omotetii ın jurul punctului fix z0.4) c 6= 0. Aici distingem doua subcazuri, dupa cum ecuatia are radacini distincte sau

nu.(i) Daca ∆ = (d− a)2 + 4bc = 0, fie z0 unicul punct fix (dublu). Atunci:

1

f(z)− z0− 1

z − z0=

1

az + b

cz + d− az0 + b

cz0 + d

− 1

z − z0=

=c(cz0 + d)z + cdz0 + d2 − ad+ bc

(ad− bc)(z − z0)

este constant, deoarece:

(−z0)c(cz0 + d)− (cdz0 + d2 − ad+ bc) = 0

folosind faptul ca z0 =a− d

2c.

(ii) Daca ∆ 6= 0, fie z1 6= z2 ∈ C radacinile ecuatiei. In loc sa verificam prin calcul ca

f(z)− z1

f(z)− z2:z − z1

z − z2

este constant, sa observam ca f este (unica) transformare omografica care aplica zk ın zkpentru k = 1, 2 iar f(∞) = a/c. Astfel, f are forma:

f(z)− z1

f(z)− z2:f(∞)− z1

f(∞)− z2=z − z1

z − z2

Page 185: Teme si probleme pentru concursurile studentesti de matematica ...

178

deci α =f(∞)− z1

f(∞)− z2.

Pentru studiul sirului, sa notam cu l1, l2 cele doua puncte fixe. Din relatia:

f(z)− l1f(z)− l2

= αz − l1z − l2

deducem ca:zn − l1zn − l2

= αnz0 − l1z0 − l2

de unde:

zn =l1(z0 − l2)− αnl2(z0 − l1)

(z0 − l2)− αn(z0 − l1)

Acest sir este convergent daca si numai daca |α| 6= 1, conditie echivalenta cu cea din enunt.

Problema 12.10 (Teorema lui Lucas) Fie P o functie polinomiala. Sa se arate caradacinile lui P ′ se gasesc ın cel mai mic poligon convex ce contine radacinile lui P .

Solutie. Fiecare poligon convex este intersectia unui numar finit de semi-plane. Estedeci suficient sa aratam ca, daca toate radacinile lui P , sa le notam z1, . . . , zn , se aflaıntr-un anumit semi-plan, atunci P ′ nu se anuleaza ın nici un punct din celalalt semi-plan.Este convenabil sa descriem semi-planul prin z ∈ C | Im a(z − b) ≤ 0, cu a, b ∈ C, a 6= 0.Presupunem deci ca Im a(zk − b) ≤ 0, ∀k = 1, n. Fie z din celalalt semi-plan, adicaIm a(z − b) > 0. Deoarece:

P ′(z)

P (z)=

n∑k=1

1

z − zk

deducem:

ImP ′(z)

aP (z)=

n∑k=1

Im1

a(z − zk)=

n∑k=1

1

|a(z − zk)|2Im a(z − zk) =

= −n∑k=1

1

|a(z − zk)|2[Im a(z − b)− Im a(zk − b)] < 0

In particular P ′(z) 6= 0.Observatie. Analizand calculul facut, constatam ca radacinile lui P ′ se afla chiar ın

interiorul poligonului, cu exceptia urmatoarelor situatii:(i) P are radacini multiple.(ii) P are toate radacinile coliniare.

Problema 12.11 Un polinom cu coeficienti reali se numeste stabil daca toate radacinilesale au partea reala negativa.

a) Daca P este stabil, atunci si P ′ este stabil.b) (i) Daca P (X) = X2 + aX + b atunci P este stabil daca si numai daca a > 0 si

b > 0.(ii) Daca P (X) = X3 + aX2 + bX + c atunci P este stabil daca si numai daca a >

0, b > 0, c > 0 si a.b > c.(iii) Daca P (X) = X4 + aX3 + bX2 + cX + d atunci P este stabil daca si numai daca

a > 0, b > 0, c > 0, d > 0 si abc > a2d+ c2.c) Fie P (X) = anX

n + an−1Xn−1 + . . .+ a1X + a0 un polinom cu coeficienti reali. Sa

se arate ca, daca P este stabil, atunci akak+3 < ak+1ak+2, pentru orice k = 0, 1, . . . , n− 3.

Page 186: Teme si probleme pentru concursurile studentesti de matematica ...

Functii complexe 179

IMC, 2003

Solutie. a) Consecinta a teoremei lui Lucas.b) (i) Daca P are radacini reale, conditia este ca ambele sa fie negative, ceea ce este

echivalent cu a > 0 si b > 0. Daca P are radacini nereale, fie acestea z1 si z2, atuncib = z1z2 > 0 iar −a = z1 + z2 = 2Re z1 < 0.

(ii) P admite cel putin o radacina reala, fie aceasta x1. Scriind:

P (X) = (X − x1)(X2 + pX + q)

deducem ca P este stabil daca si numai daca x1 < 0, p > 0, q > 0. Insa a = p − x1; b =q − px1; c = −qx1. Echivalenta cu proprietatile din enunt este acum imediata.

(iii) In acest caz scriem

P (X) = (X2 + pX + q)(X2 + rX + s),

deci conditia este p > 0, q > 0, r > 0, s > 0. Pe de alta parte, a = p+ r; b = s+ pr + q;c = ps+ qr; d = qs. Tinand seama ca

abc− a2d− c2 = pr[(q − s)2 + (p+ r)(ps+ qr)

]echivalenta cu proprietatile din enunt rezulta din nou cu usurinta.

c) Scriem descompunerea polinomului P :

P (X) =∏i

(kiX + li)∏j

(pjX2 + qjX + rj)

unde ki, li, pj , qj , rj ∈ R. Din ipoteza, pentru fiecare i, ki si li au acelasi semn; iar pentrufiecare j, pj , qj , rj au de asemenea acelasi semn. Inmultind eventual P cu −1, putempresupune ca are toti coeficientii pozitivi. Pentru simplificarea notatiilor, extindem sirulcoeficientilor astfel: an+1 = an+2 = . . . = 0 si a−1 = a−2 = ... = 0. Demonstratia seface prin inductie, pentru −1 ≤ k ≤ n − 2. Cazurile n = 2, 3 au fost deja verificate. Fien ≥ 3 si sa presupunem ca afirmatia este valabila pentru toate valorile mai mici ale lui n.Sa consideram un factor al lui P , de forma X2 + pX + q unde p si q sunt numere realepozitive. Adica P (X) = (X2 + pX + q)(bn−2X

n−2 + ...+ b1X + b0) = (X2 + pX + q)Q(X).Toate radacinile polinomului Q au partea reala negativa, deci conform ipotezei inductiveavem bk+1bk+2 < bkbk+3, ∀ − 1 ≤ k ≤ n − 4. Definind analog bn−1 = bn = ... = 0 sib−1 = b−2 = ... = 0 , inegalitatea precedenta are loc pentru toate valorile lui k. Sa aratamacum ak+1ak+2 > akak+3. Pentru k = −1 sau k = n − 2 este evident, deoarece ak+1ak+2

este pozitiv iar akak+3 = 0. Mai departe, presupunem 0 ≤ k ≤ n− 3. Dar

ak+1ak+2 − akak+3 = (qbk+1 + pbk + bk−1)(qbk+2 + pbk+1 + bk)−

−(qbk + pbk−1 + bk−2)(qbk+3 + pbk+2 + bk+1) = (bk−1bk − bk−2bk+1)+

+p(b2k − bk−2bk+2) + q(bk−1bk+2 − bk−2bk+3) + p2(bkbk+1 − bk−1bk+2)+

+q2(bk+1bk+2 − bkbk+3) + pq(b2k+1 − bk−1bk+3)

Aratam acum ca fiecare din cei sase termeni este pozitiv, iar cel putin unul este strictpozitiv. Din ipoteza inductiva p2(bkbk+1 − bk−1bk+2) > 0 iar bk−1bk − bk−2bk+1 ≥ 0 siq2(bk+1bk+2 − bkbk+3) ≥ 0. Pentru a confirma semnul expresiei p(b2k − bk−2bk+2) scriembk−1(b2k − bk−2bk+2) = bk−2(bkbk+1− bk−1bk+2) + bk(bk−1bk − bk−2bk+1) ≥ 0. If bk−1 > 0 se

Page 187: Teme si probleme pentru concursurile studentesti de matematica ...

180

obtine b2k−bk−2bk+2 ≥ 0. Daca nu, din bk−1 = 0 rezulta fie bk−2 = 0 sau bk+2 = 0. In ambelecazuri b2k−bk−2bk+2 = b2k ≥ 0. Deci, p(b2k−bk−2bk+2) ≥ 0. Similar, pq(b2k+1−bk−1bk+3) ≥ 0.Semnul expresiei q(bk−1bk+2 − bk−2bk+3) se verifica analog. Consideram bk+1(bk−1bk+2 −bk−2bk+3) = bk−1(bk+1bk+2 − bkbk+3) + bk+3(bk−1bk − bk−2bk+1) ≥ 0. Discutam din noucazurile: bk+1 > 0, cand putem ımparti prin bk+1. Altfel, fie bk−2 = 0 sau bk+3 = 0. Inambele cazuri, obtinem bk−1bk+2 − bk−2bk+3 ≥ 0, ceea ce ıncheie demonstratia.

Problema 12.12 Pentru n ≥ 1 notam:

En(z) = (1− z) exp

(z +

z2

2+ . . .+

zn

n

)Sa se arate ca |1− En(z)| ≤ |z|n+1, ∀|z| < 1

Solutie. Consideram dezvoltarea ın serie de puteri:

En(z) = 1 +

∞∑k=1

akzk

Prin derivare, se obtine:

−zn exp

(z +

z2

2+ . . .+

zn

n

)=

∞∑k=1

kakzk−1

Identificand coeficientii, deducem ca a1 = a2 = . . . = an = 0 iar an+k ≤ 0, ∀k ≥ 1. Pe dealta parte

0 = En(1) = 1 +

∞∑k=1

an+k

conduce la:∞∑k=1

|an+k| = −∞∑k=1

an+k = 1

In concluzie, daca |z| ≤ 1, obtinem:

|1− En(z)| =

∣∣∣∣∣∞∑k=1

an+kzn+k

∣∣∣∣∣ = |z|n+1

∣∣∣∣∣∞∑k=1

an+kzk−1

∣∣∣∣∣ ≤≤ |z|n+1

∞∑k=1

|an+k| = |z|n+1

Problema 12.13 Sa se justifice ca exista o functie olomorfa pe mulctimea U = z ∈C||z| > 4 a carei derivata este

z

(z − 1)(z − 2)(z − 3).

Exista o functie olomorfa ın U a carei derivata estez2

(z − 1)(z − 2)(z − 3)?

Berkeley, 1978

Page 188: Teme si probleme pentru concursurile studentesti de matematica ...

Functii complexe 181

Solutie. Deoarece daca G(z) = F(

1z

), atunci G′(z) = − 1

z2F ′(

1z

), prima chestiune

revine la a gasi o functie G, olomorfa ın discul centrat ın 0 si de raza 14 , astfel ıncat

G′(z) =1

(1− z)(1− 2z)(1− 3z)

O asemenea functie exista, orice functie olomorfa ıntr-un disc admitand primitive.La a doua ıntrebare raspunsul este negativ, caci 1

z(1−z)(1−2z)(1−3z) nu este olomorfa ındisc.

Problema 12.14 Fie f : C→ C o functie olomorfa, cu proprietatea |f(z)| = | sin z|. Sase arate ca exista o constanta C de modul 1 astfel ıncat f(z) = C sin z, ∀z ∈ C.

Solutie. Functiaf(z)

sin zeste olomorfa ın multimea deschisa si conexa D := C \ kπ|k ∈

Z . Deoarece∣∣∣ f(z)

sin z

∣∣∣ = 1, ∀z ∈ D, concluzia rezulta din urmatorul rezultat general:

Lema Fie D ⊆ C deschis conex, iar f : D → C o functie olomorfa. Daca |f | esteconstanta ın D, atunci functia f este constanta ın D.

Demonstratie. Daca |f | = 0, atunci f = 0 ın D. Fie deci |f |2 = u2 + v2 = C 6= 0.Deducem:

u∂u

∂x+ v

∂v

∂x= 0

u∂u

∂y+ v

∂v

∂y= 0

Folosind conditiile Cauchy–Riemann, se obtine:u∂u

∂x− v∂u

∂y= 0

u∂u

∂y+ v

∂u

∂x= 0

De unde (u2 + v2)∂u

∂x= 0 si deci concluzia.

La acelasi rezultat se ajunge cu mai putine calcule, daca folosim o transformare omo-grafica T , care aplica cercul |z| = C pe axa reala (de ex. Tz := i z−Cz+C ). Astfel, functiaolomorfa T f ia numai valori reale. Avand partea imaginara identic 0, conditiile Cauchy-Riemann asigura ca partea reala este constanta.

Observatie. De fapt, functiaf(z)

sin zeste olomorfa ın C, deoarece f(kπ) = 0, ceea ce

arata ca fiecare punct kπ este o singularitate aparenta. Astfel, aceasta functie olomorfaeste si maginita ın C iar concluzia rezulta pe baza teoremei lui Liouville.

Problema 12.15 Pentru fiecare numar complex z 6∈ 0, 1 definim

f(z) :=∑ 1

(log z)4

suma fiind extinsa la toate ramurile logaritmului. Sa se arate ca

f(z) =z3 + 4z2 + z

6(z − 1)4

∀z 6∈ 0, 1.

Page 189: Teme si probleme pentru concursurile studentesti de matematica ...

182

IMC, 2004

Solutia 1. Este vorba despre seria∑k∈Z

1

(ln |z|+ i arg z + 2kπi)4

cu arg z ∈ (−π, π]. Seria modulelor este∑k∈Z

1

((ln |z|)2 + (arg z + 2kπ)2)2

Aceasta serie este evident convergenta, deci ın seria initiala nu conteaza ordinea de sumare.De asemenea, seria este uniform convergenta pe compacte din C\0, 1. In adevar, pentrufiecare compact din C \ 0, 1, exista 0 < α < 1 < A < ∞ pentru care z ∈ K ⇒α ≤ |z| ≤ A. Pentru orice k 6= 0, putem majora temenul general cu

1

(C2 + (2|k| − 1)2)2

unde C := min (− lnα, lnA).Avand ın vedere ca la traversarea semiaxei negative, determinarile se permuta ciclic,

deducem ca suma seriei este chiar functie olomorfa ın C \ 0, 1.Studiem natura singularitatilor izolate 1, 0 si ∞.Observam ca 1 este pol de ordin 4: ın afara de termenul corespunzator lui k = 0, toti

sunt functii olomorfe ın discul centrat ın 1 si de raza 1.Pentru ∞ observam urmatoarea majorare, pentru |z| > 1:∑

k∈Z

1

((ln |z|)2 + (arg z + 2kπ)2)2≤ 1

∑k∈Z

∫ (2k+1)π

(2k−1)π

dt

((ln |z|)2 + t2)2=

=1

∫ ∞−∞

dt

((ln |z|)2 + t2)2=

1

2π(ln |z|)3

∫ ∞−∞

dt

(1 + t2)2≤ C

(ln |z|)3

De aici rezulta ca limz→∞

f(z) = 0 astfel ca ∞ este o singularitate aparenta pentru f , cu

valoarea 0.Pentru z = 0 este suficient sa observam ca fiecare determinare pentru log 1

z este o altadeterminare pentru (− log z). Adica are loc relatia f(1/z) = f(z), ceea ce arata ca z = 0este de asemenea o singularitate aparenta pentru f , cu valoarea 0.

Astfel f este functie meromorfa ın planul complex extins, deci este o functie rationala.Avand doar pol de ordin 4 ın z = 1, deducem ca exista P polinom astfel ıncat f(z) = P (z)

(z−1)4.

Deoarece ∞ este singularitate aparenta cu valoarea 0, rezulta ca P este polinom de grad< 4. z = 0 fiind de asemenea singularitate aparenta cu valoarea 0, rezulta ca termenul liberal lui P este 0. Tinand cont si de relatia f(1/z) = f(z), deducem ca P (z) = az3 +bz2 +az.Pentru a determina valorile a = 1/6, b = 2/3 exista mai multe posibilitati.

Prima posibilitate este sa dam valori; pentru z = −1 scriem

f(−1) =∑k∈Z

1

((2k − 1)π)4= 2π−4

∞∑n=1

1

(2n− 1)4=

1

48

de unde 2a− b = −1/3; sa observam ca putem calcula

limz→1

(z − 1)4f(z) = 1

Page 190: Teme si probleme pentru concursurile studentesti de matematica ...

Functii complexe 183

de unde 2a+ b = 1.Cu mai multe calcule, putem determina partea principala a dezvoltarii Laurent pentru

f ın z = 1. Folosind scrierea

ln(1 + w) = w − w2

2+w3

3− w4

4+ . . .

deducem partea principala pentru 1(ln(1+w))4

ca fiind

w−4 + 2w−3 +7

6w−2 +

1

6w−1

iar aceasta este si partea principala pentru f(1 + w). Astfel

f(z) =1 + 2(z − 1) + 7

6(z − 1)2 + 16(z − 1)3

(z − 1)4=z3 + 4z2 + z

6(z − 1)4

O alta posibilitate este sa observam ca are loc relatia f(z) + f(−z) = 16f(z2): de-terminarile pentru log(z2) = log((−z)2) sunt 2 log(z) si 2 log(−z). Din aceasta observatieobtinem b = 4a.

O cale complet diferita, care sugereaza si o a doua solutie, este sa plecam de la

g(z) :=∑ 1

(log z)2

Ca mai sus, obtinem

g(z) =dλz

(z − 1)2

iar λ = 1 se gaseste usor cu una din metodele deja utilizate. Mai departe se deriveaza dedoua ori.

Solutia 2. Pornim de la dezvoltarea cotangentei ın fractii simple

cotg w =1

w+

∞∑n=1

(1

w + nπ+

1

w − nπ

)care se poate scrie

1

2icotg

w

2i=

1

w+

∞∑n=1

(1

w + 2nπi+

1

w − 2nπi

)Inlocuind w = ln |z|+ i. arg z avem

1

2icotg

ln z

2i=

1

2iie2i ln z

2i + 1

e2i ln z2i − 1

=1

2+

1

z − 1

Aici definitia sumarii este esentiala!Prin derivari succesive, deducem:∑ 1

(log z)2= −z

(1

2+

1

z − 1

)′=

z

(z − 1)2

∑ 1

(log z)3=

z2 + z

2(z − 1)2∑ 1

(log z)4=z3 + 4z2 + z

6(z − 1)4

Page 191: Teme si probleme pentru concursurile studentesti de matematica ...

184

Problema 12.16 Fie p(z) = a0 + a1z + a2z2 + . . . + anz

n un polinom cu coeficienticomplecsi. Fie (ck)k=0,...,n un sir convex (adica 2ck ≤ ck−1 + ck+1 pentru fiecare k =1, 2, ..., n − 1) si 1 = c0 ≥ c1 ≥ . . . ≥ cn ≥ 0. Definim polinomul q(z) = c0a0 + c1a1z +c2a2z

2 + . . .+ cnanzn. Sa se arate ca

max|z|≤1

|q(z)| ≤ max|z|≤1

|p(z)|

IMC, 2009

Solutie. Din principiul de maxim, avem

Mp := max|z|=1

|p(z)| = max|z|≤1

|p(z)|

Avem de aratat ca Mq ≤ Mp. Observam pentru ınceput ca putem presupune cn = 0.In adevar, daca cn = 1, atunci p = q si afirmatia este banala. In rest, q(z) = cnp(z) +

(1− cn)r(z), unde r(z) =n∑j=0

cj − cn1− cn

ajzj . Sirul c′j :=

cj − cn1− cn

satisface conditiile si ın plus

c′n = 0. Iar daca stim ca Mr ≤Mp , atunci rezulta

Mq = |q(z0)| ≤ cn|p(z0)|+ (1− cn)|r(z0)| ≤ cnMp + (1− cn)Mr ≤Mp

Revenim la demonstratie, ın cazul cn = 0. Pe baza formulei lui Cauchy:

aj =1

2πi

∫|z|=1

p(z)

zj+1dz =

1

∫|z|=1

p(z)z−j |dz|

Folosind aceasta formula, putem exprima polinomul q astfel:

2πq(w) =

n∑j=0

cj

(∫|z|=1

p(z)z−j |dz|

)wj

Adunand expresii similare, dar care sunt 0 datorita olomorfiei, pentru −n ≤ j ≤ −1,obtinem ın continuare:

2πq(w) =n∑

j=−nc|j|

(∫|z|=1

p(z)z−j |dz|

)wj =

∫|z|=1

n∑j=−n

c|j|(w

z)j

p(z)|dz|

Sa introducem notatia

K(u) :=

n∑j=−n

c|j|uj = c0 +

n∑j=1

cjRe uj

Pentru a termina demonstratia, aratam ca functia K ia numai valori pozitive. Pentruaceasta, se verifica prin inductie scrierea

K(u) =n∑k=1

dkFk(u)

unde dk := ck−1 − 2ck + ck+1 ≥ 0 iar

Fk(u) :=

k−1∑j=−k+1

(k − |j|)uj

Page 192: Teme si probleme pentru concursurile studentesti de matematica ...

Functii complexe 185

In sfarsit, scrierea

Fk(u) = (1 + u+ . . .+ uk−1)(1 + u−1 + . . .+ u−(k−1)) =∣∣∣1 + u+ . . .+ uk−1

∣∣∣2 ≥ 0

arata ca Fk ≥ 0, deci K ≥ 0.Deoarece avem ∫

|z|=1|K(u)||du| =

∫|z|=1

K(u)|du| = 2πc0 = 2π

putem finaliza:

2π|q(w)| =

∣∣∣∣∣∫|z|=1

K(wz

)p(z)|dz|

∣∣∣∣∣ ≤∫|z|=1

|K(wz

)||p(z)||dz| ≤

≤Mp

∫|z|=1

|K(u)||du| = 2πMq

Problema 12.17 Notam D = C \ z ∈ C | Re z = 0, |Im z| ≥ 1 si definim f : D → Cprin:

f(z) =1

2iln

1 + iz

1− iz(determinarea principala).

(i) Sa se verifice ca functia f este olomorfa ın D iar f ′(z) =1

z2 + 1, ∀z ∈ D.

(ii) Daca x ∈ R, atunci f(x) = arctg x.(iii) Notam D1 = w ∈ C | Re w ∈ (−π/2, π/2). Sa se arate ca f = ( tg |D1)−1

Solutie. (i) Pentru ca f sa fie definita si olomorfa, se impun conditiile: z 6= −i si1 + iz

1− iz6∈ (−∞, 0], care sunt echivalente cu z ∈ D.

f ′(z) =1

2i

(1 + iz

1− iz

)′1 + iz

1− iz

=1

1 + z2

(ii) Deoarece

∣∣∣∣1 + ix

1− ix

∣∣∣∣ = 1, deducem ca:

f(x) =1

2iln

1 + ix

1− ix=

1

2i

(ln

∣∣∣∣1 + ix

1− ix

∣∣∣∣+ i arg1 + ix

1− ix

)=

1

2arg

1 + ix

1− ix

Daca notam θ = arg1 + ix

1− ix, atunci θ ∈ (−π, π), cos θ =

1− x2

1 + x2, sin θ =

2x

1 + x2, de unde

θ/2 ∈ (−π/2, π/2) si tg θ/2 =sin θ

1 + cos θ= x, adica exact θ/2 = arctg x.

(iii) Daca z ∈ D, s-a observat ca1 + iz

1− iz6∈ (−∞, 0], deci

Re f(z) =1

2arg

1 + iz

1− iz∈ (−π/2, π/2)

Page 193: Teme si probleme pentru concursurile studentesti de matematica ...

186

adica f(z) ∈ D1, ∀z ∈ D. Astfel, putem calcula:

tg f(z) =e2if(z) − 1

i(e2if(z) + 1)=

1 + iz

1− iz− 1

i

(1 + iz

1− iz+ 1

) = z, ∀z ∈ D

Reciproc, ecuatia tg w = iy (y ∈ R) conduce la e2iw =1− y1 + y

. Daca |y| ≥ 1, atunci

1− y1 + y

≤ 0, ceea ce arata ca w ∈ D1 =⇒ tg w ∈ D. Astfel, pentru w ∈ D1 putem calcula:

f(tg w) =1

2iln

1 + itg w

1− itg w=

1

2iln

cosw + i sinw

cosw + i sinw=

1

2iln e2iw = w.

Observatie. Aceste proprietati justifica definirea functiei arctg ca f de mai sus.

Problema 12.18 Sa se calculeze ∫ 2π

0cos2n xdx

Solutie. Folosind cosx =eix + e−ix

2gasim:

∫ 2π

0cos2n xdx = 2−2n

∫ 2π

0(eix + e−ix)2ndx =

= 2−2n

∫ 2π

0

2n∑k=0

Ck2nei(2n−2k)xdx = 22n−1πCn2n

Problema 12.19 Sa se gaseasca toate functiile f , definite si continue ın multimea |z| ≤ 2,olomorfe ın multimea |z| < 2 si care verifica relatia:

1

2πi

∫|ζ|=2

(ζ +1

ζ)f(ζ)

(ζ − z)2= z,∀|z| < 2

Solutie. Descompunand ın fractii simple (sau cu teorema reziduurilor) relatia dinenunt se scrie:

f(0)

z2+ (1− 1

z2)f(z) + (z +

1

z)f ′(z) = z

sau: [(z +

1

z)f(z)

]′=

(z2

2+f(0)

z

)′de unde:

(z +1

z)f(z) =

z2

2+f(0)

z+ C

deci:

f(z) =z3 + 2Cz + 2f(0)

2(z2 + 1)

Punand conditia de olomorfie (∀|z| < 2), se obtine f(0) = i(12 − C), de unde C = 1

2 sif(0) = 0. Deci f(z) = z, ∀|z| ≤ 2 este singura functie care convine.

Page 194: Teme si probleme pentru concursurile studentesti de matematica ...

Functii complexe 187

Problema 12.20 Fie dezvoltarea ın serie de puteri

ln(1 + ez) =

∞∑n=0

anzn

(i) Sa se afle raza de convergenta a seriei de puteri.(ii) Sa se arate ca a0 = ln 2; a1 = 1/2 iar a2n+1 = 0, ∀n ≥ 1.(iii) Sa se stabileasca o formula de recurenta pentru coeficientii a2n.

Solutie. (i) ln(1 + ez) este functie olomorfa ın C, mai putin semi-dreptele Re z >0, Im z = (2k + 1)π. Deducem ca raza de convergenta a seriei de puteri este π.

(ii) Prin derivare, se obtine functiaez

1 + ez, care este para, cu valoarea ın 0 egala cu 1

2 .

Sau, se poate folosi identitatea: ln(1 + ez)− ln(1 + e−z) = z.(iii) Relatia de recurenta se stabileste, identificand ın expresia derivatei:

ez

1 + ez=∞∑n=1

nanzn−1, ∀|z| < π

adica:∞∑n=0

zn

n!= (2 +

∞∑n=1

zn

n!)(∞∑n=1

nanzn−1), ∀|z| < π

de unde a2 = 1/8 iar

2na2n +n− 1

2!a2n−2 +

n− 2

4!a2n−4 + . . .+

1

(2n− 2)!a2 =

1

4(2n− 1)!

Problema 12.21 Fie f : C→ C o functie olomorfa, cu proprietatea ca exista n ∈ N,astfel ıncat f(ze2πi/n) = f(z), ∀z ∈ C. Sa se arate ca exista o functie olomorfa g : C→ C,astfel ıncat f(z) = g(zn), ∀z ∈ C

Solutie. Deoarece f(z) =

∞∑m=0

amzm, ipoteza arata ca:

f(z) = f(ze2πi/n) =

∞∑m=0

amzme2mπi/n

Identificand coeficientii, se obtine ca am 6= 0 daca si numai daca n divide m, adica f(z) =∞∑m=0

amn(zn)m.

Problema 12.22 (i) Fie n ∈ N fixat. Sa se arate ca functia:

f(z) =sinπz

πz(1− z)(1− z

2) . . . (1− z

n)

are doar singularitati aparente ın C.Rezulta ca exista o (unica) functie olomorfa g : C → C, astfel ıncat g2(z) = f(z),

∀z ∈ C si g(0) = 1.

Page 195: Teme si probleme pentru concursurile studentesti de matematica ...

188

(ii) Fie d dreptunghiul cu varfurile: −1 + δ± iA si n+ 1− δ± iA (unde δ ∈ (0, 1)). Sase arate ca: ∫

d

g(z)

sinπzdz = 2i

n∑k=0

(−1)k√Ckn

(iii) Folosind o majorare convenabila a integralei, sa se arate ca

limn→∞

n∑k=0

(−1)k√Ckn = 0

Solutie. (i) Deoarece limz→0

sinπz

πz= 1 iar

∀k = 1, n : limz→k

sinπz

1− z

k

= (−1)k+1πk

deducem ca f are doar singularitati aparente ın C. In plus,

limz→k

f(z) = Ckn

C fiind simplu conex, existenta (si unicitatea ) lui g sunt cunoscute.

(ii) Cu cele demonstrate, functiag(z)

sinπzare ın interiorul dreptunghiului d numai sin-

gularitati izolate ın k = 0, 1, . . . , n, fiecare fiind pol simplu. Deci:

Rez

(g(z)

sinπz, k

)=

g(k)

(−1)kπ=

(−1)k

π

√Ckn.

(iii) Pe laturile orizontale z = x± iA, deci:

| sinπz| ≥ 1

2(eπA − 1) si |1− z

k| ≥ A

k− 1

Rezulta: ∣∣∣∣ g(z)

sinπz

∣∣∣∣2 =

∣∣∣∣∣∣ 1

sinπz · πz(1− z) · · · (1− z

n)

∣∣∣∣∣∣ ≤ 1

12(eπA − 1)πA · · · (A

n− 1)

deci integralele pe cele doua laturi orizontale tind la 0 cand A→∞.Evaluarea pe latura verticala Re z = n+1−δ se reduce la cea pe latura Re z = −1+δ,

prin schimbarea z → n− z. Intr-adevar sinπ(n− z) = (−1)n+1 sinπz, iar:

(n− z)(1− n+ z) · · · (1− n− zn

) = (1− z

n)(1− z

n− 1) · · · z(−1)n−1.

Acum, integrala pe latura verticala ramasa, se majoreaza prin:∣∣∣∣∫ A

−A

g(−1 + δ + iy)idy

sinπ(−1 + δ + iy)

∣∣∣∣ ≤ ∫ A

−A

∣∣∣∣ g(−1 + δ + iy)

sinπ(−1 + δ + iy)

∣∣∣∣ dyDeoarece integrala

∫ ∞−∞

dy

| sinπ(−1 + δ + iy)|este convergenta, ramane sa minoram:

|z(1− z)(1− z

2) . . . (1− z

n)| ≥ (1− δ)(2− δ) . . . (1− δ − 1

k) . . . (1− δ − 1

n)

Page 196: Teme si probleme pentru concursurile studentesti de matematica ...

Functii complexe 189

Sa notam α = 1 − δ ∈ (0, 1). Folosind faptul ca ln(1 +α

k) ≥ α

k− α2

2k2, precum si faptul

ca 1 + 12 + . . . 1

n − lnn→ γ, deducem ca exista o constanta C (independenta de α), astfelıncat Cnα minoreaza produsul. Se obtine astfel ıncadrarea:

0 ≤n∑k=0

(−1)k√Ckn ≤

C√n

Observatie. Evident, pentru n imparn∑k=0

(−1)k√Ckn = 0.

Problema 12.23 Sa se arate ca:

1

2πi

∫|z|=1

e

x

2(z +

1

z)dz =

∞∑n=0

(x2

)2n+1 1

n!(n+ 1)!

Solutie. Avem:

1

2πi

∫|z|=1

e

x

2(z +

1

z)dz = Rez

ex2 (z +1

z), 0

Reziduul se determina scriind:

e

x

2(z +

1

z)

=

( ∞∑n=0

xnzn

2nn!

)( ∞∑n=0

xn

2nn!zn

)si identificand coeficientul lui z−1.

Problema 12.24 Integrand functia:1

z3 cosπzpe patratul γn de varfuri ±n ± in, sa se

determine

∞∑n=0

(−1)n

(2n+ 1)3.

Solutie. Aplicand teorema reziduurilor, avem:∫γn

dz

z3 cosπz= 2πi

(n−1∑k=−n

Rez(f, k +1

2) + Rez(f, 0)

)=

= 2πi

[−2

n−1∑k=0

(−1)k

π(k + 12)3

+π2

2

]caci:

Rez(f, 0) =1

2!

[d2

dz2

1

cosπz

]z=0

=

[π2

2 cos3 πz

]z=0

Apoi

| cosπz|2 =1

4

[e2πy + e−2πy + 2 cos 2πx

].

Deci, daca x = ±n, atunci

| cosπz|2 =1

4(eπy + e−πy)2 ≥ 1.

Daca y = ±n, atunci, pentru n suficient de mare | cosπz|2 ≥ 1. Deci∫γn

dzz3 cosπz

→ 0

pentru n→∞, de unde rezulta ca suma ceruta esteπ3

32.

Page 197: Teme si probleme pentru concursurile studentesti de matematica ...

190

Problema 12.25 Integrand functia zα−1eiz (determinarea principala, α ∈ (0, 1)) pecurba γ formata din arcele: segmentul [ε,A]; arcul de cerc de centru 0 si de raza A,situat ın primul cadran; segmentul de extremitati iA si iε; arcul de cerc de centru 0 si deraza ε (unde 0 < ε < A <∞), sa se calculeze valorile integralelor:∫ ∞

0xα−1 sinx dx ;

∫ ∞0

xα−1 cosx dx

Solutie. Functia fiind olomorfa ın interiorul curbei γ, rezulta ca:

0 =

∫γzα−1eiz dz =

∫ A

εxα−1eix dx+

∫CA

zα−1eiz dz−

−∫ A

εxα−1e−xei.απ/2 dx−

∫Cε

zα−1eiz dz

Cu lema lui Jordan, deducem ca:

limA→∞

∫CA

zα−1eiz dz = 0

Apoi: ∣∣∣∣∫Cε

zα−1eiz dz

∣∣∣∣ ≤ πεα

2→ 0

cand ε→ 0. Egaland partile reala si imaginara, se obtine:∫ ∞0

xα−1 cosx dx = cosαπ

2Γ(α)

∫ ∞0

xα−1 sinx dx = sinαπ

2Γ(α)

Problema 12.26 Fie −1 < a < b. Integrand functia

(z +

1

z

)azb−1 pe curba ınchisa γ

formata din arcele: arcul de cerc de centru 0 si de raza 1, situat ın semiplanul Re z > 0;segmentele de extremitati i(1− ε) si iε, respectiv i(−1 + ε) si −iε; arcele de cerc de centre0, i si −i de raza ε (unde 0 < ε < 1), sa se arate ca:∫ π

2

0cosa t cos bt dt =

πΓ(a+ 1)

2a+1Γ(a+b2 + 1)Γ(a−b2 + 1)

Solutie. Functia data fiind olomorfa ın domeniul limitat de curba γ, gasim:

0 = −∫γi

(z +1

z)azb−1dz −

∫ 1−ε

ε(1

x− x)axb−1eiπ/2(b−a−1)dx−

−∫γ0

(z +1

z)azb−1dz +

∫ 1−ε

ε(1

x− x)axb−1eiπ/2(a−b+1)dx+

∫ π/2

−π/2cosa t(cos(b− 1)t+ i sin(b− 1)t)i(cos t+ i sin t)dt−

∫γ−i

(z +1

z)azb−1dz

Page 198: Teme si probleme pentru concursurile studentesti de matematica ...

Functii complexe 191

Apoi: ∣∣∣∣∣∫γi

(z +1

z)azb−1dz

∣∣∣∣∣ ≤ C.π4 εεa∣∣∣∣∣∫γ−i

(z +1

z)azb−1dz

∣∣∣∣∣ ≤ Cπ4 εεa∣∣∣∣∣∫γ0

(z +1

z)a.zb−1dz

∣∣∣∣∣ ≤ Cπ2 εεb−a−1

de unde: ∫ π2

−π2

cosa t cos bt dt = 2 sinπ

2(b− a− 1)

∫ 1

0(x− 1

x)axb−1dx

si deci rezultatul propus.

Problema 12.27 (i) Integrand functia

e3iz − 3eiz + 2

z3

pe curba γ formata din arcele: segmentele [−A,−ε] si [ε,A] si semicercurile centrate ın 0,de raze respectiv ε si A, situate ın semiplanul Im z > 0,, sa se arate ca:∫ ∞

0

sin3 x

x3dx =

8

(ii) Integrand functia:

1− eiz

z2(z2 + 1)

pe curba de mai sus, sa se arate ca :∫ ∞−∞

1− cosx

x2(x2 + 1)dx =

π

e

(iii) Integrand functia:

e2aiz − e2biz

z2, a > 0, b > 0

pe curba γ de mai sus, sa se deduca valoarea integralei (Froullani):∫ ∞0

cos 2ax− cos 2bx

x2dx

Solutie. (i) Daca x ∈ R, atunci:

e3ix − 3eix + 2

x3=

cos 3x− 3 cosx+ 2

x3+ i

sin3 x

x3

Deducem ca:

0 =

∫γ

e3iz − 3eiz + 2

z3dz =

∫ −ε−A

(cos 3x− 3 cosx+ 2

x3+ i

sin3 x

x3

)dx+

Page 199: Teme si probleme pentru concursurile studentesti de matematica ...

192

+

∫ A

ε

(cos 3x− 3 cosx+ 2

x3+ i

sin3 x

x3

)dx−

∫Cε

e3iz − 3eiz + 2

z3dz+

+

∫CA

e3iz − 3eiz + 2

z3dz

Insa: ∣∣∣∣∫CA

e3iz − 3eiz + 2

z3dz

∣∣∣∣ ≤ πA 6

A3→ 0

cand A→∞. Deoarece:

e3iz − 3eiz + 2

z3= −3

z+ f(z)

cu f olomorfa ıntr-o vecinatate a originii, urmeaza (cf. teorema semireziduurilor):∫Cε

e3iz − 3eiz + 2

z3dz = (−3)πi+

∫Cε

f(z)dz

iar ∫Cε

f(z)dz → 0

cand ε→ 0. Se obtine astfel:∫ ∞0

sin3 x

x3dx =

1

2

∫ ∞−∞

sin3 x

x3dx = −1

8

∫ ∞−∞

sin 3x− 3 sinx

x3dx =

8

(ii) Aplicand teorema reziduurilor, obtinem:∫γ

1− eiz

z2(z2 + 1)dz = 2πiRez(

1− eiz

z2(z2 + 1), i) = 2πi

1− e−1

(−1)2i

Pe de alta parte:∫γ

1− eiz

z2(z2 + 1)dz =

∫ −ε−A

1− cosx− i sinx

x2(x2 + 1)dx−

∫Cε

1− eiz

z2(z2 + 1)dz+

+

∫ A

ε

1− cosx− i sinx

x2(x2 + 1)dx+

∫CA

1− eiz

z2(z2 + 1)dz

Deoarece1− eiz

z2(z2 + 1)=

1− (1 + iz + z2f(z))

z2(z2 + 1)=−iz− f(z)

z2 + 1

deducem ca: ∫Cε

1− eiz

z2(z2 + 1)dz = −i

∫Cε

dz

z−∫Cε

f(z)

z2 + 1dz

Insa: ∣∣∣∣∫Cε

f(z)

z2 + 1dz

∣∣∣∣ ≤ πεM → 0

cand ε→ 0, iar: ∫Cε

dz

z= πi

Page 200: Teme si probleme pentru concursurile studentesti de matematica ...

Functii complexe 193

Deoarece: ∣∣∣∣∫CA

1− eiz

z2(z2 + 1)dz

∣∣∣∣ ≤ πAMA4→ 0

cand A→∞, deducem ca valoarea integralei esteπ

e.

(iii) Avem:

0 =

∫γ

e2aiz − e2biz

z2dz =

∫ −ε−A

e2aix − e2bix

x2dx−

∫Cε

e2aiz − e2biz

z2dz+

+

∫ A

ε

e2aix − e2bix

x2dx+

∫CA

e2aiz − e2biz

z2dz

Deoarece:e2aiz − e2biz

z2=

2i(a− b)z

+ f(z)

cu f olomorfa ıntr-o vecinatate a originii, deducem ca:

limε→0

∫Cε

e2aiz − e2biz

z2dz = πi · 2i(a− b)

Apoi: ∣∣∣∣∫CA

e2aiz − e2biz

z2dz

∣∣∣∣ ≤ πAMA2→ 0

cand A→∞. Rezulta ca valoarea integralei propuse este π(b− a).

Problema 12.28 Integrand functiaeaz

(ez + 1)2, (a ∈ (0, 2)) pe dreptunghiul de varfuri

±A, ±A+ 2πi, sa se afle valoarea integralei:

∫ ∞−∞

eaxdx

(ex + 1)2.

Solutie. Conform teoremei reziduurilor:∫γ

eaz

(ez + 1)2dz = 2πiRez(f, πi)

Pentru a calcula reziduul ın polul de ordinul 2, se efectueaza schimbarea z − πi = w si

deci: Rez(f, πi) = eaπiRez

(eaw

(1− ew)2, 0

). Scriind:

eaw

(1− ew)2=

p

w2+q

w+ . . .

se gaseste:1 + aw + . . . = (1 + w + . . .)(p+ qw + . . .)

deci prin identificare: Rez(f, πi) = eaπi(a− 1). Pe de alta parte:∫γ

eaz

(ez + 1)2dz =

∫ A

−A

eaxdx

(ex + 1)2+

∫ 2π

0

ea(A+iy)

(eA+iy + 1)2idy−

−∫ A

−A

ea(x+2πi)

(ex+2πi + 1)2dx−

∫ 2π

0

ea(−A+iy)

(e−A+iy + 1)2idy

Ipotezele asigura ca (1−e2πia)

∫ ∞−∞

eaxdx

(ex + 1)2= 2πieaπi(a−1) deci valoarea integralei este:

π(1− a)

sin aπpentru a 6= 1. Pentru a = 1, valoarea 1 se poate gasi si prin calcul direct.

Page 201: Teme si probleme pentru concursurile studentesti de matematica ...

194

Problema 12.29 Integrand functialn z

z2 − 1(determinarea principala), pe curba γ formata

din arcele: segmentul [ε,A]; arcul de cerc de centru 0 si de raza A, situat ın primul cadran;segmentul de extremitati iA si iε; arcul de cerc de centru 0 si de raza ε (unde 0 < ε <A <∞), sa se arate ca : ∫ ∞

0

lnx

x2 − 1dx =

π2

4

Solutie. Se arata imediat ca functialn z

z2 − 1are o singularitate aparenta ın 1, deci:

0 =

∫γ

ln zdz

z2 − 1=

∫ A

ε

lnxdx

x2 − 1+

∫CA

ln zdz

z2 − 1−∫ A

ε

ln iy

−y2 − 1idy −

∫Cε

ln zdz

z2 − 1

Apoi: ∣∣∣∣∫CA

ln z

z2 − 1dz

∣∣∣∣ ≤ π

2A

lnA+ π2

A2 − 1→ 0 cand A→∞∣∣∣∣∫

ln z

z2 − 1dz

∣∣∣∣ ≤ π

2ε− ln ε+ π

2

1− ε2→ 0 cand ε→ 0

Deci: ∫ ∞0

lnx

x2 − 1dx = −i

∫ ∞0

ln y

y2 + 1dy +

π

2

∫ ∞0

dy

y2 + 1=π2

4

(prima integrala fiind nula ın mod necesar).

Problema 12.30 Integrand functialn2 z

1− z3(determinarea principala), pe cercul centrat

ın 0, de raza A, cu o taietura pe semiaxa reala negativa, sa se afle valoarea integralei∫ ∞0

lnx

x3 + 1dx.

Solutie. Pe de o parte:∫γ

ln2 z

1− z3dz = 2πi

(Rez(

ln2 z

1− z3, 1) + Rez(

ln2 z

1− z3, e

2πi3 )+

+Rez(ln2 z

1− z3, e

4πi3 )

)=

= 2πi

[(2πi

3 )2

−3e4πi3

+(−2πi

3 )2

−3e−4πi3

]=

8π3i

27· 2 cos

3= −8π3i

27

Pe de alta parte:∫γ

ln2 z

1− z3dz =

∫ −ε−A

[ln(−x) + iπ]2

1− x3(−dx)−

∫Cε

ln2 z

1− z3dz−

−∫ −ε−A

[ln(−x)− iπ]2

1− x3(−dx) +

∫CA

ln2 z

1− z3dz

Deoarece: ∣∣∣∣∫Cε

ln2 z

1− z3dz

∣∣∣∣ ≤ 2πε(− ln ε+ π)2

1− ε3→ 0 pentru ε→ 0

Page 202: Teme si probleme pentru concursurile studentesti de matematica ...

Functii complexe 195

iar ∣∣∣∣∫CA

ln2 z

1− z3dz

∣∣∣∣ ≤ 2πA(lnA+ π)2

A3 − 1→ 0 pentru A→∞

deducem ca:

−8π3i

27=

∫ 0

−∞

4πi ln(−x)

1− x3(−dx) = 4πi

∫ ∞0

lnx

x3 + 1dx

adica:

∫ ∞0

lnx

x3 + 1dx =

2π2

27.

Page 203: Teme si probleme pentru concursurile studentesti de matematica ...

Capitolul 13

Combinatorica si grafuri

Combinatorica

Notatii

• N - multimea numerelor naturale

• |A| - cardinalul multimii A

• bxc - partea ıntreaga a numarului x

•(

1 2 . . . np(1) p(2) . . . p(n)

)- permutare a multimii X = 1, 2, . . . , n, unde p : X →

X este o aplicatie bijectiva

• n! = 1 · 2 · 3 · . . . · n - numarul n-factorial, pentru n ∈ N, n ≥ 1

Prin definitie, 0! = 1.

• [n]k - numarul aranjamentelor de n luate cate k, pentru n, k ∈ N, n ≥ k

•(nk

)- numarul combinarilor de n luate cate k, pentru n, k ∈ N, n ≥ k

Definitii si rezultate

•Definitie. Se numeste permutare a multimii X = 1, 2, . . . , n orice aplicatie bijectivap : X → X.Daca X este o multime oarecare, atunci orice aplicatie bijectiva p : X → X se numestepermutare a multimii X.• Definitie. Se numeste partitie a unei multimi X, o descompunere a lui X sub formaX = A1 ∪A2 ∪ · · · ∪Ak, unde multimile nevide A1, . . . , Ak sunt doua cate doua disjunctesi se numesc clasele partitiei.

O partitie este de tipul 1k12k2 . . . nkn daca ea contine k1 clase cu un element, k2 clasecu doua elemente, . . . , kn clase cu n elemente.• Observatie. Intr-o partitie nu conteaza ordinea de scriere a claselor si nici ordinea

de scriere a elementelor ın fiecare clasa.• Definitie. Se numeste partitie a unui ıntreg n o scriere a numarului n sub forman = n1 + n2 + · · ·+ nk, unde numerele naturale n1, n2, . . . , nk se numesc partile partitieisi verifica inegalitatile n1 ≥ n2 ≥ · · · ≥ nk ≥ 1.

196

Page 204: Teme si probleme pentru concursurile studentesti de matematica ...

Combinatorica si grafuri 197

• Definitie. Pentru o multime finita X, astfel ıncat |X| = n si 1 ≤ k ≤ n, se numestearanjament de n elemente luate cate k orice submultime ordonata alcatuita din kelemente ale multimii X. Teorema. Numarul de aranjamente de n elemente luate cate k ale multimii X este

[n]k = n(n− 1) · · · (n− k + 1), 1 ≤ k ≤ n.

• Observatie. Numarul [n]k se poate exprima sub forma

[n]k =n!

(n− k)!, n, k ∈ N, 1 ≤ k ≤ n.

Prin definitie, [n]0 = 1.• Observatie. Numarul de posibilitati de a introduce k bile, numerotate de la 1 la k

ın n urne, numerotate de la 1 la n este [n]k.• Definitie. Pentru o multime finita X, astfel ıncat |X| = n, se numeste permutare den elemente un aranjament de n elemente luate cate n.• Observatie. Numarul permutarilor de n elemente este [n]n = n!.

Teorema. Daca X, Y sunt multimi finite astfel ıncat |X| = m si |Y | = n, undem,n ∈ N∗, atunci numarul functiilor f : X → Y este nm.• Observatie. Daca X este formata din m bile, iar multimea Y este formata din n

cutii, atunci nm reprezinta numarul tuturor posibilitatilor de a aseza bilele ın cutii (ınaceeasi cutie pot fi introduse mai multe bile). Teorema. Daca X, Y sunt multimi finite astfel ıncat |X| = m si |Y | = n, undem,n ∈ N∗, m ≤ n, atunci numarul functiilor injective f : X → Y este [n]m.• Observatie. In cazul particular f : X → Y cu |X| = |Y | = n, orice aplicatie

injectiva este bijectiva si deci numarul de aplicatii bijective de la X la Y este [n]n = n!.• Definitie. Pentru o multime finita X, astfel ıncat |X| = n si 0 ≤ k ≤ n, se numestecombinare de n elemente luate cate k orice submultime a lui X formata din kelemente. Teorema. Numarul de combinari de n elemente luate cate k ale multimii X este(

n

k

)=

n!

k!(n− k)!, 0 ≤ k ≤ n.

Teorema (formula de recurenta). Daca 1 ≤ k ≤ n, atunci(n

k

)=

(n− 1

k

)+

(n− 1

k − 1

)

Teorema (binomul lui Newton). Daca a, b ∈ C si n ∈ N, atunci are loc formula

(a+ b)n =

n∑k=0

(n

k

)akbn−k.

Teorema (principiul includerii si al excluderii). Pentru orice n multimi finiteX1, . . . , Xn avem

|X1 ∪ · · · ∪Xn| =∑|X1| −

∑|X1 ∩X2|+

∑|X1 ∩X2 ∩X3|+ · · ·+

Page 205: Teme si probleme pentru concursurile studentesti de matematica ...

198

(−1)n|X1 ∩ · · · ∩Xn|.

Teorema. Daca X, Y sunt multimi finite astfel ıncat |X| = m si |Y | = n, undem,n ∈ N∗, m ≥ n atunci numarul functiilor surjective f : X → Y este

nm −(n

1

)(n− 1)m +

(n

2

)(n− 2)m −

(n

3

)(n− 3)m + · · ·+ (−1)n−1

(n

n− 1

).

• Definitie. Se numeste functie generatoare asociata unui sir de numere (an) ca fiindsuma seriei

∑∞n=0 anx

n, ın ipoteza ca seria este convergenta pe R sau pe intervale din R.• Definitie. Se defineste numarul lui Catalan ca fiind numarul de moduri ın care se potpune parantezele ıntr-un produs neasociativ de n factori, scrisi ın ordinea a1, a2, . . . , an. Teorema. Numarul lui Catalan are expresia

Tn =1

n

(2n− 2

n− 1

),

pentru n ∈ N, n ≥ 1.

Grafuri

Notatii

• |V |− numarul elementelor multimii finite V

• (V,U) ≡ G− graf

• O(G)− ordinul grafului G

• dim (G)− dimensiunea grafului G

• gr (G)− grosimea grafului G

• d (G)− diametrul unui graf conex G

• ρ (G)− raza grafului G

• χ (G)− numarul cromatic al grafului G

• q (G)− indicele cromatic al grafului G

• (xi, xj)− arc al grafului G

• [xi, xj ]− muchie a grafului G

• A− multimea arcelor grafului G

• D ≡ (x1, x2, . . . , xp)− drum al grafului G

• l (D)− lungimea drumului D

• g (x)− gradul varfului x

• g− (x)− gradul de intrare al varfului x

• g+ (x)− gradul de iesire al varfului x

Page 206: Teme si probleme pentru concursurile studentesti de matematica ...

Combinatorica si grafuri 199

Definitii si rezultate

• Definitie. Consideram V = x1, . . . , xn, n ∈ N∗, o multime. Numim graf neorientatperechea ordonata de multimi (V,U) ≡ G, U ⊂ P2(V ), unde P2(V ) reprezinta multimeapartilor cu doua elemente ale lui V .

Elementele multimii V se numesc varfuri sau noduri ale grafului G iar elementelemultimii U se numesc muchii.

Numarul elementelor multimii V, notat O (G) se numeste ordinul grafului G, iarnumarul elementelor multimii U, notat dim (G) se numeste dimensiunea grafului G.

In cazul ın care xi, xj ∈ U, vom nota aceasta multime cu [xi, xj ] , iar xi si xj senumesc extremitatile acestei muchii.

Daca [xi, xj ] ∈ U, spunem ca varfurile xi si xj sunt adiacente ın graful G si ca varfurilexi si xj sunt incidente cu muchia [xi, xj ] .

• Definitie. Numim graf orientat perechea ordonata de multimi (V,U) ≡ G, unde Ueste formata din perechi ordonate de elemente din V, numite arce.

Daca xi, xj ∈ U, vom nota acest arc cu (xi, xj). De asemenea, notand cu uij ≡ (xi, xj)un arc oarecare vom spune ca xi este extremitatea initiala, iar xj este extremitateafinala a arcului uij , despre care vom spune ca este orientat de la xi la xj .

Pentru un graf orientatG, un sir de varfuriD = x1, x2, . . . , xp−1, xp cu proprietateaca: (x1, x2) ; (x2, x3) ; . . . ; (xp−1, xp) ∈ U se numeste drum.

Numarul arcelor care compun drumul D, notat cu l (D) se numeste lungimea dru-mului.

Drumul D cu proprietatea ca x1 = xp si toate arcele (x1, x2) ; (x2, x3) ; . . . ; (xp−1, xp)sunt distincte doua cate doua se numeste circuit. Un circuit format dintr-un singur arcse numeste bucla.

Pentru un graf neorientat G, un sir de varfuri L = [x1, x2, ..., xp] cu proprietatea caoricare doua varfuri vecine sunt adiacente, adica [x1, x2]; [x2, x3]; . . . ; [xp−1, xp] ∈ U senumeste lant.

Varfurile x1 si xp se numesc extremitatile lantului L, iar p− 1 este lungimea acestuilant. Daca varfurile x1, x2, . . . , xp sunt distincte doua cate doua, lantul L se numesteelementar.

Pentru un graf neorientat G, un lant L = [x1, x2, . . . , xp] cu proprietatea ca x1 = xpsi toate muchiile [x1, x2]; [x2, x3]; . . . ; [xp−1, xp] distincte doua cate doua se numeste ciclu.Daca ıntr-un graf G suprimam unul sau mai multe arce (muchii) se obtine un nou grafnumit graf partial al grafului G. Daca ıntr-un graf G suprimam unul sau mai multevarfuri, ımpreuna cu arcele (muchiile) care intra sau ies din ele, obtinem un nou grafnumit subgraf al grafului G.

Un ciclu (circuit) hamiltonian pentru un graf G este un ciclu (circuit) elementarcare contine toate varfurile grafului. (trece o data si numai o data prin toate varfurilegrafului)

Un ciclu (circuit) eulerian al unui graf G este un ciclu (circuit) care foloseste toatemuchiile (arcele) grafului G, adica trece o data si numai o data prin fiecare muchie (arc)a grafului G.• Definitie. Numim graf hamiltonian un graf neorientat care contine un ciclu hamilto-nian sau un graf orientat care contine un circuit hamiltonian.

Un lant (drum) elementar al unui grafG care contine toate varfurile grafului se numestelant (drum) hamiltonian.

Un drum se numeste eulerian daca trece o data si numai o data prin fiecare arc(muchie) al grafului.

Page 207: Teme si probleme pentru concursurile studentesti de matematica ...

200

Un drum se numeste simplu daca trece o singura data printr-un acelasi arc al grafului.• Definitie. Un graf G cu proprietatea ca oricare doua varfuri sunt extremitatile unuilant al lui G se numeste graf conex.• Definitie. Un graf G se numeste tare conex daca oricare ar fi xi si xj doua varfuri alesale, exista un drum de la xi la xj si un drum de la xj la xi.

Un graf orientat este simetric daca (xi, xj) ∈ U ⇔ (xj , xi) ∈ U, pentru orice arc algrafului si antisimetric daca (xi, xj) ∈ U ⇒ (xj , xi) /∈ U, pentru orice varfuri xi, xj ∈ V.• Definitie. Orice graf conex si fara cicluri se numeste arbore.

Daca G este un graf conex si x este un varf oarecare al acestuia prin excentrici-tatea acestuia ıntelegem numarul e (x) = max

y∈Vd (x, y) , unde d (x, y) este distanta dintre

varfurile x si y, adica lungimea minima a lanturilor dintre x si y.Diametrul unui graf conex G, notat d (G) este distanta maxima dintre perechile de

varfuri ale lui G.Centrul unui graf conex G este format din acele varfuri de excentricitate minima.Raza unui graf conex G este minimul excentricitatilor varfurilor sale, minim notat

ρ (G) .Numarul cromatic al unui graf G, notat χ (G) este numarul minim de culori cu care

pot fi colorate varfurile lui G, astfel ıncat oricare doua varfuri adiacente sa aiba culoridiferite.

Indicele cromatic al unui graf G, notat q (G) este numarul minim de culori cu carepot fi colorate muchiile grafuluiG, astfel ıncat oricare doua muchii cu o extremitate comunasa fie colorate cu culori diferite.

Pentru un graf neorientat G, gradul varfului x, notat g (x) este numarul muchiilorincidente cu varful x.

Daca G este un graf orientat, gradul de intrare al varfului x, notat g− (x) estenumarul arcelor care intra ın varful x (de forma (y, x)), iar gradul de iesire al varfuluix, notat g+ (x) este numarul arcelor care ies din varful x (de forma (x, z)) si gradul varfuluix este g (x) = g− (x) + g+ (x) .• Definitie. Grafurile G1 ≡ (V1, U1) si G2 ≡ (V2, U2) se numesc izomorfe daca exista obijectie f : V1 → V2, astfel ıncat [x, y] ∈ U1 daca si numai daca [f (x) , f (y)] ∈ U2, undeprin [x, y] este notata muchia care uneste varfurile x si y ale grafului G1.

Un izomorfism al unui graf G cu el ınsusi se numeste automorfism.O p−colorare a unui graf G este fie o partitie V = V1 ∪ V2 ∪ ... ∪ Vp a multimii

varfurilor sale astfel ıncat oricare doua varfuri din aceeasi clasa sa nu fie adiacente, fie ofunctie f : U → 1, 2, ..., p astfel ıncat [i, j] ∈ U sa implice f (i) 6= f (j) .• Definitie. Numim graf k−regulat un graf neorientat pentru care fiecare varf x ∈ Vare gradul g (x) = k sau un graf orientat cu proprietatea ca g− (x) = g+ (x) = k, pentruorice varf x.

Pentru un graf G, un varf de grad zero se numeste varf izolat, iar un varf de gradulunu se numeste varf terminal.

Reprezentand fiecare varf xi ∈ V printr-un punct ın plan si ducand arcele dupa criteriulstabilit obtinem imaginea grafului.•Definitie. Numim graf planar un graf G ale carui varfuri se pot reprezenta prin puncteın plan si ale carui muchii se pot reprezenta prin arce de curba Jordan ce unesc perechide puncte care corespund extremitatilor fiecarei muchii, astfel ıncat oricare doua arce decurba au ın comun cel mult un punct extremitate.• Definitie. Un graf G care admite o partitie a multimii varfurilor, V = V1 ∪V2 ∪ · · · ∪Vpastfel ıncat fiecare muchie sa aiba extremitatile ın doua clase distincte ale partitiei se

Page 208: Teme si probleme pentru concursurile studentesti de matematica ...

Combinatorica si grafuri 201

numeste graf multipartit.Un graf multipartit este complet daca oricare pereche de varfuri situate ın clase

diferite ale partitiei formeaza o muchie.In particular, un graf G pentru care exista o partitie a multimii varfurilor, V = V1∪V2,

V1 ∩ V2 = ∅, astfel ıncat fiecare muchie u a grafului are o extremitate ın V1 si cealalta ınV2 se numeste graf bipartit.

Graful bipartit G este complet daca el contine toate muchiile [x, y] unde x ∈ V1 siy ∈ V2.

Daca | V1 |= p si | V2 |= q, graful bipartit complet G se noteaza cu Kp,q.Prin Kn vom nota un graf complet cu n varfuri si care are

(n2

)muchii, deci pentru care

oricare doua varfuri sunt adiacente.Un graf bipartit complet de forma K1,p se numeste graf-stea.Cea mai mica lungime a unui ciclu elementar al grafului G, notata gr (G) se numeste

grosimea grafului.

Matrice asociate unui graf.Arcele unui graf care au extremitatea finala ıntr-un varf xi se numesc arce incidente

interior varfului xi, iar cele cau au extremitatea initiala ın varful xi se numesc arceincidente exterior varfului xi.

Consideram G un graf de ordinul n si sa notam cu (uj) arcele acestui graf.Matricea de incidenta asociata grafului G este matricea patratica A = (aij) , unde:

aij =

−1,daca arcul uj este incident interior varfului xi;0,daca xi nu este extremitate a arcului uj ;1,daca arcul uj este incident exterior varfului xi.

Matricea conexiunii directe asociata grafului G este matricea patratica D = (dij) ,unde:

dij =

1,daca exista arcul (xi, xj) ;0,daca nu exista arcul (xi, xj) .

Matricea conexiunii totale asociata grafului G este matricea patratica T = (tij) ,unde:

tij =

1,daca exista cel putin un drum de la xi la xj , xi 6= xj ;0, ın caz contrar.

Teorema lui Chen. Daca graful G cu n varfuri este un graf fara circuite, atunci elcontine un drum hamiltonian daca si numai daca matricea T a conexiunii totale atasatacontine exact C2

n elemente nenule.

Formula lui Cayley. Numarul arborilor cu n varfuri, x1, x2, . . . , xn este egal cunn−2, n ≥ 2.

• Definitie. Pentru un graf planar G, o componenta conexa a planului, obtinuta prinsuprimarea din plan a muchiilor si varfurilor reprezentarii plane a lui G se numeste fataa reprezentarii plane.

Frontiera oricarei fete este o curba Jordan ınchisa, constand din muchiile unui cicluelementar al grafului G.

Formula lui Euler. Daca graful conex si planar G are n varfuri si m muchii, atunciorice reprezentare planara a sa contine m− n+ 2 fete.

Page 209: Teme si probleme pentru concursurile studentesti de matematica ...

202

Probleme

Problema 13.1 Sa se arate ca oricare ar fi 52 de puncte ıntr-un patrat de latura 1, existatrei puncte care pot fi acoperite cu un disc de raza 1

7 .

Solutie. Fie punctele M1, . . . ,M52 situate ıntr-un patrat de latura 1. Vom arata caexista trei discuri D(Mik ,

17), ik ∈ 1, . . . , 52,pentru k ∈ 1, 2, 3, care sa aiba intersectia

nevida. In acest caz discul cautat are centrul ıntr-un punct din intersectie, iar raza este 17 .

Suprafata totala acoperita de cele 52 de discuri este S = 52π · 149 = 52

49π. Suprafata

acoperita de acestea, S′, este cel mult egala cu cea a unui patrat ce are latura cu 2

7 mai

mare decat cea a patratului initial, adica S′< 81

49 .Obtinem

S

S′>

52

81π > 2.

In consecinta, exista D(Mik ,17), ik ∈ 1, . . . , 52, k ∈ 1, 2, 3 cu intersectia nevida.

Problema 13.2 Pe suprafata unui poligon de arie 13 se aseaza 10 poligoane de arie 6. Sase arate ca exista 4 poligoane ce se intersecteaza dupa o arie mai mare ca 1

70 .

Solutie. Fie poligoanele P1, . . . , P10 astfel ıncat S(Pk) = 6, pentru orice k = 1, . . . 10si poligonul P cu S(P ) = 13. Definim multimile

I1 =10⋃k=1

Pk, I2 =10⋃

i6=j=1

(Pi ∩ Pj), . . . , I10 =10⋂k=1

Pk.

Se poate demonstra prin inductie relatia

10∑k=1

S(Pk) =

10∑k=1

S(Ik).

Pentru multimile Ik, k = 1, . . . 10, sunt evidente inegalitatile

S(I1) ≥ S(I2) ≥ . . . ≥ S(I10),

de unde obtinem10∑k=1

S(Pk) ≤ 3S(I1) + 7S(I4),

deci 60 ≤ 39 + 7S(I4), adica S(I4) ≥ 3.I4 contine

(104

)intersectii de cate 4 poligoane, printre care exista una de arie maxima

notata Smax, deci:

3 ≤ S(I4) ≤∑

S(P1 ∩ P2 ∩ P3 ∩ P4) ≤(

10

4

)· Smax

Din inegalitatile de mai sus deducem ca Smax ≥ 3/(

104

)= 1

70 .

Problema 13.3 Fiind date numerele naturale nenule m,n, m ≤ n, sa se demonstreze cam divide numarul

nm−1∑k=0

(−1)k(n

k

).

Page 210: Teme si probleme pentru concursurile studentesti de matematica ...

Combinatorica si grafuri 203

Hungarian Mathematical Olympiad, 2001

Solutie. Vom aplica formula de recurenta a coeficientilor binomiali.

n

m−1∑k=0

(−1)k(nk

)= n

m−1∑k=0

(−1)k(n− 1k

)+ n

m−1∑k=1

(−1)k(n− 1k − 1

)

= n[m−1∑k=0

(−1)k(n− 1k

)+

m−2∑k=0

(−1)k+1

(n− 1k

)]

= n(−1)m−1

(n− 1m− 1

)= n(−1)m−1 (n−1)!

(m−1)!(n−m)!

= m(−1)m−1

(nm

).

Problema 13.4 Sa se calculeze suma

1 · 2(n

2

)+ 2 · 3

(n

3

)+ · · ·+ (n− 1) · n

(n

n

).

Solutie. Consideram un grup de n persoane dintre care vom alege comitete de cate

k persoane, fiecare comitet avand un presedinte si un vicepresedinte. Exista

(nk

)posibilitati de a alege asemenea comitete, iar pentru fiecare comitet presedintele si vi-cepresedintele pot fi selectati ın k(k − 1) moduri. Numarul comitetelor pe care le putemforma cu un presedinte si un vicepresedinte este

1 · 2(n2

)+ 2 · 3

(n3

)+ · · ·+ (n− 1) · n

(nn

).

Selectia se poate face si ın alta maniera: putem alege presedintele si vicepresedinteleın n(n − 1) moduri, dupa care adaugam ceilalti membri ai comitetului dintre cele n − 2persoane ramase. Deoarece numarul tuturor submultimilor unei multimi cu n−2 elementeeste 2n−2, obtinem

1 · 2(n2

)+ 2 · 3

(n3

)+ · · ·+ (n− 1) · n

(nn

)= n(n− 1)2n−2.

Problema 13.5 Doua sute de studenti participa la un concurs de matematica, la careau de rezolvat 6 probleme. Se stie ca fiecare problema a fost rezolvata de cel putin 120 departicipanti. Sa se demonstreze ca exista doi participanti astfel ıncat fiecare problema afost rezolvata de cel putin unul dintre ei.

Solutie. Intrucat fiecare problema a fost rezolvata de cel putin 120 de participanti, de-ducem ca exista cel putin 720 de solutii corecte ın total. Deoarece sunt doar 200 de studenti,obtinem ca macar un student a rezolvat cel putin patru probleme. Daca a rezolvat 5 sau6 probleme, atunci el poate face pereche cu oricare alt student. Presupunem ca a rezolvatexact 4 probleme. Fiecare dintre cele doua probleme ramase a fost rezolvata de cel putin120 de participanti si cum numarul total este 200, exista cel putin un student care le-arezolvat pe amandoua.

Page 211: Teme si probleme pentru concursurile studentesti de matematica ...

204

Problema 13.6 O multime S ce contine patru numere naturale o numim conexa dacapentru orice x ∈ S, cel putin unul din numerele x − 1 sau x + 1 apartine lui S. Fie Cnnumarul de submultimi conexe ale multimii 1, 2, . . . n.

(a) Sa se determine C7;

(b) Sa se determine o formula generala pentru Cn.

Romanian Mathematical Olympiad, 2006

Solutie. Fie S = a, b, c, d multime conexa, astfel ıncat a < b < c < d. Deoarecea− 1 /∈ S rezulta ca a+ 1 ∈ S, deci b = a+ 1. Analog, ıntrucat d+ 1 /∈ S, avem d− 1 ∈ S,adica c = d − 1. Astfel o multime care sa satisfaca conditiile problemei este de formaS = a, a+ 1, d− 1, d , cu d− a > 2.

(a) Submultimile conexe ale multimii 1, 2, . . . , 7 sunt

1, 2, 3, 4; 1, 2, 4, 5; 1, 2, 5, 6; 1, 2, 6, 7;

2, 3, 4, 5; 2, 3, 5, 6; 2, 3, 6, 7; 3, 4, 5, 6; 2, 4, 6, 7; 4, 5, 6, 7.

(b) Definim diametrul multimii S = a, a+1, d−1, d astfel D = d−a+1. Este evidentca D > 3 si D ≤ n − 1 + 1 = n. In cazul D = 4 avem n − 3 multimi conexe, dacaD = 5 exista n− 4 multimi conexe s.a.m.d. Astfel obtinem

Cn = 1 + 2 + · · ·+ (n− 3) =(n− 3)(n− 2)

2.

Problema 13.7 Fie multimea A astfel ıncat |A| = n2 si F o familie de submultimi ale luiA care au n elemente. Presupunem ca oricare doua multimi din F au cel mult un elementın comun.

(a) Sa se arate ca |F| ≤ n2 + n;

(b) Pentru n = 3, sa se puna ın evidenta un exemplu ın care |F| sa ia valoarea maxima.

Romanian TST, 1985

Solutie. (a) Pentru x ∈ A, notam cu k(x) numarul multimilor B ∈ F pentru carex ∈ B. Fie aceste multimi B1, B2, . . . , Bk(x).

Atunci B1\x, B2\x, . . . , Bk(x)\x sunt submultimi disjuncte ale lui A\x, fiecare

avand n− 1 elemente. Intrucat A \ x are n2− 1 elemente, obtinem k(x) ≤ n2−1n−1 = n+ 1.

Repetand rationamentul orice x ∈ A si sumand, obtinem∑x∈A

k(x) ≤ n2(n+ 1).

Dar ∑x∈A

k(x) =∑B∈F|B| = n|F|.

Atunci n|F| ≤ n2(n+ 1), de unde |F| ≤ n2 + n.

Page 212: Teme si probleme pentru concursurile studentesti de matematica ...

Combinatorica si grafuri 205

(b) In cazul ın care |A| = 32, asezam elementele 1, 2, . . . , 9 ıntr-o matrice 1 2 34 5 67 8 9

si alegem ca multimi ale lui F cele formate cu factorii produselor care apar ın calcululdeterminantului de ordin 3 al matricei de mai sus. Deci,

F = 1, 2, 3, 4, 5, 6, 7, 8, 9, 1, 4, 7, 2, 5, 8, 3, 6, 9,

1, 5, 9, 2, 6, 7, 3, 4, 8, 3, 5, 7, 2, 4, 9, 1, 6, 8 .

In acest caz |F| = 32 + 3 = 12.

Problema 13.8 Se noteaza prin sn,m,r numarul functiilor f : X → Y, unde |X| =n, |Y | = m, cu proprietatea ca exista Z ⊂ Y, |Z| = r si astfel ıncat f(X) ⊃ Z.

Sa se arate ca

sn,m,r = mn −(r

1

)(m− 1)n +

(r

2

)(m− 2)n − · · ·+ (−1)n(m− r)n.

Solutie. Fie Z = y1, . . . yr si pentru fiecare i = 1, . . . , r notam cu Ai = f : X →Y, yi /∈ f(X). Obtinem sn,m,r = mn − |A1 ∪ A2 ∪ · · · ∪ Ar|. Dar Ai reprezinta multimeafunctiilor definite pe X, cu valori ın Y \ yi, deci |Ai| = (m − 1)n. Multimea Ai ∩ Ajcontine functiile definite pe X, cu valori ın Y \ yi, yj si deci |Ai ∩ Aj | = (m − 2)n.In general, |Ai1 ∩ Ai2 ∩ · · · ∩ Aik | = (m − k)n, unde 1 ≤ i1 < i2 < · · · < ik ≤ r.Deoarece exista

(rk

)submultimi de indici i1, i2, . . . , ik, rezulta ca fiecare suma de forma∑

1≤i1<i2<···<ik≤r |Ai1 ∩Ai2 ∩ · · · ∩Aik | contine(rk

)termeni, fiecare fiind egal cu (m− k)n.

Obtinemsn,m,r = mn − |A1 ∪A2 ∪ · · · ∪Ar|

= mn −(r1

)(m− 1)n +

(r2

)(m− 2)n − · · ·+

+(−1)r(m− r)n.

Problema 13.9 Fie A un alfabet format din n litere identice: a1, a1; a2, a2; . . . ; an, an,perechile diferite constand din litere diferite. Se formeaza toate cuvintele care folosesctoate cele 2n litere din alfabetul A, astfel sa nu apara doua litere identice vecine.

Sa se arate ca numarul acestor cuvinte este egal cu:

1

2n[(2n)!−

(n

1

)2(2n− 1)! +

(n

2

)22(2n− 2)!− · · ·+ (−1)n2nn!].

Solutie. Numarul tuturor cuvintelor care folosesc toate cele 2n litere din alfabetul A esteegal cu

(2n)!

(2!)n=

(2n)!

2n,

deoarece literele identice pot fi permutate ıntre ele ın (2!)n = 2n moduri distincte, rezultandun acelasi cuvant format cu cele 2n litere din A.

Sa notam cu Ai multimea cuvintelor formate cu cele 2n litere din A pentru care celedoua litere ai sunt vecine. Rezulta ca numarul cautat este egal cu

(2n)!

2n− |A1 ∪A2 ∪ · · · ∪An|.

Page 213: Teme si probleme pentru concursurile studentesti de matematica ...

206

Pentru a evalua |A1 ∪ A2 ∪ · · · ∪ An| aplicam principiul includerii-excluderi:|A1 ∪A2 ∪ · · · ∪An| =

∑ni=1 |Ai| −

∑1≤i<j≤n |Ai ∩Aj |

+(−1)k−1∑

1≤i1<···<ik≤n |Ai1 ∩Ai2 ∩ · · · ∩Aik |+ . . .

+(−1)n−1| ∩ni=1 Ai|.Sa calculam ın cazul general numarul de elemente din Ai1 ∩ Ai2 ∩ · · · ∩ Aik si sa

aratam ca acesta nu depinde de alegerea indicilor 1 ≤ i1 < · · · < ik ≤ n. Daca un cuvıntapartine acestei multimi ınseamna ca el apartine fiecareia din multimile Ai1 , Ai2 , . . . , Aik ,deci literele ai1 , ai1 ; ai2 , ai2 ; . . . ; aik , aik sunt vecine. Cuvintele pentru care cele k perechide litere sunt vecine se obtin ın modul urmator: se formeaza toate cuvintele avınd 2n− klitere care formeaza un alfabet obtinut din A prin suprimarea catei unei litere dintreai1 , ai2 , . . . , aik . Apoi ın fiecare cuvant astfel format se dubleaza literele ai1 , ai2 , . . . , aik ,adaugand dupa litera aij o alta litera egala cu aij , pentru fiecare j = 1, . . . , n. Deciobtinem:

|A1 ∪A2 ∪ · · · ∪An| =(2n− k)!

(2!)n−k=

2k(2n− k)!

2n.

Deoarece indicii i1, . . . , ik cu 1 ≤ i1 < · · · < ik ≤ n pot fi alesi ın

(nk

)moduri, rezulta

ca numarul cuvintelor care nu contin doua litere identice vecine este

(2n)!

2n−(n1

)2(2n− 1)!

2n+

(n2

)22(2n− 2)!

2n− · · ·+ (−1)n2nn!

2n.

Problema 13.10 Daca p este o permutare a multimiiX = 1, . . . , n, spunem ca numaruli este un punct fix al permutarii p, daca p(i) = i, 1 ≤ i ≤ n. Sa se arate ca numarul D(n)al permutarilor fara puncte fixe ale multimii X este egal cu

D(n) = n!(

1− 1

1!+

1

2!− 1

3!+ · · ·+ (−1)n

n!

).

Care este numarul permutarilor a n obiecte cu p puncte fixe ?

Solutie. Notam cu Ai multimea celor (n − 1)! permutari care admit un punct fix ın isi vom determina numarul permutarilor care admit cel putin un punct fix. Acest numareste egal cu |A1 ∪A2 ∪ · · · ∪An| si, conform principiului includerii-excluderii, se obtine dinrelatia:

|A1 ∪A2 ∪ · · · ∪An| =n∑i=1

|Ai| −∑

1≤i<j≤n|Ai ∩Aj |+ · · ·+ (−1)n−1| ∩ni=1 Ai|.

Dar |Ai1 ∩Ai2 ∩· · ·∩Aik | = (n−k)!, deoarece o permutare din multimea A1∪A2∪· · ·∪Anreprezinta puncte fixe ın pozitiile i1, i1, . . . , ik, celelalte pozitii continand o permutare acelor n− k elemente ramase, al caror numar este (n− k)!. Dar pozitiile i1, i1, . . . , ik pot fi

alese din multimea celor n pozitii ın

(nk

)moduri, deci

∣∣∣ n⋃i=1

Ai

∣∣∣ =

(n1

)(n− 1)!−

(n2

)(n− 2)! + · · ·+ (−1)n

(nn

).

Page 214: Teme si probleme pentru concursurile studentesti de matematica ...

Combinatorica si grafuri 207

Astfel,

D(n) = n!−∣∣∣ n⋃i=1

Ai

∣∣∣ = n!−(n1

)(n− 1)! + · · ·+ (−1)n

(nn

),

de unde se obtine expresia lui D(n).

Rezulta ca limn→∞

D(n)

n!=

1

e, deci pentru n suficient de mare probabilitatea ca o per-

mutare a n elemente aleasa aleator sa nu aiba puncte fixe este apropriata de 13 . Deoarece

cele p puncte fixe (0 ≤ p ≤ n) pot fi alese ın

(np

)moduri si celelalte n − p puncte

nu mai sunt fixe, rezulta ca numarul permutarilor din Sn cu p puncte fixe este egal cu(np

)D(n − p), deoarece pentru fiecare alegere a celor p puncte fixe exista D(n − p)

permutari ale obiectelor ramase, fara puncte fixe, daca se defineste D(0) = 1.

Problema 13.11 Fie X = 1, 2, . . . , n si D(n) numarul permutarilor multimii X farapuncte fixe. Daca E(n) reprezinta numarul permutarilor pare ale multimii X fara punctefixe, aratati ca

E(n) =1

2(D(n) + (−1)n−1(n− 1)).

Solutie. Conform problemei anterioare, avem

D(n) = n!(

1− 1

1!+

1

2!− 1

3!+ · · ·+ (−1)n

n!

).

Pentru a obtine expresia lui E(n) vom nota cu Ai multimea permutarilor pare p ∈ Snastfel ıncat p(i) = i. Deoarece Sn contine 1

2n! permutari pare, rezulta ca

E(n) = 12 n!−

∣∣∣⋃ni=1Ai

∣∣∣= 1

2 n!−∑n

i=1 |Ai|+∑

1≤i<j≤n |Ai ∩Aj |+ · · ·+ (−1)n∣∣∣⋂n

i=1Ai

∣∣∣.Deoarece |Ai1 ∩Ai2 ∩ · · · ∩Aik | = 1

2 (n− k)!, rezulta ın mod analog ca pentru D(n), ca

E(n) = 12 n!−

(n1

)12(n− 1)! + · · ·+ (−1)n−1

(n

n− 1

)+ (−1)n

= 12 (D(n) + (−1)n−1(n− 1)).

Problema 13.12 Verificati ca:

∑∞n=0 D(n) t

n

n! = e−t

1−t ;

D(n+ 1) = (n+ 1)D(n) + (−1)n+1;D(n+ 1) = n(D(n) +D(n− 1)).

Solutie. Conform unei probleme anterioare, avem:

D(n)

n!= 1− 1

1!+

1

2!− · · ·+ (−1)n

n!,

Pe de alta parte, se cunoaste dezvoltarea:

e−t

t− 1=(

1− t

1!+t2

2!− . . .

)(1 + t+ t2 + . . . ).

Din relatiile de mai sus deducem ca D(n)n! este coeficientul lui tn din dezvoltarea functiei

e−t

t−1 si astfel se obtine prima relatie din enuntul problemei.

Page 215: Teme si probleme pentru concursurile studentesti de matematica ...

208

Pentru a obtine relatiile de recurenta pornim de la formula

D(n) = n!(

1− 1

1!+

1

2!− 1

3!+ · · ·+ (−1)n

n!

).

Este evident caD(n+ 1) = (n+ 1)!(1− 1

1! + 12! −

13! + · · ·+ (−1)n

n! + (−1)n+1

(n+1)! )

= (n+ 1)n!(1− 11! + 1

2! −13! + · · ·+ (−1)n

n! ) + (n+ 1)! (−1)n+1

(n+1)! ,

de unde

D(n+ 1) = (n+ 1)D(n) + (−1)n+1.

In continuare, prin adunarea relatiei obtinute mai sus cu

D(n) = nD(n− 1) + (−1)n,

se obtine D(n+ 1) +D(n) = n (D(n) +D(n− 1)) +D(n) + (−1)n+1 + (−1)n, ceea ce esteechivalent cu

D(n+ 1) = n(D(n) +D(n− 1)).

Problema 13.13 Care este numarul un de moduri ın care putem urca o scara cu n trepte,stiind ca la fiecare pas urcam o treapta sau doua trepte?

Solutie. Este evident ca numarul un reprezinta ın acelasi timp numarul de scrieri alenumarului natural n ca o suma a numerelor 1 sau 2, doua scrieri fiind distincte daca eledifera si prin ordinea termenilor.

De exemplu2 = 1 + 13 = 2 + 1 = 1 + 2 = 1 + 1 + 14 = 2 + 2 = 2 + 1 + 1 = 1 + 2 + 1 = 1 + 1 + 2 = 1 + 1 + 1 + 1.

Primul termen este 1 sau 2. In primul caz numarul de scrieri este egal cu un−1, deoarecerestul termenilor sunt egali cu 1 sau cu 2 si au suma n− 1, iar ın al doilea caz numarul descrieri este egal cu un−2. Deci am obtinut recurenta

un = un−1 + un−2,

cu valorile initiale u1 = 1, u2 = 2. Definim u0 = 1 si astfel obtinem sirul lui Fibonacci,

deci un = Fn. Atasam ecuatia caracteristica r2 = r + 1, care are solutiile r1 = 1+√

52 si

r2 = 1−√

52 .

Solutia generala esteun = c1r

n1 + c2r

n2 ,

unde constantele c1 si c2 se detrmina din conditiile initiale u0 = 1 si u1 = 1.Astfel se obtine sistemulC1 + C2 = 11+√

52 C1 + 1−

√5

2 C2 = 1,

care are solutiile C1 =√

5+12√

5si C2 =

√5−1

2√

5.

Astfel obtinem

un = Fn =

√5 + 1

2√

5

(1 +√

5

2

)n+

√5− 1

2√

5

(1−√

5

2

)n=

Page 216: Teme si probleme pentru concursurile studentesti de matematica ...

Combinatorica si grafuri 209

=1√5

((1 +√

5

2

)n+1−(1−

√5

2

)n+1),

pentru orice n ≥ 0.

Daca notam f(x) =∞∑n=0

Fnxn, obtinem

xf(x) =∞∑n=1

Fn−1xn si x2f(x) =

∞∑n=2

Fn−2xn.

Tinand cont de relatia de recurenta a lui un deducem:

f(x)− xf(x)− x2f(x) = F0 + (F1 − F0)x+∞∑n=2

(Fn − Fn−1 − Fn−2)xn = F0 = 1,

de unde

f(x) =1

1− x− x2.

Observatii. 1) Functia f(x) = 11−x−x2 este functia generatoare a sirului lui Fibonacci.

2) Problema de mai sus poate fi enuntata si ın modul urmator:Sa se determine numarul de moduri de pardosire a unei alei de dimensiuni 2× n cu placide dimensiune 2× 1.

Problema 13.14 Sa se arate ca functia generatoare a numerelor lui Catalan Tn este datade egalitatea:

f(x) = T1x+ T2x2 + · · ·+ Tnx

n + · · · = 1−√

1− 4x

2.

Sa se gaseasca pe aceasta cale expresia numerelor Tn.

Solutie. Numarul lui Catalan se defineste ca fiind numarul de moduri ın care se potpune parantezele ıntr-un produs neasociativ de n factori, scrisi ın ordinea x1, x2, . . . , xn.

Daca exista o singura pereche de paranteze care nu sunt continute ın alte paranteze,atunci aceasta pereche contine ın interior produsul factorilor x2, . . . , xn, ramanand ınexterior factorul x1, sau contine produsul x1, . . . , xn−1, ramanand ınafara factorul xn.

Daca exista doua perechi de paranteze care nu sunt continute ın alte paranteze, rezultaca aceste perechi contin produsul factorilor x1, . . . xk, respectiv xk+1, . . . , xn, unde 2 ≤ k ≤n−2. Deoarece ıntr-un produs de k, respectiv n−k factori putem pune parantezele ın Tk,respectiv Tn−k moduri, rezulta relatia

Tn =n−1∑k=1

TkTn−k,

cu T1 = 1.Daca notam f(x) = T1x+ T2x

2 + · · ·+ Tnxn + · · · = 1−

√1−4x2 , atunci

f2(x) = T 21 x

2 + (T1T2 + T2T1)x3 + · · ·+ (

n−1∑k=1

TkTn−k)xn + · · · = f(x)− x,

Page 217: Teme si probleme pentru concursurile studentesti de matematica ...

210

tinand seama de relatia de recurenta obtinuta si de datele initiale T1 = T2 = 1. Solutiile

ecuatiei de gradul al doilea f2(x) − f(x) − x = 0 sunt f(x) = 1±√

1−4x2 . Presupunem ca

x < 14 si ıntrucat f(0) = 0, obtinem

f(x) =1−√

1− 4x

2.

Vom dezvolta ın serie de puteri functia√

1− 4x, folosindu-ne de formula generalizataa binomului lui Newton.

(x+ a)α = aα + αaα−1x+ α(α−1)2! aα−2x2 + · · ·+

+α(α−1)...(α−k+1)k! aα−kxk + · · · ,

unde a > 0.Aceasta serie este convergenta pentru orice x care verifica |x| < a. Daca α este un

numar ıntreg si pozitiv, numai un numar finit de termeni ai seriei sunt nenuli, iar dez-voltarea obtinuta este formula binomului lui Newton.

Pentru a dezvolta (1 − 4x)12 ın serie de puteri ale lui x vom nota y = −4x, α = 1

2 si

vom dezvolta binomul (y + 1)12 . Se obtine coeficientul lui xn egal cu

(1/2)(−1/2)(−3/2)···(1/2−n+1)n! (−4)n = (−1)n−1 1

2n1·3···(2n−3)

n! (−1)n22n =

= − (2n−2)!2n−1(n−1)!n!

2n = −2 (2n−2)!n(n−1)!(n−1)! = − 2

n

(2n− 2n− 1

).

Dar Tn este coeficientul lui xn din dezvoltarea lui f(x), deci se obtine ınmultind cu −12

coeficientul lui xn din dezvoltarea lui (1− 4x)12 . Astfel, avem Tn = 1

n

(2n− 2n− 1

).

Problema 13.15 Sa se arate ca numarul sirurilor (x1, x2, . . . , x2n−2) cu xi ∈ −1, 1,pentru i = 1, 2, . . . , 2n− 2 si care satisfac conditiile:

(a) x1 + x2 + · · ·+ xk ≥ 0 pentru orice 1 ≤ k ≤ 2n− 2;

(b) x1 + x2 + · · ·+ x2n−2 = 0

este egal cu 1n

(2n−2n−1

).

Solutie. Vom determina numarul de siruri de litere care contin litera a de k ori, literab de m ori si care au proprietatea(P ) : Pentru orice i, 1 ≤ i ≤ m+ k, numarul de litere a ın primele i litere ale sirului estemai mare sau egal cu numarul de litere b.

Numarul acestor siruri este nenul daca si numai daca este verificata conditia k ≥ m > 0.Numarul sirurilor de litere care contin litera a de k ori si litera b de m ori este egal cuP (m, k) =

(m+km

). Pentru a rezolva problema este suficient sa determinam numarul sirurilor

care nu verifica proprietatea (P ), deoarece numarul cautat se obtine prin scaderea acestuinumar din

(m+km

).

Vom arata ca numarul sirurilor formate din m litere b si k litere a care nu verificaproprietatea (P ) este egal cu P (m− 1, k+ 1) =

(m+km−1

), adica este egal cu numarul tuturor

sirurilor formate din m− 1 litere b si din k + 1 litere a. Pentru aceasta vom considera unsir format din m litere b si k litere a care nu verifica (P ). Va exista o pozitie cu numarul2s+ 1, unde s ≥ 0, astfel ıncat sirul considerat contine litera b ın pozitia 2s+ 1, iar ın fataacestei pozitii exista un numar egal de litere a si b, numar egal cu s. Vom considera celmai mic indice cu aceasta proprietate si vom adauga litera a ın fata acestui sir, obtinand

Page 218: Teme si probleme pentru concursurile studentesti de matematica ...

Combinatorica si grafuri 211

astfel un sir format din m litere b si k+ 1 litere a. Prima litera a sirului astfel obtinut estea, iar printre primele 2s+ 2 litere exista un numar egal de litere a si b. Schimbam literelea si b ıntre ele pe primele 2s+ 2 pozitii ale sirului.

Deoarece ın primele 2s+ 2 pozitii numarul de litere a a fost egal cu numarul literelorb, nu se va schimba numarul total de litere de fiecare fel si obtinem un sir format din mlitere b si k + 1 litere a. Prima litera este acum b.

Astfel am asociat fiecarui sir din m litere b si k litere a care nu satisfac (P ), un sirformat din m litere b si k + 1 litere a, care ıncep cu litera b. Se demonstreaza imediat caaceasta aplicatie este injectiva. (Se considera doua siruri diferite care nu satisfac (P ) sicare difera pe o pozitie de rang p ≤ 2s+ 1 sau p > 2s+ 1.)

Vom demonstra ca ın acest mod este posibil sa se obtina orice sir format din m litereb si k+ 1 litere a, care ıncep cu litera b, deci aplicatia este si surjectiva. Sa consideram unastfel de sir. Deoarece presupunem m ≤ k sau m < k + 1, va exista o pozitie pana la careexista un numar egal de litere a si b, pentru ca sirul ıncepe cu litera b.

Daca pana la prima pozitie cu aceste proprietati ınlocuim literele a si b ıntre ele sisuprimam prima litera a, vom gasi un sir format din k litere a si m litere b care nu verifica(P ).

Daca aplicam acestui sir transformarea descrisa gasim sirul de la care am plecat. Da-torita acestei bijectii numarul sirurilor cu m litere b si k litere a care nu verifica (P ) esteegal cu numarul sirurilor cu m litere b si k + 1 litere a care ıncep cu b. Daca suprimamprima litera b gasim toate sirurile formate din m−1 litere b si k+1 litere a, al caror numareste egal cu P (m− 1, k + 1) =

(m+km−1

).

In concluzie, numarul sirurilor care satisfac (P ) este egal cu(m+ k

m

)−(m+ k

m− 1

)=k −m+ 1

k + 1

(m+ k

m

).

Pentru k = m = n− 1 se obtine numarul Tn = 1n

(2n−2n−1

).

Acest numar reprezinta solutia problemei, deoarece daca ıl ınlocuim pe 1 cu litera asi pe −1 cu litera b, conditia (a) exprima faptul ca numarul literelor a este cel putin egalcu numarul literelor b ın primele k pozitii ale sirului, pentru 1 ≤ k ≤ 2n− 2. Conditia (b)exprima faptul ca numarul literelor a este egal cu numarul literelor b si ambele sunt egalecu n− 1.

Problema 13.16 O triangulatie a unui poligon convex A1A2 . . . An+1 cu n + 1 varfurieste o multime formata din n−2 diagonale care nu se intersecteaza ın interiorul poligonuluici numai ın varfuri si care ımpart suprafata poligonului ın n− 1 triunghiuri.

Sa se arate ca numarul de triangulatii ale unui poligon convex cu n + 1 varfuri esteegal cu

Tn =1

n

(2n− 2

n− 1

).

Solutie. Vom arata ca exista o bijectie de la triangulatiile unui poligon convex cu n + 1vırfuri la multimea scrierilor cu paranteze ale unui produs de n factori ın ordinea x1, . . . , xn.

Daca poligonul cu n+1 varfuri este A1A2 . . . An+1 ne vom deplasa din A1 ın A2 s.a.m.d.pe laturile poligonului pana cand ajungem ın An+1, obtinand o scriere cu paranteze a unuiprodus cu n factori, dupa regulile urmatoare:

-cand ne deplasam pe o latura scriem un nou factor xi, ın ordinea x1, . . . , xn;-cand ajungem ıntr-un varf la care sosesc anumite diagonale ale triangulatiei, scriem

un numar de paranteze de ınchidere egal cu numarul de diagonale care au o extremitate

Page 219: Teme si probleme pentru concursurile studentesti de matematica ...

212

ın acel varf si pentru care cealalta extremitate a fost parcursa si un numar de parantezede deschidere egal cu numarul de diagonale care sunt incidente ın acel varf si pentru carecealalta extremitate nu a fost vizitata.

Este evident ca aceasta corespondenta este injectiva.Pentru a arata ca este si surjectiva, vom considera un produs cu paranteze a n factori

ın ordinea x1, . . . , xn. Acest produs contine n− 2 paranteze de deschidere si tot atatea deınchidere.

Fiecarei paranteze de deschidere ıi corespunde o unica paranteza de ınchidere. Pentrufiecare pereche formata dintr-o paranteza de deschidere si una de ınchidere vom consideraprima litera care se gaseste la dreapta parantezei de deschidere, fie xi si prima litera carese gaseste la stanga parantezei de ınchidere, fie xj si vom duce diagonala AiAj+1.

Deoarece fiecare pereche de paranteze contine ın interior un produs a doi factori siparantezele sunt corect puse, cele n− 2 diagonale ale poligonului constituie o triangulatiea acestuia.

Aplicand acestei triangulatii corespondenta descrisa, gasim produsul cu paranteze a nfactori de la care am plecat, ceea ce demonstreaza ca ea este o bijectie.

In concluzie, numarul de triangulatii este egal cu numarul lui Catalan, Tn.

Problema 13.17 Sa se arate ca numarul functiilor crescatoare

f : 1, . . . , n → 1, . . . , n

care satisfac conditia f(x) ≤ x, pentru orice 1 ≤ x ≤ n, este egal cu numarul lui Catalan

Tn+1 =1

n+ 1

(2n

n

).

Solutie. Considerand un sistem de axe xOy, vom desena dreptele de ecuatii x = k, y = lunde 0 ≤ k, l ≤ n sunt numere ıntregi si vom considera punctele de intersectie ale acestordrepte situate ın primul cadran, pe prima bisectoare si sub prima bisectoare.

Pentru fiecare functie crescatoare f : 1, . . . , n → 1, . . . , n vom construi un drumın aceasta retea ın felul urmator: daca ne aflam ın punctul M(i, f(i)), ne deplasam ınpunctul M1(i+ 1, f(i)) pe un segment orizontal, apoi ne deplasam pe un segment verticalpana cand ajungem ın punctul M2(i+1, f(i+1)). Daca f(i+1) = f(i), obtinem M2 = M1;ın caz contrar, deplasarea are loc ın sus, deoarece f(i+ 1) > f(i).

Efectuand aceste deplasari pentru i = 1, . . . , n − 1 obtinem un drum ascendent ınaceasta retea, de extremitati (1, 0) si (n, f(n)). Unind apoi originea O cu punctul (1, 0)printr-un segment orizontal si ın cazul cand f(n) < n, punctul (n, f(n)) cu punctul A(n, n)situat pe prima bisectoare, printr-un sir de segmente verticale, obtinem un drum de ex-tremitati O(0, 0) si A(n, n).

Este evident ca drumul astfel obtinut este compus din n segmente orizontale si nsegmente verticale, nu are coborasuri cand ne deplasam de la O catre A si este situat subprima bisectoare. Am obtinut astfel o corespondenta bijectiva ıntre multimea functiilorcrescatoare f : 1, . . . , n → 1, . . . , n care satisfac conditia f(x) ≤ x, pentru orice1 ≤ x ≤ n, si multimea drumurilor de extremitati O si A cu proprietatile mentionate.Injectivitatea acestei aplicatii este evidenta, deoarece la functii diferite corespund drumuridiferite. Pentru a arata ca este si surjectiva, sa consideram un drum cu extremitatile O siA, cu proprietatile mentionate. Definim funtia fd prin

fd = max j| (i, j) ∈ d,

Page 220: Teme si probleme pentru concursurile studentesti de matematica ...

Combinatorica si grafuri 213

pentru orice i = 1, . . . , n. In acest caz imaginea functiei fd prin corepondenta descrisa estechiar drumul d, ceea ce demonstreaza surjectivitatea acestei corespondente.

Pentru a numara drumurile ascendente de lungime 2n de extremitati O si A(n, n), sit-uate sub prima bisectoare, se observa ca exista o corespondenta biunivoca ıntre multimeadrumurilor si multimea sirurilor (x1, x2, . . . , x2n) cu xi = 1 sau xi = −1, pentru 1 ≤ i ≤ 2n,care satisfac conditiile:

(a) x1 + x2 + · · ·+ xk ≥ 0, pentru orice k = 1, . . . , n si

(b) x1 + x2 + · · ·+ x2n = 0

Pentru a defini aceasta corespondenta sa ne deplasam pe un drum ascendent d de la O laA. Drumul d se poate scrie ca un sir de segmente de lungime 1, d = (s1, s2, s2n), ordineaindicilor indicand ordinea de parcurgere a segmentelor cand ne deplasam de la O la A. Sirulasociat drumului se obtine din sirul (s1, s2, s2n) scriind ın locul fiecarui segment orizontalnumarul 1 si ın locul fiecarui segment vertical numarul −1.

Conditia (a) exprima faptul ca drumul d nu poate trece prin puncte situate deasupraprimei bisectoare a axelor, iar conditia (b) faptul ca el contine n segmente orizontale si nsegmente verticale, deci faptul ca el se termina ın punctul A.

Conform unei probleme anterioare, numarul sirurilor (x1, . . . , x2n) formate din 1 si −1

cu proprietatile enuntate este egal cu Tn+1 = 1n+1

(2nn

).

Problema 13.18 Sa se arate ca numarul sirurilor (a1, a2, . . . a2n+1) formate din ıntreginenegativi, cu proprietatea ca a1 = a2n+1 = 0 si |ai − ai+1| = 1 pentru i = 1, 2, . . . , 2n,este egal cu numarul lui Catalan

Tn+1 =1

n+ 1

(2n

n

).

Solutie. Conform unei probleme anterioare, numarul sirurilor (x1, x2, . . . , x2n) cu termeni1 sau −1, care verifica conditiile x1 + x2 + · · · + xk ≥ 0 pentru orice 1 ≤ k ≤ 2n si

x1 + x2 + · · ·+ x2n = 0 este egal cu 1n+1

(2nn

).

Sa observam ca exista o bijectie de la multimea sirurilor x = (x1, x2, . . . , x2n) lamultimea sirurilor a = (a1, a2, . . . , a2n+1) care verifica conditiile date, definita prinf(x) = a daca

a1 = 0,a2 = x1 = 1,a3 = x1 + x2,. . .ak+1 = x1 + x2 + · · ·+ xk, pentru 1 ≤ k ≤ 2n,

deci a2n+1 = 0.

Problema 13.19 Fie X o colectie de n obiecte (n ≥ 1), nu neaparat distincte. Dacan ≥ a2 + 1, cu a numar ıntreg nenegativ, aratati ca are loc cel putin unul din urmatoareledoua cazuri:

(a) cel putin a+ 1 obiecte sunt identice;

(b) cel putin a+ 1 obiecte sunt distincte doua cate doua.

Page 221: Teme si probleme pentru concursurile studentesti de matematica ...

214

Solutie. Prin reducere la absurd, presupunem ca (a) si (b) nu au loc. Rezulta ca X continecel mult a obiecte distincte doua cate doua si fiecare dintre acestea sunt prezente ın celmult a copii, deci X are cel mult a2 ≤ n− 1 obiecte, ceea ce este absurd.

In consecinta, concluzia problemei este adevarata.Observatie. Problema de mai sus este de acelasi tip cu urmatoarele:

(a) Din orice sir de n2 +1 numere reale se poate extrage sau un subsir crescator cu n+1termeni, sau un subsir descrescator cu n+ 1 termeni.

(b) Se dau n2 +1 intervale pe o dreapta, atunci sau exista n+1 intervale dintre ele astfelıncat oricare doua sunt disjuncte, sau exista n+ 1 cu intersectia nevida.

Problema 13.20 Fie A = (i)1≤i≤n, B = (i)1≤i≤n, C = (i)1≤i≤n trei partitii ale uneimultimi finite M.

Daca pentru orice i, j, k exista inegalitatea

|Ai ∩Bj |+ |Ai ∩ Ck|+ |Bj ∩ Ck| ≥ n,

atunci |M | ≥ n3

3 , egalitatea putand avea loc ın cazul n ≡ 0 ( mod 3).

Solutie. Sumand inegalitatile date dupa j = 1, 2, . . . , n obtinem

|Ai|+ n|Ai ∩ Ck|+ |Ck| ≥ n2.

Adunam inegalitatile obtinute pentru i = 1, . . . , n si avem

|M |+ 2n|Ck| ≥ n3.

Insumand si dupa k deducem ca n|M |+ 2n|M | ≥ n4, de unde |M | ≥ n3

3 .

Daca n ≡ 0 (mod 3) aceasta margine inferioara poate fi atinsa. Intrucat |M | ≥ n3

3 ,consideram urmatoarea partitie a lui M :

M = A11 ∪A1

2 ∪ · · · ∪A1n ∪A2

1 ∪A22 ∪ · · · ∪A2

n ∪ · · · ∪An1 ∪An2 ∪ · · · ∪Ann,

astfel ıncat |Aji | = n/3 pentru orice 1 ≤ i, j ≤ n.

Daca notam Ai =

n⋃j=1

Aij , Bi =

n⋃j=1

Aji si Ci =

n⋃j=1

Aji+j−1 (mod n), atunci partitiile

(Ai), (Bi), (Ci), 1 ≤ i ≤ n, verifica egalitatea

|Ai ∩Bj |+ |Ai ∩ Ck|+ |Bj ∩ Ck| = n,

deoarece pentru i, j, k sunt verificate relatiile

|Ai ∩Bj | = |Ai ∩ Ck| = |Bj ∩ Ck| =n

3.

In acest caz avem |M | = n3

3 .

Problema 13.21 O functie f : X → X se numeste idempotenta daca f(f(x)) = f(x),pentru orice x ∈ X.

Daca multimea X are n elemente, sa se arate ca numarul i(n) al functiilor idempotentef : X → X este dat de relatia:

i(n) =n∑k=1

(n

k

)kn−k.

Page 222: Teme si probleme pentru concursurile studentesti de matematica ...

Combinatorica si grafuri 215

Solutie. Vom arata pentru ınceput ca f este idempotenta daca si numai daca functiag : Y → Y, unde Y = f(X) si g(x) = f(x) pentru orice x ∈ Y, este functia identica.

Consideram un element arbitrar x ∈ Y = f(X), atunci exista z ∈ X astfel ıncatx = f(z) si ıntrucat f este idempotenta avem:

g(x) = f(x) = f(f(z)) = f(z) = x,

pentru orice element arbitrar x ∈ Y.Daca g este functia identica, deducem:

f(f(x)) = f(y) = g(y) = y = f(x),

pentru orice x ∈ X, deci f este idempotenta.Daca notam |Y | = k, rezulta 1 ≤ k ≤ n.

Multimea Y poate fi aleasa ın

(nk

)moduri, restrictia functiei f la Y este functia

identica, iar numarul functiilor h : X \ Y → Y este egal cu kn−k. Deoarece f este identicape Y rezulta ca f este unic determinata de restrictia sa h la X \ Y. Astfel este justificataformula pentru i(n).

Problema 13.22 Un lant de lungime n ın familia partitiilor unei multimiX cu n elementeeste un sir de partitii distincte doua cate doua care verifica:

P1 < P2 < · · · < Pn.

Partitia P1 are o singura clasa formata din X, iar Pn are n clase care contin fiecare cateun singur element al lui X.

Sa se arate ca numarul lanturilor de lungime n ın familia partitiilor lui X cu n elementeeste egal cu

(n− 1)!n!

2n−1.

Solutie. Fiecare partitie Pk, cu 1 ≤ k ≤ n − 1, are k clase si se obtine din Pk+1 prinunificarea a doua clase ıntr-una singura. Deci, daca Pn, Pn−1, . . . , Pk+1 sunt fixate, partitiaPk poate fi aleasa ın

(k+1

2

)moduri.

Numarul lanturilor de lungime n este egal cu

n−1∏k=1

(k + 1

2

)=

(n− 1)!n!

2n−1.

Problema 13.23 Sa se arate ca:∑A1,...Ak

|A1 ∪A2 ∪ · · · ∪Ak| = n(2k − 1)2k(n−1),

unde suma se face dupa toate alegerile submultimilor A1, . . . Ak ale unei multimi X cu nelemente.

Solutie. Fie Y ⊂ X cu |Y | = p. Y poate fi scris ca o reuniune a k multimi, Y =A1 ∪A2 ∪ · · · ∪Ak ın (2k − 1)p moduri diferite. Intr-adevar, fiecare dintre cele p elementeale lui Y poate apartine unui numar de 2k − 1 familii nevide de submultimi A1, . . . Ak.

Page 223: Teme si probleme pentru concursurile studentesti de matematica ...

216

Deoarece Y poate fi aleasa ın

(np

)moduri diferite, rezulta ca

∑|A1 ∪ · · · ∪Ak| =

n∑p=1

p

(np

)(2k − 1)p

= n(2k − 1)

n∑p=1

(n− 1p− 1

)(2k − 1)p−1

= n(2k − 1)2k(n−1).

Problema 13.24 Sa se demonstreze ca:∑|A1 ∪ · · · ∪Ak| = (2k − 1)

∑|A1 ∩ · · · ∩Ak|,

unde sumele se efctueaza dupa toate alegerile posibile ale submultimilor A1, . . . Ak ale uneimultimi X.

Solutie. Tinand cont de problema anterioara, trebuie sa mai aratam ca∑|A1∩· · ·∩Ak| =

n2k(n−1), daca |X| = n.Deoarece operatia de trecere la complementara:

C(A1 ∪ · · · ∪Ak) = CA1 ∩ · · · ∩ CAk

stabileste o bijectie ıntre familia multimilor de forma A1 ∪ · · · ∪ Ak si familia multimilorde forma A1 ∩ · · · ∩Ak, iar

|CA1 ∩ · · · ∩ CAk| = n− |A1 ∪ · · · ∪Ak|,

putem scrie∑|A1 ∩ · · · ∩Ak| =

∑(n− |A1 ∪ · · · ∪Ak|) = n2nk − n(2k − 1)2k(n−1) = n2k(n−1),

adica ceea ce trebuie aratat.S-a tinut seama ca fiecare din sumele scrise contine 2nk termeni, iar fiecare din multimile

A1, . . . Ak poate fi aleasa dintre submultimile lui X ın 2n moduri.

Problema 13.25 Fie X o multime finita si E1, . . . , Em o familie de submultimi ale luiX cu proprietatea ca oricare doua multimi distincte Ei si Ej nu au exact un element ıncomun si |Ei| ≥ 2 pentru i = 1, . . . ,m.

Sa se arate ca putem colora elementele din X cu doua culori, astfel ıncat nicio multimeEi sa nu aiba toate elementele de o aceeasi culoare, pentru 1 ≤ i ≤ m.

Solutie. Fie X = x1, . . . , xn. Vom colora aceste elemente cu una din culorile a saub, astfel ıncat nicio multime Ei sa nu fie monocromatica. Sa coloram elementul x1 cuculoarea a. Presupunand ca am colorat elementele x1, . . . , xi, cu 1 < i < n, cu culorile asau b astfel ıncat sa nu apara nicio multime Ek monocromatica, sa consideram cazul candacest proces nu mai poate continua. Deci nu putem colora elementul xi+1 cu culoarea a,deoarece exista o multime E ⊂ x1, . . . , xi+1 cu xi+1 ∈ E care are toate elementele diferitede xi+1 colorate cu culoarea a. Elementul xi+1 nu poate fi colorat nici cu culoarea b, deciexista o multime F ⊂ x1, . . . , xi+1 cu xi+1 ∈ F, care are toate elementele diferite de xi+1

colorate cu culoarea b. Obtinem ca E,F sunt doua multimi distincte dintre E1, . . . , Em si

Page 224: Teme si probleme pentru concursurile studentesti de matematica ...

Combinatorica si grafuri 217

E ∩F = xi+1, ceea ce contrazice ipoteza. Deci putem colora elementul xi+1 cu culoareaa sau cu culoarea b, astfel ıncat sa nu apara multimi Ek monocromatice.

Am demonstrat astfel prin inductie ca acest proces de colorare poate continua panacand reusim sa coloram toate elementele din X cu doua culori, astfel ıncat sa nu aparanicio multime monocromatica.

Problema 13.26 Sa se arate ca numarul P (n,m) al partitiilor ıntregului n ın m partisatisface recurenta:

P (n+ k, k) = P (n, 1) + P (n, 2) + · · ·+ P (n, k),

iar P (n, 1) = P (n, n) = 1

Solutie. Partitiile numarului n cu cel mult k parti formeaza o multime cu P (n, 1) +P (n, 2) + · · ·+P (n, k) elemente. Fiecare partitie a lui n cu cel mult k parti poate fi scrisasub forma

n = a1 + a2 + · · ·+ am + 0 + · · ·+ 0,

unde suma contine k termeni si a1 ≥ a2 ≥ · · · ≥ am ≥ 1, cu 1 ≤ m ≤ k.Din aceasta exprimare a lui n putem obtine o partitie a lui n+ k cu k parti astfel:

n+ k = (a1 + 1) + (a2 + 1) + · · ·+ (am + 1) + 1 + · · ·+ 1,

unde suma contine k termeni si a1 + 1 ≥ a2 + 1 ≥ · · · ≥ am + 1 ≥ 1.Aplicatia astfel definita este o injectie, deoarece unor partitii diferite ale lui n cu cel

mult k parti le corespund partitii diferite ale lui n + k cu k parti. Ea este si surjectie,deoarece orice partitie a lui n + k cu k parti provine din acea partitie a lui n cu m ≤ kparti obtinuta prin scaderea unei unitati din fiecare termen al partitiei lui n+k si retinandprimii termeni diferiti de zero. Existenta unei bijectii ıntre multimea partitiilor lui n cucel mult k parti si multimea partitiilor lui n + k cu k parti justifica recurenta data,care permite calculul prin recurenta al tuturor numerelor P (n, k), plecand de la valorileP (n, 1) = P (n, n) = 1, pentru orice n, si P (n, k) = 0 pentru n < k.

Problema 13.27 Sa se arate ca numarul P (n) al partitiilor lui n si P (n,m) al partitiilorıntregului n ın m parti, verifica relatia:

P (n,m) = P (n−m) pentru m ≥ n

2.

Solutie. Fiecarei partitii a lui n ın m parti, de forma n = n1 + n2 + · · · + nm, cu n1 ≥n2 ≥ · · · ≥ nm ≥ 1, ıi corespunde o partitie a lui n−m, obtinuta din scrierea

n−m = (n1 − 1) + (n2 − 1) + · · ·+ (nm − 1),

prin suprimarea eventualilor termeni nuli.Aplicatia astfel definita este injectiva. Vom demonstra ca ea este si surjectiva pentru

m ≥ n2 . Intr-adevar, plecand de la o partitie

n−m = r1 + · · ·+ rk

a lui n −m, obtinem k ≤ m, deoarece ın caz contrar am avea k ≥ m + 1, deci n −m ≥k ≥ m+ 1, ceea ce implica m ≤ n−1

2 . Dar acest rezultat contrazice ipoteza m ≥ n2 .

Adunand cate o unitate la fiecare din termenii de mai sus si adaugand m − k ≥ 0termeni egali cu 1, obtinem o partitie a lui n cu m parti

n = (r1 + 1) + · · ·+ (rk + 1) + 1 + · · ·+ 1,

a carei imagine prin aplicatia descrisa este partitia lui n−m de la care am plecat.

Page 225: Teme si probleme pentru concursurile studentesti de matematica ...

218

Problema 13.28 Sa se justifice expresiile urmatoarelor functii generatoare:

(a) Functia generatoare a numerelor P (n) ale tuturor partitiior ıntregului n este:

∞∑n=0

P (n)xn =1

(1− x)(1− x2)(1− x3) · · ·,

unde P (0) = 1;

(b) Functia generatoare a numerelor P (n,m) ale partitiior ıntregului n ın m parti este:

∞∑n=0

P (n,m)xn =xm

(1− x)(1− x2)(1− x3) · · · (1− xm);

(c) Functia generatoare a numerelor partitiilor lui n ın parti impare este

1

(1− x)(1− x3)(1− x5) · · ·;

(d) Functia generatoare a numerelor partitiilor lui n ın parti distincte doua cate douaeste

(1 + x)(1 + x2)(1 + x3) · · · ;

(e) Functia generatoare a numerelor partitiilor lui n ın parti impare, distincte doua catedoua este

(1 + x)(1 + x3)(1 + x5) · · · .

Solutie. Vom da demonstratia doar ın cazul (a), ın celelalte cazuri ea facandu-se analog.Fie dezvoltarea

1

(1− a1x)(1− a2x2) · · · (1− akxk) · · ·=

(1 + a1x+ a21x

2 + · · · )(1 + a2x2 + a2

2x4 + · · · ) . . . (1 + akx

k + a2kx

2k + · · · ) =

1 + a1x+ (a21 + a2)x2 + · · ·+ (aλ11 aλ22 · · · a

λkk + · · · )xn + · · ·

Se observa ca termenul aλ11 aλ22 · · · aλkk care intra ın coeficientul lui xn are proprietatea

ca λ1 + 2λ2 + · · ·+ kλk = n, deci el defineste o partitie a lui n si anume:

n = k + k + · · ·+ k︸ ︷︷ ︸λk

+ · · ·+ 2 + 2 + · · ·+ 2︸ ︷︷ ︸λ2

+ 1 + 1 + · · ·+ 1︸ ︷︷ ︸λ1

.

Tinand cont de modul de desfacere a parantezelor, se observa ca ın acest fel exponentiiliterelor care apar ın coeficientul lui xn genereaza fara repetitie toate partitiile lui n, decidaca facem a1 = a2 = · · · = 1, coeficientul lui xn va fi tocmai numarul partitiilor lui n,adica P (n).

Din (a) deducem (c) deoarece λ2 = λ4 = · · · = 0Demonstratia punctelor (d) si (e) se face ın mod analog.

Problema 13.29 Sa se determine toate perechile de numere ıntregi pozitive (a, b) cu pro-prietatea ca exista o descompunere a multimii numerelor ıntregi pozitive ın doua multimiA si B, astfel ıncat aA = bB.

Page 226: Teme si probleme pentru concursurile studentesti de matematica ...

Combinatorica si grafuri 219

IMC, 2003

Solutie. Este evident ca a 6= b, deoarece A si B sunt disjuncte.Fie a, b o solutie a problemei, pentru care avem o descompunere a multimii numerelor

ıntregi data de multimile A si B, astfel ıncat aA = bB. Daca notam d = (a, b) cel mai maredivizor comun al numerelor a si b, atunci a = da1, b = bb1, (a1, b1) = 1 si a1A = b1B.Rezulta ca a1, b1 este o solutie, deci este suficient sa determina solutiile a, b doar ıncazul (a, b) = 1.

Daca 1 ∈ A, atunci a ∈ aA = bB, de unde obtinem ca b este un divizor al lui a.Analog, daca 1 ∈ B, atunci a este un divizor al lui b. Deci, pentru toate solutiile, unuldintre numerele a, b este un divizor al celuilalt.

Sa demonstram ca daca n ≥ 2, atunci perechea (1, n) este o solutie a problemei.Pentru fiecare ıntreg pozitiv k, fie f(k) cel mai mare ıntreg nenegativ astfel ıncat nf(k) | k.Consideram multimile

A = k : f(k) este impar, B = k : f(k) este par.

Astfel am obtinut o descompunere a multimii numerelor ıntregi nenegative ce satisfaceconditia A = nB.

Problema 13.30 Pentru un numar ıntreg n ≥ 3, se definesc multimile,

Sn = (x1, x2, . . . , xn);∀i, xi ∈ 0, 1, 2

An = (x1, x2, . . . , xn) ∈ Sn;∀i ≤ n− 2, |xi, xi+1, xi+2| 6= 1

siBn = (x1, x2, . . . , xn) ∈ Sn;∀i ≤ n− 1, (xi = xi+1 ⇒ xi 6= 0) .

Sa se arate ca |An+1| = 3|Bn|.

IMC, 2005

Solutia 1. Extindem definitiile ın pentru n = 1, 2. Consideram urmatoarele multimi

A′n = (x1, x2, . . . , xn) ∈ An;xn−1 = xn , A

′′n = An \A

′n

B′n = (x1, x2, . . . , xn) ∈ Bn;xn = 0 , B′′n = Bn \B

′n

si notam an = |An|, a′n = |A′n|, a

′′n = |A′′n|, bn = |Bn|, b

′n = |B′n|, b

′′n = |B′′n|.

Urmatoarele relatii pentru a−siruri sunt evidente:an = a

′n + a

′′n

a′n+1 = a

′′n

a′′n+1 = 2a

′n + 2a

′′n,

ceea ce ne conduce la an+1 = 2an + 2an−1.Pentru b−siruri sunt adevarate relatiile:

bn = b′n + b

′′n

b′n+1 = b

′′n

b′′n+1 = 2b

′n + 2b

′′n,

de unde obtinem bn+1 = 2bn + 2bn−1.Pentru primele valori ale sirurilor (an) si (bn) avema1 = 3, a2 = 9, a3 = 24,b1 = 3, b2 = 8,

Page 227: Teme si probleme pentru concursurile studentesti de matematica ...

220

de undea2 = 3b1a3 = 3b2.

In continuare, se demonstreaza prin inductie ca an+1 = 3bn, pentru orice n ≥ 1.Solutia 2. Considerand xi ca elemente ale lui Z3 si lucrand ”modulo 3”, obtinem(x1, x2, . . . , xn) ∈ An ⇒ (x1 + 1, x2 + 1, . . . , xn + 1) ∈ An,

(x1 + 2, x2 + 2, . . . , xn + 2) ∈ An,ceea ce ınseamna ca 1/3 dintre elementele lui An ıncep cu 0. Stabilim astfel o bijectie

ıntre submultimea vectorilor lui An+1 care ıncep cu 0 si multimea Bn prin

(0, x1, x2, . . . , xn) ∈ An+1 7→ (y1, y2, . . . , yn) ∈ Bn

y1 = x1, y2 = x2 − x1, y3 = x3 − x2, . . . , yn = xn − xn−1.

(daca yk = yk+1 = 0⇒ xk −xk−1 = xk+1−xk = 0, cu x0 = 0, de unde xk−1 = xk = xk+1,ceea ce contrazice definitia lui Ak−1.)

Aplicatia inversa este definita prin

(y1, y2, . . . , yn) ∈ Bn 7→ (0, x1, x2, . . . , xn) ∈ An+1

x1 = y1, x2 = y1 + y2, . . . , xn = y1 + y2 + · · ·+ yn.

Problema 13.31 Fie V un poligon convex cu n varfuri.

(a) Sa se demonstreze ca daca n este divizibil cu 3, atunci V poate fi triangulat (adicaımpartit ın triunghiuri disjuncte doua cate doua, cu pastrarea varfurilor) astfel ıncatfiecare varf al lui V devine varf al unui numar impar de triunghiuri.

(a) Daca n nu este divizibil cu 3, atunci V poate fi triangulat astfel ıncat exact douavarfuri ale sale devin varfuri ale unui numar par de triunghiuri.

IMC, 2006

Solutie. Vom rezolva problema prin inductie dupa n. In cazurile n = 3, 4, 5 concluziaeste evidenta.

Presupunem ca este adevarata concluzia problemei pentru n = k si consideram cazuln = k + 3. Notam varfurile lui V cu P1, . . . , Pk+3. Conform ipotezei inductive pentrupoligonul P1 . . . Pk, fiecare dintre varfurile acestuia apartine unui numar impar de tri-unghiuri, cu exceptia a doua varfuri daca n nu este divizibil cu 3. Adaugam triunghiurileP1PkPk+2, PkPk+1Pk+2 si P1Pk+2Pk+3. Astfel atasam doua noi triunghiuri cu varfurile ınP1 si Pk, deci paritatea este mentinuta.

Varfurile Pk+1, Pk+2, Pk+3 apartin unui numar impar de triunghiuri. Atunci numarulvarfurilor care apartin unui numar par de triunghiuri ramane acelasi ca si pentru poligonulP1P2 . . . Pk.

Problema 13.32 Un produs trebuie sa treaca prin sase faze de prelucrare (operatii)pentru a ajunge finit. Fie xi, i = 1, 6 aceste operatii. Conditiile tehnice impun respectareaurmatoarelor restrictii:

a) Operatia x1 trebuie efectuata ınaintea operatiilor x2 si x4;b) Operatia x2 trebuie efectuata ınaintea operatiei x3 si dupa operatia x5;c) Operatia x4 trebuie efectuata ınaintea operatiilor x3 si x5;d) Operatia x6 trebuie efectuata ultima.Sa se indice ordinea efectuarii operatiilor astfel ıncat, trecand o data si numai o data

prin fiecare faza de prelucrare, sa se obtina produsul finit.

Page 228: Teme si probleme pentru concursurile studentesti de matematica ...

Combinatorica si grafuri 221

Solutie. Vom considera cele sase operatii drept varfuri ale unui graf, iar faptul caoperatia xi trebuie efectuata ınaintea operatiei xj va fi desemnat prin arcul (xi, xj) .

In aceste conditii se obtine graful

Metoda 1.Utilizam matricea conexiunii directe.In prima etapa eliminam varful corespunzator coloanei formate numai din zerouri, iar

din matrice linia si coloana corespunzatoare acestuia. Continuam procedeul cu matriceaastfel obtinuta pana la ultimul varf eliminat.

Ordinea fazelor de prelucrare va fi indicata de varfurile grafului, ın ordinea eliminarii.

x1

x2

x3

x4

x5

x6

x1 x2 x3 x4 x5 x6

0 1 0 1 0 1

0 0 1 0 0 10 0 0 0 0 10 0 1 0 1 00 1 1 0 0 10 0 0 0 0 0

x1→

x2

x3

x4

x5

x6

x2 x3 x4 x5 x60 1 0 0 10 0 0 0 10 1 0 1 0

1 1 0 0 10 0 0 0 0

x4→

x2

x3

x5

x6

x2 x3 x5 x60 1 0 10 0 0 11 1 0 1

0 0 0 0

x5→x2

x3

x6

x2 x3 x6 0 1 1

0 0 10 0 0

x2→ x3

x6

x3 x6(0 1

0 0

)x3→

Asadar ordinea fazelor de prelucrare este indicata de drumul hamiltonian:

DH = (x1, x4, x5, x2, x3, x6) .

Metoda 2.Utilizam matricea conexiunii totale.

T =

0 1 1 1 1 10 0 1 0 0 10 0 0 0 0 10 1 1 0 1 10 1 1 0 0 10 0 0 0 0 0

x1 x2 x3 x4 x5 x6

x1

x2

x3

x4

x5

x6

Observam ca tii = 0, i = 1, 6 si deci graful nu are cicluri.Numarul elementelor nenule din matricea T este C2

6 si deci, ın graf exista un drumhamiltonian (conform teoremei lui Chen).

Asezam liniile matricei T dupa ,,puterile” de atingere a varfurilor si obtinem matricea

Page 229: Teme si probleme pentru concursurile studentesti de matematica ...

222

T ∗ =

0 1 1 1 1 10 1 1 0 1 10 1 1 0 0 10 0 1 0 0 10 0 0 0 0 10 0 0 0 0 0

x1

x4

x5

x2

x3

x6

Drumul hamiltonian este DH = (x1, x4, x5, x2, x3, x6) .

Problema 13.33 Intr-o camera se afla sase persoane. Utilizand teoria grafurilor,demonstrati ca exista 3 persoane care se cunosc ıntre ele sau 3 persoane care nu se cunoscıntre ele.

Solutie. Consideram ca cele sase persoane sunt situate ın nodurile unui graf conex cusase varfuri.Luam ca referinta nodul A (persoana A).

F

E

D

C

A

B

Unim doua noduri printr-o muchie verde daca persoanele corespunzatoare se cunosc siprintr-o linie rosie daca ele nu se cunosc.

In camera cele 6 persoane pot fi ın situatia X cunoaste pe Y sau X nu cunoaste pe Y.Din A pleaca 5 muchii ⇒ sunt cel putin 3 muchii de aceeasi culoare (presupunem

verde).Izolam subgraful A,F,E,C .Daca F si E s-ar cunoaste (linia verde), atunci avem 3 persoane care se cunosc: A,F,E.Asadar F si E nu se cunosc (linia rosie).Rationam analog pentru muchiile CE si CF .Obtinem astfel triunghiul CEF, ın varfurile caruia se afla persoane care nu se cunosc

ıntre ele.

Problema 13.34 Intr-un plan trasam un numar oarecare de drepte, astfel ıncat oricaretrei dintre ele sa nu fie concurente. Obtinem un graf planar G considerand punctele deintersectie ale dreptelor ca varfuri ale grafului si segmentele dintre punctele de intersectievecine drept muchii ale acestui graf.Sa se demonstreze ca χ (G) ≤ 3.

Solutie. Notam coordonatele punctelor de intersectie, ıntr-un sistem ortogonal de axeın plan cu (x1, y1) ; (x2, y2) ; . . . ; (xn, yn) .

Putem presupune ca directiile axelor de coordonate sunt astfel alese ıncat absciselex1, x2, . . . , xn sa fie diferite doua cate doua.

Nu restrangem generalitatea presupunand ca x1 < x2 < · · · < xn.Vom colora varfurile grafului, ın aceasta ordine, succesiv cu trei culori. Daca am colorat

cu 3 culori varfurile de abscise x1, x2, . . . , xi−1, varful (xi, yi) are cel mult doua varfuriadiacente lui care au fost deja colorate, deoarece nu exista trei drepte concurente.

Page 230: Teme si probleme pentru concursurile studentesti de matematica ...

Combinatorica si grafuri 223

Asadar pentru varful (xi, yi) gasim o a treia culoare disponibila, pentru i = 2, n, ceeace verifica inegalitatea ceruta ın enunt.

Problema 13.35 Consideram un graf conex G si notam cu d (x, y) distanta dintrevarfurile x si y, adica numarul muchiilor din cel mai scurt lant care uneste pe x cu ysi cu e (x) = max

y∈Vd (x, y) excentricitatea varfului x.

Centrul grafului G este format din acele varfuri ce au excentricitatea minima, minimnotat cu ρ (G) si numit raza grafului G.

a) Sa se demonstreze ca centrul oricarui arbore este format dintr-un varf sau din douavarfuri adiacente.

b) Daca G este arbore, sa se demonstreze ca e (x) este o functie convexa, ın sensul cadaca y si z sunt varfuri adiacente cu x, atunci:

e (x) ≤ e (y) + e (z)

2

c) Demonstrati ca pentru orice graf conex avem d (G) ≤ 2ρ (G), unde d (G) este di-ametrul grafului G.

Solutie. a) Demonstram mai ıntai ca daca suprimam toate varfurile de grad 1 alearborelui G, atunci e (x) descreste cu o unitate pentru toate varfurile subgrafului astfelobtinut.

Toate varfurile la distanta e (x) de x sunt de gradul 1, deci prin suprimarea acestorae (x) descreste pentru toate varfurile ramase. Prin aceasta operatie e (x) descreste cu exacto unitate, deoarece cel mai lung lant care pleaca din x se termina ıntr-un varf de gradul 1al lui G, care este apoi suprimat. Proprietatea este evidenta pentru grafurile cu un singurvarf sau doua si sa presupunem ca ea este adevarata pentru toti arborii cu cel mult n− 1varfuri.

Consideram ca G are n > 2 varfuri.Notam cu C0 multimea varfurilor x care formeaza centrul lui G. Daca C0 nu contine

nici un varf de gradul 1, suprimam toate varfurile de grad 1 ale arborelui G. Deoarecepentru toate varfurile ramase x, valoarea e (x) scade cu o unitate, rezulta ca prin aceastaoperatie se obtine un nou arbore G∗ cu acelasi centru C0.

Deoarece G∗ are cel mult n− 2 varfuri, rezulta conform ipotezei de inductie ca C0 esteformat dintr-un varf sau din doua varfuri adiacente si demonstratia este ıncheiata. DacaC0 contine un varf x de gradul g (x) = 1 rezulta ca x este adiacent cu un singur varf y.Evident ca y este strict mai apropiat decat x de oricare varf alt al lui G.

Deci e (x) poate fi minim numai daca e (x) = 1 si G este un arbore format din varfurilex si y legate printr-o muchie. In acest caz, C0 = x, y si proprietatea este demonstrata.Lanturile de lungime maxima ale unui arbore au intersectia nevida si aceasta continecentrul arborelui.

b) Fie L un lant de lungime e (x) care pleaca din x. Daca L nu contine nici unul dintrevarfurile y sau z, atunci:

e (y) = 1 + e (x) si e (z) = 1 + e (x)

Din acestea obtinem:

e (x) <e (y) + e (z)

2

Lantul L nu poate sa contina si varful y si varful z deoarece ambele varfuri sunt vecinecu x.

Page 231: Teme si probleme pentru concursurile studentesti de matematica ...

224

Daca L contine pe y, obtinem: e (y) ≥ e (x)− 1 si e (z) = e (x) + 1, de unde obtinem:

e (x) ≤ e (y) + e (z)

2

c) Aplicatia d : V × V → N, d = d (x, y) , definita ın enunt pentru orice x, y ∈ V esteo metrica pe aceasta multime si deci ea satisface inegalitatea triunghiulara. Fie x, y douavarfuri astfel ıncat d (x, y) = d (G) si z un varf de excentricitate minima, egal cu ρ (G) .

Avem:d (G) = d (x, y) ≤ d (x, z) + d (z, y) ≤ ρ (G) + ρ (G) = 2ρ (G)

Problema 13.36 Daca G ≡ (V,U) este un arbore si f : V → V este o aplicatie cu pro-prietatea ca daca [x, y] ∈ U, atunci f (x) = f (y), sau [f (x) , f (y)] ∈ U, sa se demonstrezeca f are un punct fix sau o muchie fixa.

Solutie. Demonstram proprietatea prin inductie, dupa numarul varfurilor arboreluiG.

Daca |V | ∈ 1, 2, proprietatea este evidenta. Presupunem ca proprietatea esteadevarata pentru orice arbore cu cel mult n − 1 varfuri si sa o demonstram pentru unarbore G cu n ≥ 3 varfuri.

Daca f este o bijectie, atunci pentru orice x 6= y avem f (x) 6= f (y) si [x, y] ∈ U implica[f (x) , f (y)] ∈ U, deci f este un automorfism al lui G. Orice varf terminal este dus de ftot ıntr-un varf terminal. Notam cu G∗ subarborele obtinut din G prin suprimarea tuturorvarfurilor terminale.

Evident G∗ este nevid deoarece n ≥ 3. Notam cu V ∗ multimea varfurilor lui G∗.Rezulta ca f (V ∗) = V ∗ si restrictia lui f la V ∗, notata f∗ are aceeasi proprietate ca sif. Asadar f∗ si ın consecinta f are un punct fix sau o muchie fixa, conform ipotezei deinductie (|V ∗| ≤ n− 2) .Daca f nu este o bijectie, atunci f (V ) este o submultime proprie de varfuri ale lui G.

Aceste varfuri induc un subgraf conex al lui G din conditia impusa lui f, deci f (V )este multimea varfurilor unui arbore si |f (V ) | ≤ n− 1.

Deoarece f (f (V )) ⊂ f (V ) putem considera restrictia lui f la subarborele general demultimea de varfuri f (V ) care are aceeasi proprietate ca f. Conform ipotezei de inductie,aceasta restrictie, deci si f, are un punct fix sau o muchie fixa.

Observatie. Proprietatea nu mai este adevarata daca G este un graf cu cicluri.De exemplu, daca G ≡ K3 contine varfurile x, y, z, definim f prin f (x) = y, f (y) = z,f (z) = x. In acest caz, aplicatia f nu are nici puncte fixe, nici muchii fixe.

Problema 13.37 Se noteaza cu an numarul arborilor cu varfurile x1, x2, . . . , xn.a) Sa se demonstreze ca:

an =n−1∑k=1

k

(n− 2

k − 1

)akan−k;

b) Sa se demonstreze formula lui Cayley plecand de la aceasta identitate si folosinduna din identitatile lui Abel.

O. Dziobek, Sitzungsber. Berl. Math. G., 17 (1917), 64-67

Solutie. a) Pentru orice arbore A cu n varfuri x1, x2, . . . , xn, daca suprimam o muchieoarecare obtinem doi arbori disjuncti care contin ımpreuna toate varfurile lui A. Marcamextremitatile muchiei suprimate. Deoarece A are n − 1 muchii, plecand de la toti cei an

Page 232: Teme si probleme pentru concursurile studentesti de matematica ...

Combinatorica si grafuri 225

arbori cu n varfuri obtinem (n− 1) an perechi de astfel de arbori cu cate un varf marcatın fiecare arbore. Daca A1 si A2 sunt doi arbori disjuncti cu k respectiv n−k varfuri, carecontin ımpreuna toate varfurile x1, x2, . . . , xn, putem marca un varf al lui A1 si un varf allui A2 ın k (n− k) moduri pentru k = 1, n− 1.

Multimea varfurilor lui A1 si A2 pot fi alese ın(n−1k−1

)moduri cu conditia ca un varf

fixat, fie acesta x1 sa apartina arborelui A1, pentru a elimina repetitiile.Putem gasi ak, respectiv an−k arbori cu multimea varfurilor ın A1, respectiv A2, deci

numarand ın doua moduri perechile de arbori disjuncti care contin ımpreuna varfurilex1, x2, . . . , xn si au cate un varf marcat ın fiecare arbore, obtinem:

n−1∑k=1

(n− 1

k − 1

)akan−kk (n− k) = (n− 1) an. (13.1)

Cum(n−1k−1

)= n−1

n−k(n−2k−1

), din (13.1) obtinem imediat identitatea ceruta ın enunt.

b) In identitatea (13.1) schimbam ıntre ei indicii k si n − k, tinem cont de faptul ca(n−1k−1

)=(n−1n−k)

si aceasta devine:

n−1∑k=1

(n− 1

k

)akan−kk (n− k) = (n− 1) an. (13.2)

Adunand membru cu membru (13.1) cu (13.2), obtinem:

n−1∑k=1

(n

k

)akan−kk (n− k) = 2 (n− 1) an. (13.3)

Observatie. Aceasta identitate se poate deduce si direct daca nu mai fixam varful x1

ın A1.Demonstram ca an = nn−2, prin inductie dupa n.Daca n = 1⇒ exista un singur arbore cu un varf si formula se verifica.Presupunem ca am = mm−2, (∀)m = 1, n− 1.Pentru a demonstra ca an = nn−2, conform cu (13.3) este suficient sa demonstram ca:

n−1∑k=1

(n

k

)kk−1 (n− k)n−k−1 = 2 (n− 1)nn−2,

egalitate care este una dintre identitatile lui Abel.

Problema 13.38 Se da graful G cu n2 varfuri care corespund patratelor unei table desah cu n linii si n coloane (n ∈ N∗ impar).

Un cal de pe tabla de sah este mutat pe un drum ın forma de L (doua varfuri seconsidera adiacente daca exista o ,,saritura” ın L a calului de pe un patrat pe celalalt)

Poate un cal de pe tabla de sah, ,,sarind” ın forma de L, sa treaca o data si numai odata prin toate cele n2 patrate ale tablei si sa se ıntoarca ın punctul de plecare?

Solutie. Deoarece calul sare de pe un patrat alb pe unul negru si de pe un patratnegru pe unul alb, rezulta ca acest graf este bipartit.

Graful G considerat contine doar cicluri elementare cu un numar impar de varfuri,deci nu exista un ciclu elementar cu n2 varfuri.

In concluzie, raspunsul la ıntrebare este nu.

Page 233: Teme si probleme pentru concursurile studentesti de matematica ...

226

Problema 13.39 Fie A1 ≡ (V,U1) si A2 ≡ (V,U2) doi arbori care au aceeasi multimede varfuri V. Daca pentru orice varf x ∈ V, subgraful obtinut din A1 prin suprimareavarfului x si a muchiilor incidente cu x este izomorf cu subgraful obtinut din A2 prinaceeasi operatie, sa se demonstreze ca arborii A1 si A2 au acelasi diametru.

P.J. Kelly, Pacific J. Math. 7 (1957), 961-968

Solutie. Notam grafurile obtinute din A1, respectiv A2 prin suprimarea varfului x sia muchiilor incidente cu x prin A1x, respectiv A2x.

Deoarece un arbore cu n varfuri are (n− 1) muchii, gradul varfului x ın arborele A1

este:[1] g1 (x) = |V | − 1 − m (A1x) , unde prin m (A1x) am notat numarul muchiilor din

graful A1x.Analog obtinem:[2] g2 (x) = |V | − 1−m (A2x)Deoarece A1x si A2x sunt izomorfe ⇒ m (A1x) = m (A2x) .Cu aceasta din [1] si [2] deducem ca g1 (x) = g2 (x) , pentru orice varf x, deci arborii

A1 si A2 au aceleasi varfuri terminale (de gradul unu).Notam cu T multimea varfurilor terminale pentru arborii A1 si A2.Daca |T | = 2, rezulta ca A1 si A2 sunt lanturi de lungime |V | − 1, deci A1 si A2 au

acelasi diametru.Presupunem acum ca |T | ≥ 3.Consideram L un lant elementar de lungime maxima ın arborele A1, lungime care este

prin definitie diametrul lui A1, adica d (A1) .Extremitatile acestui lant sunt doua varfuri terminale din multimea T.Multimea T mai contine cel putin un alt varf terminal x care nu apartine lantului L.Conform ipotezei A1x este izomorf cu A2x. Deoarece lantul L este continut ın arborele

A1x, rezulta ca exista un lant elementar L∗, de aceeasi lungime cu lantul L si care estecontinut ın arborele A2x.

Asadar si arborele A2 contine lantul elementar L∗, de aceeasi lungime cu lantul L.Deducem ca d (A2) ≥ d (A1) .

In rationamentul precedent schimbam rolul arborilor A1 si A2 si deducem ca d (A1) ≥d (A2) .

Asadar arborii A1 si A2 au acelasi diametru.Observatie. In conditiile date chiar arborii A1 si A2 sunt izomorfi.

Problema 13.40 Intr-o tara sunt n orase, oricare doua fiind unite fie printr-o autostrada,fie prin cale ferata. Un turist doreste sa faca un tur complet prin aceasta tara (sa vizitezefiecare oras o singura data si sa se ıntoarca ın orasul de unde a pornit). Demonstrati caturistul poate alege orasul de pornire si traseul astfel ıncat sa nu schimbe mijlocul detransport ales mai mult de o data.

Solutie. Reprezentam tara printr-un graf complet G cu n varfuri, avand muchiilecolorate cu doua culori ın functie de traseul ales dintre orasele corespunzatoare varfurilor.

Notam cu c1, respectiv cu c2 culorile alese pentru muchiile corespunzatoare au-tostrazilor, respectiv caii ferate.

Convenim sa numim un ciclu ,,bun” daca el este hamiltonian si are sau toate muchiilela fel colorate, sau parcurgand ciclul dintr-un oarecare oras ales, culoarea muchiilor se vaschimba o singura data, pentru ca ciclul sa corespunda cerintei din enunt.

Folosim inductia dupa n. Pentru n ∈ 2, 3 cerinta este ındeplinita.

Page 234: Teme si probleme pentru concursurile studentesti de matematica ...

Combinatorica si grafuri 227

Consideram afirmatia adevarata pentru orice k ≤ n si sa demonstram aceasta pentruk = n+ 1. Fie X un varf oarecare al grafului G.

Graful G∗, format din varfurile lui G, diferite de X, contine un ciclu bun, notat C∗

(conform ipotezei de inductie).Daca acest ciclu este monocromatic, atunci inserand varful X ıntre oricare doua varfuri

ale lui C∗ obtinem un ciclu ”bun” C al lui G. Daca C∗ este format din muchii de douaculori, atunci el are forma V1V2 . . . VkVk+1 . . . VnV1, unde muchiile V1V2, V2V3, . . . , Vk−1Vksunt de culoare c1, iar muchiile VkVk+1, Vk+1Vk+2, . . . , VnV1 sunt de culoare c2.

Consideram muchia VkX din G.Daca aceasta are culoarea c1, atunci ciclul C ≡ V1−V2−· · ·−Vk−X−Vk+1−· · ·−Vn−V1

este bun.Daca VkX are culoarea c2, atunci ciclul C ≡ V1 − V2 − · · · − Vk−1 −X − Vk − Vk+1 −

· · · − Vn − V1 este bun.

Problema 13.41 La o petrecere au venit n barbati si n femei. Daca un barbat doreste sadanseze cu o femeie, cuplul acesta ıl vom numi ,,compatibil”. Compatibilitatea se considerareciproca. Demonstrati ca este posibil sa formam n cupluri compatibile daca si numai dacapentru orice grup de k barbati exista cel putin k femei compatibile cu cel putin un barbatdintre cei k barbati.

P. Hall, 1935

Solutie. Transpunem enuntul ın limbajul teoriei grafurilor:Avem un graf bipartit G cu 2n varfuri, ımpartite ın doua multimi A si B cu cate n

elemente.Din enunt deducem ca pentru oricare k varfuri din A, exista cel putin k varfuri ın B

unite cu cel putin unul din cele k varfuri din A. (Vom numi aceasta proprietate ,,conditialemei mariajelor”.)

Trebuie sa demonstram ca putem uni fiecare varf din A cu un varf din B, astfel ıncatdin fiecare varf sa porneasca exact o muchie.

Procedam prin inductie dupa n.Pentru n ∈ 1, 2 , cerinta problemei este evidenta. Presupunem afirmatia adevarata

pentru orice graf bipartit cu 2m varfuri (m < n) si sa demonstram ca ea este adevaratapentru m = n.

Consideram un varf oarecare X ∈ B.Fie C ⊆ A, multimea varfurilor din A unite cu X.Daca unind un varf Y ∈ C cu X, conditia lemei mariajelor se pastreaza pentru graful

ramas (fara varfurile X si Y ), atunci aplicand inductia, obtinem concluzia pentru G.Daca unind Y cu X, conditia lemei mariajelor nu se pastreaza pentru graful ramas,

atunci exista o submultime C∗ ⊆ A \ Y pentru care exista o submultime D∗ ⊆ B \ Xcu mai putin de |C∗| varfuri din B unite cu cel putin un varf din C∗. Insa conform conditieilemei mariajelor pentru C∗, avem ca ın graful initial G exista o submultime D ⊆ B cu celputin |C∗| varfuri unite cu cel putin un varf din C∗. Rezulta ca diferenta dintre D si D∗

consta ın varful X, de unde |D∗| = |C∗| − 1 si apoi |D| = |C∗|.Tinand cont de faptul ca C∗ ⊆ A si D ⊆ B, ultima relatie afirma faptul ca exista

cateva, sa zicem k < n varfuri din A, astfel ıncat exista exact k varfuri ın B care pot fiunite cu exact cate un varf din cele k varfuri din A.

Deoarece conditia lemei mariajelor este adevarata pentru G si |D| = |C∗| deducem caea este adevarata si pentru acest subgraf G∗ cu 2k varfuri.

Asadar putem aplica inductia pentru aceste 2k varfuri si le putem uni ın k perechi.

Page 235: Teme si probleme pentru concursurile studentesti de matematica ...

228

Ramane sa demonstram ca este adevarata conditia lemei mariajelor si pentru subgrafulramas H, format din 2n− 2k varfuri. Folosim metoda reducerii la absurd.

Fie A∗ si B∗ cele doua multimi cu cate n − k varfuri ale grafului bipartit H. Atunciconditia lemei mariajelor nu are loc ın acest subgraf.

Fie E ⊂ A∗ o submultime a varfurilor lui H pentru care nu are loc lema mariajelor.Daca |E| = p, atunci exista cel mult p− 1 varfuri ın B∗ care sunt unite cu un varf din

E.In graful G (unde conditia lemei mariajelor are loc) considerand multimea de varfuri

E ∪ C∗ deducem ca exista cel mult k + |E| − 1 varfuri din B care pot fi unite cu un varfdin E ∪ C∗, ceea ce contrazice conditia lemei mariajelor.

Problema 13.42 Intr-un oras, dintr-o statie de metrou se poate ajunge ın orice altastatie (eventual prin intermediul altor statii).

Demonstrati ca exista o statie ce poate fi ınchisa pentru reparatii, astfel ıncat din oricestatie ramasa sa putem ajunge ın oricare alta.

Solutie. Consideram harta orasului ca fiind un graf, ale carui varfuri sunt statiile demetrou.

Luam un varf oarecare X0 si construim un sir (xn)n∈N ın modul urmator:Initial xn = −1, pentru orice n ≥ 1, cu exceptia lui x0, care este zero.Apoi pentru k = 1, 2, . . . efectuam urmatorul algoritm: tuturor varfurilor Xn pentru

care xn = −1 si care sunt unite cu cel putin un varf al carui xn este (k − 1) li se asociazaun xn = k. Dupa ce am asociat fiecarui varf un xn, luam un varf Xi astfel ıncat xi estemaximal.

Vom demonstra ca Xi poate fi sters (ımpreuna cu muchiile incidente lui) astfel ıncatgraful ramas este tot conex.

Intr-adevar luam doua varfuri oarecare Xp si Xq. Deoarece xp, xq ≤ xi, atunci se poateajunge din X0 ın Xp si Xq fara a trece prin Xi.

Atunci ıntre Xp si Xq exista drumul:

Xp → X0 → Xq.

Problema 13.43 Intr-o multime de 2n+ 1 persoane, pentru oricare n dintre acestea maiexista una (nu dintre cele n) care le cunoaste pe toate cele n persoane.Demonstrati ca ın multime exista o persoana care le cunoaste pe toate celelalte.

Rusia, 2001

Solutie. Corespunzator problemei consideram graful G, ale carui varfuri sunt per-soanele multimii.

Demonstram mai ıntai ca graful G contine un subgraf complet cu cel putin n + 1varfuri. Sa presupunem, prin reducere la absurd, ca aceasta afirmatie nu este adevarata.

Fie k, numarul de varfuri ale subgrafului maximal si complet G∗ al grafului G.In cazul k ≤ n, adaugam la aceste k persoane alese ın mod arbitrar ınca n−k persoane.Conform enuntului mai exista o persoana P care le cunoaste pe toate aceste n persoane

(inclusiv pe primele k).Dar, atunci varfurile lui G∗ ımpreuna cu P formeaza un subgraf complet cu k + 1

varfuri, contradictie cu alegerea lui G∗.Asadar k ≥ n + 1. Consideram acum un subgraf complet cu n + 1 varfuri. Conform

enuntului exista o persoana care le cunoaste pe toate cele n persoane ramase. Evident caaceasta persoana trebuie sa apartina subgrafului complet ales, deci ea cunoaste si restulpersoanelor din multime.

Page 236: Teme si probleme pentru concursurile studentesti de matematica ...

Combinatorica si grafuri 229

Problema 13.44 17 savanti tin legatura prin e-mail. Ei comunica despre unul dintre treisubiecte convenite. Demonstrati ca exista trei savanti care discuta ıntre ei acelasi subiect.

OIM, 1964

Solutie. Consideram savantii ca varfuri ale unui graf complet G si ,,subiectele” lor dediscutie ca reprezentand trei culori c1, c2, c3.

Fiecare muchie a acestui graf este colorata cu una dintre aceste culori.Trebuie sa demonstram ca exista un triunghi monocromatic ın G. Consideram, ın mod

arbitrar un varf X al lui G.X are 16 vecini si conform principiului lui Dirichlet, exista cel putin sase muchii ce

pornesc din X, colorate cu aceeasi culoare (presupunem ca aceasta este culoarea c1).Consideram ca aceste muchii unesc varful X cu varfurile X1, X2, . . . , X6.Daca ar exista o muchie XiXj , i 6= j, i, j = 1, 6 de culoarea c1, atunci varfurile X, Xi,

Xj formeaza un triunghi monocromatic.Daca nu exista o astfel de muchie, atunci muchiile dintre X1, X2, . . . , X6 sunt colorate

cu doar doua culori diferite (c2 si c3).Demonstram acum ca exista un triunghi monocolor cu varfurile printre X1, X2, . . . , X6.Consideram ca acestea sunt varfurile unui hexagon convex, avand laturile si diagonalele

colorate cu una dintre cele doua culori.Graful corespunzator este complet.Considerand varfulX1, dintre cele cinci muchii care pornesc din el, conform principiului

lui Dirichlet, cel putin trei au aceeasi culoare.Fie ca X1X2, X1X3, si X1X4 au culoarea c2.Considerand triunghiul X2X3X4, daca X2X3, X2X4 sau X3X4 au culoarea c2, atunci

ımpreuna cu varful X1 avem un triunghi cu laturile de culoarea c2.Daca nu, rezulta ca X2X3, X2X4 si X3X4 sunt de culoarea c3 si deci, triunghiul

X2X3X4 este cel cautat.

Problema 13.45 Se considera un poligon convex cu n laturi si ın varfurile sale se scriu nu-merele x1, x2, . . . , xn, iar pe fiecare latura se scrie produsul numerelor scrise ın extremitatileacestei laturi. Fie S suma numerelor scrise pe toate laturile. Demonstrati inegalitatea:

√n− 1

(x2

1 + x22 + · · ·+ x2

n

)≥ 2S.

Rusia, 2003

Solutie. Consideram poligonul convex drept un arbore cu n ≥ 3 varfuri.Justificam mai ıntai faptul ca orice arbore are un varf terminal (de grad 1), adica un

varf care are un singur vecin.Un arbore cu n varfuri are n− 1 muchii.Daca din fiecare varf ar porni cel putin doua muchii, atunci numarul minim de muchii

ce l-ar putea avea arborele este jumatate din 2 + 2 + · · ·+ 2︸ ︷︷ ︸n−ori

, deoarece fiecare muchie este

numarata de doua ori.Am obtine n− 1 ≥ n (fals).Stergand un varf terminal si muchia adiacenta acestuia, se obtine tot un arbore.Vom demonstra inegalitatea din enunt prin inductie.Pentru n = 3, inegalitatea devine:

x21 + x2

2 + x23 ≥√

2 (x1x2 + x1x3 + x2x3) ,

Page 237: Teme si probleme pentru concursurile studentesti de matematica ...

230

care se demonstreaza prin calcul direct.Presupunem ca inegalitatea este adevarata pentru orice arbore cu n varfuri si sa o

demonstram pentru un arbore cu n+ 1 varfuri.Consideram un varf terminal al grafului G si fie x numarul scris ın acest varf.Notam cu y ≡ xi, numarul scris ın varful adiacent varfului terminal considerat.Fie G∗ graful obtinut din G, fara acest varf si fara aceasta muchie.Notam cu S∗ suma tuturor numerelor de pe muchiile lui G∗. Trebuie sa demonstram

ca: √n(x2

1 + x22 + · · ·+ x2

)≥ 2S∗ + 2xy

Folosind ipoteza de inductie, este suficient sa demonstram ca:(√n−√n− 1

) (x2

1 + x22 + . . . x2

n

)+√nx2 ≥ 2xy.

Cum√n −√n− 1 = 1√

n+√n−1≥ 1

2√n

si x21 + x2

2 + · · · + x2n ≥ y2, este suficient sa

demonstram ca:y2

2√n

+√nx2 ≥ 2xy,

inegalitate care se deduce usor din inegalitatea mediilor.

Problema 13.46 Consideram un poligon convex cu n laturi si ın care trasam toate diago-nalele acestuia. Putem sa coloram toate varfurile, laturile si diagonalele acestuia cu catevaculori, astfel ıncat oricare doua laturi sau doua diagonale sau o latura si o diagonala cuo extremitate comuna sa aiba culori diferite si oricare latura sau diagonala are culoareadiferita de culoarea varfului ce le uneste.

Determinati numarul minim posibil de culori necesare.Italia, 2007

Solutie. Consideram poligonul ca un graf G complet si neorientat avand n varfuri.Numarul de culori este mai mare sau egal cu n.

Intr-adevar, din fiecare varf pornesc n− 1 muchii, colorate ın n− 1 culori diferite. Inplus, toate aceste n−1 culori diferite sunt diferite si de culoarea varfului din care pornesc.

Initial sa numerotam varfurile de la 1 la n. Apoi pe fiecare muchie dintre doua varfurinumerotate i si j scriem restul ımpartirii lui i + j la n (daca acest rest este zero, scriemn).

In final, din fiecare varf pornesc n−1 muchii de culori diferite. Atunci ın varful respectivscriem a n−a culoare (eventual unele varfuri vor fi colorate la fel, ceea ce nu este ıncontradictie cu enuntul problemei).

Asadar numarul minim de culori este n.

Problema 13.47 Intre cele 2000 de orase ale unei tari nu exista drumuri. Demonstratica ıntre orasele tarii pot fi construite drumuri bidirectionale astfel ıncat din doua orase saporneasca cate un drum, din alte doua orase cate doua drumuri, . . . , si din ultimile douaorase cate 1000 de drumuri.

Sankt-Petersburg, 2001

Solutia 1. Problema devine un caz particular daca demonstram afirmatia generala:Exista un graf cu 4n varfuri astfel ıncat gradul a doua varfuri sa fie 1, gradul altor

doua varfuri sa fie 2, . . . , gradul ultimilor doua varfuri sa fie 2n.Demonstram prin inductie dupa numarul de varfuri.

Page 238: Teme si probleme pentru concursurile studentesti de matematica ...

Combinatorica si grafuri 231

Cazul n = 1: Luam 4 varfuri A,B,C,D si unim A cu B, B cu C si C cu D. Astfel Asi D au gradul 1 iar B si C au gradul 2.

Presupunem acum ca afirmatia este adevarata pentru un graf cu 4n varfuri si o vomdemonstra pentru un graf cu 4n + 4 varfuri. Fie G un graf cu 4n varfuri care satisfaceproprietatea enuntata.

Notam cu X1, X2, . . . , X4n aceste varfuri. Presupunem ca X2k−1 si X2k au gradul k,pentru k = 1, 2n. Consideram alte patru varfuri A,B,C,D. Unim A cu B, C cu D si Bcu D. Apoi varful B ıl unim cu 2n varfuri Xi, unde i este impar, iar pe D ıl unim cu 2nvarfuri Xi ale lui G cu i numar par.

Fie G∗ graful astfel obtinut.Demonstram ca acesta satisface proprietatea enuntata.Intr-adevar el are doua varfuri de gradul 1 (A si C), doua varfuri de gradul 2 (X1 unit

cu B si X2, respectiv X2 unit cu D si X1), . . . , doua varfuri de gradul 2n+1, acestea fiindX4n−1 si X4n si doua varfuri de gradul 2n+ 2, acestea fiind B si D care sunt unite ıntreele, apoi cu cate 2n varfuri ale lui G si ın plus B este unit cu A, iar D este unit cu C.

Solutia 2. Impartim cele 2000 orase ın 1000 perechi. Unim orasele din aceeasi pereche.Numerotam perechile de la 1 la 1000.Al doilea oras din fiecare pereche i = 1, 1000 ıl unim cu primul oras din fiecare pereche

j, cu i < j ≤ 1000.Dupa efectuarea acestui procedeu constatam ca din primul oras din perechea k si din

al doilea oras din perechea 1001− k vor porni exact k drumuri.

Problema 13.48 Fiecare dintre cei 2012 deputati ai parlamentului unei tari i-a dat opalma unui alt deputat. Demonstrati ca se poate forma o comisie parlamentara din 671deputati care nu au primit nici o palma.

Moscova, 1994

Solutie. Consideram deputatii ca varfurile unui graf orientat G. Un arc ıntre varfurileA si B ınseamna ca deputatul A i-a tras o palma deputatului B.

Demonstram cazul general: In orice graf orientat G cu cel putin 3n− 1 varfuri existaun subgraf cu n varfuri, care nu contine niciun arc incident interior cu macar unul dintrecele n varfuri.

Demonstram prin inductie dupa n.Cazul n = 2 este evident.Presupunem afirmatia adevarata pentru un oarecare n si o demonstram pentru n+ 1.

Consideram ca avem cel putin 3n+ 2 varfuri.Cazul 1. Daca exista un varf X ın care nu ajunge nici un arc (ın problema aceasta

ınseamna ca deputatul X nu a primit nici o palma), atunci consideram graful G∗ fara

Page 239: Teme si probleme pentru concursurile studentesti de matematica ...

232

acest varf X si varful X∗ adiacent lui X (si fara muchiile care pornesc din X∗). G∗ are celputin 3n > 3n− 1 varfuri deci contine n varfuri neunite oricare doua printr-un arc.

Adaugand varful X obtinem n+ 1 varfuri neunite oricare doua.Cazul 2. Consideram ca ın orice varf ajunge cel putin un arc.Deoarece din fiecare varf porneste exact un arc, rezulta ca ın fiecare varf ajunge exact

un arc.Luam oricare varf X, varful A spre care porneste un arc din X si varful B spre care

porneste un arc din X.Conform inductiei, din varfurile ramase putem selecta n varfuri neunite oricare doua.

Atunci la aceste varfuri ıl adaugam pe X.Problema este ın cazul particular n = 671.

Page 240: Teme si probleme pentru concursurile studentesti de matematica ...

Capitolul 14

Aritmetica si teoria numerelor

Definitii si rezultate

Definitie. Fie a, b, n ∈ Z cu n 6= 0. Spunem ca a este congruent cu b modulo n, si

notam a ≡ b (mod n), daca a− b... n.

Observatie. Pentru a, b ∈ Z si n ∈ N∗, a ≡ b (mod n) daca si numai daca a = b ın Zn.

Mica teorema a lui Fermat. Daca p este un numar prim, iar a ∈ Z cu (a, p) = 1,atunci ap−1 ≡ 1 (mod p).

Lema chineza a resturilor. Fie n1, n2, . . . , nk ∈ Z∗ cu proprietatea ca oricare douasunt relativ prime si a1, a2, . . . , ak ∈ Z. Sistemul de congruente

x ≡ a1 (mod n1)x ≡ a2 (mod n2)............................x ≡ ak (mod nk)

admite solutii, si, data fiind o solutie x0, multimea solutiilor sistemului estex0 + λn1n2 · · ·nk |λ ∈ Z.

Definitie. Fie a, n ∈ Z cu n 6= 0. Spunem ca a este rest patratic modulo n daca existax ∈ Z cu proprietatea x2 ≡ a (mod n).

Definitie. Pentru p > 2 numar prim si a ∈ Z cu (a, p) = 1, definim simbolul lui

Legendre

(a

p

)astfel:

(a

p

)=

1, daca a este rest patratic modulo p−1, daca a nu este rest patratic modulo p.

233

Page 241: Teme si probleme pentru concursurile studentesti de matematica ...

234

Proprietati ale simbolului lui Legendre. Fie a, b ∈ Z si p ∈ N prim impar.

1)(ap

)≡ a

p−12 (mod p). (Euler)

2) Daca a ≡ b (mod p), atunci(ap

)=(bp

).

3)(abp

)=(ap

)(bp

).

4)(−1p

)= (−1)

p−12 .

5)(

2p

)= (−1)

p2−18 .

Legea de reciprocitate patratica (Gauss). Daca p, q ∈ N sunt numere prime,impare si distincte, atunci (

p

q

)·(q

p

)= (−1)

p−12· q−1

2 .

Observatie. Proprietatile 2) si 3) arata ca putem interpreta simbolul lui Legendre cafiind un morfism de grupuri de la Zp \ 0 la C∗.

Observatie. Daca p este numar prim impar, a ∈ Z si p | a, vom folosi uneori, pentru

o mai buna sistematizare a calculelor, conventia

(a

p

)= 0.

Definitie. Functia lui Legendre asociaza fiecarei perechi alcatuite dintr-un numarprim p si un numar natural n exponentul ep(n) la care apare p ın descompunerea ınfactori primi a lui n!.

Teorema. ep(n) =

[logp n]∑k=1

[n

pk

]=∑k≥1

[n

pk

].

Probleme

Problema 14.1 a) Aratati ca pentru n ∈ N suficient de mare putem partitiona un patratdat ın n patrate.b) Fie d ≥ 2. Aratati ca exista o constanta N(d) cu proprietatea ca pentru orice numarnatural n ≥ N(d) putem partitiona un cub d-dimensional dat ın n cuburi d-dimensionale.

IMC, 2000

Solutie. Incepem cu urmatoareaLema. Daca numerele naturale a si b sunt prime ıntre ele, atunci orice numar natural

n > ab− a− b se poate scrie sub forma n = ax+ by, cu x, y ∈ N.Demonstratie. Sa presupunem a ≥ b. Daca b = 1, afirmatia lemei este evidenta.Presupunem ın continuare ca b > 1. Fie n ≥ ab − a. Atunci numerele n, n − a, n −2a, . . . , n − (b − 1)a parcurg un sistem complet si independent de resturi modulo b, deciexista x ∈ 0, 1, . . . , b − 1 si y ∈ N astfel ıncat n = ax + by. Daca n = ab − a − k,k ∈ 1, 2, . . . , b−1, atunci alegem z ∈ 1, 2, . . . , b−1 pentru care z = ka−1 ın Zb si avem

az−k... b, deci exista y ∈ N pentru care az−k = by. De aici rezulta ca n = a(b−1−z)+by.

Revenind la solutia propriu-zisa, sa observam ca orice partitie a unui ,,d-cub” (d ≥ 2)ın n d-cuburi poate fi rafinata la o partitie a sa ın n + (ad − 1) d-cuburi pentru oricea ∈ N∗. Acest lucru se poate realiza pur si simplu prin alegerea unui d-cub din partitie

Page 242: Teme si probleme pentru concursurile studentesti de matematica ...

Aritmetica si teoria numerelor 235

si partitionarea acestuia ın ad d-cuburi. Conform lemei, pentru a demonstra afirmatiileproblemei este suficient sa gasim o pereche de numere naturale relativ prime de formaad − 1; 2d − 1 si (2d − 1)d − 1 reprezinta o astfel de pereche.

Problema 14.2 Fie n ∈ N∗. Aratati ca 2n−1 divide numarul∑0≤k<n

2

(n

2k + 1

)· 5k.

IMC, 2008

Solutie. Se stie ca numarul Fibonacci Fn este dat de formula

Fn =1√5

((1 +√

5

2

)n−

(1−√

5

2

)n).

Folosind formula binomului lui Newton, obtinem

Fn =1

2n−1

((n

1

)+

(n

3

)· 5 + · · ·+

(n

t

)· 5

t−12

), unde t = 2

[n− 1

2

]+ 1.

Obtinem 2n−1Fn =∑

0≤k<n2

(n

2k + 1

)· 5k si problema este rezolvata.

Problema 14.3 Consideram numerele D = d1d2 . . . d9, E = e1e2 . . . e9 si F = f1f2 . . . f9

scrise ın baza 10. Pentru orice i ∈ 0, 1, . . . 9, daca ınlocuim cifra di a lui D cu ei obtinemun numar divizibil cu 7. De asemenea, pentru orice i ∈ 0, 1, . . . 9, daca ınlocuim cifraei a lui E cu fi obtinem un numar divizibil cu 7. Aratati ca pentru orice i ∈ 0, 1, . . . 9

avem di − fi... 7.

Putnam, 1995

Solutie. Conform enuntului, pentru orice i ∈ 1, 2, . . . , 9 avem

(ei − di) · 109−i +D ≡ 0 (mod 7) si (14.1)

(fi − ei) · 109−i + E ≡ 0 (mod 7). (14.2)

Sumand relatiile din (14.1) pentru i ∈ 1, 2, . . . , 9, obtinem E − D + 9D ≡ 0 (mod 7),adica

E +D ≡ 0 (mod 7). (14.3)

Adunand pentru fiecare i ∈ 1, 2, . . . , 9 relatiile (14.1) si (14.2), obtinem (fi−di) ·109−i+E + D ≡ 0 (mod 7). De aici si din (14.3) obtinem, tinand cont de faptul ca (10, 7) = 1,

ca di − fi... 7.

Problema 14.4 Aratati ca nu exista patru puncte ın spatiul euclidian astfel ıncatdistanta dintre oricare doua sa fie numar impar.

Putnam, 1993

Page 243: Teme si probleme pentru concursurile studentesti de matematica ...

236

Solutia 1. Presupunem ca exista patru astfel de puncte. Pentru x, y ∈ R si n ∈ N∗,vom folosi notatia ,,x ≡ y (mod n)” cu sensul ,,x− y este un numar ıntreg divizibil cu n”.Alegem un sistem de coordonate ın care coordonatele celor patru puncte sunt (0, 0), (a, 0),(r, s) si (x, y), cu a ∈ 2N+ 1. Patratele numerelor impare fiind congruente cu 1 modulo 8,presupunerea facuta implica relatiile:

r2 + s2 ≡ 1 (mod 8)(r − a)2 + s2 ≡ 1 (mod 8)

x2 + y2 ≡ 1 (mod 8)(x− a)2 + y2 ≡ 1 (mod 8)

(x− r)2 + (y − s)2 ≡ 1 (mod 8).

(14.4)

Scazand primele doua dintre aceste relatii se obtine 2ar ≡ a2 (mod 8). Prin urmare, reste un numar rational al carui numitor este par si divide 2a. Se arata ın mod analog ca xare aceeasi proprietate. Inmultim toate coordonatele cu a, reducandu-ne astfel la cazul ıncare r si x au numitorul 2. Atunci, congruenta 2ar ≡ a2 (mod 8) este de numere ıntregi;din ea va rezulta r ≡ a

2 (mod 4). Scriind r = a2 + 4b, b ∈ Z, obtinem

r2 =a2

4+ 4ab+ 16b2 ≡ a2

4(mod 4);

de aici, folosind si prima relatie din (14.4), deducem ca

s2 ≡ 1− r2 ≡ 1− a2

4(mod 4).

In mod analog se arata ca x ≡ a2 (mod 4) si ca y2 ≡ 1− a2

4 (mod 4); din penultima relatiesi din r ≡ a

2 (mod 4) obtinem x− r ≡ a2 −

a2 ≡ 0 (mod 4), deci (x− r)2 ∈ 16Z. De aici si

din ultima relatie din (14.4) deducem ca (y − s)2 ≡ 1− (x− r)2 ≡ 1 (mod 8). Obtinem

(y + s)2 ≡ 2y2 + 2s2 − (y − s)2 ≡ 2(1− a2

4) + 2(1− a2

4)− 1 ≡

≡ 3− a2 ≡ 2 (mod 4).Inmultind membru cu membru aceasta congruenta de numere ıntregi cu (y − s)2 ≡ 1(mod 8), obtinem (y2 − s2)2 ≡ 2 (mod 4). Daca y2 − s2 = u

v , u, v ∈ Z, v 6= 0, rezulta caexista k ∈ Z astfel ıncat u2−2v2 = 4kv2. De aici, u = 2u1, u1 ∈ Z; obtinem 2u2

1−v2 = 2kv2,deci v = 2v1, v1 ∈ Z, apoi u2

1 − 2v21 = 4kv2

1. Continuand inductiv, obtinem pentru fiecarej ∈ N numerele uj , vj ∈ Z cu proprietatea u2

j − 2v2j = 4kv2

j . Din definitia acestor numere,

avem vj = 2vj+1 pentru toti j ∈ N. De aici, 2j | v pentru orice j ∈ N, deci v = 0,contradictie.

Solutia 2. Sa presupunem ca exista patru puncte O, A, B, C ın plan astfel ıncatnumerele a = OA, b = OB, c = OC, x = BC, y = CA, z = AB sa fie ıntregi siimpare. Notam cu α, respectiv β masurile unghiurilor orientate AOB si BOC; masuraunghiului orientat AOC este deci α+β. Cum cos(α+β) = cosα cosβ− sinα sinβ, rezulta(cos(α+ β)− cosα cosβ)2 = (1− cos2 α)(1− cos2 β), de unde

1− cos2 α− cos2 β − cos2(α+ β) + 2 cosα cosβ cos(α+ β) = 0. (14.5)

Conform teoremei cosinusului, avem cosα =a2 + b2 − z2

2ab, cosβ =

b2 + c2 − x2

2bcsi cos(α+

β) =c2 + a2 − y2

2ca. Inlocuind ın relatia (14.5) si ınmultind cu 4a2b2c2, obtinem

4a2b2c2 − c2(a2 + b2 − z2)2 − a2(b2 + c2 − x2)2 − b2(c2 + a2 − y2)2+

Page 244: Teme si probleme pentru concursurile studentesti de matematica ...

Aritmetica si teoria numerelor 237

+(a2 + b2 − z2)(b2 + c2 − x2)(c2 + a2 − y2) = 0. De aici rezulta 4 − 1 − 1 − 1 + 1 ≡ 0(mod 4), contradictie.

Solutia 3. Presupunem ca exista patru puncte cu proprietatea din enunt; consideramun sistem de coordonate cu originea ın unul dintre ele, si notam cu −→v1 , −→v2 si −→v3 vectoriicare unesc originea cu celelalte trei puncte; volumul V al paralelipipedului determinat deacesti vectori este nul, deoarece ei sunt coplanari. Deci,

0 = V 2 =

∣∣∣∣∣∣−→v1 · −→v1

−→v1 · −→v2−→v1 · −→v3−→v2 · −→v1

−→v2 · −→v2−→v2 · −→v3−→v3 · −→v1

−→v3 · −→v2−→v3 · −→v3

∣∣∣∣∣∣ .Daca punem a = |−→v1 |, b = |−→v2 |, c = |−→v3 |, x = |−→v2 − −→v3 |, y = |−→v3 − −→v1 | si z = |−→v1 − −→v2 | siavem ın vedere relatia

2−→vi · −→vj = |−→vi |2 + |−→vj |2 − |−→vi −−→vj |2, (14.6)

obtinem

8V 2 =

∣∣∣∣∣∣2a2 a2 + b2 − z2 a2 + c2 − y2

a2 + b2 − z2 2b2 b2 + c2 − x2

a2 + c2 − y2 b2 + c2 − x2 2c2

∣∣∣∣∣∣ ,de unde

0 = 8V 2 ≡

∣∣∣∣∣∣2 1 11 2 11 1 2

∣∣∣∣∣∣ ≡ 4 (mod 8),

contradictie.Solutia 4. Presupunem ca exista patru puncte ca ın enunt si definim −→v1 , −→v2 si −→v3 ca

ın solutia 3. Este clar ca −→vi ·−→vi ≡ 1 (mod 8); conform (14.6), avem si 2−→vi ·−→vj ≡ 1 (mod 8)pentru i 6= j.Cum printre punctele considerate nu pot exista trei coliniare, rezulta ca vectorii −→v1 , −→v2

si −→v3 sunt doi cate doi necoliniari. Exista prin urmare scalari r, s ∈ R cu proprietatea ca−→v3 = r−→v1 + s−→v2 . Atunci,

2−→v1 · −→v3 = 2r−→v1 · −→v1 + 2s−→v1 · −→v2

2−→v2 · −→v3 = 2r−→v2 · −→v1 + 2s−→v2 · −→v2

2−→v3 · −→v3 = 2r−→v3 · −→v1 + 2s−→v3 · −→v2 .(14.7)

Cum −→v1 si −→v2 sunt necoliniari, avem∣∣∣∣ −→v1 · −→v1−→v1 · −→v2−→v2 · −→v1−→v2 · −→v2

∣∣∣∣ 6= 0.

Prin urmare, primele doua ecuatii din (14.7), privite ca ecuatii ın r si s, admit solutie unica

ın numere rationale. Fie r =R

T, s =

S

Taceasta solutie, cu R,S, T ∈ Z si (R,S, T ) = 1.

Inmultind cu T ecuatiile din (14.7), obtinem T ≡ 2R + S (mod 8), T ≡ R + 2S (mod 8)si 2T ≡ R + S (mod 8). Adunand primele doua dintre aceste congruente si scazand-ope a treia, gasim ca T este par. Utilizand aceasta informatie si primele doua congruente,deducem ca R si S sunt si ele pare, contradictie.

Problema 14.5 Fie x, y, z ∈ Z cu proprietatea ca S = x4 + y4 + z4 se divide prin 29.Aratati ca S se divide prin 294.

IMC, 2007

Page 245: Teme si probleme pentru concursurile studentesti de matematica ...

238

Solutie. Vom arata ca 29 divide x, y si z, de unde rezulta imediat concluzia problemei.Presupunem ca 29 nu divide x, y si z; pentru a fixa ideile, sa consideram ca 29 - x. Atunci,x este inversabil ın corpul Z29; fie w inversul sau. Atunci, numarul (xw)4 + (yw)4 + (zw)4

este la randul sau divizibil cu 29, deci

(yw)4 ≡ −1− (zw)4 (mod 29). (14.8)

Pe de alta parte, exista doar opt puteri 4 modulo 29:

0 ≡ 04 (mod 29),1 ≡ 14 ≡ 124 ≡ 174 ≡ 284 (mod 29),7 ≡ 84 ≡ 94 ≡ 204 ≡ 214 (mod 29),

16 ≡ 24 ≡ 54 ≡ 244 ≡ 274 (mod 29),20 ≡ 64 ≡ 144 ≡ 154 ≡ 234 (mod 29),23 ≡ 34 ≡ 74 ≡ 224 ≡ 264 (mod 29),24 ≡ 44 ≡ 104 ≡ 194 ≡ 254 (mod 29),25 ≡ 114 ≡ 134 ≡ 164 ≡ 184 (mod 29).

De aici rezulta ca (yw)4 ∈ 0, 1, 7, 16, 20, 23, 24, 25, ın timp ce−1 − (zw)4 ∈ 28, 27, 21, 12, 8, 5, 4, 3 (toate clasele fiind modulo 29), ceea ce contrazice(14.8).Ramane asadar ca 29 divide x, y si z, ceea ce ıncheie demonstratia.

Problema 14.6 Fie p > 3 un numar prim si n =22p − 1

3. Aratati ca n divide 2n − 2.

Vojtech Jarnik, 2002

Solutie. Cum n = 4p−1 + 4p−2 + · · · + 1, putem scrie n = 10101 . . . 101 (p cifre 1) ınbinar. Obtinem prin urmare reprezentarea binara

3n = 1111 . . . 111︸ ︷︷ ︸2p cifre

. (14.9)

Pe de alta parte,

2n − 2 = 1111 . . . 111︸ ︷︷ ︸n−1 cifre

0. (14.10)

Conform micii teoreme a lui Fermat, 2p−1 ≡ 1 (mod p), deci p|22p−2 − 1. Rezulta ca p

divide22p − 4

3=

22p − 1

3− 1, adica p|n− 1; cum n este impar, avem si 2p|n− 1. Aceasta

relatie, ımpreuna cu (14.9) si (14.10), arata ca 3n|2n − 2.

Problema 14.7 Aratati ca exista o infinitate de perechi (m,n) de numere naturale primeıntre ele pentru care ecuatia ın x

(x+m)3 = nx

are trei radacini ıntregi distincte.

IMC, 2006

Page 246: Teme si probleme pentru concursurile studentesti de matematica ...

Aritmetica si teoria numerelor 239

Solutie. Notand y = x+m, ecuatia devine

y3 − ny +mn = 0.

Notam doua dintre radacinile ei cu u si v; conform relatiilor ıntre radacini si coeficienti,cea de-a treia radacina va fi w = −(u+v), si sunt verificate si relatiile uv+uw+vw = −n(de unde u2 + uv + v2 = n) si −uv(u + v) = uvw = −mn. Prin urmare, uv(u + v) estedivizibil prin u2 +uv+v2. Punand u = kp, v = kq, avem u2 +uv+v2 = k2(p2 +pq+ q2) siuv(u+ v) = k3pq(p+ q). Alegand p, q prime ıntre ele si punand k = p2 + pq+ q2, conditiau2 + uv + v2|uv(u+ v) este ındeplinita. Cu aceste constatari, revenim la notatiile initialesi punem pentru fiecare pereche p, q de numere naturale prime ıntre ele m = pq(p + q) sin = (p2 + pq + q2)3. Obtinem astfel o infinitate de ecuatii (x + m)3 = nx, fiecare avandtrei radacini ıntregi distincte: p3, q3 si −(p+ q)3.

Problema 14.8 Demonstrati ca ecuatia

an+1 − (a+ 1)n = 2001 (14.11)

are solutie unica ın numere naturale nenule.

Putnam, 2001

Solutie. Fie a, n ∈ N∗ astfel ıncat an+1 − (a+ 1)n = 2001. Cum a divide an+1 − [(a+1)n − 1], rezulta ca a | 2002 = 2 · 7 · 11 · 13.

Cum 2001... 3, (14.11) implica a ≡ 1 (mod 3). De aici, an+1 ≡ 1 (mod 3), deci (a+1)n ≡ 1

(mod 3). Prin urmare, n este par.Daca a ar fi par, atunci an+1 − (a + 1)n ≡ −(a + 1)n (mod 4). Cum n este par, avem si−(a+ 1)n ≡ −1 (mod 4). Folosind (14.11), obtinem

1 ≡ 2001 ≡ an+1 − (a+ 1)n ≡ −(a+ 1)n ≡ −1 (mod 4),

contradictie. Ramane ca a este impar, deci a | 1001 = 7 · 11 · 13. Cum a ≡ 1 (mod 3),rezulta ca a | 7 · 13. Pe de alta parte, n ∈ N∗ fiind par, avem n ≥ 2, de unde a ≡ an+1 −(a+ 1)n = 2001 ≡ 1 (mod 4). Prin urmare, a ∈ 1, 13. Cum pentru niciun n ∈ N nu areloc 1 − 2n = 2001, rezulta a = 13. Este imediat ca a = 13, n = 2 verifica (14.11). Pe dealta parte, daca n > 2 este par, se obtine

13n+1 − (13 + 1)n ≡ 13n+1 ≡ 13 6≡ 1 ≡ 2001 (mod 8),

deci singura solutie a ecuatiei (14.11) este a = 13, n = 2.

Problema 14.9 Pentru n ∈ N∗ notam cu d(n) numarul divizorilor sai naturali. Aratatica sirul (d(n2 + 1))n≥n0 nu este strict monoton pentru niciun n0 ∈ N.

Vojtech Jarnik, 2003

Solutie. Sa observam pentru ınceput ca daca n este par are loc inegalitatea

d(n2 + 1) < n. (14.12)

Intr-adevar, ın aceasta situatie putem grupa divizorii lui n2 +1 ın perechi(d, n

2+1d

), unde

d < n este impar, ceea ce conduce (ıntrucat n2 + 1 nu este patrat perfect) la inegalitateadorita.Sa presupunem acum ca exista n0 ∈ N astfel ıncat (d(n2 + 1))n≥n0 este strict monoton.Cum d(n2+1) este par, obtinem d((n+1)2+1) ≥ d(n2+1)+2 si, inductiv, d((n+k)2+1) ≥d(n2 +1)+2k pentru orice n ≥ n0 si k ∈ N∗. Obtinem d(4n2

0 +1) ≥ d(n20 +1)+2n0 ≥ 2n0,

ceea ce contrazice (14.12).

Page 247: Teme si probleme pentru concursurile studentesti de matematica ...

240

Problema 14.10 Aratati ca orice numar natural nenul se poate scrie ca suma de unulsau mai multe numere de forma 2r3s, cu r, s ∈ N, astfel ıncat niciunul dintre termenii uneiastfel de sume sa nu fie divizor al altuia.

Putnam, 2005

Solutie. Demonstram afirmatia problemei prin inductie dupa n.Numarul 1 se reprezinta ca 2030.Fie n > 1. Presupunem ca toate numerele naturale nenule mai mici sau egale cu n − 1admit reprezentari de tipul din enunt. Daca n este par, atunci obtinem o reprezentare alui n ınmultind cu 2 o reprezentare a lui n

2 . Daca n este impar, notam m = [log3 n]. Dacan = 3m, am terminat. Altfel, consideram o reprezentare s1+s2+· · ·+sk de tipul din enunt anumarului n−3m

2 . Atunci, n = 3m+2s1+2s2+· · ·+2sk. Este clar ca niciunul dintre termenii

2si nu divide alt termen de acest tip sau pe 3m. In plus, cum 2si ≤ n− 3m < 3m+1 − 3m,obtinem si < 3m, deci 3m - 2si. Prin urmare, n admite reprezentari de tipul cerut, ceea ceıncheie pasul de inductie si demonstratia.

Problema 14.11 Pentru n ∈ N∗ notam cu σ(n) suma divizorilor naturali ai lui n. Vomspune ca numarul n este ,,straniu” daca σ(n) ≥ 2n si nu exista nicio reprezentare de tipuln = d1 + d2 + · · ·+ dr, unde r > 1, iar d1, d2, . . . , dr sunt divizori distincti ai lui n.Aratati ca exista o infinitate de numere stranii.

Vojtech Jarnik, 2010

Solutie. Fie n un numar straniu si p > σ(n) un numar prim. Presupunem ca pn nueste straniu. Daca 1 = d1, d2, . . . , dk = n sunt divizorii naturali ai lui n, atunci cei ai lui pnsunt d1, d2, . . . , dk, pd1, pd2, . . . , pdk. Acestia din urma sunt si distincti, ıntrucat (p, n) = 1.Daca am avea pn = di1 + · · ·+ dir + p(dj1 + · · ·+ djs), ik, jl ∈ 1, . . . , k, ar rezulta

di1 + · · ·+ dir = p(n− dj1 − · · · − djs). (14.13)

Dar n 6∈ dj1 , . . . , djs, deoarece reprezentarile numerelor stranii nu pot consta ıntr-unsingur sumand. In plus, n fiind straniu, avem si n−dj1−· · ·−djs 6= 0. Rezulta ca membruldrept din (14.13) este multiplu nenul de p; la fel este prin urmare si di1 + · · ·+ dir . Acestlucru este ınsa ın contradictie cu p > σ(n). Ramane deci ca pn este straniu. Prin urmare,daca am avea un numar straniu n, punand n1 = n si presupunand construit nk, am luap > σ(nk) si am obtine ca nk+1 = pnk este straniu. De aici ar rezulta existenta unui sirstrict crescator de numere stranii, ceea ce ar rezolva problema.Mai ramane de gasit un exemplu de numar straniu. Sa observam ca, daca consideram unnumar n cu σ(n) = 2n+ 4 si care nu este divizibil nici prin 3, nici prin 4, atunci nu vomputea sa-l reprezentam pe 4 (deci, nici pe n) ca suma de divizori distincti ai lui n. n = 2pqare proprietatile de mai sus (unde p si q sunt numere prime impare distincte) daca sinumai daca 3(p+ 1)(q+ 1) = σ(2pq) = 4pq+ 4; ne este deci suficient ca (p− 3)(q− 3) = 8.Aceasta egalitate este verificata de p = 5 si q = 7, valori care conduc la numarul straniu70.

Problema 14.12 Determinati |S|, unde

S = x ∈ N∗| x < 102006 si x2 − x... 102006

IMC, 2006

Page 248: Teme si probleme pentru concursurile studentesti de matematica ...

Aritmetica si teoria numerelor 241

Solutia 1. Pentru k ∈ N∗ notam Sk = x ∈ N∗| x < 10k si x2−x... 10k si s(k) = |Sk|.

Fie x ∈ Sk+1 si akak−1 . . . a0 scrierea sa zecimala. Este imediat ca ak−1ak−2 . . . a0 ∈ Sk.Fixam acum y = ak−1ak−2 . . . a0 ∈ Sk, si consideram x = akak−1 . . . a0, ak ∈ 0, 1, . . . , 9.Avem x2 − x = (ak · 10k + y)2 − (ak · 10k + y) = (y2 − y) + ak · 10k(2y − 1) + a2

k+1 · 102k.

Cum exista z ∈ Z astfel ıncat y2 − y = 10kz, rezulta ca x2 − x este divizibil prin 10k+1

daca si numai dacaz + ak(2y − 1) ≡ 0 (mod 10). (14.14)

Cum y 6≡ 3 sau 8 (mod 10), congruenta (14.14) are solutie unica, ceea ce arata ca fiecarey ∈ Sk provine din exact un element x ∈ Sk+1 prin ınlaturarea cifrei (eventual, nule)initiale. Prin urmare, pentru orice k ∈ N∗ exista o corespondenta bijectiva ıntre Sk siSk+1. Rezulta ca s(2006) = s(1). Cum ınsa S1 = 1, 5, 6, avem |S| = s(2006) = s(1) = 3.

Solutia 2. Fie x ∈ S. Cum x2 − x = x(x − 1), iar numerele x si x − 1 sunt primeıntre ele, unul dintre ele trebuie sa fie divizibil cu 22006 si unul dintre ele (eventual, acelasi)trebuie sa fie divizibil cu 52006. Prin urmare, x trebuie sa satisfaca conditiile x ≡ 0 sau1 (mod 22006) si x ≡ 0 sau 1 (mod 52006). Conform lemei chineze a resturilor, fiecare dincele 4 cazuri conduce la o unica solutie din multimea 0, 1, . . . , 102006 − 1. Aceste solutiisunt distincte doua cate doua, deoarece dau resturi diferite modulo 22006 sau 52006. Unadintre solutii este 0, care nu se ıncadreaza ın conditiile din enunt. Prin urmare, |S| = 3.

Problema 14.13 Pentru a ∈ N, notam na = 101a− 100 · 2a. Aratati ca pentru a, b, c, d ∈0, 1, . . . , 99, na + nb ≡ nc + nd (mod 10100) implica a, b = c, d.

Putnam, 1994

Solutie. Cum 220 ≡ 95 (mod 101), iar 250 ≡ 100 (mod 101), rezulta ca ordinul lui 2ın grupul Z101 \ 0 este 100 (deci, 2 este radacina primitiva modulo 101).

Lema. Daca a, b ∈ N sunt astfel ıncat 2a ≡ 2b (mod 101), atunci a ≡ b (mod 100).Demonstratie. Sa presupunem, pentru a fixa ideile, ca a ≥ b. Daca 101 | 2a−2b = 2b(2a−b−1), atunci 2a−b ≡ 1 (mod 101), deci, conform consideratiilor anterioare lemei, a − b ≡ 0(mod 100).

Revenind la solutia propriu-zisa, observam ca, ın conformitate cu lema chineza a res-turilor, relatia na + nb ≡ nc + nd (mod 10100) este echivalenta cu sistemul

a+ b ≡ c+ d (mod 100)2a + 2b ≡ 2c + 2d (mod 101).

(14.15)

Cum ordinul lui 2 ın Z101 este 100, din prima relatie din (14.15) obtinem 2a+b ≡ 2c+d

(mod 101), deci2a2b ≡ 2c2d (mod 101). (14.16)

Din (14.16) si a doua relatie din (14.15) obtinem 2a(2c + 2d − 2a) ≡ 2c2d (mod 101), deunde (2a−2c)(2a−2d) ≡ 0 (mod 101). Rezulta 2a ≡ 2c (mod 101) sau 2a ≡ 2d (mod 101).Aplicand lema, obtinem ca a este congruent cu c sau cu d modulo 100; din (14.15) deducemca b este congruent modulo 100 cu cealalta valoare din c, d. Cum a, b, c, d ∈ 0, 1, . . . , 99,congruentele obtinute sunt de fapt egalitati.

Problema 14.14 Fie a, b ∈ Z si n ∈ N∗ cu proprietatea ca multimea Z\axn+byn | x, y ∈Z este finita. Aratati ca n = 1.

IMC, 2010

Page 249: Teme si probleme pentru concursurile studentesti de matematica ...

242

Solutie. Presupunem ca n > 1. Observam ca a si b trebuie sa fie relativ prime, deoareceın caz contrar numerele care nu sunt divizibile prin (a, b) nu pot fi reprezentate sub formaaxn + byn. Remarcam si faptul ca putem ınlocui n cu orice divizor prim p al sau.Daca p = 2, atunci expresia ax2 + by2 nu poate avea orice rest modulo 8 (daca b este par,atunci ax2 are cel mult trei resturi posibile(mod 8), iar by2 cel mult doua. Deci, ax2 + by2

poate avea cel mult sase resturi modulo 8. Daca a e par, procedam analog, iar daca abeste impar, atunci ax2 + by2 ≡ ±(x2 ± y2) (mod 4); ±(x2 + y2) nu poate da restul ±3modulo 4, iar ±(x2− y2) nu poate da restul ±2 modulo 4). Acest caz duce prin urmare lacontradictie.Daca p ≥ 3, cum 0p, 1p, . . . , (p − 1)p dau resturi diferite modulo p, iar xp ≡ (x + kp)p

(mod p2), rezulta ca puterile p de numere ıntregi dau exact p resturi modulo p2. Prinurmare, numerele de forma axp + byp pot da cel mult p2 resturi modulo p2. Daca ele nudau restul k modulo p2, atunci niciun numar din k + p2Z nu e reprezentabil sub formadin enunt, contradictie. Daca axp + byp da toate resturile modulo p2, atunci p2|axp + byp

daca si numai daca p|x si p|y. Rezulta ca p2|axp + byp daca si numai daca pp|axp + byp.De aici deducem ca niciun numar din p2 + p3Z nu este reprezentabil sub forma axp + byp,contradictie.

Problema 14.15 Aratati ca, date fiind numerele a, b, c ∈ Z, exista n ∈ Z pentru care√n3 + an2 + bn+ c 6∈ Z.

Putnam, 1998

Solutia 1. Presupunem ca P (n) = n3 +an2 + bn+ c este patrat perfect pentru pentrun ∈ 1, 2, 3, 4. Cum P (2) si P (4) sunt patrate perfecte de aceeasi paritate, diferenta lor,adica 56 + 12a + 2b, trebuie sa fie multiplu de 4. Prin urmare, b trebuie sa fie par. Pede alta parte, si P (1) si P (3) sunt patrate perfecte de aceeasi paritate, deci diferenta lor,adica 26 + 8a + 2b, trebuie sa fie si ea multiplu de 4. De aici rezulta ca b trebuie sa fieimpar, contradictie.

Solutia 2. Daca 4b− a2 = 0 si c = 0, atunci n3 + an2 + bn + c = n(n + a2 )2, care nu

este patrat perfect pentru niciun n 6= a2 care nu este patrat perfect.

Daca 4b− a2 si c nu sunt ambele 0, luam n = 4m2. Atunci,

n3 + an2 + bn+ c = (8m3 + am)2 + (4b− a2)m2 + c.

Daca 4b − a2 > 0 sau 4b − a2 = 0 si c > 0, atunci pentru m suficient de mare vom avea(4b − a2)m2 + c > 0 si (4b − a2)m2 + c < 2(8m3 + am) − 1, deci n3 + an2 + bn + c seafla ıntre (8m3 + am)2 si (8m3 + am + 1)2, nefiind prin urmare patrat perfect. Daca4b− a2 < 0 sau 4b− a2 = 0 si c < 0, se arata ın mod analog ca n3 + an2 + bn+ c se aflaıntre (8m3 + am− 1)2 si (8m3 + am)2, nefiind prin urmare patrat perfect.

In problema 14.16 prezentam o generalizare a afirmatiei problemei 14.15:

Problema 14.16 Daca polinomul P ∈ Z[X] are proprietatea ca P (n) este patrat perfectpentru orice n ∈ Z, atunci P este patratul unui polinom din Z[X].

Solutie. Presupunem ca exista polinoame P care au toate valorile P (n), cu n ∈ Z,patrate perfecte, dar nu sunt ele ınsele patrate de polinoame din Z[X]. Este suficientsa consideram polinoame care nu se divid prin patratul niciunui polinom din Z[X]; pre-supunem deci ca P are aceasta proprietate. Atunci, discriminantul D al lui P este nenul.

Page 250: Teme si probleme pentru concursurile studentesti de matematica ...

Aritmetica si teoria numerelor 243

Fie m ∈ Z pentru care P (m) 6= 0; pentru fiecare numar prim pi care divide D, notam

ei = maxi ∈ N | P (m)... peii . Cum P este neconstant, P (m+λ

∏pi|D

pei+1i ) ia valori oricat

de mari pe masura ce creste λ ∈ Z. In plus, cum

P (m)− P (m+ λ∏pi|D

pei+1i )

...∏pi|D

pei+1i ,

niciuna dintre aceste valori nu este divizibila cu vreun factor de tipul pei+1i . In consecinta,

exista numere ıntregi n oricat de mari pentru care P (n) are macar un factor prim p - D.Sa consideram o astfel de situatie. Daca p2 - P (n), atunci P (n) nu este patrat perfect,contradictie. Daca p2 | P (n), atunci P (n + p) ≡ P (n) + pP ′(n) ≡ pP ′(n) (mod p2), iar

p - P ′(n), deoarece p - D. De aici, P (n + p)... p, dar P (n + p)

... p2, de unde P (n + p) nueste patrat perfect, contradictie.

Observatie. Demonstratia arata ceva mai mult decat s-a afirmat ın enunt, si anume:Daca P ∈ Z[X] nu este patratul unui polinom din Z[X], atunci exista numere n ∈ Z oricatde mari pentru care P (n) nu este patrat perfect.

Problema 14.17 Demonstrati ca exista o infinitate de perechi ordonate (a, b) de numereıntregi cu proprietatea ca pentru orice t ∈ N numarul at+b este triunghiular daca si numaidaca t este triunghiular.

Putnam, 1988

Solutia 1. Reamintim ca numerele triunghiulare sunt cele de tipul tn = n(n+1)2 , n ∈ N.

Se vede usor ca t3n+1 = 9tn + 1, iar t3n ≡ t3n+2 ≡ 0 (mod 3) pentru orice n ∈ N. Prinurmare, numarul natural t este triunghiular daca si numai daca 9t+1 este triunghiular (∗).Consideram functia f : R → R, f(x) = 9x + 1. Notam fk = f f . . . f︸ ︷︷ ︸

k factori

; fie ak, bk ∈ N

pentru care fk(x) = akx + bk. Cum ak = 9k, aceste perechi sunt distincte. Aplicandinductiv (∗), deducem ca toate perechile (ak, bk) au proprietatea ceruta.

Solutia 2. Daca t = n(n+1)2 , atunci 8t + 1 = (2n + 1)2. Reciproc, daca t ∈ N are

proprietatea ca 8t + 1 este patrat perfect, acest patrat este al unui numar impar, fie el2n + 1. Rezulta ca t = n(n+1)

2 . Prin urmare, t ∈ N este triunghiular daca si numai daca8t+ 1 este patrat perfect.Fie un numar natural impar k. Avem k2 ≡ 1 (mod 8); de aici deducem ca, pentru t ∈N, t este triunghiular ⇔ 8t + 1 este patrat perfect ⇔ k2(8t + 1) este patrat perfect ⇔8(k2t+ k2−1

8

)+ 1 este patrat perfect ⇔

(k2t+ k2−1

8

)este triunghiular. Prin urmare,

orice pereche (a, b) =(k2, k

2−18

), k ∈ 2N + 1, are proprietatea ceruta.

Observatie. Vom numi ad-hoc o pereche (a, b) de numere ıntregi triunghiulara dacaare proprietatea ca, pentru orice numar natural t, t este triunghiular daca si numai dacaat + b este triunghiular. Solutia 2 arata ca pentru orice numar ıntreg impar k perechea(k2, k

2−18

)este triunghiulara. Cu alte cuvinte, perechile triunghiulare sunt de forma ((2m+

1)2, tm), m ∈ N.Vom arata ca, reciproc, orice pereche triunghiulara este de aceasta forma: Fie (a, b) opereche triunghiulara. Atunci, pentru orice n ∈ N∗, atn + b este triunghiular, deci 8(atn +b) + 1 = 4an2 + 4an+ (8b+ 1) este patrat perfect. Conform observatiei din finalul solutieiproblemei 14.16, exista l ∈ Z[X] cu proprietatea

Page 251: Teme si probleme pentru concursurile studentesti de matematica ...

244

4an2 + 4an+ (8b+ 1) = l(n)2.

Evident, l este de gradul 1; identitatea anterioara arata ca l = 2√ax+√a. Notand k = l(0),

obtinem a = k2 = 8b+ 1, deci b = k2−18 . Pentru ca b sa fie ıntreg, este necesar sa avem k

impar.

Problema 14.18 Definim sirul (xn)n≥1 astfel: xn = n pentru n ∈ 1, 2, . . . , 2006 sixn+1 = xn + xn−2005 pentru n ≥ 2006. Aratati ca sirul (xn)n contine 2005 termeni con-secutivi divizibili cu 2006.

Putnam, 2006

Solutie. Incepem prin a observa ca, daca un sir de numere ıntregi (xn)n≥1 satisface orelatie de recurenta de tipul

xn = f(xn−1, xn−2, . . . , xn−k) pentru orice n > k (14.17)

(unde k ∈ N∗ si f ∈ Z[X1, X2, . . . , Xn] sunt fixate), atunci sirul (xn)n este periodic moduloorice N ∈ N∗ de la un rang ıncolo. Acest lucru se ıntampla deoarece, fixand N ∈ N∗ sinotand cu HN numarul k-uplurilor ordonate de elemente din ZN , printre sistemele Σi =(xi+1, xi+2, . . . , xi+k), i ∈ 0, 1, . . . ,HN (toate clasele sunt modulo N) vor exista, conformprincipiului cutiei, cel putin doua egale. Daca acestea sunt Σi si Σj , sa presupunem, pentrua fixa ideile, ca i < j. Atunci,

xj+k+1 = f(xj+k, xj+k−1, . . . , xj+1) =

= f(xi+k, xi+k−1, . . . , xi+1) = xi+k+1.

Inductiv, se arata ca xr+j−i = xr pentru orice r ≥ i+ 1.Mai observam ca, daca relatia de recurenta (14.17) poate fi rescrisa sub forma xn−k =g(xn−k+1, xn−k+2, . . . , xn), g ∈ Z[X1, X2, . . . , Xn], atunci putem extinde sirul initial la. . . , x−2, x−1, x0, x1, x2, . . ., acest ,,sir” fiind, cu argumente similare celor de mai sus, peri-odic modulo orice N ∈ N∗.Cum relatia de recurenta din enunt se poate rescrie

xn−2005 = xn+1 − xn,

sirul dat se prelungeste la . . . , x−2, x−1, x0, x1, x2, . . ., care este periodic modulo 2006. Seconstata ınsa cu usurinta ca

x1 = x0 = · · · = x−2004 = 1 si

x−2005 = x−2006 = · · · = x−4009 = 0.

De aici si din periodicitatea modulo 2006 a ,,sirului” . . . , x−2, x−1, x0, x1, x2, . . . rezultaafirmatia problemei.

Problema 14.19 Fie a ∈ Z. Aratati ca pentru orice numar prim p polinomul X4 + a2 ∈Zp[X] este reductibil.

Solutie. Fie p ∈ N un numar prim. Notam f = X4 + a2 ∈ Zp[X].Daca p | a, atunci f = X4, deci f este reductibil peste Zp.Daca p - a, iar p = 2, atunci f = (X + 1)4, deci este reductibil.

Page 252: Teme si probleme pentru concursurile studentesti de matematica ...

Aritmetica si teoria numerelor 245

Daca p - a, iar p > 2, pot aparea cazurile:

1.

(−1

p

)= 1. Atunci, exista b ∈ Zp cu b2 = −1. Rezulta ca f = (X2 + ab)(X2 − ab), deci

f este reductibil peste Zp.

2.

(2a

p

)= 1. Atunci, exista c ∈ Zp cu c2 = 2a. Rezulta f = (X2 + a)2 − 2aX2 =

(X2 − cX + a)(X2 + cX + a), deci f este reductibil peste Zp.

3.

(−1

p

)=

(2a

p

)= −1. Atunci,

(−2a

p

)=

(−1

p

)(2a

p

)= 1. Exista prin urmare d ∈ Zp

cu d2 = −2a. Atunci, f = (X2 − a)2 + 2aX2 = (X2 − dX − a)(X2 + dX − a), deci f estereductibil peste Zp.

Observatie. Se constata, aplicand de pilda criteriul lui Eisenstein lui f(X + 1), capolinomul f = X4 + 1 ∈ Z[X] este ireductibil. Tinand cont si de afirmatia problemei,concluzionam ca exista polinoame ireductibile f ∈ Z[X] care sunt reductibile ın Zp[X]pentru orice numar prim p.

Problema 14.20 Fie p un numar prim impar. Cate elemente are multimea

x2 | x ∈ Zp ∩ y2 + 1 | y ∈ Zp ?

Putnam, 1991

Solutia 1. Notam cu S multimea solutiilor ecuatiei x2 = y2 + 1 (S ⊂ Zp × Zp).Schimbarea liniara de coordonate (u, v) = (x + y, x − y) a lui Zp × Zp este inversabila,

deoarece

∣∣∣∣ 1 11 −1

∣∣∣∣ = −2 6≡ 0 (mod p). Prin urmare, |S| este egal cu numarul solutiilor

ecuatiei uv = 1 peste Zp, adica p− 1.Problema cere numarul de elemente al imaginii functiei φ : S → Zp, φ(x, y) = x2. Dacaz = x2 pentru o pereche (x, y) ∈ S, atunci φ−1(z) = (±x,±y). Deci, φ−1(z) are patruelemente daca z 6∈ 0, 1, doua elemente daca z = 1, si, numai ın eventualitatea ın care−1 este patrat ın Zp, doua elemente daca z = 0. In consecinta, |S| = 4|φ(S)| − 2 − 2c,unde c este 1 sau 0 dupa cum −1 este sau nu patrat ın Zp. De aici,

|φ(S)| = p+ 1 + 2c

4. (14.18)

Cum |φ(S)| ∈ Z, rezulta ca avem c = 1 daca p ≡ 1 (mod 4), respectiv c = 0 daca p ≡ 3

(mod 4). Inlocuind ın (14.18), deducem ca multimea din enunt are 1 +[p

4

]elemente.

Solutia 2. Extinzand definitia simbolului lui Legendre

(a

p

)prin

(a

p

)= 0 daca p | a

si folosind faptul ca numarul de resturi patratice (nenule) modulo p este egal cu cel alneresturilor patratice, obtinem relatia

p−1∑a=0

(a− kp

)pentru orice k ∈ Z. (14.19)

In continuarea acestei solutii, vom nota cu [P ] ∈ 0, 1 valoarea de adevar a propozitieiP . Notam si Z2

p = x2 | x ∈ Zp. Cu aceste notatii, avem de determinat

N =

p−1∑a=0

[a ∈ Z2p] · [a− 1 ∈ Z2

p].

Page 253: Teme si probleme pentru concursurile studentesti de matematica ...

246

Observand ca

[a ∈ Z2p] =

1

2

(1 +

(a

p

)+ [a = 0]

),

obtinem

N =1

4

(1 +

(−1

p

)+ 1 +

(1

p

)+

p−1∑a=0

(1 +

(a

p

))(1 +

(a− 1

p

)))=

=1

2

[(−1

p

)= 1

]+

1

2+

1

4

p−1∑a=0

(1 +

(a

p

)+

(a− 1

p

)+

(a

p

)(a− 1

p

)).

Aplicand de doua ori relatia (14.19), gasim

N =1

2

[(−1

p

)= 1

]+

1

2+p

4+

1

4

p−1∑a=0

(a

p

)(a− 1

p

). (14.20)

Pentru k ∈ Z, notam S(k) =

p−1∑a=0

(a

p

)(a− kp

). Pentru a determina N , avem nevoie de

S(1). Daca p - k, atunci

S(k) =

p−1∑b=0

(kb

p

)(k(b− 1)

p

)=

p−1∑b=0

(k

p

)(b

p

)(k

p

)(b− 1

p

)=

=

p−1∑b=0

(b

p

)(b− 1

p

)= S(1). In plus,

p−1∑k=0

S(k) =

p−1∑a=0

(a

p

) p−1∑k=0

(a− kp

)= 0. Prin urmare,

S(1) = − S(0)

p− 1= −p− 1

p− 1= −1. Introducand ın (14.20), obtinem

N =1

2

[(−1

p

)= 1

]+p+ 1

4. (14.21)

Cum N ∈ Z, rezulta ca daca p ≡ 1 (mod 4), atunci(−1p

)= 1, iar daca p ≡ 3 (mod 4),

atunci(−1p

)= −1; introducand aceste valori ın (14.21) obtinem N = 1 +

[p4

].

Observatie. Ambele metode de abordare au condus la redemonstrarea proprietatii(−1p

)= (−1)

p−12 , prezentata ın introducerea capitolului, si care ar fi putut fi utilizata

pentru a trage concluzia direct din relatiile (14.18), respectiv (14.21). Am preferat abor-darea prezentata tocmai pentru a sublinia ca proprietatea mentionata nu este necesarapentru completarea rationamentului, ci consecinta a acestuia.

Problema 14.21 Aratati ca pentru orice n ∈ N∗ are loc relatia

n! =n∏i=1

L(

1, 2, . . . ,[ni

]), (14.22)

L(a1, a2 . . . , ak) desemnand ın aceasta problema cel mai mic multiplu comun al numerelorıntregi a1, a2 . . . , ak.

Putnam, 2003

Page 254: Teme si probleme pentru concursurile studentesti de matematica ...

Aritmetica si teoria numerelor 247

Solutia 1. Este suficient sa aratam ca pentru orice numar prim p exponentii lui p dindescompunerile ın factori primi ale celor doi membri ai relatiei (14.22) coincid.Conform proprietatilor functiei lui Legendre (a se vedea introducerea capitolului), expo-

nentul la care apare p ın descompunerea lui n! este

n∑i=1

[n

pi

]. Acest numar poate fi inter-

pretat ca fiind cardinalul multimii S a punctelor din primul cadran care au coordonateleın N∗ si se afla fie pe curba C data de ecuatia y = np−x, fie ıntre C si axele de coordonate;

fiecare termen

[n

pi

]al sumei reprezinta numarul de puncte din S care au abscisa i.

Pe de alta parte, exponentul lui p din descompunerea ın factori primi a lui

m(

1, 2, . . . ,[ni

])este

[logp

[ni

]]=[logp

(ni

)]. Acesta este ınsa exact numarul de puncte

din S care au ordonata i. In concluzie,

n∑i=1

[logp

[ni

]]=

n∑i=1

[n

pi

], adica egalitatea dorita.

Solutia 2. Vom demonstra relatia ceruta prin inductie dupa n. Ea este evidenta pentrun = 1. Pasul de inductie este imediat daca utilizam identitatea

n =n−1∏i=1

L(1, 2, . . . ,

[ni

])L(1, 2, . . . ,

[n−1i

]) . (14.23)

Ramane deci sa probam aceasta identitate; notam cu P produsul din membrul sau drept.Remarcam ca al i-lea factor din P este 1 daca n

i 6∈ N (deci, daca ni nu este divizor al lui n)

sau daca ni este divizor al lui n, dar nu este putere de numar prim (deoarece orice numar

k ∈ N∗ \1 care nu este putere de numar prim divide L(1, 2, . . . , k−1)). Pe de alta parte,daca n

i este putere a numarului prim p, atunci cel de-al i-lea factor al lui P este egal cu p.

Fie acum p ∈ N un numar prim arbitrar. Intrucat ni parcurge toti divizorii proprii ai lui

n, P contine cate un factor egal cu p pentru fiecare k ∈ N∗ pentru care pk | n. In plus, Pnu contine alti factori divizibili prin p. Din aceste motive, P coincide cu descompunereaın factori primi a lui n, ceea ce ıncheie demonstratia.

Problema 14.22 Pentru α ∈ R∗+ definim S(α) = [nα] | n ∈ N∗. Aratati ca N∗ nu sepoate scrie ca reuniunea disjuncta a trei multimi nevide S(α), S(β) si S(γ) cu α, β, γ ∈ R∗+.

Putnam, 1995

Solutie. Presupunem ca exista α, β, γ ∈ R∗+ astfel ıncat S(α), S(β) si S(γ) sa repre-zinte o partitie a lui N∗. Atunci, 1 apartine uneia dintre aceste multimi, fie ea S(α). Prinurmare, exista n ∈ N∗ astfel ıncat nα < 2. De aici rezuta α < 2; pe de alta parte, α > 1,deoarece altfel am avea S(α) = N∗, ın contradictie cu ipoteza.Consideram m ∈ N, m ≥ 2, pentru care 1+ 1

m ≤ α < 1+ 1m−1 . Atunci, [kα] = k pentru orice

k ∈ 1, 2, . . . ,m− 1, iar [mα] = m+ 1, deci m este cel mai mic element din S(β)∪ S(γ).In plus, orice doua elemente consecutive ale multimii S(β) ∪ S(γ) difera prin m sau prinm+ 1.Putem presupune, fara a restrange generalitatea, ca m ∈ S(β). Vom avea deci [β] = m.Fie n un element din S(γ). Conform celor de mai sus, elemente cele mai apropiate de nale lui S(β) se afla la distanta cel putin m de n, deci distanta dintre ele este de cel putin2m. Cum ınsa [β] = m, doua elemente consecutive din S(β) difera prin cel mult m + 1,contradictie.

Observatie. De fapt, nu exista trei multimi S(α), S(β) si S(γ) ca ın enunt care sa fiedisjuncte doua cate doua.

Page 255: Teme si probleme pentru concursurile studentesti de matematica ...

248

Fie N ∈ N, N > maxα, β, γ. Consideram tripletele vn =(

,nβ

,nγ

)∈ R3,

n ∈ 0, 1, . . . , N3. Daca divizam cubul unitate din R3 ın N3 cuburi de latura 1N , atunci,

conform principiului cutiei, cel putin doua dintre aceste triplete, fie ele vi si vj , se vor gasiın acelasi cub din diviziune. Fie k = |i − j|. Atunci, k

α este la distanta cel mult 1N de un

numar ıntreg m, deci [mα] este egal cu k− 1 sau cu k. Altfel spus, S(α) contine fie k− 1,fie k. Se arata ın mod analog ca S(β) si S(γ) contin k − 1 sau k. Asadar, cel putin unuldintre numerele k−1 si k se gaseste ın cel putin doua dintre multimile S(α), S(β) si S(γ).Prin urmare, aceste multimi nu pot fi disjuncte doua cate doua.

Problema 14.23 Fie A ⊂ N∗ nevida si N(x) = a ∈ A| a ≤ x. Notam cu B multimeanumerelor naturale nenule b care pot fi scrise sub forma a − a′ cu a, a′ ∈ A. ScriemB = b1, b2, . . ., cu bi < bi+1 pentru orice i ≥ 1. Aratati ca daca sirul (bi+1 − bi)i∈N∗ este

nemarginit, atunci limx→∞

N(x)

x= 0.

Putnam, 2004

Solutia 1. Sa presupunem pentru ınceput ca exista numere naturale nenule b0 = 1,b1, . . . , bn cu proprietatile:

(pn) Pentru orice i ∈ 1, 2, . . . , n, numarul ci = bi2bi−1

este natural.

(qn) Oricare ar fi e1, . . . , en ∈ −1, 0, 1, |e1b1 + e2b2 + · · ·+ enbn| 6∈ B.Atunci, fiecare numar natural a va admite o scriere unica de tipul

a = a0b0 + a1b1 + · · ·+ an−1bn−1 +mbn

cu 0 ≤ ai < 2ci+1 pentru orice i ∈ 0, 1, . . . , n − 1. Prin urmare, oricare ar fi di ∈0, 1, . . . , ci − 1 (i ∈ 1, 2, . . . , n− 1), m0 ∈ 0, 1, . . . , 2c0 − 1 si mn ∈ N, multimea

m0b0 + (2d1 + e1)b1 + · · ·+ (2dn−1 + en−1)bn−1 + (2mn + en)bn,

unde ei ∈ 0, 1 pentru orice i ∈ 1, 2, . . . , n, contine cel mult un element din A. De

aici rezulta ca lim supx→∞

N(x)

x≤ 1

2n. Aceasta relatie arata ca, daca reusim sa construim

pentru fiecare n ∈ N∗ cate un sistem b0 = 1, b1, . . . , bn cu proprietatile (pn) si (qn), atunci

0 ≤ lim supx→∞

N(x)

x≤ 1

2npentru orice n ∈ N∗, de unde rezulta afirmatia problemei.

Vom construi inductiv sisteme b0 = 1, b1, . . . , bn cu proprietatile (pn) si (qn). Sistemulformat din b0 = 1 are ın mod trivial proprietatile (p0) si (q0). Fie acum sistemul b0 =1, b1, . . . , bn cu proprietatile (pn) si (qn). Este imediat faptul ca b0 + b1 + · · ·+ bn−1 < bn.Conform ipotezei problemei, putem gasi o multime Sn ⊂ N \ B formata din 6n numereconsecutive. Notam cu bn+1 cel de-al doilea (ın ordine crescatoare) multiplu de 2bn dinSn. Atunci, este evident ca pentru orice x ∈ −2bn,−2bn + 1, . . . , 0 avem bn+1 + x ∈Sn. Din definitia lui Sn obtinem si bn+1 + x ∈ Sn pentru orice x ∈ 0, 1, . . . , 2bn. Deaici si din faptul ca sistemul b0, b1, . . . , bn are proprietatea (qn), rezulta ca sistemul b0 =1, b1, . . . , bn, bn+1 are proprietatea (qn+1). Dar b0 = 1, b1, . . . , bn, bn+1 are proprietatea(pn+1) prin constructie, deci e un sistem de tipul cautat. Acest fapt ıncheie pasul deinductie si solutia.

Solutia 2. Fie S multimea valorilor pe care le poate lua lim supx→∞

N(x)

xpentru diversele

multimi A; cum S ⊂ [0, 1], ea este marginita; punem L = supS.

Presupunem ca L > 0. Exista atunci A si B ca ın enunt astfel ıncat lim supx→∞

N(x)

x>

Page 256: Teme si probleme pentru concursurile studentesti de matematica ...

Aritmetica si teoria numerelor 249

3L

4. Din conditia de nemarginire din enunt rezulta ca exista m ∈ N∗ \ B. Atunci, A si

A + m sunt disjuncte. Notam A′ = A ∪ (A + m) si N ′(x) = |1, 2, . . . , x ∩ A′|. Atunci,

lim supx→∞

N ′(x)

x>

3L

2> L, deci A′ nu poate verifica conditiile din enunt. Prin urmare, daca

notam B′ = a′−a′′ | a′, a′′ ∈ A′, rezulta ca exista N ∈ N∗ astfel ıncat din orice N numerenaturale consecutive sa se gaseasca macar unul ın B′. Dar

B′ ⊂ b+ em | b ∈ B, e ∈ −1, 0, 1,

de unde rezulta ca din orice n+ 2m numere naturale consecutive se va gasi ın B cel putinunul, contradictie.

Ramane deci ca nu putem avea L > 0. Prin urmare, L = 0, de unde limx→∞

N(x)

x= 0.

Page 257: Teme si probleme pentru concursurile studentesti de matematica ...

Bibliografie

[1] L. Ahlfors, Complex Analysis, McGraw-Hill, 1979.

[2] T. Andreescu, R. Gelca, Putnam and Beyond, Springer, 2007.

[3] G. Berge, Graphs, North-Holland, 1985.

[4] Gh. Bucur, E. Campu, S. Gaina, Culegere de probleme de calcul diferential si integral,III, Ed. Tehnica, Bucuresti, 1967.

[5] R. Diestel, Graph Theory, Springer, 1997.

[6] S. Francinou, H. Gianella, S. Nicolas, Exercices de mathematiques des oraux del’Ecole Polytechnique et des Ecoles Normale Superieures, Cassini, Paris, 2001.

[7] D. Flondor, N. Donciu, Algebra si analiza matematica. Culgere de probleme, vol. Isi II, Ed. Didactica si Pedagogica, Bucuresti, 1979.

[8] L.C. Florescu, Analiza matematica, Ed. Universitatii ,,Al.I. Cuza” Iasi, 1999.

[9] A. Gibbons, Algorithmic Graph Theory, Cambridge Univ. Press, 1985.

[10] N.V. Ghircoiasu, C. Miron, Grafuri de fluenta si aplicatii ın tehnica, EdituraTehnica, Bucuresti, 1974.

[11] M. Ivan, Elemente de calcul integral, Ed. Mediamira, 2003.

[12] W. Kaczor, M. Nowak, Problems in Mathematical Analysis, vol. I, II, III, A.M.S.,2003.

[13] G. Klambauer, Problems and Propositions in Analysis, Marcel Dekker, 1979.

[14] B. Makarov, M. Goluzina, A. Lodkin, A. Podkorytov, Selected Problems in RealAnalysis, A.M.S., 2000.

[15] R. Merris, Combinatorics, Willey-Interscience, 2003.

[16] T. Needham, Visual Complex Analysis, Clarendon Press, 1997.

[17] L. Panaitopol, A. Gica, O introducere ın aritmetica si teoria numerelor, Ed. Univer-sitatii din Bucuresti, 2001

[18] L. Panaitopol, A. Gica, Probleme de aritmetica si teoria numerelor, Ed. Universitatiidin Bucuresti, 2006.

[19] G. Pavel, F.I. Tomuta, I. Gavrea, Matematici speciale, Ed. Dacia, Cluj-Napoca, 1981.

250

Page 258: Teme si probleme pentru concursurile studentesti de matematica ...

BIBLIOGRAFIE 251

[20] V. Pop, Geometrie combinatorica, Editura Mediamira, Cluj-Napoca, 2010.

[21] D. Popa, Calcul integral, Ed. Mediamira, 2005.

[22] E. Popa, Introducere ın teoria functiilor de o variabila complexa, Ed. Universitatii,,Al. I. Cuza” Iasi, 2001.

[23] E. Popa, Analiza matematica, Ed. GIL, 2005.

[24] A. Precupanu, Bazele analizei matematice, Ed. Canova, Iasi, 1995.

[25] E. Ramis, C. Deschamps, J. Odoux, Analyse, vol. 1 si 2, Ed. Masson, Paris, 1993.

[26] V. Rudner, C. Nicolescu, Probleme de matematici speciale, Ed. Didactica si Peda-gogica, Bucuresti, 1982.

[27] P.N. de Souza, J.-N. Silva, Berkeley Problems in Mathematics, Springer, 2004.

[28] G. Szekely (ed.), Contests in Higher Mathematics, Miklos Schweitzer Competitions1962-1991, Springer, 1996.

[29] T. Trif, Probleme de calcul diferential si integral ın Rn, Univ. Babes-Bolyai, 2003.

[30] I. Tomescu, Introducere ın combinatorica, Ed. Tehnica, 1972.

[31] I. Tomescu, Probleme de combinatorica si teoria grafurilor, Ed. Didactica si Peda-gogica, Bucuresti, 1981.

[32] N. Vornicescu, M. Ivan, V. Popa, V. Pop, Calcul diferential, Ed. Mediamira, 2004.

Concursuri

[33] Ariel: Internet Mathematical Olympiad for Students, 2008-2011.

[34] IMC: International Mathematics Competition for University Students, 1994-2011.

[35] Iran: Iranian University Students Mathematics Competitions, 1973-2011.

[36] Vojtech Jarnik: Vojtech Jarnik International Mathematical Competition, 1991-2011.

[37] Putnam: William Lowell Putnam Mathematical Competition, 1938-2010.

[38] SEEMOUS: South Eastern European Mathematical Olympiad for University Stu-dents, 2007-2011.